Вы находитесь на странице: 1из 284

IJSO (STAGE-1 & 2)

CHEMISTRY
PREFACE

Dear Student,

In the current competitive environment, there are certain students who always aspire for more.
These students are those who have extremely good fundamentals and do not face any problem in
attempting the regular school book problems. This study material is for those students.

This material is aimed at equipping the students with first and second stage of International

Junior Science Olympiad (IJSO) i.e. National Standard Examination in Junior Science (NSEJS)
and INJSO (Indian National Junior Science Olympiad). The best use of this material is for those
students who have natural interest in problem solving and want to aspire for more.

As this material is exam oriented rather than class oriented, there is no specific class wis target.
However, based on our prior experience, this material can be best utilized by students of class
VIII, IX and X.

We hope you this material in its accuracy and appropriateness.

Team Resonance.

Pre-foundation Career Care Programmes (PCCP) Division

Every effort has been taken to make our study material error free, however any suggestion to
improve is welcome in this regard.
SUBJECT : CHEMISTRY IJSO (STAGE-1 & 2)

CONTENTS
S. NO. TOPICS PAGE NO.

1. Mole concept 1 - 22

2. Equivalent concept 23 - 36

3. Structure of Atom 37 - 50

4. Periodic table 51 - 69

5. Matter 70 - 91

6. Acids bases and salts 92 - 107

7. Metals and non metals 108 - 126

8. Carbon and its compounds 127 - 146

9. Study of gas laws 147 - 159

10. Chemical and ionic equilibrium 160 - 177

11. Chemical kinetics 178 - 190

12. Nuclear chemistry 191 - 203

13. Thermodynamics 204 - 220

14. Chemical bonding 221 - 235

15. Qualitative analysis 236 - 264

16. Electrochemistry 265 - 275

17. Answer key 276 - 281

© Copyright reserved

All right reserved. Any photocopying, publishing or reproduction of full or any part of this study
material is strictly prohibited. This material belongs to only the enrolled student of RESONANCE.
Any sale/resale of this material is punishable under law.
MOLE CONCEPT

Mass of one atom of an element


DALTON 'S ATOMIC THEORY 1
 mass of one oxygen atom
16
  The present standard unit which was adopted
internationlly in 1961, is based on the mass of one
Dalton’s atomic theory was given by John Dalton in 1808.
carbon-12 atom.
The main postulates of Dalton’s atomic theory are:
1. Elements consist of indivisible, indestructible particles Atomic Mass Unit (or amu) :
called atom. The atomic mass unit (amu) is equal to one twelth
2. Atoms of an element are indentical to each other. They  1
have the same mass and size.   the mass of one atom of carbon-12 isotope.
3. Atoms of different elements differ in properties and have
 12 
different masses and sizes. 1
 1 amu = × mass of one C-12 atom
4. Compounds are formed when atoms of different 12
elements combine with each other in simple ~ mass of one nucleon in C-12 atom.
numerical ratio such as 1 : 1, 2 : 2, 2 : 3 and so on. = 1.66 × 10–24 gm or 1.66 × 10–27 kg
5. Atoms cannot be created, destroyed or transofrmed into  one amu is also called one Dalton (Da).
atoms of other elements. Today, amu has been replaced by ‘U’ which is known
as unified mass
The main failures of Dalton’s atomic theory are :
1. It failed to explain how atoms of different elements differ Now the relative atomic mass is given as
from each other i.e., did not tell anything about structure Relative atomic mass
of the atom. mass of one atom of the element
2. It could not explain how and why atoms of different = 1
element combine with each other to form compound.  mass of one C  12 atom
12
3. It failed to explain the nature of forces that bind together
different atoms in a molecule. Atomic mass
4. It failed to explain Gay Lussac’s law of combining R.A.M. =
1 amu
volumes. Atomic mass = R.A.M × 1 amu
5. It did not make any distinction between ultimate particle Relative molecular mass
of an element that takes part in reaction (atoms) a n d
ultimate particle that has independent existence mass of one molecule of the subs tan ce
(molecules). =
1
 mass of one  C  12 atom
Significant Figures : The uncertainty in the experimental 12
or the calculated values is indicated by mentioning the  Molecular mass = Relative molecular mass × 1 amu
number of significant figures. Significant figures are
meaningful digits which are known with certainty. The Atomic mass : It is the mass of 1 atom of a substance it is
uncertainty is indicated by writing the certain digits and expressed in amu.
the last uncertain digit. ATOMIC MASS = R.A.M × 1 amu
Precision : Precision refers to the closeness of various Note : Relative atomic mass is nothing but the number of
measurements for the same quantity. nucleons present in the atom.
Accuracy : Accuracy is the agreement of a particular
Ex.1 Find the relative atomic mass of ‘O’ atom and its atomic
value to the true value of the result.
mass.
Relative Atomic Mass : Sol. The number of nucleons present in ‘O’ atom is 16.
One of the most important concept which come out from  relative atomic mass of ‘O’ atom = 16.
Dalton’s atomic theory was that of relative atomic mass Atomic mass = R.A.M × 1 amu = 16 × 1 amu = 16 amu
or relative atomic weight. This is done by expressing
Average / Mean Atomic Mass :
mass of one atom with respect to a fixed standard. Dalton
The weighted average of the isotopic masses of the
used hydrogen as the standard (H = 1). Later on oxygen
element’s naturally occuring isotopes.
(O = 16) replaced hydrogen as the reference. Therefore
Mathematically, average atomic mass of X (A x) =
relative atomic mass is given as
Relative atomic mass (R.A.M) = a1x 1  a 2 x 2  .....  a n x n
Mass of one atom of an element 100
mass of one hydrogen atom =
a1 = atomic mass ; x1 % occurence in nature

Ex.1 Naturally occuring chlorine is 75% Cl35 which has an


atomic mass of 35 amu and 25% Cl37 which has a

PAGE # 1
Pre-foundation Career Care Programmes (PCCP) Division
mass of 37 amu. Calculate the average atomic mass of Example : O2, H2, Cl2 etc.
chlorine - (ii) Heteroatomic molecules : – Molecules of compounds
(1) 35.5 amu (B) 36.5 amu containing more than one type of atoms.
(C) 71 amu (D) 72 amu Example : NH3, H2O, CH4 etc.
Sol. (1) Average atomic mass = Ex.1.The molecular mass of H2SO4 is 98 amu. Calculate the
75 x 35  25 x 37 number of moles of each element in 294 g of H2SO4.
= = 35.5 amu Sol. Gram molecular mass of H2SO4 = 98 gm
100
Note :(a) In all calculations we use this mass. 294
(b) In periodic table we report this mass only. moles of H2SO4 = = 3 moles
98
H2SO4 H S O
One molecule 2 atom one atom 4 atom
1 × NA 2 × NA atoms 1 × NA atoms 4 × NA atoms
MOLE  one mole 2 mole one mole 4 mole
 3 mole 6 mole 3 mole 12 mole
Mole : The Mass / Number Relationship
Mole is a chemical counting SI unit and defined as follows : Gram Molecular Mass :
The molecular mass of a substance expressed in gram
A mole is the amount of a substance that contains as
is called the gram-molecular mass of the substance.
many entities (atoms, molecules or other particles) as
or
there are atoms in exactly 0.012 kg (or 12 gm) of the
It is also defined as mass of 6.02 × 1023 molecules
carbon-12 isotope.
or
From mass spectrometer we found that there are 6.023 It is also defined as the mass of one mole of molecules.
× 1023 atoms present in 12 gm of C-12 isotope.
The number of entities in 1 mol is so important that it is For example for ‘O2’ molecule :
given a separate name and symbol known as Avogadro Molecular mass of ‘O 2’ molecule = mass of one ‘O 2 ’
constant denoted by NA. molecule
Note : In modern practice gram-atom and gram- = 2 × mass of one ‘O’ atom
molecule are termed as mole. = 2 × 16 amu = 32 amu
gram molecular mass = mass of 6.02 × 10 23 ‘O 2 ’
Gram Atomic Mass : molecules = 32 amu × 6.02 × 1023
The atomic mass of an element expressed in gram is = 32 × 1.66 × 10–24 gm × 6.02 × 1023 = 32 gm
called gram atomic mass of the element.
Mean Molar Mass or Molecular Mass :
or
The average molar mass of the different substance
It is also defined as mass of 6.02 × 1023 atoms.
or n1M1  n2M2  ......nnMn
It is also defined as the mass of one mole of atoms. present in the container =
n1  n 2  ....nn
For example for oxygen atom : n1 = Number of moles ; M = Molar mass

Atomic mass of ‘O’ atom = mass of one ‘O’ atom = 16 Solved Examples
amu 1. The molar composition of polluted air is as follows :
gram atomic mass = mass of 6.02 × 1023 ‘O’ atoms Gas At. wt. mole percentage composition
= 16 amu × 6.02 × 1023 Oxygen 16 16%
= 16 × 1.66 × 10–24 g × 6.02 ×1023 = 16 g Nitrogen 14 80%
( 1.66 × 10–24 × 6.02 × 1023 ~ 1 ) Carbon dioxide - 03%
Sulphurdioxide - 01%
Ex.1 How many atoms of oxygen are their in 16 g oxygen. W hat is the average molecular weight of the given
Sol. Let x atoms of oxygen are present polluted air ? (Given, atomic weights of C and S are 12
So, 16 × 1.66 × 10–24 × x = 16 g and 32 respectively.
1 j n
x= = NA
1.66 x 10 24 n M
j1
j j j n
MOLECULE : It is the smallest particle of matter which is
capable of independent existence.
Sol. Mavg = jn Here n j1
j = 100

 A molecule is generally an assembly of two or more n


j 1
j
tightly bonded atoms.
 Molecules are of two type on the basis of elemental
atoms.
16 x 32  80 x 28  44 x 3  64 x 1
 Mavg =
(i) Homo atomic molecules : Molecules of an element 100
containing one type of atoms only.

PAGE # 2
Pre-foundation Career Care Programmes (PCCP) Division
Since atomicity of H2SO4 = 7
512  2240  132  64 2948
= = = 29.48 Ans. Then total number of moles of atoms = n × atomicity =
100 100 0.5 × 7 = 3.5 moles
3. What is the mass of 1 molecule of CO
AVOGADRO'S HYPOTHESIS
(1) 4.65 × 10–23 gm (B) .465 × 10–23 gm
Equal volume of all gases have equal number of (C) 4.65 × 10–22 gm (D) 4.65 × 10–24 gm
molecules (not atoms) at same temperature and Ans. (1)
pressure condition. Sol. Gram molecular weight of CO = 12 + 16 = 28 g
6.023 × 1023 molecules of CO weight 28 g
S.T.P. (Standard Temperature and Pressure) 1 molecule of CO weighs
At S.T.P. condition : temperature = 0°C or 273 K
pressure = 1 atm = 760 mm of Hg 28 4.65  10 23 g
and volume of one mole of gas at STP is found to be = 6.021023
experimentally equal to 22.4 litres which is known as
molar volume. 4. Calculate the volume at STP occupied by 240 gm of SO2
(1) 84 (B) 44.8
Note : Measuring the volume is equivalent to counting the
(C) 22.4 (D) 168 Ans. (1)
number of molecules of the gas.
Sol. Molecular weight of SO2 = 32 + 2 × 16 = 64
Ex.1 Calculate the volume in litres of 20 g hydrogen gas at 64 g of SO2 occupies 22.4 litre at STP
STP.
Sol. No. of moles of hydrogen gas = mass  atomic weight = 22.4
240 g of SO2 occupies =  240  84 litre at STP
64
20 gm
= 10 mol 5. 6 × 10 20 molecules of CO 2 are removed from 220
2 gm
milligram of CO2. What are the remaining moles of CO2
volume of hydrogen gas at STP = 10 × 22.4 lt. (1) 4 × 10–3 moles (B) 5 × 10–3 moles
–3
Y-map : Interconversion of mole - volume, mass and (C) 2 × 10 moles (D) 6 × 10–3 moles Ans. (1)

number of particles : 220  10 3


Sol. Mole of 220 mg. of CO2 = = 5 × 10–3 moles;
44
Number Volume at STP
×N
2.4
lt 6  10 20
A
×2 Moles of CO2 removed are = = 10–3 moles
 lt 6  10 23
N 2.4
A
2
Mole Remaining moles of CO2 = [5 × 10–3 – 10–3] = 4 × 10–3
moles
Density :
 mol. wt. × mol. wt. It is of two type.
 At. wt. × At. wt.
 Absolute density
 Relative density
Mass For Liquid and Solids
mass
 Absolute density =
Solved Examples volume
 Relative density or specific gravity =
1. Calculate the number of gram atoms for 2 × 1023 atoms.
If atomic weight of element is 24, then calculate mass of density of the subs tance
sample density of water at 4C
(1) 7.92 gm (B) 79.2 gm We know that density of water at 4ºC = 1 g/ml.
(C) 0.792 gm (D) 0.0792 gm Ans. (1)
For Gases :
X 2  10 23 Molar mass
Sol. Number of gram atoms of element = 
N A 6.023  10 23  Absolute density (mass/volume) =
Molar volume
= 0.33 moles. where P is pressure of gas, M = mol. wt. of gas, R is the
Mass of 2 × 1023 atoms = n × GAM gas constant, T is the temperature.
= 0.33 × 24 = 7.92 gm
RELATIVE DENSITY OR VAPOUR DENSITY
2. Calculate total number of moles of atoms present in 49 Vapour density is defined as the density of the gas with
gm H2SO4. respect to hydrogen gas at the same temperature and
(1) 0.5 (B) 3.5 pressure.
(C) 5 (D) 4 Ans. (B)
dgas PM gas / RT
w 49 Vapour density = d = PM / RT
Sol. Number of mole for 49 gm H2SO4 = = = 0.5 H2 H 2
GMM 98
mole Mgas M gas
V.D. = M =
H2 2
PAGE # 3
Pre-foundation Career Care Programmes (PCCP) Division
Mgas = 2 V.D. Solved Examples
1. What weight of BaCl2 would react with 24.4 g of sodium
Relative density can be calculated w.r.t. to other gases sulphate to produce 46.6 g of barium sulphate and 23.4
also. g ofsodium chloride ?
Solved Examples Ans. Barium chloride and sodium sulphate react to produce
1. What is the V.D. of SO2 with respect to CH4 barium sulphate and sodium chloride according to the
equation :
M.W. SO 2 64
Sol. V.D. = V.D = =4 BaCl2 + Na2SO4  BaSO4 + 2NaCl
M.W . CH4 16
xg 24.4 g 46.5 g 23.4 g
Let the weight of BaCl2 be x g. According to law of
2. 7.5 litre of the particular gas at S.T.P. weighs 16 gram. conservation of mass :
What is the V.D. of gas Mass of reactants = Mass of products
Sol. 7.5 litre = 16 gram Mass of reactants = (x + 24.4) g
Mass of products = (46.6 + 23.4) g
7.5 16 48
moles =  M = 48 gram V.D. = 24 Equating the two masses
22.4 M 2 x + 24.4 = 46.6 + 23.4
x = 46.6 + 23.4 – 24.4 or x = 45.6 g
LAWS OF CHEMICAL COMBINATION hence, the weight of BaCl2 is 45.6 g
(A) law of conservation of mass/Law of indestructibility
of matter 2. 10 g of CaCO 3 on heating gives 4.4 g of CO 2 then
Given by – Lavoisier determine weight of produced CaO in quintal.
Tested by – Landolt Ans. Total mass of reactant = 10 g
Statement : According to law of conservation of mass in Mass of CO2 = 4.4 g
all physical & chemical changes total mass of the Mass of produced CaO = x
system remains constant. or According to law of conservation of mass
In a physical or chemical change mass is neither be 10 = 4.4 + x
created nor destroyed. 10 – 4.4 = x
i.e. Total mass of reactant = Total mass of the product x = 5.6 g
 This relationship hold good when reactant are  1 quintal = 100 kg
completely converted into products.  1 Kg = 1000 g
 If reactant are not completely consume then the Kg
= 5.6 g  = 5.6  10–3  Kg
relationship will be : 1000
Total mass of reactant = Total mass of product + Mass 1
= 5.6  10–3  quintal = 5.6  10–5 quintal
of unreacted reaction 100
Exception : Nuclear reaction are excepton of law of (B) Law of definite proportions/contant compostion :
conservation of mass. In nuclear reaction mass + energy Given by  Joseph Proust
is conserved. Statement : A chemical compounds always contains
Modern View : According to the modern views, the law same elements combined together in same proportion
of conservation of mass is not always valid. The law by mass. i,e, chemical compound has a fixed
hold good only in case of such chemical reactions where composition & it does not depends on the method of its
there is no evolution of heat or light. preperation or the source from which it has been
According to Einstein’s theory of relativity, mass and obtained.
energy are interconvertable according to the equation. For example, molecule of ammonia always has the
E = mc2 formula NH3. That is one molecule of ammonia always
where E = Energy content of the body contains. One atom of nitrogen and three atoms of
m = Mass of the body. hydrogen or 17.0 g of NH3 always contains 14.0 g of
C = Velocity of light (3  1010 cm/sec) nitrogen and 3 g of hydrogen.
E = (Change in mass)  c2 For Example : CO2 can be obtained by using any one of
As c is very large, change in mass is found to be the following method.
extermely small in normal reactions. (1) By burning of carbon in the presence of O2 :
During chemical processes, the loss of mass is C + O2  CO2
negligible. But in nuclear reactions, tremendous amount (B) By decomposition of CaCO3
of energy is evolved. Consequently, the change in mass
CaCO3  CaO + CO2 
is quite significant. Thus, it is clear that the law of
conservation of mass and law of conservation of energy (C) CaCO3 + 2 HCl  CaCl2 + H2O + CO2 
are two ways of looking at the same law. In CO2 wt. ratio = C : O
Therefore, combining the two we get general law known = 12 : 32
as law of conservation of mass energy. It states that, =3:8
Mass and energy are inter convertible. But the total sum W hatever sample of CO 2 is taken it is observed that
of mass and energy of the system remains constant. carbon & oxygen are always combined in the ratio of 12
: 32 i.e. 3 : 8.

PAGE # 4
Pre-foundation Career Care Programmes (PCCP) Division
Solved Examples N 2 O3 28 : 48 = 14 : 24
1. Weight of copper oxide obtained by treating 2.16 g of N 2 O4 28 : 64 = 14 : 32
metallic copper with nitric acid and subsequent ignition N 2 O5 28 : 80 = 14 : 40
was 2.70 g. In another experiment, 1.15 g of copper oxide Number of parts by weight of oxygen which combine
on reduction yielded 0.92 g of copper. Show that the with 14 parts by weight of nitrogen from the above are
results illustrate the law of constant composition. 8,16,24,32 and 40 respectively. Their ratio is 1 : 2 : 3 : 4
Ans. Ist experiment : 5, which is a simple ratio. Hence the law illustrated.
Mass of copper oxide = 2.70 g (ii) Sulphur combines with oxygen to from two oxides,
 Mass of oxygen = 2.70 – 2.16 = 0.54 g viz SO2 and SO3. The weights of oxygen which combine
The ratio of the masses of copper and oxygen in the Ist with a fixed weight, i.e. 32 parts by weight of sulphur in
experiment is 2.16 : 0.54 = 4 : 1 two oxides are in the ratio of 32 :48 or 2 : 3 which is
IInd experiment simple ratio. Hence the law of multiple proportions is
Mass of copper oxide = 1.15 g ;Mass of copper = 0.92 g illustrated.
 Mass of oxygen = 1.15 – 0.92 = 0.23 g Solved Examples
The ratio of the masses of copper and oxygen in the IInd 1. On analysis it was found that the black oxide of copper
experiment is 0.92 : 0.23 = 4 : 1 and the red oxide of copper contain 79.9% and 88.8%
Thus the ratio of the masses of copper and oxygen in metal respectively. Establish the law of multiple
the two experiment is the same, i.e. 4 : 1. Hence , the law proportions with the help of this data.
of constant composition is proved. Ans. In the black oxide,
2. Illustrate the law of definite proportions from the following 79.9 g copper combines with (100 – 79.9), i.e. 20.1 g
data : oxygen
(a) 0.32 g of sulphur on burning in air produced 224 ml  In red oxide 88.8 g copper will combine with 100 –
of SO2 at NTP. 88.8 = 11.2 g
(b) Sulphur dioxide obtained by the decomposition of  According to red oxide 79.9 copper will combine with
sulphate contains 50% of sulphur 11.2  79.9
Ans. Molecular weight of SO2 = 32 + 32 = 64 = 10.08 g oxygen
88.8
224  64 Thus the weights of oxygen that combine with the same
224 ml of SO2 at NTP will weight = = 0.64 79.9 g copper are 20.1 g and 10.08 respectively. These
22400
(a) In first Experiment are in the ratio 20.1 : 10.08 = 2 : 1
64 g of SO2 contain = 32 g sulphur It is a simple whole number ratio. Hence, the law of
0.64  32 multiple proportions is established.
 0.64 g of SO2 will contain = = 0.32 g (D) law of reciprocal proportion :
64 Given by  Richter.
sulphur
The ratio of the weights of two elements A and B which
Similarly 64 g of SO2 contain = 32 g oxygen
combine separately with fixed weight of the third
0.64  32 element C is either the same or some simple multiple
0.64 g of SO2 will contain = = 0.32 Oxygen of this ratio of the weights in which A and B combine
64
directly with each other.
(b) As sulphur and oxygen are equal in amount in SO2
This law is illustrated with the help of the following
Percentage of sulphur in SO2 = 50% ; Percentage of
examples :
oxygen in SO2 = 50%
The elements C and O combine separately with the
Thus, the percentage of sulphur and oxygen in both the
third element H to form CH4 and H2O and they combine
experiments is same. This illustrates the law of constant
directly with each other to form CO2 as shown in the
composition.
adjoining figure.
(C) Law of multiple proportion : Given by  Daltons
Statement : When two elements combine to form more
than one compound than the differnt mass of one
element which combine with a fixed mass of other
element bear a simple ratio to one another.
The following examples will illustrate.
(i) Nitrogen and oxygen combine to form five oxides, In CH4, 12 parts by weight of carbon combine with 4
which are : parts by weight of hydrogen. In H2O, 2 parts by weight of
Nitrous oxide (N 2 O), nitric oxide (NO), nitrogen hydrogen combine with 16 parts by weights of oxygen.
trioxide(N2 O3), nitrogen tetraoxide (N2O4 ) and nitrogen Thus the weights of C and O which combine with fixed
pentoxide (N2O5). weight of hydrogen (say 4 parts ofweight) are 12 and
weights of oxygen which combine with the fixed weight of 32, i.e. they are in the ratio 12 : 32 or 3 : 8.
nitrogen in these oxides are calculatd as under : Now in CO 2 , 12 parts by weight of carbon combine
Oxide Ratio of weights of nitrogen and oxygen in each directly with 32 parts by weight of oxygen, i.e. they
compound combine directly in the ratio 12 : 32 or 3 : 8 which is the
N2 O 28 : 16 = 14 : 8 same as the first ratio.
NO 14 : 16 = 14 : 16 Solved Examples
1. Copper sulphide contains 66.6% Cu, copper oxide
contains 79.9% copper and sulphur trioxide contains

PAGE # 5
Pre-foundation Career Care Programmes (PCCP) Division
40% sulphur. Show that these data illustrate law of  40 ml of H2 will react with 40 ml of Cl2 and 80 ml of
reciprocal proportions. HCl will produce.
Sol. In copper sulphide, Required vol. of Cl2 = 40 ml
Cu : S mass ratio is 66.6 : 33.4 produced vol. of HCl = 80 ml.
In sulphur trioxide,
O : S mass ratio is 60 : 40
PERCENTAGE COMPOSITION
Now in copper sulphide Here we are going to find out the percentage of each
33.4 parts of sulphur combines with Cu = 66.6 parts element in the compound by knowing the molecular
40.0 parts of sulphur combines with Cu. formula of compound.
We know that according to law of definite proportions
66.6  40 any sample of a pure compound always possess
= = 79.8 parts
33.4 constant ratio with their combining elements.
Now ratio of the masses of Cu and O which combines
with same mas (40 parts) of sulphur separately is Ex.1 Every molecule of ammonia always has formula NH 3
79.8 : 60 irrespective of method of preparation or sources. i.e. 1
Cu : O ratio by mass in CuO is mole of ammonia always contains 1 mole of N and 3
79.9 : 20.1 mole of H. In other words 17 gm of NH3 always contains
14 gm of N and 3 gm of H. Now find out % of each
79.8 20.1 element in the compound.
Ratio I : Ratio II =  =3:1
60 79.9 Mass of N in 1 mol NH3
Sol. Mass % of N in NH3 =  100 =
Which is simple whole number ratio. Mass of 1 mol of NH3
hence, law of reciprocal proportion is proved.
14 gm
(E) Gay–Lussac’s law of gaseous Volume : 17 gm × 100 = 82.35 %
Given by  Gay Lussac
Mass of H in 1 mol NH3
Statement : When gases react with each other in simple Mass % of H in NH3 =  100 =
ratio of their volume & if product is also in gaseous state Mass of 1 mol e of NH3
then the volume of the product is also bear a simple 3
ratio to the volume of gaseous reactant when all volumes × 100 = 17.65 %
17
are measured under similar condition of temperature &
pressure. Chemical formula
The following exapmles illustrate the law : It is of two types :
(i) One volume of hydrogen combines with one volume (a) Molecular formulae : Chemical formulae that indicate
of chlorine to produce 2 volumes of hydrogen chloride. the actual number and type of atoms in a molecule called
are molecular formulae.
H2(g) + Cl2(g)  2HCl(g) Example : Molecular formula of Benzene is C6H 6
W e can see that ratio of the volumes of gaseous (b) Empirical formulae : Chemical formulae that indicate
reactants and products is simple. only the relative number of atoms of each type in a
(ii) 2 volumes of carbon monooxide combine with 1 molecule are called empirical formulae.
volume of oxygen to give 2 volumes of carbon dioxide Solved Examples
1. Acetylene and benzene both have the empirical formula
2CO + O2  2CO2
CH. The molecular masses of acetylene and benzene
(iii) One volume of nitrogen combines with 3 volumes of are 26 and 78 respectively. Deduce their molecular
hydrogen to form 2 volumes of ammonia. formulae.
N2 + 3H2  2NH3 Sol.  Empirical Formula is CH
Step-1 The empirical formula of the compound is CH
In all above examples, we can see that ratio of volumes
 Empirical formula mass = (1 × 12) + 1 = 13.
between the reactants and products is simple.
Molecular mass = 26
Thus, the law of gaseous volume is established. This
Step-2 To calculate the value of ‘n’
law is purely a generalisation from experimental data.
This law can not be proved from Dalton’s atomic theory Molecular mass 26
while others could be proved. n= = =2
Empirical formula mass 13
Step-3 To calculate the molecular formula of the compound.
Solved Examples
Molecular formula = n × (Empirical formula of the
1. For the gaseous reaction H2 + Cl2  2HCl compound)
If 40 ml of hydrogen completely reacts with chlorine then = 2 × CH = C2 H2
find out the required volume of chlorine and volume of Thus the molecular formula is C2 H 2
produced HCl ? Similarly for benzene
Ans. According to Guy Lussac’s Law : To calculate the value of ‘n’
H2 + Cl2  2HCl
Molecular mass 78
 1 ml of H2 will react will 1 ml of CL2 and 2 ml of HCl n = Empirical formula mass = =6
will he produced. 13
thus the molecular formula is 6 × CH = C6H6

PAGE # 6
Pre-foundation Career Care Programmes (PCCP) Division
2. Phosgene, a poisonous gas used during W orld war-I, Mass of KClO3 2  122.5
contains 12.1% C, 16.2% O and 71.7% Cl by mass. What
Mass of O2 = 3  32
is the empirical formula of phosgene.
(1) COCl2 (B) COCl  Mass - volume analysis :
(C) CHCl3 (D) C2O2Cl4 Ans (1) Now again consider decomposition of KClO3
2KClO3  2KCl
+ 3O2
Sol. mass volume ratio : 2 × 122.5 g : 2 × 74.5 g : 3
× 22.4 litre at STP
we can use two relation for volume of oxygen

Mass of KClO3 2  122 .5


=
volume of O 2 at STP 3  22.4 lt
...(i)
Then empirical formulae = COCl2 Mass of KCl 2  74 .5
and =
volume of O 2 at STP 3  22.4 lt
Chemical Reaction :
It is the process in which two or more than two substances ...(ii)
interact with each other where old bonds are broken and  Mole-mole analysis :
new bonds are formed. This analysis is very much important for quantative
analysis point of view. Students are advised to clearly
CHEMICAL EQUATION understand this analysis.
All chemical reaction are represented by chemical Now consider again the decomposition of KClO3 .
equations by using chemical formula of reactants and 2KClO3  2KCl + 3O2
products. Qualitatively a chemical equation simply In very first step of mole-mole analysis you should read
describes what the reactants and products are. However, the balanced chemical equation like
a balanced chemical equation gives us a lot of quantitative 2 moles KClO3 on decomposition gives you 2 moles
information. mainly the molar ratio in which reactants KCl and 3 moles O 2. and from the stoichiometry of
combine and the molar ratio in which products are reaction we can write
formed.
Moles of KClO3 Moles of KCl Moles of O 2
= =
Attributes of a balanced chemical equation: 2 2 3
(a) It contains an equal number of atoms of each element Now for any general balance chemical equation like
on both sides of equation.(POAC)
(b) It should follow law of charge conservation on either a A + b B  c C + d D
side. you can write.
(c) Physical states of all the reagents should be included
in brackets.
Moles of A reacted moles of B reacted
= =
(d) All reagents should be written in thier standard a b
molecular forms (not as atoms )
(e) The coefficients give the relative molar ratios of each moles of C reacted moles of D reacted
=
reagent. c d
Note : In fact mass-mass and mass-vol analysis are also
Interpretation of balanced chemical equations : interpreted in terms of mole-mole analysis you can use
Once we get a balanced chemical equation then we can following chart also.
interpret a chemical equation by following ways
 Mass - mass analysis
 Mass - volume analysis
 Mole - mole analysis
 Vol - Vol analysis (separately discussed as
eudiometry or gas analysis)
Now you can understand the above analysis by
following example
 Mass-mass analysis :
Consider the reaction
2KClO3  2KCl + 3O2
Solved Examples
According to stoichiometry of the reaction
1. 367.5 gram KClO3 (M = 122.5) when heated how many
mass-mass ratio : 2 × 122.5 2 × 74.5 : 3 × 32
gram of KCl and oxygen is produced.
Mass of KClO3 2  122.5 Sol. Balance chemical equation for heating of KClO3 is
or Mass of KCl = 2  74 .5
2KClO3  2KCl + 3O2

PAGE # 7
Pre-foundation Career Care Programmes (PCCP) Division
mass-mass ratio :2 × 122.5 g : 2 × 74.5 g : 3 × 32 g Solved Examples
1. Three moles of Na2 CO 3 are reacted with 6 moles of
mass of KClO 3 2  122.5 367.5 122.5
=  = HCl solution. Find the volume of CO2 gas produced at
mass of KCl 2  74.5 W 74.5 STP. The reaction is
W = 3 × 74.5 = 223.5 g Na2 CO3 + 2HCl  2 NaCl + CO2 + H2O
Mass of KClO3 2  122.5 367.5 2  122.5 Sol. From the reaction :
=  = Na2 CO3 + 2HCl  2 NaCl + CO2 + H2O
Mass of O 2 3  32 W 3  32
given moles 3 mol 6 mol
W = 144 g
given mole ratio 1 : 2
Stoichiometric coefficient ratio 1 : 2
2. 367.5 g KClO3 (M = 122.5) when heated, how many litre
See here given moles of reactant are in stoichiometric
of oxygen gas is produced at STP
coefficient ratio therefore no reactant is left over.
Sol. You can use here equation (1)
Now use Mole-mole analysis to calculate volume of CO2
mass of KClO3 2  122 .5 prdouced at STP
=  Moles of Na 2CO 3 Mole of CO 2 Pr oduced
volume of O 2 at STP 3  22.4 lt
=
1 1
367.5 2  122 .5 Moles of CO2 produced = 3
= V = 3 × 3 × 11.2 V = 100.8 lt volume of CO2 produced at STP = 3 × 22.4 L = 67.2 L
V 3  22.4 lt
2. 6 moles of Na2 CO 3 is reacted with 4 moles of HCl
3. 10 ml of liquid carbon disulphide (sp. gravity 2.63) is solution. Find the volume of CO2 gas produced at STP.
burnt in oxygen. Find the volume of the resulting gases The reaction is
measured at STP.
Na2 CO3 + 2HCl  2 NaCl + CO2 + H2O
Ans. 23.26
Sol. From the reaction :
Sol. 1 ml of CS2 Weighs 2.63 g
10 ml of CS2 weighs 26.3 g Na2 CO3 + 2HCl  2 NaCl + CO2 + H2O
CS 2 + 3O2  CO2 + 2SO2 given mole of reactant 6 : 4
12+(2×32) 22.4 L 44.8 L give molar ratio 3 : 2
76 gm 67.2 L (total volume at Stoichiometric coefficient ratio 1 : 2
STP) See here given number of moles of reactants are not in
 76 g of CS2 will yield 67.2 L of a mixture of CO2 and stoichiometric coefficient ratio. Therefore there should
SO2 at STP be one reactant which consumed first and becomes
limiting reagent.
67.2 But the question is how to find which reactant is limiting,
 26.3 g of CS2 will yield  26.3 = 23.26 lit.
76 it is not very difficult you can easily find it. According to the
following method.
4. 12 L of H2 and 11.2 L of Cl2 are mixed and exploded. The How to find limiting reagent :
composition by volume of mixture is - Step :  Divide the given moles of reactant by the
Ans. 0.8 L of H2, 22.4 L of HCl respective stoichiometric coefficient of that reactant.
Sol. H2 + Cl2  2HCl
1L 1L 2L Step : See for which reactant this division come out to
11.2 L 11.2 L 22.4L be minimum. The reactant having minimum value is
Volume of H2 = [12 – 11.2] = 0.8 L, Volume of Cl2 = Zero, limiting reagent for you.
Volume of HCl = 22.4 L Step :  Now once you find limiting reagent then your
focus should be on limiting reagent
From Step  &  Na2 CO3 HCl
LIMITING REAGENT
6 4
=6 = 2 (division is minimum)
The reactant which is consumed first and limits the 1 2
amount of product formed in the reaction, and is therefore,  HCl is limiting reagent
called limiting reagent. From Step 
Limiting reagent is present in least stoichiometric Mole of HCl Moles of CO2 produced
amount and therefore, controls amount of product. From =
2 1
The remaining or leftout reactant is called the excess
mole of CO2 produced = 2 moles
reagent.
volume of CO2 produced at S.T.P. = 2 × 22.4 = 44.8 lt.
When you are dealing with balance chemical equation
then if number of moles of reactants are not in the ratio
Solved Examples
of stoichiometric coefficient of balalnced chemical
1. 10 moles SO2 and 15 moles O2 were allowed to react
equation, then there should be one reactant which
over a suitable catalyst. 8 moles of SO3 were formed.
should be limiting reactant.
The remaining moles of SO2 and O2 respectively are -

PAGE # 8
Pre-foundation Career Care Programmes (PCCP) Division
Ans. 2 moles, 11 moles w
Sol. 2SO2 + O2  2SO3  Number of moles of solute dissolved =
M
10 15 0
w
10 – 2x 15 – x 2x  V ml water have mole of solute
2x = 8 x = 4 M
Hence, remaining, SO2 = 10 – 8 = 2 moles, O2 = 15 – 4 w  1000
= 11 moles  1000 ml water have 
M  Vinml
2. If 0.5 mol of BaCl2 is mixed with 0.2 mole of Na3PO4, the w  1000
maximum amount of Ba3(PO4)2 that can be formed is : Molarity (M) =
(Mol. wt of solute)  Vinml
Ans. 0.10 mol Some other relations may also be useful.
3 BaCl2 + 2 Na3PO4  6 NaCl + Ba3(PO4)2
mass of solute
molar ratio 3 2 6 1 Number of millimoles =  1000 =
initial moles 0.5 0.2 0 0 (Mol. wt. of solute )
Limiting reagent is Na 3 PO 4 hence it would be (Molarity of solution × Vinml)
consumed, and the yield would be decided by it initial  Molarity of solution may also given as :
moles. Number of millimole of solute
2 moles of Na3PO4 give 1 mole of Ba3 (PO4)2 , Total volume of solution in ml
 0.2 moles of Na3 PO 4 would give 0.1 mole of of
Ba3(PO4)2  Molarity is an unit that depends upon temperature
Solutions : .it varies inversely with temperature .
A mixture of two or more substances can be a solution. Mathematically : Molarity decreases as temperature
W e can also say that a solution is a homogeneous increases.
mixture of two or more substances. ‘Homogeneous’ 1 1
means ‘uniform throughout’. Thus a homogeneous
Molarity  temperature  volume
mixture, i.e., a solution, will have uniform composition
throughout.
 If a particulars solution having volume V 1 and
molarity = M1 is diluted to V2 mL the
Properties of a solution :
M1V1 = M2V2
 A solution is clear and transparent. For example, a
M2 : Resultant molarity
solution of sodium chloride in water is clear and
tranparent.
 If a solution having volume V 1 and molarity M 1 is
 The solute in a solution does not settle down even
mixed with another solution of same solute
after the solution is kept undisturbed for some time.
having volume V2 & molarity M2
 In a solution, the solute particle cannot be distinguished
then M1V1 + M2V2 = MR (V1 + V2)
from the solvent particles or molecules even under a
microscope. In a true solution, the particles of the solute M1V1  M2 V2
disappear into the space between the solvent molecules. MR = Resultant molarity =
V1  V2
 The components of a solution cannot be separated
by filtration. Ex.1 149 gm of potassium chloride (KCl) is dissolved in 10
Concentration terms : Lt of an aqueous solution. Determine the molarity of the
The following concentration terms are used to solution (K = 39, Cl = 35.5)
expressed the concentration of a solution. These are Sol. Molecular mass of KCl = 39 + 35.5 = 74.5 gm
 Molarity (M)  149 gm
Molality (m)  Mole fraction (x)  Moles of KCl = 74.5 gm = 2
 % calculation 
Normality (N)  ppm 2
 Remember that all of these concentration terms are  Molarity of the solution = = 0.2 M
10
related to one another. By knowing one concentration
term you can also find the other concentration terms. Let MOLALITY (M)
us discuss all of them one by one.
Molality (m) :
MOLARITY (M) The number of moles of solute dissolved in1000 g (1
The number of moles of a solute dissolved in 1 L kg) of a solvent is known as the molality of the solution.
(1000 ml) of the solution is known as the molarity i.e., molality =
of the solution. number of moles of solute
 1000
i.e., Molarity of solution mass of solvent in gram
number of moles Let y g of a solute is dissolved in x g of a solvent. The
= volume of solution in litre molecular mass of the solute is m. Then Y/m mole of
Let a solution is prepared by dissolving w gm of the solute are dissolved in x g of the solvent. Hence
solute of mol.wt. M in V ml water. Y
Molality =  1000
m x

PAGE # 9
Pre-foundation Career Care Programmes (PCCP) Division
 Molality is independent of temperature changes. Sol. Mass of substance = 0.5 g
Mass of solvent = 25 g
Ex.1 255 g of an aqueous solution contains 5 g of urea. What  percentage of the substance (w/w) =
is the concentration of the solution in terms of molality.
0 .5
(Mol. wt. of urea = 60)  100 = 1.96.
Sol. Mass of urea = 5 g 0.5  25
Molecular mass of urea = 60
2. 20 cm 3 of an alcohol is dissolved in80 cm 3 of water.
5 Calculate the percentage of alcohol in solution.
Number of moles of urea = = 0.083
60 Sol. Volume of alcohol = 20 cm3
Mass of solvent = (255 – 5) = 250 g Volume of water = 80 cm3
 Molality of the solution
20
 percentage of alcohol =  100 = 20.
Number of moles of solute 20  80
= × 1000
Mass of solvent in gram 3. What is the concentration of sugar (C12 H22 O11) in mole
L–1 if its 20g are dissolved in enough water to make a
0.083 final volume upto 2L ?
= × 1000= 0.332. Sol. Molarity of solution (mol L–1)
250

MOLE FRACTION(X) mass of solute (g) 1000


= 
M. Mass V in ML
The ratio of number of moles of the solute or solvent
present in the solution and the total number of moles
20 1000
present in the solution is known as the mole fraction Conc. of sugar = × = 0.0292 mol L–1
of substances concerned. 342 2000
Let number of moles of solute in solution = n
Number of moles of solvent in solution = N 4. If the density of methanol is 0.793 kg L –1 , what is its
n volume needed for making 2.5 L of its 0.25 M solution?
 Mole fraction of Solute (x1) = Sol. Amount of methanol required m(g) =
nN
Molarity  M. Mass  Vin ML
N
 Mole fraction of solvent (x2) = 1000
nN
also x1 + x2 = 1 0.25  32  2500
= = 20g
 Mole fraction is a pure number. It will remain 1000
independent of temperature changes.
Mass
Since, density = 
% calculation : Volume

Mass 20
The concentration of a solution may also expressed in Volume of CH3 OH required = density = = 25.22 mL
terms of percentage in the following way. 0.793

 % weight by weight (w/w) : It is given as mass of 5. A given solution of NaOH contains 2.00 g of NaOH per
solute present per 100 g of solution. litre of solution. Calculate the molarity of this solution.
Sol. Molar mass of NaOH = 40.00 g
mass of solute in g 2 .00
i.e. % w/w =  100
mass of solution in g 2.00 g NaOH = mole NaOH = 0.05 mole NaOH
40 .00
 % weight by volume (w/v) : It is given as mass of Mole of solute 0.05
solute present per 100 ml of solution. Molarity = Volume of solution in litres = = 0.05 M
1

mass of solute in g 6. 2.8 g of KOH is dissolved in water to give 200 cm3 of


i.e., % w/v =  100
Volume of solution in ml solution. Calculate the molarity of KOH in the solution.
 % volume by volume (V/V) : It is given as volume of Sol. Molar mass of KOH = 56.00 g
solute present per 100 ml solution. 2. 8
i.e., % V/V 2.8 g of KOH = moles of KOH = 0.05 mole of KOH
56
Volume of solute in mL Mole of solute 0.05
=  100 Molarity = Volume of solution in litres = =
Volume of solution in mL 200 / 1000
Solved Examples 0.25 M
1. 0.5 g of a substance is dissolved in 25 g of a solvent.
Calculate the percentage amount of the substance in 7. Calculate the number of moles and the amount in gram
the solution. of NaOH in 250 cm3 of a 0.100 M NaOH solution.

PAGE # 10
Pre-foundation Career Care Programmes (PCCP) Division
Sol. A 0.100 M NaOH solution contains 0.1 moles of NaOH in  Mass of sodium acetate required =
1 L or 1000 cm3 of solution.
0.375  82.0245  500
Number of moles of NaOH in 250 cm 3 of solution = = 15.379 = 15.38 g
0.1 250 1000
= 0.025 mole NaOH
1000
PH OF SOLUTION
Molar mass of NaOH = 40.00 g
Mass of NaOH in 250 cm3 of solution = 0.025 × 40.00 = According to Arrhenius theory, a acid releases H+ ion in
1.0 g aqueous solution. The concentration of these ions is
expressed by enclosing H+ in square bracket i.e. as [H+].
8. How much volume of 10 M HCl should be diluted with Thus, greater the [H + ] ions, stronger will be the acid.
water to prepare 2.00 L of 5 M HCl ? However, according to pH scale, lesser the pH value,
Sol. In dilution the following equation is applicable : stronger will be the acid. From the above discussion,
M1 V1 = M2 V2 we can conclude that pH value and H+ ion concentration
10 M HCl 5 M HCl are inversely proportional to each other.
10 × V1 = 5 × 2.00
The relation between them can also be expressed as-
5  2.00
V1 = L = 1.00 L  1 
10 pH = – log [H+] = log  
 H 
9. What volume of 6 M HCl and 2 M HCl should be mixed to
So, negative logarithm of hydrogen ion concentration is
get one litre of 3 M HCl ?
known as pH.
Sol. Suppose the volume of 6 M HCl required to obtain 1 L of
e.g.
3 M HCl = x L
Let the [H+] of an acid solution be 10–3 M. Its pH can be
 Volume of 2 M HCl required = (1 – x) L
calculated as -
Applying the molarity equation
pH = – log [H+]
M1 V1 + M2 V2 = M3 V3
= – log [10–3]
6 M HCl 2 M HCl 3 M HCl
= (–) (–3) log 10
6x + 2 (1 – x) = 3 × 1
= 3 ( log 10 = 1)
6x + 2 – 2x = 3
 Note :
4x = 1
Just as the [H+] of a solution can be expressed in terms
x = 0.25 L
of pH value, the [OH–] can be expressed as pOH.
Hence volume of 6 M HCl required = 0.25 L
1
Volume of 2 M HCl required = 0.75 L Mathematically , pOH = – log [OH–] = log
[ OH – ]
10. (a) A sample of NaOH weighing 38 g is dissolved in Moreover, pH + pOH = 14.
water and the solution is made to 50.0 mL in a volumetric Thus, if pH value of solution is known, its pOH value can
flask. What is the molarity of the resulting solutions ? be calculated.
(b) How many moles of NaOH are contained in 27 mL of Ex. Find the pH of 0.1 M of NaOH
0.15 M NaOH ? pOH = – log [OH–]
= – log [10–1]
38 = (–) (–1) log 10
Sol. (a) Moles of NaOH in 50.0 mL solution =
40 = 1 ( log 10 = 1)
pH + pOH = 14.
38 1000
 moles of NaOH in 1.0 L solution = × = 19 M pH = 13
40 50
(b) 1.0 L of solution contains moles = 0.15  Note :
0.15 There are some solutions which have definite pH i.e.,
 27.0 mL of solution contains moles = × 27 = their pH do not change on dilution or on standing for
1000
long. Such solutions are called buffer solutions.
4.05 × 10–3 moles.
Ex-1. Calculate pH of
11. Calculate the mass of sodium acetate (CH 3 COO Na)
(a) 10 –2 M H 2SO 4
required to make 500 mL of 0.375 molar aqueous
(b) 10 –2 NH 2SO 4
solutions. Molar mass of sodium acetate is 82.0245 g
(c) 10 –2 M Ca(OH)2
mole–1 .
(d) 10 –2 NCa(OH)2
mass of solute (g) 1000 Ans. (a) 1.699 (b) 2
Sol. Molarity of solution (M) = × (c) 12.3010 (d) 12
M. Mass V in mL
Hence, mass of solute = Ex-2. Calculate pH of each of the following solutions :
(a) a sample of seawater that has [OH –] = 1.60 x 10 –6 M
Molarity  M.Mass  Volume of solution
(b) a sample of acid rain that has [H 3O + ] = 6.0 x 10 –6 M
1000 (c) HCl solution of concentration 4 x 10 –7 M
(d) 1 M HCl

PAGE # 11
Pre-foundation Career Care Programmes (PCCP) Division
(e) a solution of lime containing 0.56 g CaO(lime) in 100
mL solution. 7. Which of the following has the maximum mass ?
Ans.(a) 8.204 (b) 5.22 (A) 25 g of Hg
(c) 6.37 (d) 0 (e) 13.3 (B) 2 moles of H2O
(C) 2 moles of CO2
Ex-3. W hat is pH of the resulting solution (acidic/basic/ (D) 4 g atom of oxygen
neutral) when following solution are mixed ?
(I) : 0.1 M H 2SO 4 , (II) : 0.1 M HCl, 8. Total mass of neutrons in 7mg of 14C is -
(III) : 0.1 M Ba(OH)2 (IV) : 0.1 M NaOH (A) 3 × 1020 kg (B) 4 × 10–6 kg
–7
(a) I and II, in 1 : 1 volume (C) 5 × 10 kg (D) 4 × 10–7 kg
(b) I and II in 1 : 2 volume
(c) I and IV in 1 : 2 volume 9. If the atomic mass of Sodium is 23, the number of moles
(d) I and III in 1 : 1 volume in 46 g of sodium is :
(e) II and III in 1 : 1 volume (A) 1 (B) 2
(f) II and III in 2 :1 volume (C) 2.3 (D) 4.6
(g) II and IV in 1 : 2 volume 10. Which of the following contains the greatest number of
Ans. (a) 0.823 acidic (b) 0.69 acidic atoms ?
(c) neutral (d) neutral (A) 1.0 g of butane (C4H10) (B) 1.0 g of nitrogen (N2)
(e) 13.18 basic (f) neutral (C) 1.0 g of silver (Ag) (D) 1.0 g of water (H2O)
(g) 13.18 basic
11. A sample of aluminium has a mass of 54.0 g. What is
Ex.-4. What is the pH of 0.01 M H2 SO4 ? the mass of the same number of magnesium atoms?
(A) 1.0 (B) 2.0 (At. wt. Al = 27, Mg=24)
(C) 3.0 (D) 4.0 (A) 12 g (B) 24 g
Ans. (B) (C) 48 g (D) 96 g.

EXERCISE-1 12. The weight of a molecule of the compound C6H12O6 is :


(A) 1.09 × 10–21 g (B) 2.988 × 10–22 g
–23
(C) 5.025 × 10 g (D) 16.023 × 10–23 g
I. Laws of chemical combination, atoms, molecules, moles
& avogadro's hypothesis 13. Four 1-1 litre flasks are seperately filled with the gases
N 2, Ne, N 2O and SO 3 at the same temperature and
1. Which of the following sample contains the maximum pressure. The ratio of total number of atoms of these
number of atoms - gases present in different flask would be :
(A) 1 mg of C4H10 (B) 1 mg of N2 (A) 1 : 1 : 1 : 1 (B) 1 : 2 : 2 : 3
(C) 1 mg of Na (D) 1 mL of water (C) 2 : 1 : 3 : 4 (D) 3 : 2 : 2 : 1

2. The total number of protons, electrons and neutrons in 14. The total number of g-molecules of SO2Cl2 in 13.5 g of
12 sulphuryl chloride is
12 g of 6 C is
-
(A) 0.1 (B) 0.2
25
(A) 1.084 × 10 (B) 6.022 × 1023
(C) 0.3 (D) 0.4
(C) 6.022 × 1022 (D) 18
15. The number of sodium atoms in 2 moles of sodium
3. 4.4 g of CO2 and 2.24 litre of H2 at STP are mixed in a
ferrocyanide (Na4[Fe(CN)6]) is :
container. The total number of molecules present in the
(A) 12 × 23 (B) 26 × 1023
container will be - 23
(C) 34 × 10 (D) 48 × 1023
(A) 6.022 × 1023 (B) 1.2044 × 1023
(C) 2 mole (D) 6.023 × 1024
16. 4.4 g of an unknown gas occupies 2.24 litres of volume
at STP, the gas may be :
4. How many atoms are contained in a mole of Ca(OH)2 -
(A) N2O (B) CO
(A) 30 × 6.02 × 1023 atoms/mol
(C) CO2 (D) 1 & 3 Both
(B) 5 × 6.02 × 1023 atoms/mol
(C) 6 × 6.02 × 1023 atoms/mol
17. 5.6 litre of oxygen at STP contains :
(D) None of these
(A) 6.02 × 1023 atoms
(B) 3.01 × 1023 atoms
5. Number of moles present in 1 m3 of a gas at NTP are -
(C) 1.505 × 1023 atoms
(A) 44.6 (B) 40.6
(D) 0.7525 × 1023 atoms
(C) 42.6 (D) 48.6
18. The volume of a gas in discharge tube is 1.12 × 10–7
6. The volume occupied by one molecule of water (density
ml. at STP then the number of molecule of gas in the
1 g/cm3) is -
tube is :
(A) 18 cm3 (B) 22400 cm3
–23
(A) 3.01 × 104 (B) 3.01 × 1015
(C) 6.023 × 10 (D) 3.0 × 10–23 cm3 12
(C) 3.01 × 10 (D) 3.01 × 1016

PAGE # 12
Pre-foundation Career Care Programmes (PCCP) Division
19. A person adds 1.71 gram of sugar (C12H22O11) in order 30. A compound of X and Y has equal mass of them. If their
to sweeten his tea. The number of carbon atoms added atomic weights are 30 and 20 respectively. Molecular
are (mol. mass of sugar = 342) formula of that compound (its mol wt. is 120) could be
(A) 3.6 × 1022 (B) 7.2 × 1021 (A) X2Y2 (B) X3Y3
(C) 0.05 (D) 6.6 × 1022 (C) X2Y3 (D) X3Y2

20. If V ml of the vapours of substance at NTP weight W g. 31. 1 litre of a hydrocarbon weights as much as one litre of
Then mol w. of substance is : CO 2.Then the molecular formula of the hydrocarbon is

V
(A) (W/V) × 22400 (B) × 22.4 (A) C3H8 (B) C2H6
W (C) C2H4 (D) C3H6
W 1
(C) (W–V) × 22400 (D) 32. Percentage of Se in peroxidase anhydrous enzyme is
V  22400
0.5% by weight (at. wt. = 78.4) then min. mol. wt. of
II. Percentage composition and molecular formula peroxidase anhydrous enzymes is :
(A) 1.568 × 104 (B) 1.568 × 103
21. Two elements A (at. wt. 75) and B (at. wt. 16) combine to
(C) 15.68 (D) 2.136 × 104
yield a compound. The % by weight of A in the compound
was found to be 75.08. The empirical formula of the 33. Which of the following compounds has same empirical
compound is - formula as that of glucose :
(A) A2B (B) A2B3 (A) CH3CHO (B) CH3COOH
(C) AB (D) AB2 (C) CH3OH (D) C2H6

22. The hydrated salt, Na2SO4. nH2O undergoes 55.9% loss 34. Caffine has a molecular weight of 194. It contains 28.9%
in weight on heating and becomes anhydrous. The value by mass of nitrogen number of atoms of nitrogen in
of n will be - one molecule of it :
(A) 5 (B) 3 (A) 2 (B) 3
(C) 7 (D) 10 (C) 4 (D) 5

23. How many gram of KCl would have to be dissolved in 60 35. Insulin constans 3.4% sulphur. The minimum mol. wt.
g of H2O to give 20% by weight of solution - of insulin is –
(A) 15 g (B) 1.5 g (A) 941.176 (B) 944
(C) 11.5 g (D) 31.5 g (C) 945.27 (D) None

24. The total number of AlF3 molecule in a sample of AlF 3 36.The percentage value of nitrogen in urea is about
containing 3.01 × 1023 ions of F– is - (A) 46 (B) 85
(A) 9.0 × 1024 (B) 3.0 × 1024 (C) 18 (D) 28
(C) 7.5 × 1023 (D)10 23
III. Density and vapour density
25. Equal masses of O2, H2 and CH4 are taken in a container.
The respective mole ratio of these gases in container is 37. 22.4 litre of water vapour at NTP, when condensed to
(A) 1 : 16 : 2 (B) 16 : 1 : 2 water occupies an approximate volume of :
(C) 1 : 2 : 16 (D) 16 : 2 : 1 (Given : density of water = 1 g/ml)
(A) 18 litre (B) 1 litre
26. The volume of CO2 (in litres) liberated at STP when 10 g (C) 1 ml (D) 18 ml
of 90% pure limestone is heated completely, is-
(A) 22.4 L (B) 2.24 L 38. Vapour density of a gas if its density is 0.178 g/L at NTP
(C) 20.16 L (D) 2.016 L is :
(A) 0.178 (B) 2
27. The number of atoms of Cr and O in a compound are 4.8 (C) 4 (D) 0.089
× 1010 & 9.6  1010 respectively. Its empirical formula is–
(A) Cr2O3 (B) CrO2 39. Vapour density of a volatile substance w.r.t. CH 4 is
(C) Cr2O4 (D) None 4(CH4 = 1). Its molecular weight would be –
(A) 8 (B) 32
28. The empirical formula of a compound of molecular mass (C) 64 (D) 128
120 is CH2O. The molecular formula of the compound is
(A) C2H4O2 (B) C4H8O4 40. A gas is found to have the formula (CO)x. It’s VD is 70
(C) C3H6O3 (D) all of these the value of x must be :
(A) 7 (B) 4
29. A hydrocarbon contains 75% of carbon. Then its (C) 5 (D) 6
molecular formula is :
(A) CH4 (B) C2H4
(C) C2H6 (D) C2H2

PAGE # 13
Pre-foundation Career Care Programmes (PCCP) Division
IV : Balanced chemical equation analysis 52. A metal oxide has the formula Z2O3. It can be reduced by
41. 8g of sulphur are burnt to form SO2, which is oxidised by hydrogen to give free metal and water. 0.1596 g of the
Cl2 water. The solution is treated with BaCl2 solution. metal requires 6 mg of hydrogen for complete reduction.
The amount of BaSO4 precipitated is - The atomic mass of the metal is -
(A) 1.0 mole (B) 0.5 mole
(A) 27.9 (B) 159.6
(C) 0.75 mole (D) 0.25 mole
(C) 79.8 (D) 55.8
42. Weight of oxygen in Fe2O3 and FeO is in the simple ratio
of - Question number 60, 61, 62 and 63 are based on the
(A) 3 : 2 (B) 1 : 2 following information :
(C) 2 : 1 (D) 3 : 1
43. 2.76 g of silver carbonate on being strongly heated yield Q. Dissolved oxygen in water is determined by using a
a residue weighing - redox reaction. Following equations describe the
(A) 2.16g (B) 2.48 g procedure -
(C) 2.32 g (D) 2.64 g I 2Mn 2+ (aq) + 4OH –(aq) + O 2 (g)  2MnO 2(s) +
2H 2O(  )
44. Amount of oxygen required for combustion of 1 kg of a
mixture of butane and isobutane is - II MnO 2 (s)+2I–(aq)+4H + (aq)  Mn 2+ (aq)+I2(aq) +
(A) 1.8 kg (B) 2.7 kg 2H2O(  )
(C) 4.5 kg (D) 3.58 kg III 2S2O 23– (aq) + I2(aq)  2S4O 26– (aq) + 2I–(aq)
45. Rakesh needs 1.71 g of sugar (C12H22O11) to sweeten
his tea. W hat would be the number of carbon atoms 53. How many moles of S 2 O23 – are equivalent to each mole
present in his tea ?
(A) 3.6 × 1022 (B) 7.2 × 1021 of O2 ?
(C) 0.05 × 10 23
(D) 6.6 × 1022 (A) 0.5 B) 1
(C) 2 (D) 4
46. 224 mL of a triatomic gas weigh 1 g at 273 K and 1 atm.
The mass of one atom of this gas is - 54. What amount of I2 will be liberated from 8 g dissolved
(A) 8.30 × 10–23 g (B) 2.08 × 10–23 g oxygen ?
–23
(C) 5.53 × 10 g (D) 6.24 × 10–23 g (A) 127 g (B) 254 g
47. Amount of BaSO4 formed on mixing the aqueous solution (C) 504 g (D) 1008 g
of 2.08 g BaCl2 and excess of dilute H2SO4 is -
(A) 2.33 g (B) 2.08 g 55. 3 × 10–3 moles O2 is dissolved per litre of water, then what
(C) 1.04 g (D) 1.165 g will be molarity of I– produced in the given reaction ?
(A) 3 × 10–3 M (B) 4 × 3 × 10–3 M
48. Mg (OH)2 in the form of milk of magnesia is used to
1
neutralize excess stomach acid. How many moles of (C) 2 × 3 × 10–3 M (D)  3  10 – 3 M
stomach acid can be neutralized by 1 g of Mg(OH)2 ? 2
(Molar mass of Mg(OH)2 = 58.33)
56. 8 mg dissolved oxygen will consume -
(A) 0.0171 (B) 0.0343
(A) 5 × 10–4 mol Mn+2 (B) 2.5 × 10–4 mol Mn2+
(C) 0.686 (D) 1.25 2+
(C) 10 mol Mn (D) 2 mol Mn2+
49. PH 3(g) decomposes on heating to produce
phosphorous and hydrogen. The change in volume 3
57. The equation : 2Al(S) + O (g) Al2O3(S) show that
when 100 mL of such gas decomposed is - 2 2
(A) + 50 mL (B) + 500 mL
3 7
(C) – 50 mL (D) – 500 mL (A) 2 mole of Al reacts with mole of O2 to produce
2 2
50. In the reaction , CrO5 + H2SO4  Cr2(SO4)3 + H2O + O2 mole of Al2O3
one mole of CrO5 will liberate how many moles of O2 ? 3
(B) 2g of Al reacts with g of O2 to produce one mole of
(A) 5/2 (B) 5/4 2
(C) 9/2 (D) None Al2O3
3
51. Calcium carbonate decomposes on heating according (C) 2g mole of Al reacts with litre of O2 to produce 1
2
to the equation - mole of Al2O3
CaCO3(s)  CaO(s) + CO2(g)
3
At STP the volume of CO 2 obtained by thermal (D) 2 mole of Al reacts with mole of O2 to produce 1
2
decomposition of 50 g of CaCO3 will be -
mole of Al2O3
(A) 22.4 litre (B) 44 litre
(C) 11.2 litre (D) 1 litre

PAGE # 14
Pre-foundation Career Care Programmes (PCCP) Division
58. Volume of CO 2 obtained at STP by the complete V. Limiting reagent
decompoisition of 9.85 g. BaCO3 is –
(At. wt. of Ba = 137) 67. If 0.5 mol of BaCl2 is mixed with 0.1 mole of Na3PO 4 ,
(A) 2.24 lit (B) 1.12 lit the maximum number of mole of Ba3(PO4)2 that can be
(C) 0.84 lit (D) 0.56 lit formed is :
(A) 0.7 (B) 0.05
59. 500 ml of a gaseous hydrocarbon when burnt in excess (C) 0.30 (D) 0.10
of O2 gave 2.5 litre of CO2 and 3.0 litre of water vapours
under same conditions. Molecular formula of the 68. For the reaction 2P + Q  R, 8 mol of P and 5 mol of Q
hydrocarbon is : will produce
(A) C4H8 (B) C4H10 (A) 8 mol of R (B) 5 mol of R
(C) C5H10 (D) C5H12 (C) 4 mol of R (D) 13 mol of R

60. 1.2 g of Mg (At mass 24) will produce MgO equal to : 69. 4 g of hydrogen is ignited with 4 g of oxygen, the amount
(A) 0.05 mol (B) 40 g of water formed is ?
(C) 40 mg (D) 4 g (A) 2.5 g (B) 0.5 g
(C) 4.5 g (D) 8 g
61. The moles of O 2 required for reacting with 6.8 g of
ammonia 70. 0.5 mole of H2SO 4 is mixed with 0.2 mole of Ca(OH)2.
(......NH3 + ....... O2  ...... NO + ..... H2O) is The maximum number of moles of CaSO4 formed is:
(A) 5 (B) 2.5 (A) 0.2 (B) 0.5
(C) 1 (D) 0.5 (C) 0.4 (D) 1.5

62. The volume of oxygen required for complete combus- 71. For the reaction : A + 2B  C
tion of 20 ml of ethene is 5 mole of A and 8 mole of B will produce :
(A) 30 ml (B) 60 ml (A) 5 mole of C (B) 4 mole of C
(C) 40 ml (D) 50 ml (C) 8 mole of C (D) 12 mole of C

63. What weight of CaCO3 must be decomposed to produce 72. Calculate the amount of H2 which is left unreacted in the
the sufficient quantity of carbon dioxide to convert 21.2 given reaction :
kg of Na2CO 3 completely in to NaHCO3. [Atomic mass
Na = 23, Ca = 40] 2H2 + O2  2H2O
If 8 g of H2 is mixed with 16 g O2 ?
CaCO3  CaO + CO2
(A) 3 g (B) 6 g
Na2 CO3 + CO2 + H2O  2NaHCO3 (C) 1 g (D) 4 g
(A) 100 Kg (B) 20 Kg
(C) 120 Kg (D) 30 Kg 73. Zinc and hydrochloric acid react according to the reac-
tion.
64. The volume of gas at NTP produced by 100g of CaC2 Zn(s) + 2HCl(aq.)  ZnCl2(aq.) + H2(g)
with water : If 0.30 mole of Zn are added to hydrochloric acid contain-
(A) 70 litre (B) 35 litre ing 0.52 mole HCl, how many moles of H2 are produced
(C) 17.5 litre (D) 22.4 litre ?
(A) 0.26 (B) 1.04
65. Sulphur trioxide is prepared by the following two (C) 0.52 (D) 0.13
reactions
S8(s) + 8O2(g)  8SO2(g) 74. A mixture of 1.0 mole of Al and 3.0 mole of Cl2 are al-
2SO2 (g) + O2(g)  2SO3(g) lowed to react as :
How many grams of SO3 are produced from 1 mol of S8 ? 2Al (s) + 3Cl2 (g)  2AlCl3 (s)
(A) 1280.0 (B) 640.0 (a) Which is limiting reagent ?
(C) 960.0 (D) 320.0 (b) How many moles of AlCl3 are formed
(c) Moles of excess reagent left unreacted is
66. Butane, C4H10, burns with the oxygen in air to give carbon (A) (a) Al, (b) 1.0 (c) 1.5
dioxide and water. (B) (a) Cl2, (b) 2.0 (c) 2.0
W hat is the amount (in moles) of carbon dioxide pro- (C) (a) Al, (b) 0.5 (c) 1.5
duced from 0.15 mol C4H10 ? (D) (a) Cl2, (b) 1.0 (d) 1.5
C4H10(g) + O2(g)  CO2(g) + H2O(g) (not bal-
anced) VI. Principle of Atom Conservation (POAC)
(A) 0.15 mol CO2 (B) 0.30 mol CO2
(C) 0.45 mol CO2 (D) 0.60 mol CO2 75. 25.4 g of iodine and 14.2g of chlorine are made to react
completely to yield a mixture of Cl and Cl3. Calculate
the number of moles of Cl and Cl3 formed.
(A) 0.1 mole, 0.1 mole (B) 0.1 mole, 0.2 mole
(C) 0.5 mole, 0.5 mole (D) 0.2 mole, 0.2 mole

PAGE # 15
Pre-foundation Career Care Programmes (PCCP) Division
76. 21.6 g of silver coin is dissolved in HNO3. When NaCl is 86. What volume of 0.10 M H2SO4 must be added to 50 mL
added to this solution, all silver is precipitated as AgCl. of a 0.10 M NaOH solution to make a solution in which
The weight of AgCl is found to be 14.35 g then % of silver the molarity of the H2SO4 is 0.050 M ?
in coin is : (A) 400 mL (B) 50 mL
(A) 50% (B) 75% (C) 100 mL (D) 150 mL
(C) 100% (D) 15%
87. What approximate volume of 0.40 M Ba(OH)2 must be
VII. Concentration terms added to 50.0 mL of 0.30 M NaOH to get a solution in
which the molarity of the OH– ions is 0.50 M?
77. Molarity of H2SO4 (density 1.8g/mL) is 18M. The molality (A) 33 mL (B) 66 mL
of this solution is - (C) 133 mL (D) 100 mL
(A)36 (B) 200
(C) 500 (D) 18 88. What volume of a 0.8 M solution contains 100 millimoles
of the solute?
78. If 250 mL of a solution contains 24.5 g H2SO4 the molarity (A) 100 mL (B) 125 mL
and normality respectively are - (C) 500 mL (D) 62.5 mL
(A) 1 M, 2 N (B) 1M,0.5 N
(C) 0.5 M, 1N (D) 2M, 1N 89. 500 mL of a glucose solution contains 6.02 × 10 22
molecules. The concentration of the solution is
79. The mole fraction of NaCl, in a solution containing 1 (A) 0.1 M (B) 1.0 M
mole of NaCl in 1000 g of water is - (C) 0.2 M (D) 2.0 M
(A) 0.0177 (B) 0.001
(C) 0.5 (D) 0.244 90. What is the concentration of nitrate ions if equal volumes
of 0.1 M AgNO3 and 0.1 M NaCl are mixed together :
80. 3.0 molal NaOH solution has a density of 1.110 g/mL. (A) 0.1 M (B) 0.2 M
The molarity of the solution is - (C) 0.05 M (D) 0.25 M
(A) 2.9732 (B) 3.05
(C) 3.64 (D) 3.0504 91. The volume of water that must be added to a mixture of
250 ml of 0.6 M HCl and 750 ml of 0.2 M HCl to obtain
81. The mole fraction of water in 20% (wt. /wt.) aqueous 0.25 M solution of HCl is :
solution of H2O2 is - (A) 750 ml (B) 100 ml
77 68 (C) 200 ml (D) 300 ml
(A) (B)
68 77
92. The molarity of the solution containing 2.8% mass-vol-
20 80 ume solution of KOH is
(C) (D)
80 20 (A) M/10 (B) M/2
(C) M/5 (D) 1 M
82. When FeCl3 is ignited in an atmosphere of pure oxygen,
the following reaction takes place- 93. The mole fraction of water in a solution containing 117
4FeCl3(s) + 3O2(g)  2Fe2O3(s) + 6Cl2(g) g sodium chloride and 900 g of water is ?
If 3 moles of FeCl3 are ignited in the presence of 2 moles (A) 0.0632 (B) 0.038
of O2 gas, how much of which reagent is present in excess (C) 0.9615 (D) 1.000
and therefore, remains unreacted ?
(A) 0.33 mole FeCl3 remains unreacted 94. 250 ml of 0.5 M KCl is diluted with water to 500 ml of
(B) 0.67 mole FeCl3 remains unreacted solution, the number of chloride ions in the resulting
(C) 0.25 mole O2 remains unreacted solution are
(D) 0.50 mole O2 remains unreacted (A) 6.02 × 1023 (B) 3.76 × 1022
24
(C) 1 × 10 (D) 3.76 × 1023
83. Which of the following has the highest normality ?
(A) 1 M H2SO4 (B) 1 M H3PO3 95. 300 ml of 3.0 M NaCl is added to 200 ml of 4.0 M BaCl2
(C) 1 M H3PO4 (D) 1 M HNO3 solution. The concentration of Cl¯ ions in the resulting
solution is
84. The molarity of 98% H2SO4(d = 1.8g/mL) by wt. is - (A) 7 M (B) 1.6 M
(A) 6 M (B) 18.74 M (C) 1.8 M (D) 5 M
(C) 10 M (D) 4 M
96. The molality of a sulphuric acid solution is 0.2. Calculate
85. If 500 ml of 1 M solution of glucose is mixed with 500 the total weight of the solution having 1000 gm of solvent.
ml of 1 M solution of glucose final molarity of solution (A) 1000 g (B) 1098.6 g
will be : (C) 980.4 g (D) 1019.6g
(A) 1 M (B) 0.5 M
(C) 2 M (D) 1.5 M

PAGE # 16
Pre-foundation Career Care Programmes (PCCP) Division
VIII : Calculation of oxidation number (C) Bromine is neither reduced nor oxidized.
(D) Bromine is both reduced and oxidized.
97. The oxidation number of Oxygen in Na2O2 is :
(A) + 1 (B) + 2 108. Which of the following is a redox reaction:
(C) – 2 (D) – 1 (A) 2 CrO42– + 2H+ Cr2O72– + H2 O
(B) CuSO4 + 4 NH3  [Cu (NH3)4] SO4
98. One of the following has both positive and negative oxi- (C) 2Na2 S2O3 + 2  Na2S4O6 + 2Na
dation states (D) Cr2O72– + 2OH–  2 CrO42– + H2 O
(A) F (B) Cl
(C) He (D) Na 109. Consider the reaction, Zn + Cu2+  Zn2+ + Cu
With reference to the above, which one of the following
99. The oxidation state of osmium (Os) in OsO4 is
is the correct statement ?
(A) + 7 (B) + 6
(A) Zn is reduced to Zn2+
(C) + 4 (D) + 8
(B) Zn is oxidised to Zn2+
(C) Zn2+ is oxidised to Zn
100. Oxidation number of nitrogen in (NH4)2SO4 is
(D) Cu2+ is oxidised to Cu.
1
(A) – (B) – 1 110. W hich reaction does not represent auto redox or
3 disproportionation reaction :
(C) + 1 (D) – 3 (A) Cl2 + OH–  Cl– + ClO3– + H2O
101. In which of the following compounds, the oxidation num-
ber of iodine is fractional ? (B) 2H2O2  H2O + O2
(A) F7 (B) 3– (C) 2Cu+  Cu2+ + Cu
(C) F5 (D) F3 (D) (NH4)2Cr2O7  N2 + Cr2O3 + 4H2O

102. The oxidation number of cobalt in K3[Co(NO2)6] is


111. In the reaction X – + XO3– + H+  X2 + H2O, the molar
(A) 0 (B) + 4
ratio in which X – and XO3– react is :
(C) + 3 (D) + 6
(A) 1 : 5 (B) 5 : 1
(C) 2 : 3 (D) 3 : 2
103. Phosphorus has the oxidation state of +3 in
(A) Phosphorous acid
112.The compound that can work both as an oxidising as
(B) Orthophosphoric acid
well as a reducing agent is :
(C) Hypophosphorous acid
(A) KMnO4 (B) H2O2
(D) Metaphosphoric acid
(C) Fe2(SO4)3 (D) K2Cr2O7
104. The oxidation number of Phosphorus in Mg2P2O7 is :
113.W hich of the following behaves as both oxidising and
(A) + 3 (B) + 2
reducing agents ?
(C) + 5 (D) – 3
(A) H2SO4 (B) SO2
(C) H2S (D) HNO3
105. In which of the following compounds, nitrogen has an
oxidation state of –1 ? X. PH of solution :
(A) N2O (B) NO2–
114.The pH of 0.001 M solution of HCl is -
(C) NH2OH (D) N2H4
(A) 1.0 (B) 3
(C) 4.0 (D) 5.0
IX : Balancing of redox reactions
115.The pH of a solution containing 0.1 N NaOH solution is :
106. A reducing agent is a substance :
(A) 1 (B) 10–1
(A) in which an element undergoes increase in oxidation
(C) 13 (D) 10–13
number.
(B) in which an element undergoes decrease in
116.When 0.4 g of NaOH is dissolved in one litre of solution,
oxidation number.
the pH of solution is -
(C) which gains electron(s)
(A) 12 (B) 2
(D) which shares electron(s)
(C) 6 (D) 10
107. Consider the following reaction: 117.The hydrogen ion concentration and pH of the solution
3Br2 + 6CO32 – + 3H2O  5Br – + BrO3– + 6 HCO3– made by mixing 100 mL of 1.0 M HNO3 with 100 mL of
Which of the following statements is true regarding this 0.8 M KOH are -
reaction: (A) [H+] = 0.1, pH = 1
(A) Bromine is oxidized and the carbonate radical is (B) [H+] = 0.01, pH = 2
reduced. (C) [H+] = 1 × 10–12, pH = 12
(B) Bromine is reduced and the carbonate radical is (D) [H+] = 1 × 10–7, pH = 7
oxidized.

PAGE # 17
Pre-foundation Career Care Programmes (PCCP) Division
118.The pH and pOH of 0.1 M aqueous solution of HNO3 are 3. The volume of 0.5 M aqueous NaOH solution required to
: neutralize 10 ml of 2 M aqueous HCl solution is :
(A) 0,14 (B) 14,0 [KVPY-Part-I-2008]
(C) 13,1 (D) 1,13 (A) 20ml (B) 40ml
(C) 80ml (D) 120ml
119.20 mL of 0.1 N HCl is mixed with 20 mL of 0.1 N KOH 4. 3.01×1023 molecules of elemental Sulphur will react with
solution ; the pH of the solution will be - 0.5 mole of oxygen gas completely to produce
(A) 0 (B) 2 [KVPY-Part-I-2008]
(C) 7 (D) 9 (A) 6.02 × 1023 molecules of SO3
(B) 6.02 × 1023 molecules of SO2
120. When the pH changes from 4 to 2, the hydrogen ion (C) 3.01 × 1023 molecules of SO3
concentration will increase by a factor - (D) 3.01 x 1023 molecules of SO2
(A) 2 (B) 1/2
5. The solubility of a gas in a solution is measured in three
(C) 102 (D) 100.5
cases as shown in the figure given below where w is the
weight of a solid slab placed on the top of the cylinder lid.
121. 100 mL of 0.2 N HCl is added to 100 mL of 0.18 N NaOH The solubility will follow the order :
and the whole volume is made two litre. The pH of [KVPY-Part-I-2008]
resulting solution is - w w w w w w
(A) 1 (B) 2
(C) 3 (D) 4
122. 10 mL of 0.1 N HCl is added to 990 mL solution of NaCl. gas gas
gas
The pH of the resulting solution is-
(A) zero (B) 3
(C) 7 (D) 10
solution solution solution
123. The pH of a solution is 6.0. To this solution, sufficient
acid is added to decrease the pH to 3.0. The increase in
hydrogen ion concentration is -
(A) 100 times (B) 10 times (A) a > b > c (B) a < b < c
(C) 1000 times (D) 2.5 times (C) a = b = c (D) a >b < c
6. The density of a salt solution is1.13 g cm–3 and it con-
EXERCISE-2 tains 18% of NaCI by weight. The volume of the solution
containing 36.0 g of the salt will be :
[KVPY-Part-II-2008]
COMPETITIVE EXAM PREVIOUS YEARS’ QUESTIONS :
(A) 200 cm3 (B) 217 cm3
1. A 3 N solution of H2SO4 in water is prepared from Conc. (C) 177 cm3 (D) 157cm3
H2SO4 (36 N) by diluting [KVPY-Part-II-2007] 7. One mole of nitrogen gas on reaction with 3.01 x 1023
(A) 20 ml of the conc. H2SO4 to 240 ml molecules of hydrogen gas produces-
(B) 10 ml of the conc. H2SO4 to 240 ml [KVPY-Part-I-2009]
(C) 1 ml of the conc. H2SO4 to 36 ml (A) one mole of ammonia
(D) 20 ml of the conc. H2SO4 to 36 ml (B) 2.0 x 1023 molecules of ammonia
(C) 2 moles of ammonia
2. The solubility curve of KNO3 as a function of temperature (D) 3.01 × 1023 molecules of ammonia
is given below [KVPY-Part-II-2007]
8. [KVPY-Part-I-2009]
250 Solubility
Solubility (g/100 ml water)

g/I
250
200 KNO3
200
150
150
100 KCl
100
50
50
0
20 40 60 80 100
0 20 40 60 80 100
Temperature (ºC)
Temperature (°C)
Given the solubility curves of KNO3 and KCl, which of the
The amount of KNO 3 that will crystallize when a satu- following statements is not true ?
rated solution of KNO3 in 100 ml of water is cooled from (A) At room temperature the solubility of KNO3 and KCI
90°C to 30 °C, is are not equal
(A) 16 g (B) 100 g (B) The solubilities of both KNO3 and KCI increase with
(C) 56 g (D)160 g temperature
(C) The solubility of KCI decreases with temperature
(D) The solubility of KNO3 increases much more com-
pared to that of KCl with increase in temperature
PAGE # 18
Pre-foundation Career Care Programmes (PCCP) Division
9. 10 ml of an aqueous solution containing 222 mg of cal- ume of it will be required to react with the above solu-
cium chloride (mol. wt. = 111) is diluted to 100 ml. The tion? [IJSO-Stage-II/2013]
concentration of chloride ion in the resulting solution is - (A) 40 ml. (B) 10 ml.
[KVPY-Part-II-2009] (C) 20 ml. (D) 80 ml.
(A) 0.02 mol/lit. (B) 0.01 mol/lit.
(C) 0.04 mol/lit (D) 2.0 mol/lit. 19. X is a member of alkene series with a molecular mass
28 amu. 200 cm3 X is burnt in just sufficient air (contain-
10. Aluminium reduces manganese dioxide to manganese ing 20% oxygen) to form carbon dioxide and steam. If all
at high temperature. The amount of aluminium required the measurements are made at constant pressure and
to reduce one gram mole of manganese dioxide is - 100º C, find the composition of the products formed and
[KVPY-Part-II-2009] the unreacted air. [IJSO-Stage-II/2013]
(A) 1/2 gram mole (B) 1 gram mole (A) 400 cm3, 400 cm3, 600 cm3 respectively
(C) 3/4 gram mole (D) 4/3 gram mole (B) 200 cm3, 200cm3, 2400cm3 respectively
(C) 200 cm3, 400 cm3, 2400 cm3 respectively
11. One mole of oxalic acid is equivalent to (D) 400 cm3, 400 cm3, 2400 cm3 respectively
[IJSO-2009] 20. 1.84 g of Dolomite CaMg(CO3)2 ore was heated result-
(A) 0.5 mole of NaOH (B) 1 mole of NaOH ing in a residue of constant weight 0.96 g. During heat-
(C) 1.5 mole of NaOH (D) 2 mole of NaOH ing the metal of one of the product burnt with a dazzling
white flame . The approximate percentage composition
12. Oxygen exhibits (–1) oxidation state in of the two products in the residue are respectively :
[IJSO-Stage-I/2011] [IJSO-Stage-II/2013]
(A) OF2 (B) H2O (A) 54 and 46 (B) 46 and 54
(C) H2O2 (D) HCIO (C) 42 and 58 (D) 58 and 42

13. 8 Grams of oxygen at NTP contain 21. Haber’s process for the production of ammonia is an
[IJSO-Stage-I/2012] industrially important process used mainly in fertilizer
(A) 1.5 × 1023 molecules industry. Due to some accident only 10% conversion
(B) 3.0 × 1023 molecules took place. If all the volumes are measured in litres at
(C) 6.023 × 1023 molecules STP, what volume of reactants are needed for this con-
(D) 1.5 × 1022 molecules version? [IJSO-Stage-II/2013]
(A) 1/2 of the total volume of hydrogen and nitrogen.
14. When 1g of CaCO3 reacts with 50 ml of 0.1 M HCI, the (B) 1/5th of the total volume of hydrogen and nitrogen
volume of CO2 produced is - [IJSO-Stage-I/2012] (C) 1/10th of the total volume of hydrogen and
(A) 11.2 mL (B) 22.4 mL nitrogen
(C) 112 mL (D) 56 mL (D) 1/5th of total volume of nitrogen

15. Molality of a solution is the number of - 22. Rahul dropped 150 g of marble chips into 1 kg of HCl
[IJSO-Stage-I/2012] solution containing 0.1 of its weight of pure acid. The
(A) moles of the solute per 1000 mL of the solution. mass of marble chips that remained undissolved will
(B) moles of the solute per 1000 mL of the solvent. approximately be : [IJSO-Stage-II/2013]
(C) moles of the solute per 1000 g of the solvent. (A) 6.50 g (B) 13 g
(D) moles of the solute per 100g of the solvent. (C) 26 g. (D) 15 g.

16. The oxidation number of chlorine in CaOCI2 is - 23. How many H atoms are in 3.4 g of C12H22O11 ?
[IJSO-Stage-I/2012] [IJSO-Stage-I/2013]
(A) 0 (B) –1 (A) 6.0 × 1023 (B) 1.3 × 1023
22
(C) +1 (D) +3 (C) 3.8 × 10 (D) 6.0 × 1023

17. They density of water at room temperature is 1 g/ml (mili- 24. W hat is the molality of a solution made by dissolving
litre). Consider a spherical drop of water having volume 100 g of bromothymol blue (C27H28Br2O5S) in 1.00 L of
0.05ml. The drop evaporates at a uniform rate in one ethanol on a winter’s day at 10ºC ? The density of etha-
hour. The number of molecules leaving the liquid sur- nol at this temperature is 0.7979 kg L–1 :
face per second is approximately [IJSO-Stage-I/2013]
[IJSO-Stage-II/2013] (A) 0.100 mol kg–1 (B) 0.128 mol kg–1
–1
(A) Zero (B) 5 × 1017 (C) 0.160 mol kg (D) 0.201 mol kg–1
14
(C) 3 × 10 (D) 2 × 1021
25. 0.5755 g of a compound, containing sulfur and fluorine
18. 50 ml of 0.20 M solution of washing soda reacts with one only, has a volume of 255.0 mL at 288.0 K and 50.01
of the acids in aqua regia. One of the products is Chile kPa. What is the molecular formula of this compound ?
saltpetre. If the strength of the acid is 0.25 M, what vol- [IJSO-Stage-I/2013]
(A) S2F2 (B) SF2
(C) SF4 (D) SF6

PAGE # 19
Pre-foundation Career Care Programmes (PCCP) Division
26. The mass of 0.2 mole of Oxygen molecule is : 35. The oxidation number of sulphur thiosulphate (Na2S2O3)
[IJSO-Stage-I/2013] is : [IJSO-Stage-I/2014-15]
(A) 6.4g (B) 3.2g (A) +1 (B) +3
(C) 1.6g (D) 7.75g (C) +2 (D) +4
36. Sulphuric acid is manufactured by the contact process
27. The solubility of a salt B2D3 is X mole L–1. Its solubility in which sulphur dioxide reacts with oxygen in presence
product is [IJSO-Stage-I/2012-13] of a catalys. If 5.6 moles of SO2 reacts with 4.8 moles of
(A) X5 (B) 6X5 O2 and a large excess of water, the maximum number of
5
(C) 36X (D) 108X5 moles of H2SO4 that can be obtained is :
28. 8 grams of oxygen at NTP contain [IJSO-Stage-I/2014-15]
[IJSO-Stage-I/2012-13] (A) 11.2 (B) 5.6
(A) 1.5 × 1023 molecules (C) 4.8 (D) 1.4
(B) 3.0 × 1023 molecules 37. W hich of the following contains the same number of
(C) 6.023 × 1023 molecules atoms as 13.5 grams of aluminium ?
(D) 1.5 × 1022 molecules [IJSO-Stage-I/2014-15]
(A) 20 g of calcium (B) 10g of magnesium
29. When 1g of CaCO3 reacts with 50 ml of 0.1 M HCI, the (C) 20 g of potassium (D) 10 of sodium
volume of CO2 produced is [IJSO-Stage-I/2012-13]
(A) 11.2 mL (B) 22.4 mL 38. How many molecules of water of hydration are present
(C) 112 mL (D) 224 mL. in 252mg of oxalic acid (H2C2O4. 2H2O) ?
[IJSO-Stage-II/2014]
30. Molality of a solution is the number of (A) 2.68× 1018 (B) 2.52 × 10221
[IJSO-Stage-I/2012-13] (C) 1.83 × 1024 (D) 2.4× 1021
(A) moles of the solute per 1000 mL of the solution. 39. Help Sachin to calculate the osmotic pressure of 0.9%
(B) moles of the solute per 1000 mL of the solvent. aqueous solution of solute X at 25ºC. Molar mass of the
(C) moles of the solute per 1000 g of the solvent. solute is 60g/mol.
(D) moles of the solute per 100g of the solvent. ( R= 0.0820 lit-atm K–1 mol-1 ) [IJSO-Stage-II/2014]
(A) 3.66 x 10–3 atm (B) 3.66 atm.
31. 1.000 mL of 0.100 mol L–1 hydrochloric acid was diluted (C) 2.44 atm (D) 1.63 x 10–3 atm
to 100.0 mL with deionised water. 10.00 mL of this solu-
tion was diluted to 100.0 mL again using deionised water. 40. Assertion (A) : Sodium carbonate can be titrated against
What is the pH of the final solution ? sulphuric acid by using either phenolphthalein or methyl
[IJSO-Stage-I/2013] orange as indicator.
(A) 2 (B) 3 Reason (R) : The volume of sulphuric acid required to
(C) 4 (D) 8 produce colour change for two indicators is different.
32. Amol took 10 mL of 2.2 x 10 –5 M hydrochloric acid [IJSO-Stage-II/2014]
solution. He then diluted it to 1 litre. He found that the pH (A) Both (A) and (R) are true and (R) is the correct expla-
of diluted solution is : [IJSO-Stage-II/2014] nation of (A).
(A) 4.7 (B) 6.7 (B) Both (A) and (R) are true and (R) is not the correct
(C) 4.5 (D)) 6.5 explanation of (A).
(C) (A) is true but (R) is false.
33. Anand wanted to prepare a salt solution of pH 1.0. For (D) (A) is false but (R) is true
that he used solution of a strong monovalent base and
acid. He tried different combination to obtain the required 41. A chemist's report on a batch of pharmaceutical products,
Aspirin (C 9H 8O 4) (250 mg tablets) and paracetamol
solution. Can you help him to decide which one is the
(C 8H 9NO 2) (500 mg tablets) indicated a '+0.5% weigh
most correct combination of statements given below ?
error in each tablet. Due to this error, the consumer gets
[IJSO-Stage-2/2015]
extra 'x' molecules of aspirin per tablet and extra 'y'
I. 100ml of (M/10) acid and 100ml of (M/10) alkali.
molecules of paracetamol per tablet, Choose the 'best'
II. 55ml of (M/10) acid and 45ml of (M/10) alkali. relation between x and y IJSO Stage-1/2015
III. 10ml of (M/10) acid and 90ml of (M/10) alkali. (A) x = y (B) x > y
IV. 75ml of (M/5) acid and 25ml of (M/5) alkali. (C) y > x (D) x = 2y
(A) I and III (B) II and IV
(C) Only II (D) Only IV 42. Sodium reacts with excess oxygen to form sodium oxide.
A student wants to prepare 1.24g of sodium oxide. While
34. In the reaction,
2KCl3  2KCl + 3O2 doing the calculations, he uses atomic number of
What is the volume of oxygen released under NTP con- sodium instead of atomic mass. What is the approximate
ditions when 36.75g of KClO3 is heated ? percentage error in the mass of sodium oxide obtained
[IJSO-Stage-I/2014-15]
due to this mistake? IJSO Stage-1/2015
(A) 3.6 litres (B) 7.2 litres
(C) 18 litres (D) 10 litres (A) 11% (B) 23%
(C) 48% (D) 60%

PAGE # 20
Pre-foundation Career Care Programmes (PCCP) Division
43. The composition (v/v) of air is found to be 78% nitrogen, 49. When 1 gram of a mixture of aluminium and zinc was
21% oxygen, and 1% argon. The density of air at STP is treated with HCl, a gas was liberated. At the end of the
IJSO Stage-2/2016 reaction, the volume of the liberated gas was found to
(A) 2.24 mg/cm3 (B) 0.65 mg/cm3 be 524 cm 3, under STP conditions. The individual
(C) 0.39mg/cm3 (D) 1.3 mg/cm3 weights of aluminium and zinc in the mixture, respectively,
are : [IJSO-Stage-I/2016-17]
44. Suhita took out 100cm3 of X molar aqueous solution of (A) 0.2 g and 0.8 g (B) 0.8 g and 0.2 g
hydrogen peroxide from the refrigerator. On warming to (C) 0.5 g and 0.5 g (D) 0.322g and 0.678 g
19.5ºC, she observed 3dm3 of oxygen gas was produced
(at 1 atm pressure). Assuming complete decomposition
50. Acid base reaction are extremely common in nature and
of hydrogen peroxide, the value of X is approximately
therefore it is of utmost interest to a chemist. A chemist
IJSO Stage-2/2016
gets a sample of drain cleaner and wants to find out the
(A) 1.25 (B) 2.5
exact amount of sodium hydroxide present in it. He uses
(C) 0.8 (D) 0.5
23 mL of 0.9M Phosphoric acid to completely neutralize
45. Weights of a metal ball recorded in air, in water and in a the base. IJSO Stage-2/2016
liquid are 56N, 49N and 42N respectively. Specific gravity (I) Write the balanced chemical equation for the reaction.
(or relative density) of the solid and that of the liquid is (II) How many grams of sodium hydroxide did the
respectively IJSO Stage-2/2016 chemist find in the sample ?
(A) 8 & 6 (B) 8 & 2 (III) A chemist needs to find the molarity of 10% w/w
(C) 8 & 1.4 (D) 7 & 0.6 HCl. What is the molarity of the solution if the density of
the solution is 1.047 g/cm3 and molecular weight of HCl
46. Miska combined one liter oxygen gas and one liter is 36.5g/mole
hydrogen gas by applying an electric spark in a vessel at (IV) There are many acid base reactions that take place
STP. She observed that water is formed. Find the weight in the body. If the acid content in the stomach is increased,
of water and the weight of unreacted component antacids are used to neutralize it. A particular sample of
respectively. IJSO Stage-2/2016 antacid tablet contains sodium bicarbonate. W hen
(A) 0.4017 g and 0.7143 g ingested, it reacts with the gastric juice (hydrochloric
(B) 0.8036 g and 1.4286 g acid) in the stomach to give off carbon dioxide gas. When
(C) 0.4017 g and 1.4286 g a 1.2 g tablet reacted with 40.00 mL of hydrochloric acid
(D) 0.8036 g and 0.7143 g (density : 1.140 g/mL), carbon dioxide gas released at
STP if its density is 1.98 g/L.
47. It 0.50 mole of a monovalent metal (M+1) halide is mixed
with 0.2 mole of a divalent metal (L +2) phosphate, the 51. Potassium chlorate (KClO 3) decomposes on heating
maximum number of moles of M3PO4 that can be formed into potassium chloride (KCl) and oxygen (O 2) gas.
is [IJSO-Stage-I/2016-17] Potassium bicarbonate (KHCO 3) decomposes on
(A) 0.25 (B) 0.30 heating to give potassium carbonate (K2CO3) and release
(C) 0.16 (D) 0.20 water vapour and carbon dioxide. Potassium carbonate
(K 2CO 3) on further heating decomposes to potassium
48. Every major city in India has a pollution control board to oxide (K 2O) and carbon dioxide. However, potassium
monitor air and water pollution. The following data is chloride (KCl) does not decompose on further heating.
from three different localities in Bangalore city from the Dina has a mixture of potassium chlorate, potassium
year 2015. bicarbonate and potassium carbonate. W hen she
heated 1000g of the mixture, she observed than 18g
Locality Annual average of SO 2 in the air (volume/volume) water vapour, 290g carbon dioxide and 40g oxygen gas.
P 16.3 mL/m 3 Assuming complete decomposition, what is the
Y 16.3 ppb (m 3/ m 3 )
composition of the initial mixture in weight% ?
IJSO Stage-2/2016
Z 16.3 ppm (m 3 / m 3)

ppb stands for parts per billion and ppm stands for parts 52. The purpose of an air bag is to slow the passenger’s
per million.These are different units to forward movement into the steering wheel (or dash
express.concentration.They are very similar to percent- board) during a collision and also to provide a
age (which is actually parts per hundred). Based on the cushion between the passenger and the steering wheel.
above data, which place will you choose to live in ? The goal of an air bag is to help the passenger come to
[IJSO-Stage-I/2016-17] a stop with minimum damage. One of the ways an air
(A) All localities are equally polluted, so I have no prefer- bag helps reduce injury is by spreading the force of im-
ence. pact with the dashboard or steering wheel over a larger
(B) P is the more polluted tha Y and Z, hence I will live in area, as illustrated in Figure 1. When the force is spread
either Y or Z. over a larger area of the body, the injuries are less
(C) Locality Y is least polluted , hence I will live in Y. severe. ` [IJSO-Stage-II/2014]
(D) Z and Y are more polluted than P, hence I will live in
P.

PAGE # 21
Pre-foundation Career Care Programmes (PCCP) Division
Figure Force distribution during collision
comparing air bag to no air bag.
A certain model of car is equipped with 65.0 liter air bag
that inflates to 89.4 m/s in 40 milliseconds. The weight
of air bag is 2.0 Kg and the thickness of fully inflated air
bag is 30.0 cm.
One method used to inflate air bags in cars is to use
nitrogen produced chemically from the decomposition
of sodium azide:
2NaN3(s)  2Na (s) + 3N2 (g) ————— (A)
The sodium formed reacts with potassium nitrate to give
more nitrogen:
10 Na(s) + 2KNO3(s)  K2O(s) + 5Na2O(s) + N2(g) —
———— (B)
i. Calculate the ratio (by mass) in which the sodium azide
and potassium nitrate should be mixed in order that no
metallic sodium remains after the
reaction.
Na2O and K2O2, are highly reactive, so it would be
unsafe to allow them to be the end product of the airbag
detonation. These metal oxides react with silicon diox-
ide (SiO 2) in a final reaction to produce silicate glass,
which is harmless and stable.
ii. Write the reaction of SiO2 with sodium oxide and po-
tassium oxide.
iii. Calculate the total mass of the solid mixture of so-
dium azide and potassium nitrate needed to inflate a 72
dm3 air bag is filled with nitrogen gas at of 1 atm and at
room temperature (27oC).
Consider the molar volume of nitrogen gas as 24.0 dm3
at 300 K and universal gas constant, R = 0.0821 liter.
atm.mole–1K –1
(Important : Show all your calculation step clearly)
The sodium azide is prepared commercially by the re-
action between dinitrogen monoxide and sodium amide.
N2O(g) + 2NaNH2(s)  NaN3(s) + NaOH(s) + NH3(g)
.Hr = 55.8 KJ / mol
iv. Calculate Hr for reaction (A) above, the decomposi-
tion of sodium azide.
Given :
Compound N2O(g) NaNH2(s) NaOH(s) NH3(g)
Hf (KJ/mol) +82.0 –123.7 – 425.2 –46.1

PAGE # 22
Pre-foundation Career Care Programmes (PCCP) Division
EQUIVALENT CONCEPT

OXIDATION & REDUCTION has oxidation number equal to –2.


In case of
Let us do a comparative study of oxidation and reduction
(i) peroxide (e.g. H2O2, , Na2O2 ) is –1,
:
(ii) super oxide (e.g. KO2) is –1/2
Oxidation Reduction (iii) ozonide (e.g. KO3) is –1/3
1. Addition of Oxygen 1. Removal of Oxygen (iv) in OF2 is + 2 & in O2F2 is +1
e.g. 2Mg + O2  2MgO e.g. CuO + C  Cu + CO  Hydrogen atom :
2. Removal of Hydrogen 2. Addition of Hydrogen In general, H atom has oxidation number equal to +1.
e.g. H2S+Cl22HCl + S e.g. S + H2  H2S But in metallic hydrides ( e.g. NaH, KH), it is –1.

3. Increase in positive 3. Decrease in positive charge  Halogen atom :


charge charge In general, all halogen atoms (Cl, Br , I) have oxidation
e.g. Fe2+  Fe3+ + e– e.g. Fe3+ + e–  Fe2+ number equal to –1.
But if halogen atom is attached with a more
4. Increase in oxidation 4. Decrease in oxidation electronegative atom than halogen atom, then it will
number number show positive oxidation numbers.
(+2) (+4) (+7) (+2)
5 5 7 5
e.g. SnCl2  SnCl4 e.g. MnO4–  Mn2+ e.g. K ClO , HI O , HCI O , KBrO
3 3 4 3

5. Removal of electron 5. Addition of electron  Metals :


e.g. Sn2+  Sn4+ + 2e– e.g. Fe3+ + e–  Fe2+ (a) Alkali metal (Li , Na, K, Rb, .......) always have
oxidation number +1
Oxidation Number
(b) Alkaline earth metal (Be , Mg , Ca .......) always
 It is an imaginary or apparent charge developed over have oxidation number +2.
atom of an element when it goes from its elemental (c) Aluminium always has +3 oxidation number
free state to combined state in molecules.
Note : Metal may have negative or zero oxidation
 It is calculated on basis of an arbitrary set of rules. number
 It is a relative charge in a particular bonded state.  Oxidation number of an element in free state or in
allotropic forms is always zero
 In order to keep track of electron-shifts in chemical
reactions involving formation of compounds, a more 0 0 0 0
e.g. O 2 , S 8 , P4 , O3
practical method of using oxidation number has been
developed.  Sum of the oxidation numbers of atoms of all elements
 In this method, it is always assumed that there is a in a molecule is zero.
complete transf er of electron f rom a less  Sum of the oxidation numbers of atoms of all elements
electronegative atom to a more electronegative atom. in an ion is equal to the charge on the ion .
 If the group number of an element in modern periodic
Rules governing oxidation number table is n, then its oxidation number may vary from
The following rules are helpful in calculating oxidation (n – 10) to (n – 18) (but it is mainly
number of the elements in their different compounds. applicable for p-block elements )
It is to be remembered that the basis of these rule is e.g. N- atom belongs to 15th group in the periodic
the electronegativity of the element . table, therefore as per rule, its oxidation number may
vary from
 Fluorine atom :
3 2 3 4 5
Fluorine is most electronegative atom (known). It –3 to +5 ( N H3 ,NO , N 2 O3 , N O 2 , N 2 O 5 )
always has oxidation number equal to –1 in all its
compounds  The maximum possible oxidation number of any
element in a compound is never more than the
 Oxygen atom : number of electrons in valence shell.(but it is
In general and as well as in its oxides , oxygen atom mainly applicable for p-block elements )

PAGE # 23
Calculation of average oxidation number : 2. If there is a bond between different type of atoms :
Solved Example e.g. A – B (if B is more electronegative than A)
Then after bonding, bonded pair of electrons are
Example-1 : Calculate oxidation number of underlined counted with B - atom .
element :
(a) Na2 S2O3 (b) Na2 S 4O6 Example : Calculate oxidation number of each atom
in HCl molecule
Solution. (a) Let oxidation number of S-atom is x. Now
work accordingly with the rules given before .
(+1) × 2 + (x) × 2 + (–2) ×3 =0
x=+2 Structure :

(b) Let oxidation number of S-atom is x


 (+1) × 2 + (x) × 4 + (–2) × 6 = 0 Note : Electron of H-atom is now counted with
x = + 2.5 Cl-atom, because Cl-atom is more electronegative
than H-atom
 It is important to note here that Na2S2O3 have
two S-atoms and there are four S-atom in Na2S4O6. H : Number of electrons in the valence shell = 1
However none of the sulphur atoms in both the Number of electrons taken up after bonding = 0
compounds have + 2 or + 2.5 oxidation number, it is Oxidation number of H = 1 – 0 = + 1
the average of oxidation number, which reside on each Cl : Number of electrons in the valence shell = 7
sulphur atom. Therefore, we should work to calculate Number of electrons taken up after bonding = 8
the individual oxidation number of each sulphur atom Oxidation number of Cl = 7– 8 = – 1
in these compounds.
Solved Example
Calculation of individual oxidation number Example-2 Calculate individual oxidation number of each
It is important to note that to calculate individual S-atom in Na2S2O3 (sodium thiosulphate) with the
oxidation number of the element in its compound one help of its structure .
should know the structure of the compound and use Solution. Structure :
the following guidelines.
Formula :
Oxidation Number = Number of electrons in the
valence shell – Number of electrons taken up after
bonding
Guidelines : It is based on electronegativity of
elements.
1. If there is a bond between similar type of atom
and each atom has same type of hybridisation, then
bonded pair electrons are equally shared by each
element.
Example : Calculate oxidation number of each Cl-
atom in Cl2 molecule Note :  (central S-atom) is sp3 hybridised (25%
Structure :
s-character) and  (terminal S-atom) is sp 2
hydbridised (33% s-character). Therefore,
terminal sulphur atom is more electronegative
than central sulphur atom. Now, the shared pair
of electrons are counted with terminal S-atom.
 , S-atom : Number of electrons in the valence
shell = 6
Number of electrons left after bonding = 0
 : Number of electrons in the valence shell = 7. Oxidation number of central S-atom = 6 – 0 = + 6
Number of electrons taken up after bonding = 7. , S-atom : Number of electrons in the valence shell
 oxidation number = 7 – 7 = 0. =6
 : similarly, oxidation number = 7 – 7 = 0 Number of electrons left after bonding = 8

PAGE # 24
Oxidation number of terminal S-atom = 6 – 8 = – 2 oxidation number increases or which undergoes loss
Now, you can also calculate Average Oxidation of electrons in a redox reaction are termed as
reductants.
6  ( 2)
number of S = = + 2 (as we have calculated e.g. K , Na2S2O3 etc are the powerful reducing
2
agents.
before)
Note : There are some compounds also which can
Miscellaneous Examples : work both as oxidising agent and reducing agent
In order to determine the exact or individual e.g. H2O2, NO2–
oxidation number we need to take help from
the structures of the molecules. Some special HOW TO IDENTIFY WHETHER A PARTICULAR
cases are discussed as follows: SUBSTANCE IS AN OXIDISING OR A REDUCING
O O AGENT
 The structure of CrO5 is Cr Find the oxidation state
(O.S.) of the central atom
||
O O O

From the structure, it is evident that in CrO5 there are If O.S. If O.S. If O.S. = Intermediate
= Maximum = Minimum between maximum
two peroxide linkages and one double bond. The O.S. O.S. & minimum
contribution of each peroxide linkage is –2. Let the
oxidation number of Cr is x. It is oxidizing It is a reducing It can act both as
agent agent
 x + (–2)2 + (–2) = 0 or x = 6 reducing agent
& oxidising agent
 Oxidation number of Cr = + 6 Ans
O It can
disproportionate
 The structure of H2SO5 is H O O S as well

O REDOX REACTION
H O
A reaction in which oxidation and reduction
From the structure, it is evident that in H2SO5, there
simultaneously take place is called a redox reaction
is one peroxide linkage, two sulphur-oxygen double
In all redox reactions, the total increase in oxidation
bonds and one OH group. Let the oxidation number
number must be equal to the total decrease in
of S = x.
oxidation number.
 (+ 1) + (– 2) + x + (–2) 2+ (–2) + 1 = 0
2 5
or x + 2 – 8 = 0 or x–6=0
e.g. 10 Fe SO 4 + 2KMnO4 + 8H 2 SO 4 
or x=6
 Oxidation number of S in H2SO5 is + 6 Ans. 3 2
5 Fe2 SO 4 3 + 2 Mn SO 4 + K2SO4 + 8H2O
OXIDISING & REDUCING AGENT
DISPROPORTINATION REACTION
Oxidising and reducing agent
A redox reaction in which same element present in a
Oxidising agent or Oxidant :
particular compound in a definite oxidation state is
Oxidising agents are those compounds which
oxidized as well as reduced simultaneously is a
can oxidise others and reduce itself during the
disproportionation reaction.
chemical reaction. Those reagents in which for an
Disproportionation reactions are a special type of
element, oxidation number decreases or which
redox reactions. One of the reactants in a
undergoes gain of electrons in a redox reaction are
disproportionation reaction always contains an
termed as oxidants.
element that can exist in at least three oxidation
e.g. KMnO 4 , K 2 Cr 2 O 7 , HNO 3 ,
states. The element in the form of reacting substance
conc.H2SO4 etc are powerful oxidising agents .
is in the intermediate oxidation state and both higher
Reducing agent or Reductant : and lower oxidation states of that element are formed
Reducing agents are those compounds which can in the reaction. For example :
reduce other and oxidise itself during the chemical 1 2 0
reaction. Those reagents in which for an element, 2H2O 2 (aq)  2H2O () + O 2 (g)

0 2 2
S 8 (s) +12OH¯(aq) 4S 2 (aq) + 2S 2 O 23 ( aq) +6H2O ()

PAGE # 25
0 1 1 ClO2–  Cl– + ClO3–
Cl2 (g) + 2OH¯(aq)  ClO  (aq) + Cl  ( aq) + H2O ()
Consider the following reactions : ClO3–  Cl– + ClO4–

(a) 2KClO3  2KCl + 3O2 11. HNO2  NO + HNO3


KClO3 plays a role of oxidant and reductant both.  Rev erse of disproportionation is called
Here, Cl present in KClO3 is reduced and O present Comproportionation. In some of the
in KClO3 is oxidized. Since same element is not disproportionation reactions, by changing the medium
oxidized and reduced, so it is not a (from acidic to basic or reverse), the reaction goes in
disproportionation reaction, although it looks like backward direction and can be taken as an example
one. of Comproportionation reaction.

(b) NH4NO2  N2 + 2H2O ¯ + O3¯ + H+  2 + H2O


Nitrogen in this compound has -3 and +3 oxidation Balancing of redox reactions
number, which is not a definite value. So it is not a All balanced equations must satisfy two criteria.
disproportionation reaction. It is an example of 1. Atom balance (mass balance ) :
comproportionation reaction, which is a class of redox There should be the same number of atoms of
reaction in which an element from two different each kind on reactant and product side.
oxidation state gets converted into a single oxidation
state. 2. Charge balance :
5 7
The sum of actual charges on both sides of the
–1
(c) 4KClO3  3KClO4 + KCl equation must be equal.
There are two methods for balancing the redox
It is a case of disproportionation reaction and Cl equations :
atom is disproportionating. 1. Oxidation - number change method
List of some important disproportionation 2. Ion electron method or half cell method
reactions  Since First method is not very much fruitful for
the balancing of redox reactions, students are advised
1. H2O2  H2O + O2
to use second method (Ion electron method ) to
2. X2 + OH–(dil.)  X¯ + XO¯ (X = Cl, Br, I) balance the redox reactions

3. X2 + OH–(conc.)  X¯ + XO3¯ Ion electron method :


By this method redox equations are balanced in two
F2 does not undergo disproportionation as it is different medium.
the most electronegative element. (a) Acidic medium (b) Basic medium
F2 + NaOH(dil.)  F– + OF2  Balancing in acidic medium
F2 + NaOH(conc.)  F– + O2 Students are adviced to follow the following steps
to balance the redox reactions by Ion electron method
4. (CN)2 + OH–  CN– + OCN– in acidic medium
5. P4 + OH–  PH3 + H2PO2¯
Solved Example
6. S8 + OH–  S2– + S2O32–
Example-3 Balance the following redox reaction :
7. MnO42–  MnO4¯ + MnO2 FeSO4 + KMnO4 + H2SO4  Fe2(SO4)3 + MnSO4 + H2O +
8. NH2OH  N2O + NH3 K2SO4
Sol.Step– Assign the oxidation number to each element
NH2OH  N2 + NH3 present in the reaction.
9. Oxyacids of Phosphorus ( +1, +3 oxidation 2 6 2 1  7 – 2 1 6 2

number) Fe S O 4 + K MnO 4 + H 2 S O 4
3 6 2 2 6 2 1 2
H3PO2  PH3 + H3PO3  Fe 2 (S O 4 )3 + Mn S O 4 + H 2 O
H3PO3  PH3 + H3PO4 Step  :
Now convert the reaction in Ionic form by eliminating
10. Oxyacids of Chlorine( Halogens)( +1, +3, +5
the elements or species, which are not undergoing
Oxidation number)
either oxidation or reduction.
ClO–  Cl– + ClO2–

PAGE # 26
7 Step X :
Fe2+ + Mn O 4  Fe3+ + Mn2+ Now convert the Ionic reaction into molecular form
Step  : by adding the elements or species, which are removed
Now identify the oxidation / reduction occuring in the in step (2).
reaction Now, by some manipulation, you will get :
5
5 FeSO4 + KMnO4 + 4H2SO4  Fe2 (SO4)3 +
2
1
MnSO4 + 4H2O + K SO or
2 2 4
Step V : Spilt the Ionic reaction in two half, one for 10FeSO4 + 2KMnO4 + 8H2SO4  5Fe2(SO4)3 +
oxidation and other for reduction. 2MnSO4 + 8H2O + K2SO4.

oxidation  Balancing in basic medium :


Fe2+    Fe3+ MnO 4  
Re duction
  Mn 2 In this case, except step VI, all the steps are
same. We can understand it by the following example:
Step V :
Balance the atom other than oxygen and hydrogen Solved Example
atom in both half reactions Example-4 Balance the following redox reaction in
basic medium :
 Fe3+ ClO– + CrO2– + OH–  Cl– + CrO42– + H2O
Fe2+ MnO4 –  Mn2+
Sol. By using upto step V, we will get :
Fe & Mn atoms are balanced on both side. 1 3 6
Re duction Oxidation
Cl O     Cl – Cr O 2    Cr O 24
Step V :
Now balance O & H atom by H2O & H+ respectively Now, students are advised to follow step VI to
by the following way : For one excess oxygen atom, balance ‘O’ and ‘H’ atom.
add one H2O on the other side and two H+ on the 2H+ + ClO–  Cl– + H2O | 2H2O+ CrO2–  CrO42– + 4H+
same side.
 Now, since we are balancing in basic medium,
Fe2+  Fe3+ (no oxygen atom ) ......(i)
therefore add as many as OH– on both side of equation
8H+ + MnO4–  Mn2+ + 4H2O ....(ii) as there are H+ ions in the equation.
Step V : 2OH– + 2H+ + ClO–  Cl– + H2O +2OH–
Equation (i) & (ii) are balanced atomwise. Now 4OH– + 2H2O + CrO2–  CrO42– + 4H+ + 4OH–
balance both equations chargewise. To balance the Finally you will get
charge, add electrons to the electrically positive side. Finally you will get
Fe2+  oxidation H2O + ClO–  Cl– + 2OH– ...........(i)
   Fe3+ + e– ......(1)

5e– + 8H+ + MnO4–Re duction 2+ 4OH– + CrO2–  CrO42– + 2H2O ........... (ii)
   Mn + 4H2O ....(2)
Now see equation (i) and (ii) in which O and H
Step V : atoms are balanced by OH– and H2O
The number of electrons gained and lost in each half Now from step VIII
-reaction are equalised by multiplying both the half
reactions with a suitable factor and finally the half 2e– + H2O + ClO–  Cl– + 2OH– ..... (i) ×3
reactions are added to give the overall balanced 4OH– + CrO2–  CrO42– + 2H2O + 3e–.... (ii) ×2
reaction.
–––––––––––––––––––––––––––––––––––––––––––
Here, we multiply equation (1) by 5 and (2) by 1 and
add them : Adding : 3ClO– + 2CrO2– + 2OH–  3Cl– +
2CrO42– + H2O
Fe2+  Fe3+ + e– ..........(1) × 5
CONCEPT OF EQUIVALENT

5e   8H  MnO 4  Mn 2  4H2O .........( 2) 1

Equivalent mass of element
5Fe 2   8H  MnO 4  5Fe 3   Mn 2  4H2 O

Number of parts by mass of an element which
(Here, at his stage, you will get balanced redox reacts or displaces from a compound 1.008 parts
reaction in Ionic form) by mass of hydrogen, 8 parts by mass of oxygen
and 35.5 parts by mass of chlorine, is known as
the equivalent weight of that element.

PAGE # 27
e.g.2Mg + O 2  2MgO acid molecule
Solved Example
48g 32g Ex- 5 HCl, H2SO4 H3PO4
12g 8g
 32 g of O 2 reacts with 48 g of Mg
48  8
 8 g of O 2 = = 12 g
32 H3PO3
 Equivalent weight of Mg = 12
Similarly, Zn + H2SO 4  ZnSO 4 + H2
65.5 g 32.75
{see there are only two replaceable H+ions}
65.5 Sol. Valency factor  1 2 3 2
 Equivalent weight of Zn = = 32.75 g
2 (assume 100% dissicoiation)

3 M M M M
Al + Cl  AlCl3 Eq. wt. (E) 
2 2 1 2 3 2
 Replaceable hydrogen atoms are those hydrogen
3
27 g × 71 g atoms which are attached with the atoms of group VI
2 and group VII i.e. O,S,Se,Te, & F, Cl ,Br ,I.
 111.5 g chlorine reacts with 27 g of Al.
27  35.5  For Bases :
 35.5 chlorine reacts with = 9.0 g of Al Valency factor = number of replacable OH– ions per
111.5
base molecule.
27
 Equivalent weight of aluminium = = 9.0 Solved Example
3
As we can see from the above examples that Ex-6 NaOH, KOH
equivalent weight is the ratio of atomic weight and Sol. v .f.  1 1
a factor (say n-factor or valency factor) which is in M M
above three cases is their respective valencies. Eq. wt. 
1 1
Equivalent weight (E) :  Bases may be defined as the substances in
In general, Eq. wt. (E) which OH group is/are directly attached with group I
Atomic weight elements (Li,Na, K,Rb,Cs), group II elements (Be,
Mg,Ca,Ba ) or group III elements (Al, Ga,In,Tl),
= or Molecular weight  Mol. wt.  M transition metals, non-metallic cations like PH4+ ,
valency factor( v.f ) n  factor x NH4+ etc.

mass of species  Acid - base reaction :


Number of Equivalents = eq. wt. of that species
In case of acid base reaction, the valence factor
For a solution, Number of equivalents = N1V1, is the actual number of H+ or OH– replaced in the
where N is the normality and V is the volume in litres reaction. The acid or base may contain more number
of replaceble H+ or OH– than actually replaced in
 Equivalent mass is a pure number which, when reaction.
expressed in gram, is called gram equivalent mass.  v. f. for base is the number of H+ ion from the
acid replaced by each molecule of the base
 The equivalent mass of substance may have different
values under different conditions.
Solved Example
 There in no hard and fast rule that equivalent weight Ex-7. 2NaOH + H2 SO4  Na2 SO4 + 2H2O
will be always less than the molecular
mass. Base Acid
Sol. Valency factor of base = 1
Valency factor calculation : Here, two molecule of NaOH replaced 2H+ ion from
 For Elements : the H2 SO4. Therefore, each molecule of NaOH
Valency factor = valency of the element. replaced only one H+ ion of acid, so v.f. = 1.
 For Acids :
 v. f. for acid is the number of OH– replaced from the
Valency factor = number of replaceable H+ ions per
base by each molecule of acid

PAGE # 28
Solved Example
Solution. Mn in KMnO4 is going from +7 to +2 , so
Ex-8 NaOH + H2SO4  NaHSO4 + H2O change in oxidation number per molecule of KMnO4
is 5. So the valency factor of KMnO 4 is 5 and
Base Acid
M
Sol. Valency factor of acid = 1 equivalent weight is .
5
Here, one of molecule of H2SO4 replaced one OH–
from NaOH. Therefore, valency factor for H2SO4 is Normality :
one Normality of a solution is defined as the number of
equivalents of solute present in one litre (1000 mL)
Mol.wt solution.
 Eq. wt. of H2SO4 =
1
Let V mL of a solution is prepared by dissolving W g
of solute of equivalent weight E in water.
 Salts :
W
(a) In non-reacting condition  Number of equivalents of solute =
E

 Valency factor = Total number of positive charge W


or negative charge present in the compound. VmL of solution contain equivalents of solute
E

Solved Example W  1000


 1000 mL solution will contain EV
Ex-9 Na2 CO3 , Fe2(SO4)3 FeSO4.7H2O
equivalents of solute.

Sol. V.f. 2 2×3 = 6 W  1000


2  Normality (N) = E V

M M M  Normality (N) = Molarity x Valency factor


Eq.wt.
2 6 2
N × V (in mL) = M × V (in mL) × n
or
Note : In case of hydrated salt, positive/negative
charge of water molecule is not counted.  milliequivalents = millimoles × n

Solved Example
(b) In reacting condition
Solved Example Ex-12 Calculate the normality of a solution containing
Ex-10 Na2 CO3 + HCl  NaHCO3 + NaCl 15.8 g of KMnO 4 in 50 mL acidic solution.

Base Acid W  1000


Sol. Normality (N) =
E V
So. It is an acid base reaction, therefore valency factor Here W = 15.8 g , V = 50 mL
for Na2CO3 is one while in non-reacting condition, it
will be two. molar mass of KMnO 4
E= = 158/5 = 31.6
Solved Example Valency factor

So, normality = 10 N
(c) Equivalent weight of oxidising / reducing
agents in a redox reaction
Ex-13 Calculate the normality of a solution containing
In case of redox change , v.f. = Total change in
50 mL of 5 M solution of K2Cr2O 7 in acidic
oxidation number per molecule .
medium.

Solved Example Sol. Normality (N) = Molarity × valency factor


= 5 x 6 = 30 N
Example-11 KMnO4 + H2O2  Mn2+ + O2

PAGE # 29
LAW OF EQUIVALENCE V(0.02) ( 25 )(0.2)
  = (from (1) & (2))
The law states that one equivalent of an element 1 5
combine with one equivalent of the other. In a  V = 50 mL.
chemical reaction, equivalents and milli equivalents
Method -2 : Equivalent Method :
of reactants react in equal amount to give same
number of equivalents or milli equivalents of products At the equivalence point,
separately. milliequivalents of MnO4¯ = milliequivalents of Fe2+
Accordingly M1 × vf 1 × V1 = M2 × vf 2 × V2
(i) aA + bB  mM + nN
0.02 × 5 × V1 = 0.2 × 1 × 25
meq of A = meq of B = meq of M = m.eq. of N
(ii) In a compound MxNy ( MnO4–  Mn2+; v.f. = 5, Fe2+  Fe3+ ; v.f. = 1)
meq of MxNy = meq of M = meq of N  V1 = 50 mL.
Solved Example TITRATIONS
Ex-14 Find the number of moles of KMnO4 needed to
oxidise one mole Cu2S in acidic medium. Titration is a procedure f or determining the
concentration of a solution by allowing a carefully
The reaction is KMnO4 + Cu2S  Mn2+ + Cu2+ + SO2
measured volume to react with a standard solution of
Sol. From law of equivalence,
another substance, whose concentration is known.
equivalents of Cu2S = equivalents of KMnO4
moles of Cu2S × v.f. = moles of kMnO4 × v.f. Standard solution - It is a solution whose
1 × 8 = moles of KMnO4 × 5  moles of KMnO4 = 8/5 concentration is known and is taken in burette. It is
( v.f. of Cu2S = 2 (2 – 1) + 1 (4 – (–2))) = 8 and v.f. of also called Titrant.
KMnO4 = 1 (7 –2) = 5)
There are two type of titrants :
Ex-15 The number of moles of oxalate ions oxidized by  Primary titrants/standard - These reagents can be
one mole of MnO4– ion in acidic medium are : accurately weighed and their solutions are not to be
standardised before use.
5 2
(A) (B) Ex : Oxalic acid, K2Cr2O7, AgNO3, CuSO4, ferrous
2 5
ammonium sulphate, hypo etc.
3 5
(C) (D)  Secondary titrants/standard : These reagents
5 3
cannot be accurately weighed and their solutions are
Sol. Equivalents of C2O4 = equivalents of MnO4–
2–
to be standardised before use.
x(mole) × 2 = 1 × 5
Ex : NaOH, KOH, HCl, H2SO4, 2, KMnO4 etc.
( v.f. of C2O42– = 2 (4 – 3) = 2
and v.f. of MnO4– = 1 (7 – 2) = 5). Titrate : Solution consisting of substance to be
estimated, generally taken in a beaker .
5
x= mole of C2O42– ions. Equivalence point : It is the point when number of
2
equivalents of titrant added becomes equal to number
Solved Example of equivalents of titrate.
Ex-16 How many millilitres of 0.02 M KMnO4 solution At equivalence point :
would be required to exactly titrate 25 mL of 0.2 n1V1M1 = n2V2M2
M Fe(NO3)2 solution in acidic medium ?
Sol. Method -1 : Mole concept method Indicator : An auxiliary substance added for physical
Starting with 25 mL of 0.2 M Fe2+, we can write : detection of completion of titration at equivalence
Millimoles of Fe2+ = 25 x 0.2 ........(1) point. It generally show colour change on completion
and in volume V (in milliliters) of the KMnO4, of titration.
Millimoles of MnO4¯ = V (0.02) ........(2)
The balanced reaction is : Type of Titrations :

MnO4¯ + 5Fe2+ + 8H+  Mn2+ + 5Fe3+ + 4H2O  Acid-base titrations (to be studided in Ionic
This requires that at the equivalent point, equilibrium)

m. moles of MnO –4 m.moles of Fe 2


=
1 5

PAGE # 30
 Redox Titrations
Some Common Redox Titrations
Table of Redox Titrations : (Excluding Iodometric / Iodimetric titrations)
––––––––––––––––––––––––––––––––––––––––––––––––––––––––––––––––––––––––––––––––––––––––––––––––––––––––––––––––––––––––––––––––––––
Estimation By titrating Reactions Relation*between
of with OA and RA
––––––––––––––––––––––––––––––––––––––––––––––––––––––––––––––––––––––––––––––––––––––––––––––––––––––––––––––––––––––––––––––––––––
1. Fe2+ MnO4¯ Fe2+  Fe3+ + e– 5Fe2+  MnO4¯
MnO4– + 8H+ + 5e–  Mn2+ + 4H2O Eq. wt. of Fe2+ = M/1
2. Fe2+ Cr2O72– Fe2+  Fe3+ + e– 6Fe2+  Cr2O72–
Cr2O72– + 14H+ + 6e–  2Cr3+ + 7H2O Eq.wt. of Cr2O72– = M/6
3. C2O42– MnO4¯ C2O42–  2CO2 + 2e– 5C2O42–  2MnO4¯
MnO4– + 8H+ + 5e–  Mn2+ + 4H2O Eq. wt. of C2O42– = M/2
4. H2O2 MnO4¯ H2O2  2H+ + O2 + 2e– 5H2O2  2MnO4¯
MnO4– + 8H+ + 5e–  Mn2+ + 4H2O Eq.wt. of H2O2 = M/2
5. As2O3 MnO4– As2O3 + 5H2O  2AsO43– + 10H+ + 4e– Eq. wt. of As2O3 = M/4
MnO4– + 8H+ + 5e–  Mn2+ + 4H2O
6. AsO33– BrO3– AsO33– + H2O  AsO43– + 2H+ + 2e– Eq. wt. of AsO33– = M/2
BrO3– + 6H+ + 6e–  Br– + 3H2O Eq.wt. of BrO3– = M/6
Permanganate Titrations :

 KMnO4 is generally used as oxidising agent in acidic medium, generally provided by dilute H2SO4 .

 KMnO4 works as self indicator persistent pink color is indication of end point.

 Mainly used for estimation of Fe2+ , oxalic acid ,oxalates, H2O2 etc.

Solved Example

Ex-17 Write the balanced reaction of titration of KMnO4 Vs oxalic acid in presence of H2SO4.
Sol. Reaction : 2KMnO4 + 3H2SO4 + 5H2C2O4  K2SO4 + 2MnSO4 + 8H2O + 10CO2
 M
Redox Changes : C23+  2C4+ +2e  EH2C2O 4  
 2

 M
5e + Mn7+  Mn2+  EKMnO 4  
 5
Indicator : KMnO4 acts as self indicator.

Ex-18 Write the balanced reaction of titration of KMnO4 Vs ferrous ammonium sulphate in presence of H2SO4.
Sol. Reaction : 2KMnO4 + 10[FeSO4(NH4)2SO4. 6H2O] + 8H2SO4 
5Fe2(SO4)3 + 10(NH4)2SO4 + K2SO4 + 2MnSO4 + 68H2O
 M
Redox Changes : Fe2+  Fe3+ + e  EFeSO 4  
 1

 M
Mn7+ + 5e  Mn2+  EKMnO 4  
 5
Indicator : KMnO4 acts as self indicator

PAGE # 31
HYDROGEN PEROXIDE (H2O2) Volume strength of H2 O 2
11.2
H2O2 can behave both like oxidising and reducing Strength (in g/L) : Denoted by S
agent in both the mediums (acidic and basic). Strength = Molarity × Mol. wt = Molarity × 34
Strength = Normality × Eq. weight = Normality × 17

Solved Example
Ex-19 20 mL of H2O2 after acidification with dilute
N
H2SO4 required 30 mL of KMnO4 for complete
 Oxidising agent : (H2O2  H2O) 12
(a) Acidic medium : 2e– + 2H+ + H2O2  2H2O oxidation. Final the strength of H2O2 solution.
v.f. = 2 [Molar mass of H2O2 = 34]
(b) Basic medium : 2e– + H2O2  2OH– Sol. meq. of KMnO4 = meq. of H2O2
v.f = 2
1
 Reducing agent : (H2O2  O2) 30 × = 20 × N
12
(a) Acidic medium : H2O2  O2 + 2H+ + 2e–
v.f = 2 30 1
(b) Basic medium : 2OH– + H2O2  O2 + 2H2O + 2e– N = = N
12  20 8
v.f = 2
Note : Valency factor of H2O2 is always equal to 2. 1
strength = N × equivalent mass = ×17 = 2.12 g/L.
8
Volume strength of H2O2 : Strength of H2O2 is
represented as 10V , 20 V , 30 V etc.
Hardness of water (Hard water does not
20V H2O2 means one litre of this sample of H2O2
give lather with soap)
on decomposition gives 20L of O2 gas at STP.
Temporary hardness - due to bicarbonates of Ca &
Decomposition of H2O2 is given as :
Mg
1
H2O2  H2O + O Permanent hardness - due to chlorides & sulphates
2 2 of Ca & Mg. There are some method by which we
1 can soften the water sample.
1 mole × 22.4 L O2 at STP = 34g
2
= 11.2 L O2 at STP (a) By boiling : 2HCO3–  H2O + CO2 + CO32–
To obtain 11.2 litre O2 at STP, at least 34 g H2O2 must or
be decomposed. By Slaked lime : Ca(HCO3)2 + Ca(OH)2  CaCO3
34 + 2H2O
For 20 L O2 , we should decompose atleast ×
11.2 Ca2+ + CO32–  CaCO3
20 g H2O2
(b) By Washing Soda :
34
 1 L solution of H2O2 contains ×20 g H2O2 CaCl2 + Na2CO3  CaCO3 + 2NaCl
11.2
(c) By ion exchange resins :
34 20
 1 L solution of H2O2 contains × Na2R + Ca2+  CaR + 2Na+
11.2 17
(d) By adding chelating agents like (PO3–)3 etc.
M 34
equivalents of H2O2 ( EH 2 O2   = 17) PARTS PER MILLION (ppm)
2 2
When the solute is present in very less amount, then
34 20 20 this concentration term is used. It is defined as the
Normality of H2O2 = × =
11.2 17 5. 6 number of parts of the solute present in every 1 million
 Normality of H2O2 (N) = parts of the solution. ppm can both be in terms of
mass or in terms of moles. If nothing has been
Volume strength of H2 O 2 specified, we take ppm to be in terms of mass. Hence,
5.6 a 100 ppm solution means that 100 g of solute is
NH2O2 NH2O2 present in every 1000000 g of solution.
 MH2O2 = =
v.f . 2 mass of A
ppmA = 10 6 = mass fraction × 106
 Molarity of H2O2 (M) = Total mass

PAGE # 32
Measurement of Hardness :
Hardness is measured in terms of ppm (parts per
million) of CaCO3 or equivalent to it. EXERCISE-1
mass of CaCO I. Calculation of equivalent weight
Hardness in ppm = 3 10 6
Total mass of solution
1. When N2 is converted into NH3, the equivalent weight of
nitrogen will be :
Solved Example (A) 1.67 (B) 2.67
Ex-20. 0.00012% MgSO 4 and 0.000111% CaCl 2 is (C) 3.67 (D) 4.67
present in water. What is the measured hardness
2. When HNO3 is converted into NH3, the equivalent weight
of water and millimoles of washing soda required
of HNO3 will be :
to purify water 1000 L water ? (M = molecular weight of HNO3)
Sol. Basis of calculation = 100 g hard water (A) M/2 (B) M/1
(C) M/6 (D) M/8
0.00012
MgSO4 = 0.00012g = mole
120 3. In the ionic equation 2K+BrO3– + 12H+ + 10e–  Br2
0.000111 + 6H2O + 2K+,
CaCl2 = 0.000111g = mole the equivalent weight of KBrO 3 will be : (where M =
111
molecular weight of KBrO3)
 equivalent moles of CaCO3 = (A) M/5 (B) M/2
(C) M/6 (D) M/4
 0.00012 0.000111 
   mole
 120 111  4. If molecular weight of KMnO4 is 'M', then its equivalent
weight in acidic medium would be :
 0.00012 0.000111  (A) M (B) M/2
 mass of CaCO3 =    ×
 120 111  (C) M/5 (D) M/4

100 = 2 × 10–4 g
5. In the conversion NH2OH  N2O,
Hardness (in terms of ppm of CaCO3)
the equivalent weight of NH2OH will be :
2  10 4 (M = molecular weight of NH2OH)
=  10 6 = 2 ppm (A) M/4 (B) M/2
100
(C) M/5 (D) M/1
CaCl2 + Na2CO3  CaCO3 + 2NaCl
6. In the reaction between SO2 and O3, the equivalent weight
NaSO4 + Na2CO3  MgCO3 + Na2SO4 of ozone is :
 Required Na2CO3 for 100g of water = (A) the same as its molecular weight
(B) half the molecular weight
 0.00012 0.000111  (C) one-third of the molecular weight
   mole = 2 × 10–6 mole (D) one-fourth of the molecular weight
 120 111 
 Required Na2CO3 for 1000 litre water = 7. The equivalent weight of phosphoric acid (H3PO4) in the
reaction :
2  10 6 2
 10 6  mole ( d = 1g/mL) NaOH + H3PO4  NaH2PO4 + H2O
100 100
(A) 59 (B) 49
20 (C) 25 (D) 98
= mole = 20 m mole
1000
8. The equivalent weight of MnSO 4 is half its molecular
weight when it is converted into
(A) Mn2O3 (B) MnO4–
(C) MnO2 (D) MnO42–

9. The equivalent weight of Mohr's salt,


FeSO4.(NH4)2SO4.6H2O is equal to
(A) Its molecular weight
(B) Atomic weight
(C) half-its molecular weight
(D) one-third its molecular weight

PAGE # 33
10. Vapour density of a metal chloride is 66. Its oxide contains 21. 250 ml of 0.1 N solution of AgNO3 are added to 250 ml of
53% metal. The atomic weight of metal is - a 0.1 N solution of NaCl. The concentration of nitrate ion
(A) 21 (B) 54 in the resulting solution will be -
(C) 26.74 (D) 2.086 (A) 0.1N (B) 1.2 N
(C) 0.01 N (D) 0.05 N
11. Equivalent mass of KMnO4, when it is converted to MnSO4
is - 22. 2g of NaOH and 4.9 g of H2SO4 were mixed and volume
(A) M/5 (B) M/3 is made 1 litre. The normality of the resulting solution
(C) M/6 (D) M/2 will be -
(A) 1N (B) 0.05 N
12. In a compound AxBy -
(C) 0.5 N (D) 0.1N
(A) Mole of A = Mole of B = mole of AxBy
(B) Eq. of A = Eq. of B = Eq. of AxBy
23. 1g of a metal carbonate neutralises completely 200 mL
(C) X × mole of A = y × mole of B = (x + y) × mole of AxBy
of 0.1N HCl. The equivalent weight of metal carbonate is
(D) X × mole of A = y × mole of B
(A) 25 (B) 50
II. Titration (C) 100 (D) 75

13. How many millilitres of 0.1N H 2 SO 4 solution will be 24. 100 mL of 0.5 N NaOH were added to 20 ml of 1N HCl
required for complete reaction with a solution containing and 10 mL of 3 N H2SO4. The solution is -
0.125 g of pure Na2CO3 : (A) acidic (B) basic
(A) 23.6 mL (B) 25.6 mL (C) neutral (D) none of these
(C) 26.3 mL (D) 32.6 mL
25. 1M solution of H2SO4 is diluted from 1 litre to 5 litres , the
14. If 25 mL of a H2SO4 solution reacts completely with 1.06 normality of the resulting solution will be -
g of pure Na2CO 3, what is the normality of this acid (A) 0.2 N (B) 0.1 N
solution : (C) 0.4 N (D) 0.5 N
(A) 1 N (B) 0.5 N
(C) 1.8 N (D) 0.8 N 26. 2 g of a base whose eq. wt. is 40 reacts with 3 g of an
acid. The eq. wt. of the acid is-
15. A certain weight of pure CaCO 3 is made to react (A) 40 (B) 60
completely with 200 mL of a HCl solution to give 224 mL (C) 10 (D) 80
of CO2 gas at STP. The normality of the HCl solution is:
(A) 0.05N (B) 0.1 N 27. The volume of water to be added to 200 mL of
(C) 1.0 N (D) 0.2 N seminormal HCl solution to make it decinormal is -
(A) 200 mL (B) 400 mL
16. 0.2 g of a sample of H2O2 required 10 mL of 1N KMnO4 in (C) 600 mL (D) 800 mL
a titration in the presence of H2SO4. Purity of H2O2 is-
(A) 25% (B) 85% 28. 0.7 g of Na2CO3 . xH2O is dissolved in 100 mL. 20 mL of
(C) 65% (D) 95% which required to neutralize 19.8 mL of 0.1 N HCl. The
value of x is -
N (A) 4 (B) 3
17. 150 ml of HCl is required to react completely with (C) 2 (D) 1
10
1.0 g of a sample of limestone. The percentage purity of
calcium carbonate is - 29. 0.45 g of an acid of molecular weight 90 was neutralised
(A) 75% (B) 50% by 20 mL of 0.5 N caustic potash. The basicity of the acid
(C) 80% (D) 90% is -
(A) 1 (B) 2
18. Normality of 1% H2SO4 solution is nearly - (C) 3 (D) 4
(A) 2.5 (B) 0.1
(C) 0.2 (D) 1 30. 1 litre of 18 molar H2SO4 has been diluted to 100 litres.
The normality of the resulting solution is -
19. An oxide of metal have 20% oxygen, the eq. wt. of metal (A) 0.09 N (B) 0.18
oxide is - (C) 1800 N (D) 0.36
(A) 32 (B) 40
(C) 48 (D) 52 N N
31. 50 ml of HCl is treated with 70 ml NaOH. Result-
10 10
20. How much water is to be added to dilute 10 mL of 10 N ant solution is neutralized by 100 ml of sulphuric acid.
HCl to make it decinormal ? The normality of H2SO4 -
(A) 990 mL (B) 1010 mL (A) N/50 (B) N/25
(C) 100 mL (D) 1000 mL (C) N/30 (D) N/10

PAGE # 34
N
32. 200 mL of
10
HCl were added to 1 g calcium carbon- EXERCISE-2
ate, what would remain after the reaction ?
(A) CaCO3 (B) HCl COMPETITIVE EXAM PREVIOUS YEARS’ QUESTIONS :
(C) Neither of the two (D) Part of both
1. W hat is the ratio of number of electrons gamed by
III. Hydrogen peroxide, Hardness of water, % strength of acidified KMnO 4 and acidified K 2Cr2O 7 in the reaction
oleum, Available chlorine given below
 
H
KMnO4  2+ 2 H 3+
33. The volume strength of 1.5 N H2O2 solution is :  Mn ; K2Cr O7 
 Cr
(A) 4.8 V (B) 8.4 V
[IJSO-Stage-I/2015-16]
(C) 3 V (D) 8 V
(A) 5:6 (B) 6:5
34. Find the volume strength of H2O2 solution prepared by (C) 3:5 (D) 5:3
mixing of 250 mL of 3N H 2O 2 & 750 mL of 1N H 2O 2
solution : 2. Sati was studying neutralisation reaction. She
(A) 1.5 V (B) 8.4 V accidentally dropped 'x' grams of a sodium carbonate
(C) 5.6 V (D) 11.2 V monohydrate into a 100 mL solution of HCI whose
35. Temporary hardness is due to bicarbonates of Mg2+ and concentration was 0.25 M. 10mL from this final solution
Ca2+. It is removed by addition of CaO as follows : was taken and titrated against 0.05M NaOH solution to
Ca(HCO3)2 + CaO  2CaCO3 + H2O yield a titre value of 20mL. What is the value of 'x'.
[IJSO-Stage-I/2015-16]
Mass of CaO required to precipitate 2 g CaCO3 is :
(A) 2 g (B) 0.56 g (A) 186mg (B) 93mg
(C) 0.28 g (D) 1.12 g (C) 1860mg (D) 930mg

36. The mass of oxalic acid crystals (H2C2O4 . 2H2O) required


3. Oxygen exhibits (–1) oxidation state in
to prepare 50 mL of a 0.2 N solution is :
(A) 4.5 g (B) 6.3 g [IJSO-Stage-I/2011-12]
(C) 0.63 g (D) 0.45 g (A) OF2 (B) H2O
(C) H2O2 (D) HCIO
37. 125 mL of 63% (w/v) H2C2O4 . 2H2O solution is made to
react with 125 mL of a 40%(w/v) NaOH solution. The
4. Oxidation number and co-ordinaiton number of Pt in
resulting solution is: (ignoring hydrolysis of ions)
cisplatin PtCl2 (NH 3)2 are respectively
(A) neutral (B) acidic
[IJSO-Stage-I/2013]
(C) strongly acidic (D) alkaline
(A) +4 and 2 (B) +2 and 4
38. If equal volumes of 0.1 M KMnO 4 and 0.1 M K 2Cr2O 7 (C) 0 and 4 (D) +2 and 6
solutions are allowed to oxidise Fe2+ to Fe3+ in acidic
medium, then Fe2+ oxidised will be : 5. Mayuri was performing thermometric titration and she
(A) more by KMnO4 took 100 ml of 1 M sulphuric acid and started adding 1M
(B) more by K2Cr2O7 calcium hydroxide and she plotted a graph of tempera-
(C) equal in both cases ture vs volume of the titrant added. In that experiment
(D) cannot be determined. she found that temperature was initially increasing and
then it started decreasing. The maximum of the graph is
39. An element A in a compound ABD has oxidation number obtained at 100 ml. Calcium hydroxide. What will be the
–n. It is oxidised by Cr 2O 72– in acid medium. In the enthalpy change of this reaction.[ Given . H = -13.7 kcal
experiment, 1.68 × 10–3 moles of K2Cr2O7 were used for for equivalent.]
3.36 × 10–3 moles of ABD. The new oxidation number of [IJSO-Stage-II/2014]
A after oxidation is :
(A) 3 (B) 3 – n
(C) n – 3 (D) +n

40. The volume strength of 1.5 N H2O2 solution is :


(A) 4.8 V (B) 8.4 V
(C) 3 V (D) 8 V

41. Find the volume strength of H2O2 solution prepared by


mixing of 250 mL of 3N H 2O 2 & 750 mL of 1N H 2O 2
solution :
(A) 1.5 V (B) 8.4 V (A) –13.7 kcal (B) – 27.4 kcal
(C) 5.6 V (D) 11.2 V (C) – 1.37 kcal (D) – 2.74 kcal

PAGE # 35
6. The mass percent of MnO2 in a sample of a mineral is
determined by reacting it with a measured excess of
As 2O 3 in acid solution, and then titrating the
remaining As2O3 with standard KMnO4. A 0.225 g sample
of the mineral is ground and boiled with 75.0 mL of 0.0125
M As2O 3 solution containing 10mL of concentrated sul-
furic acid. After the reaction is complete, the solution is
cooled, diluted with water, and titrated with 2.25 x10–3M
KMnO4, requiring 16.00ml to reach the endpoint.
Note: 5 mol of As2O3 react with 4 mol of MnO4–
i. Write a balanced equation for the reaction of As2O3 with
MnO2 in acid solution. The products are Mn2+ and AsO43–.
[IJSO-Stage-II/2014]
ii. Calculate the number of moles of
i. As2O3 added initially.
ii. MnO4– used to titrate the excess As2O3.
iii. MnO2 in the sample.
iii. Determine the mass percent of MnO2 in the sample.
iv. Describe how the endpoint is selected in the KMnO4
titration

PAGE # 36
STRUCTURE OF ATOM
mm of Hg, the cathode glow moves away from the
ELECTRON cathode, creating a dark space between cathode and
the cathode glow. This dark space is called Crookes
Electrons are the fundamental particles of all
dark space.
substances.
(iv) The Crookes dark space expands with further fall
(a) Cathode Rays - Discovery of Electron : in pressure at 0.1 mm of Hg. The positive column
The nature and existence of electron was established gets split into a number of bands called striations.
by experiments on conduction of electricity through (v) At pressure 0.01 mm of Hg or less, the striations
gases. move towards the anode and vanish finally. At this
stage the glass tubes begins to glow at the end
 Note : opposite to the cathode. This phenomenon is called
In 1859, Julius Plucker started the study of conduction fluorescence.
of electricity through gases at low pressure in a Thus, some sort of invisible rays travel from the
discharge tube. negative electrode to the positive electrode. Since the
A number of interesting things happen when a high negative electrode is called cathode, these rays were
voltage (say, 10,000 V) is applied across the called cathode rays. The colour of glow depends upon
the nature of the glass used. For soda glass the
electrodes of the discharge tube, and the pressure of
the gas inside the tube is lowered. fluorescence is of yellowish green colour.

(i) W hen the pressure of the gas in the discharge (b) Properties of cathode rays :
tube is at atmospheric pressure and a high voltage (i) Cathode rays travel in a straight line at a high
is applied across the electrodes, nothing noticeable velocity and generate normally from the surface of
happens. But as we lower the pressure and increase the cathode. If an opaque object is placed in the path
the voltage, sparking or irregular streaks of light are of cathode rays its shadow falls on opposite side of
seen in the tube. This is called positive column. the cathode. It shows that cathode rays travel in
(ii) As the pressure of gas is reduced further, the length straight lines.
of the positive column reduces, a fine glow can be
To vacuum
seen at the cathode. The dark space or gap left
pump Shadow
between the cathode and the positive column is
called the Faraday’s dark space.
Cathode

Object
High
+ Anode
voltage

 Note :
Cathode rays travel with very high velocities ranging
from 109 to 1011 cm per second.
(ii) They are a beam of minute material particles having
definite mass and velocity. When a light paddle wheel
is placed in the path of the cathode rays, the blades
of the paddle wheel begin to rotate. This also proves
that cathode rays have mechanical energy.

Light paddle wheel


– +

Cathode Anode

High voltage source


(iii) When the pressure of gas is reduced to about 1

PAGE # 37
(iii) They consist of negatively charged particles. When (iii) Charge on the electron :
the cathode rays pass through an electric field, they The charge (e) on an electron was determined by
bent towards the positive plate of the electric field. Robert Millikan in 1909. Millikan found the charge on
This indicates that cathode rays are negatively oil drops to be -1.6 × 10-19 C or its multiples. So, the
charged. charge on an electron is to be -1.6 × 10-19 coulombs / unit.
(iv) Mass of an electron :
By Thomson’s experiment e/m = 1.76 × 1011 C/kg
By Millikan’s experiment e = – 1.6 × 10-19 C

1.6  10 19
So mass of electron (m) =
1.76  1011
= 9.1096 × 10-31 kg
Mass of an electron in amu = 0.000549
(iv) Cathode rays can affect the photographic plate.
(v) Mass of electron in comparison to that of
(v) The nature of cathode rays is independent of the
hydrogen :
nature of gas used in discharge tube or material of
Mass of hydrogen = 1.008 amu
cathode.
= 1.008 × 1.66 × 10-24 g ( since 1 amu = 1.66 × 10-24 g )
(vi) Cathode rays are deflected in the magnetic field = 1.673 × 10-24 g
also.
24
Mass of hydrogen atom 1.6 73  10
S = = 1837
Mass of electron 9.1096  10 – 28
– +
 Note :

N Deflection of 1
cathode rays Thus, the mass of an electron is times the
1837
in magnetic field
High mass of a hydrogen atom.
voltage

PROTON
(vii) If cathode rays are focused on a thin metal foil,
the metal foil gets heated up to incandescence.
(a) Anode Rays (Canal rays) :
(viii) When cathode rays fall on materials having high
It has been established that electron is a negatively
atomic mass, new type of penetrating rays of very
small wavelength are emitted which are called X - charged particle and present in all the atoms. As an
rays. atom is electrically neutral, there must be some
positively charged particles present in the atom to
Thus, investigations on cathode rays showed that neutralize the negative charges of the electrons. It
these consisted of negatively charged particles.
has been confirmed by experiments. Scientist
 Note : Goldstein in 1886 discovered the existence of a new
The negatively charged particles of cathode rays were type of rays in the discharge tube. He carried out the
called ‘negatrons‘ by Thomson. The name negatron experiment in discharge tube containing perforated
was changed to ‘electron‘ by Stoney. cathode. It was observed that when high potential
difference was applied between the electrodes, not
(c) Characteristics of electron : only cathode rays were produced but also a new type
(i) Electrons are sub - atomic particles which of rays were produced simultaneously from anode,
constitute cathode rays. moving from anode towards cathode and passed
through the holes of cathode.
(ii) In 1897, J.J.Thomson determined the charge to
mass (e/m) ratio of electron by studying the deflections
of cathode rays in electric and magnetic fields. The Anode rays
value of e/m has been found to be 1.7588 × 10 8
Fluorescence
coulombs/g. The e/m for electrons from different
gases was found to be the same. This indicates that
atoms of all kinds have the same kind of negatively Anode Perforated cathode
charged particles. Thus electrons are the common
constituents of all atoms.
 Note :
A cathode ray tube is used to measure the charge to High voltage source
mass ratio of the electrons.

PAGE # 38
 Note : (iv) Mass of proton relative to mass of electron :
Anode rays are called canal rays because they pass 24
Mass of a proton 1.673 10 g = 1837
through the canals or holes of the cathode. These =  28
Mass of an electron
rays are also called anode rays since they originate 9.110 g
from the anode side. Anode rays are produced from a Thus, the mass of a proton is 1837 times larger than
positively charged electrode, therefore these were the mass of an electron.
named positive rays by J.J.Thomson. (v) Charge to mass ratio for a proton : The e/m of
particles constituting the anode rays is different for
(b) Characteristics of Anode Rays : different gases.

(i) Anode rays travel in straight lines. e 1.602  10 19


of proton = = 9.579 × 104 C/g
m 1.673  10 24
(ii) These rays rotate the light paddle wheel placed in
their path. This shows that anode rays are made up
of material particles. THOMSON MODEL OF AN ATOM

(iii) Anode rays are deflected by magnetic or electric J.J. Thomson (1898) tried to explain the structure of
field. In the electric field they get deflected towards atom. He proposed that an atom consists of a sphere
negatively charged plate. This indicates that these of positive electricity in which electrons are embedded
rays are positively charged. like plum in pudding or seeds evenly distributed in
red spongy mass in case of a watermelon. The radius
(iv) The anode rays affect photographic plate. of the sphere is of the order 10–8 cm.
(v) The nature of anode rays depend upon the type of
gas used.

(c) Discovery of Proton :


J.J.Thomson in 1906, found that particles obtained
in the discharge tube containing hydrogen have e/m
value as 9.579 × 10 4 coulomb/g. This was the (a) Merits :
maximum value of e/m observed for any positive (i) Thomson’s model could explain the electrical
particle. It was thus assumed that the positive neutrality of an atom.
particles given by hydrogen represent fundamental (ii) Thomson’s model could explain why only
particle of positive charge. This particle was named negatively charged particles are emitted when a metal
proton. is heated as he considered the positive charge to be
immovable by assuming it to be spread over the total
H e +
 H (Proton)
 volume of the atom.

 Note : (iii) He could explain the formation of ions and ionic


compounds.
The name ‘proton’ was given by Rutherford in 1911.
(b) Demerits :
(d) Characteristics of Proton : This model could not satisfy the facts proposed by
Rutherford through his alpha particle scattering
(i) A proton is a sub - atomic particle which constitute experiment and hence was discarded.
anode rays produced when hydrogen is taken in the
discharge tube. RUTHERFORD MODEL OF AN ATOM
(ii) Charge of a proton : (a) Rutherford’s Alpha Particle Scattering
Proton is a positively charged particle. The charge on Experiment (1909) :
a proton is equal but opposite to that on an electron.
Ernest Rutherford and his coworkers performed
Thus, the charge on a proton is +1.602 × 10 –19
numerous experiments in which - particles emitted
coulombs/ unit.
from a radioactive element such as polonium were
(iii) Mass of a proton : allowed to strike thin sheets of metals such as gold or
platinum.
The mass of a proton is equal to the mass of a
hydrogen atom. (i) A beam of -particles (He2+) was obtained by placing
polonium in a lead box and letting the alpha particles
m p = 1.0073 amu
come out of a pinhole in the lead box. This beam of -
= 1.673 × 10-24 g
rays was directed against a thin gold foil (0.0004 cm).
= 1.673 × 10-27 kg A circular screen coated with zinc sulphide was placed
on the other side of the foil.

PAGE # 39
(ii) About 99.0% of the -particles passed undeflected (iii) The number of electrons in the orbits is equal to
through the gold foil and caused illumination of zinc the number of positive charges (protons) in the
sulphide screen. nucleus.
(iii) Very few -particles underwent small and large (iv) Volume of nucleus is very small as compared to
deflections after passing through the gold foil. the volume of atom.
(iv) A very few (about 1 in 20,000) were deflected
backward on their path at an angle of 180º. (v) Most of the space in the atom is empty.

 Note :
Rutherford’s model is also called “Planetary model’.
(c) Defects in Rutherford’s Model :

(i) Rutherford did not specify the number of electrons


in each orbit.

(ii) According to electromagnetic theory, if a charged


particle (like electron) is accelerated around another
charged particle (like protons in nucleus) then there
Rutherford was able to explain these observations would be continuous loss of energy due to continuous
as follows: emission of radiations. This loss of energy would
slow down the speed of electron and eventually the
(i) Since a large number of -particles pass through
electron would fall into the nucleus. But such a
the atom undeflected, hence, there must be large
collapse does not occur. Rutherford’s model could
empty space within the atom.
not explain this theory.
(ii) As some of the -particles got deflected, therefore,
there must be something massive and positively (iii) If the electron loses energy continuously, the
charged structure present in the atom. observed spectrum should be continuous but the
actual observed spectrum consists of well defined
(iii) The number of -particles which get deflected is lines of definite frequencies. Hence the loss of energy
very small, therefore, the whole positive charge in the is not continuous in an atom.
atom is concentrated in a very small space.
(iv) Some of the -particles retracted their path i.e. BOHR MODEL OF AN ATOM (1913)
came almost straight back towards the sources as a
result of their direct collisions with the heavy mass. To overcome the objections to Rutherford’s model
and to explain the hydrogen spectrum, Bohr proposed
a quantum mechanical model of the atom.
The important postulates on which Bohr’s model is
based are the following -
(i) The atom has a nucleus where all the protons are
present. The size of the nucleus is very small. It is
present at the centre of the atom.
(ii) Each stationary orbit is associated with a definite
amount of energy. The greater is distance of the orbit
from the nucleus, more shall be the energy associated
 Note : with it. These orbits are also called energy levels and
 - particles are made up of two protons and two are numbered as 1, 2, 3, 4 ------or K, L, M, N ---- from
neutrons and are Helium (He) nuclei. nucleus to outwards.
(iii) By the time, the electron remains in any one of the
(b) Rutherford Nuclear Model of Atom (1911) :
allowed stationary orbits, it does not lose energy.
Rutherford proposed a new picture of the structure of Such a state is called ground or normal state.
atom.
(iv) The emission or absorption of energy in the form
Main features of this model are as follows- of radiation can only occur when an electron jumps
(i) The atom of an element consists of a small from one stationary orbit to another.
positively charged “Nucleus” which is situated at the E = Efinal - Einitial = h
centre of the atom and which carries almost the entire Where h is Planck’s constant (h = 6.625 × 10–34 Js)
mass of the atom. Energy is absorbed when the electron jumps from
(ii) The electrons are distributed in the empty space lower to higher orbit and is emitted when it moves
of the atom around the nucleus in different concentric from higher to lower orbit.
circular paths (orbits).

PAGE # 40
When the electron moves from inner to outer orbit by 1312
absorbing definite amount of energy, the new state of =– kJ/mol
n2
the electron is said to be excited state.
Substituting the values of n = 1,2,3,4, ---- etc. , the
(v) Negatively charged electrons revolves around the energy of electron in various energy shell in hydrogen
nucleus in circular path. The force of attraction atom can be calculated.
between the nucleus and the electron is equal to
E1
centrifugal force of the moving electron. En = (for hydrogen atom)
Force of attraction towards nucleus = Centrifugal force n2
(vi) Out of infinite number of possible circular orbits Z2
and En = E1 × (for hydrogen atom like species)
around the nucleus, the electron can revolve only in n2
those orbits whose angular momentum is an integral where E1 = energy of electron in first orbit of hydrogen.
h h Since, n can have only integral values, it follows that
multiple of , i.e. mvr = n total energy of the electron is quantised.
2 2
where :  Note :
m = mass of the electron The negative sign indicates that the electron is under
v = velocity of electron attraction towards nucleus, i.e. it is bound to the
r = radius of the orbit, and nucleus.
n =1,2,3 ---- number of the orbit.
The electron has minimum energy in the first orbit
The angular momentum can have values such as and its energy increases as n increases, i.e., it
h 2h 3h becomes less negative. The electron can have a
, , , but it cannot have a fractional value.
2 2 2 maximum energy value of zero when n =  . The zero
Thus, the angular momentum is quantized. The energy means that the electron is no longer bound to
specified circular orbits (quantized) are called the nucleus , i.e. , it is not under the force of attraction
stationary orbits. towards nucleus.

NEUTRONS

In 1932, James Chadwick bombarded the element


Let the total energy of the electron be E. It is the sum beryllium with  - particles. He observed the emission
of kinetic energy and potential energy. of a radiation with the following properties -
E = kinetic energy + potential energy (i) The radiation was highly penetrating.
(ii) The radiation remained unaffected in the electric
1  KZe2 
= mv2 +  –  or magnetic field i.e. the radiation was neutral.
2  r 
(iii) The particle constituting the radiation had the same
Putting the value of mv2 from eq. (i), mass as that of the proton. These neutral particles
were called neutrons.
KZe2 KZe2 KZe2
E – – 9 4
2r r 2r 4 Be + 2 He  12
6C + 1
0n
Putting the value of r from eq. (iii). (Beryllium ) (α  particle) (Neutron)
(Carbon)
kZe 2 42mKZe 2 2 2 Z 2K 2me 4
E  2 2
– ---- (iv)
2 n h n 2h2 COMPARATIVE STUDY OF ELECTRON,
For hydrogen atom, Z = 1 PROTON AND NEUTRON
So,
2 2k 2me 4 Property Electron Proton Neutron
E–
n 2h2
Putting the values of , k, m, e and h. Symbol e p n

2  (3.14) 2  (9  10 9 ) 2  (9.1 10 –31 )  (1.6  10 –19 ) 4 Nature Negatively charged Positively charged Neutral
E=–
n2  ( 6.625  10 – 34 )2
Relative
-1 +1 0
21.79  10 –19 charge
=– J per atom Absolute
n2 charge
–1.602 × 10-19 C +1.602 × 10-19 C 0
13.6
= – eV per atom (1 J = 6.2419 × 1018 eV) Relative 1
n2 mass 1837
1 1

313.6 Absolute
=– 2 kcal/mol (1 eV = 23.06 kcal/mol) 9.109 × 10-28 g 1.6725 × 10-24 g 1.6748 × 10-24 g
n mass

PAGE # 41
ATOMIC STRUCTURE ELECTRONIC CONFIGURATION OF AN ATOM

An atom consists of two parts - (i) The arrangement of the electrons in different shells
(a) Nucleus is known as the electronic configuration of the
(b) Extra - nuclear region element.
(a) Nucleus : (ii) Each of the orbits can accommodate a fixed
Nucleus is situated at the centre of an atom. All the number of electrons. Maximum number of electrons
protons & neutrons are situated in the nucleus, in an orbit is equal to 2n2, where ‘n’ is the number of
therefore, the entire mass of an atom is almost the orbit.
concentrated in the nucleus. The overall charge of (iii) Electrons are filled in the increasing order of
nucleus is positive due to the presence of positively energy, i.e. K < L < M < N ......
charged protons (neutrons have no charge). The
(iv) In the outermost shell of any atom, the maximum
protons & neutrons are collectively called nucleons.
possible number of electrons is 8, except in the first
 Note : shell which can have at the most 2 electrons.
The radius of the nucleus of an atom is of the order of  Note :
10–13 cm and its density is of the order of 1014 g/cm3. If the outermost shell has its full quota of 8 electrons
(b) Extra Nuclear Region : it is said to be an octet. If the first shell has its full
In extra nuclear part or in the region outside the quota of 2 electrons, it is said to be duplet.
nucleus, electrons are present which revolve around The pictorial representation of Bohr’s model of
the nucleus in orbits of fixed energies. These orbits hydrogen, helium, carbon, sodium and calcium atoms
are called energy levels. These energy levels are having 1, 2, 6, 11 and 20 electrons respectively are
designated as K, L, M, N & so on. shown in the figure where the centre of the circle
(i) The maximum number of electrons that can be represents the nucleus.
accommodated in a shell is given by the formula
2n2.(n = number of shells i.e. 1,2,3 -------)

Shell n 2n2 max. no.of electrons


2
K 1 2(1) 2
2
L 2 2(2) 8
2
M 3 2(3) 18
2
N 4 2(4) 32

ucle K L M N Electron shells


us
N

First energy level


Second energy level
+ 2 8 18 32 Maximum number of
Third energy level electrons which can be
Fourth energy level accommodated in the
various shells

(ii) Each energy level is further divided into subshells


designated as s,p,d,f .
1st shell (K) contains 1 subshell (s)
2nd shell (L) contains 2 subshells (s,p)
3rd shell (M) contains 3 subshells (s,p,d)
4th shell (N) contains 4 subshells (s,p,d,f).
(iii) Shells are divided into sub-shells, sub shells (a) Significance of Electronic Configuration :
further contain orbitals. The electronic configuration of an atom plays an
(A) An orbital may be defined as important role in determining the chemical behaviour
of an element.
“A region in the three - dimensional space around the
nucleus where the probability of finding the electron is (i) W hen the atoms of an element have completely
maximum.”
filled outermost shell, the element will be chemically
(B)The maximum capacity of each orbital is that of two unreactive. For example the noble gases (He, Ne, Ar,
electrons.
Kr, Xe and Rn) have completely filled outermost shell
 Note : i.e. contains 8 electrons (except helium which has
The maximum number of orbitals that can be present two valence electrons) in outermost shell.
in a shell is given by the formula n2.

PAGE # 42
(ii) W hen the atom of an element has less than 8  = 0 represents s sub shell.
electrons in its outermost shell, the element will be  = 1 represents p sub shell.
reactive.  = 2 represents d sub shell.
 = 3 represents f sub shell.
QUANTUM NUMBERS
 Note :
To describe the position and energy of electron in an s,p,d and f signify sharp, principal, diffused and
atom, four numbers are required, which are known fundamental respectively.
as quantum numbers.
(iv) Number of sub-shell in a shell = Principal quantum
Four quantum numbers are :
number of shell.
(a) Principal quantum number
(b) Azimuthal quantum number (v) Maximum value of  is always less than the value
(c) Magnetic quantum number of n. So 1p, 1f, 2d, 2f, 3f subshells are not possible.
(d) Spin quantum number s will start from 1 shell
p will start from 2 shell
(a) Principal Quantum Number : d will start from 3 shell
(i) It is denoted by ‘n’. f will start from 4 shell

(ii) It represents the name, size and energy of the (vi) Relative energy of various sub-shell in a shell are
as follows -
orbit or shell to which the electron belongs.
s<p<d<f
(iii) Higher is the value of ‘n’ , greater is the distance
of the shell from the nucleus. (vii) Subshells having equal values but with different n
r1 < r2 < r3 < r4 < r5 < ---- values have similar shapes but their sizes increases
as the value of ‘n’ increases. 2s-subshell is greater in
(iv) Higher is the value of ‘n’, greater is the magnitude size than 1s- subshell. Similarly 2p, 3p, 4p subshells
of energy. have similar shapes but their sizes increase in
E1 < E2 < E3 < E4 < E5 ---- order 2p < 3p < 4p.
(v) Maximum number of electrons in a shell is given (viii) Maximum no. of electrons present in a subshell
by 2n2. = 2 (2 +1)
Shell Max. number of electrons Subshell Max. electrons
First (n =1) 2 × 12 = 2 s ( = 0) 2 (2 × 0 +1) = 2
Second (n = 2) 2 × 22 = 8
p ( = 1) 2 (2 × 1 +1) = 6
Third ( n = 3) 2 × 32 = 18
d ( = 2) 2 (2 × 2 +1) = 10
Fourth ( n = 4) 2 × 42 = 32
f ( = 3) 2 (2 × 3 +1) = 14
(vi) Angular momentum can also be calculated using
 Note :
principal quantum number.
Azimuthal quantum number was given by
nh Sommerfeld.
mvr 

(c) Magnetic quantum number :
(vii) Value of n is from 1 to 
(i) It is denoted by ‘m’.
(viii) Every shell is given a specific alphabetic name.
(ii) It represents the orbitals present in sub-shell. An
First shell (n = 1) is known as K shell.
orbital can be defined as :
Second shell (n = 2) is known as L shell.
Third shell (n = 3) is known as M shell and so on. “Region in the three - dimensional space around the
nucleus where the probability of finding an electron is
 Note :
maximum”.
Principal quantum number was given by Bohr.
(iii) For a given value of , values of m are from –
(b) Azimuthal Quantum Number : through 0 to +.
(i) It is represented by ‘’.  m
0 0
 Note :
1 –1, 0, +1
Azimuthal quantum number is also called angular 2 –2, –1, 0, +1, +2
quantum number, subsidiary quantum number or 3 –3, –2, –1, 0, +1, +2, +3
secondary quantum number. (iv) Maximum number of orbitals in a sub-shell
(ii) For a given value of n values of  is 0 to n – 1 = (2+1)
Value of n Values of  Sub shell Orbitals
1 (1st shell) 0 s ( = 0) (2 × 0 +1) = 1
2 (2nd shell) 0,1 p ( = 1) (2 × 1 +1) = 3
3 (3rd shell) 0,1,2 d ( = 2) (2 × 2 +1) = 5
4 (4th shell) 0,1,2,3 f ( = 3) (2 × 3 +1) = 7
(iii) It represents the sub-shell present in shell.

PAGE # 43
(v) Maximum number of orbitals in a shell = n2 (vii) Characteristics of orbitals :
Shell Max. orbitals (A) All orbitals of a subshell possess same energy
First (n = 1) 12 = 1 i.e., they are degenerate.
Second (n = 2) 22 = 4
Third (n = 3) 32 = 9 (B) All orbitals of the same shell differ in the direction
Fourth (n = 4) 42 = 16 of their space orientation.

(vi) It represents the orientation of orbital in three (C) Total number of orbitals in a shell is equal to n2.
dimensional space.  Note :
When l = 0, m = 0, i.e. one value implies that ‘s’ Magnetic quantum number was given by Zeeman.
subshell has only one space orientation and hence,
it can be arranged in space only in one way along x,y (d) Spin Quantum Number :
or z axis. Thus, ‘s’ orbital has a symmetrical spherical (i) It is denoted by ‘s’.
shape.
Y (ii) It represents the direction of spin of electron around
Z its own axis.
X (iii) Clockwise spin is represented by +1/2 or  and
anticlockwise by –1/2 or .
s- orbital (iv) Maximum two electrons with opposite spin can
be placed in an orbital.
When  = 1,’m’ has three values –1, 0, +1 . It implies
that ‘p’ subshell of any energy shell has three space
orientations, i.e. three orbitals. Each p-orbital has
dumb-bell shape. Each one is disposed (v) Electrons with same spin are called spin parallel
symmetrically along one of the three axis. p orbitals and those with opposite spin are called spin paired.
have directional character.
 Note :
orbital Pz Px Py Spin quantum number was given by Gold Schmidt.
m 0 ±1 ±1
z z z ORDER OF FILLING OF ELECTRONS IN SUBSHELLS
There are different rules governing the filling of
subshells. They are described briefly as follows -
x x x
(a) Aufbau Principle :
y y y y
The filling of subshells in atoms is based on their
px py pz
energies. Electrons first occupy the subshell with
lowest energy and progressively fill the other
When  = 2 ‘m’ has five values –2, –1, 0, +1, +2. It
subshells in increasing order of energy.
implies that d-subshell of any energy shell has five
 Note :
orientations, i.e. five orbitals. All the five orbitals are
not identical in shape. Four of the d-orbitals The subshell with lowest energy is filled first.
The order of energy of different subshells of an atom is -
dxy, dyz, dzx, d contain four lobes while fifth orbital
x2 – y2 1s < 2s < 2p < 3s < 3p < 4s < 3d < 4p < 5s < 4d < 5p
dz2 consists of only two lobes. < 6s < 4f < 5d < 6p < 7s < 5f < 6d < 7p.
z z z The number present before the subshells like 1,2,3
------ represents the number of the shell i.e. n.
The order of filling of different sub-shells is
represented diagrammatically as follows :

x y x y x y 1s
dxy dxz dyz
z z 2s 2p

3s 3p 3d

x y x y 4s 4p 4d 4f
2
dx – y 2

dz2
5s 5p 5d 5f
There are seven f-orbitals designated as
f f f , fx ( x 2 – y 2 ), fy ( x 2 – y 2 ), fz(x2 –y2 ), and fxyz .
yz 2 , xz 2 , z3 6s 6p 6d
Their shapes are complicated ones.

7s 7p

PAGE # 44
(b) Pauli’s Exclusion Principle : (a) Significance of Valence Electrons :
According to Pauli’s exclusion principle “an orbital (i) The valence electrons of an atom are responsible
cannot accommodate more than two electrons. If there for, and take part in, chemical changes.
are two electrons in an orbital they must have opposite (ii) The valence electrons in an atom determine the
spins.” mode of chemical combination.

(c) Hund’s Rule of Maximum Multiplicity : (iii) The valence electrons determine the combining
capacity or the valency of the atom. The number of
According to this rule : electrons in an atom that actually take part in bond
“no electron pairing takes place in the orbitals with formation is known as the valency of the element.
equivalent energy until each orbital in the given e.g. In the carbon atom, there are four valence
subshell contains one electron & the spins of all electrons.
C-K L
unpaired electrons are parallel i.e. in the same 6
2 4
direction”.
The carbon atom is capable of forming four bonds.
Electronic configuration of some elements - Hence, the valency of carbon is four.
Atom ic Sym bol of Nam e of Electronic
num ber the elem ent the elem ent configuration (iv) If the outermost shell of an atom is completely
1 H Hydrogen 1s 1 filled, its valency is zero.
2 He Helium 1s 2 The outermost shells of helium, neon, argon, krypton
3 Li Lithium 1s 2, 2s 1 etc. are completely filled. Hence the valency of these
4 Be Beryllium 1s 2, 2s 2 elements is zero.
5 B Boron 1s 2, 2s 2 , 2p 1
6 C Carbon 1s 2, 2s 2 , 2p 2 (v) Elements having the same number of valence
7 N Nitrogen 1s 2, 2s 2 , 2p 3 electrons in their atoms possess similar chemical
8 O Oxygen 1s 2, 2s 2 , 2p 4 properties.
9 F Fluorine 1s 2, 2s 2 , 2p 5
e.g. All alkali metals have one valence electron in
10 Ne Neon 1s 2, 2s 2 , 2p 6
11 Na Sodium 1s 2, 2s 2 , 2p 6 ,3s 1 their atoms. Hence, their chemical properties are
12 Mg Magnes ium 1s 2, 2s 2 , 2p 6 ,3s 2 similar.
13 Al Alum inium 1s 2, 2s 2 , 2p 6 ,3s 2 ,3p 1
(vi) Elements having different number of valence
14 Si Silicon 1s 2, 2s 2 , 2p 6 ,3s 2 ,3p 2
2 2 6 2 3
electrons in their atoms show different chemical
15 P Phos phorus 1s , 2s , 2p ,3s ,3p
16 S Sulphur 1s 2, 2s 2 , 2p 6 ,3s 2 ,3p 4
properties.
17 Cl Chlorine 1s 2, 2s 2 , 2p 6 ,3s 2 ,3p 5 e.g. Let us consider the electronic configuration of
18 Ar Argon 1s 2, 2s 2 , 2p 6 ,3s 2 ,3p 6 alkali metals and halogens. Alkali metal atoms have
19 K Potas sium 1s 2, 2s 2 , 2p 6 ,3s 2 ,3p 6, , 4s 1
single valence electron whereas halogen atoms have
20 Ca Calcium 1s 2, 2s 2 , 2p 6 ,3s 2 ,3p 6, , 4s 2
seven valence electrons. It is observed that the
21 Sc Scandium 1s 2, 2s 2 , 2p 6 ,3s 2 ,3p 6, 3d 1, 4s 2
22 Ti Titanium 1s 2, 2s 2 , 2p 6 ,3s 2 ,3p 6, 3d 2, 4s 2
chemical properties of the alkali metals are entirely
23 V Vanadium 1s 2, 2s 2 , 2p 6 ,3s 2 ,3p 6, 3d 3, 4s 2 different from those of halogens.
24 Cr Chrom ium 1s 2, 2s 2 , 2p 6 ,3s 2 ,3p 6, 3d 5, 4s 1 (vii) The number of the valence shell in the atom of an
25 Mn Manganes e 1s 2, 2s 2 , 2p 6 ,3s 2 ,3p 6, 3d 5, 4s 2 element determines the period number of the
26 Fe Iron 1s 2, 2s 2 , 2p 6 ,3s 2 ,3p 6, 3d 6, 4s 2 element in the periodic table.
27 Co Cobalt 1s 2, 2s 2 , 2p 6 ,3s 2 ,3p 6, 3d 7, 4s 2 e.g. Sodium (Na) :
28 Ni Nickel 1s 2, 2s 2 , 2p 6 ,3s 2 ,3p 6, 3d 8, 4s 2 Valence shell number = 3.
29 Cu Copper 1s 2, 2s 2 , 2p 6 ,3s 2 ,3p 6, 3d 10, 4s 1
period number = 3
30 Zn Zinc 1s 2, 2s 2 , 2p 6 ,3s 2 ,3p 6, 3d 10, 4s 2
Calcium (Ca)
Valence shell number = 4
VALENCE SHELL AND VALENCE ELECTRONS period number = 4
(viii) Elements with 1, 2 or 3 valence electrons in their
The outermost shell of an atom is known as the atoms are metals.
valence shell. The electrons present in the valence
shell of an atom are known as valence electrons.  Note :
Hydrogen and helium are exceptions to this rule.
The remainder of the atom i.e. the nucleus and other
Hydrogen and helium atoms have 1 and 2 valence
electrons is called the core of the atom. Electrons
electrons respectively, but they are non-metals.
present in the core of an atom are known as core
electrons. (ix) Elements with 4, 5, 6, 7 or 8 valence electrons in
e.g. their atoms are non metals.
The electronic configuration of the sodium (Na) atom e.g. carbon (C), nitrogen (N) and oxygen (O) are non
is :- metals.
Na (11) K L M 6
C = 2,4
2 8 1 N = 2,5
7
Thus, valence electron in Na atom = 1 and core 8
O = 2,6
electrons in Na atom = 2 + 8 = 10

PAGE # 45
 Note : (a) Characteristics of Isotopes :
Whether the atom while combining with other atoms
(i) The physical properties of the isotopes of an
can form ionic or covalent bonds is determined by
element are different. This is due to the fact that
the number of valence electrons present in the atom.
isotopes have different numbers of neutrons in their
nuclei. Hence mass, density and other physical
VALENCY
properties of the isotopes of an element are different.
Valency of an element is the combining capacity of
(ii) All the isotopes of an element contain the same
the atoms of the element with atoms of the same or
number of electrons. So, they have the same
different elements. The combining capacity of the
electronic configuration with the same number of
atoms was explained in terms of their tendency to
valence electrons. Since the chemical properties of
attain a fully filled outermost shell (stable octet or an element are determined by the number of valence
duplet) electrons in its atom, all the isotopes of an element
 Note : have identical chemical properties.
The number of electrons gained, lost or contributed
(b) Reason for the Fractional Atomic Masses
for sharing by an atom of the element gives us directly
the combining capacity or valency of the element. of Elements :

Valency of an element is determined by the number The atomic masses of many elements are in fraction
of valence electrons in an atom of the element. and not whole number. The fractional atomic masses
The valency of an element = number of valence of elements are due to the existence of their isotopes
electrons (if the number of valence electrons is1 to 4) having different masses.
The valency of an element = 8– number of valence e.g.
electrons. (if the number of valence electrons is 5 to 8) The atomic mass of chlorine is 35.5 u. Chlorine has
35 37
two isotopes 17 Cl and 17 Cl with abundance of 75%
ISOTOPES
and 25% respectively. Thus the average mass of a
The isotopes of an element have the same atomic chlorine atom will be 75% of 35 and 25% of 37, which
number but different atomic masses. is 35.5 u.
So, Average atomic mass of chlorine
 Note :
75 25
The term isotope was given by Margaret Todd. = 35 × + 37 ×
100 100
The difference in their masses is due to the presence 2625 925
= +
of different number of neutrons. 100 100
e.g. Isotopes of hydrogen : = 26.25 + 9.25
= 35.5 u.
Thus, the average atomic mass of chlorine is 35.5 u.
Hydrogen Pr ot iu m De u t e r iu m Tr itium
isotopes 1 2 3 (c) Applications of Radioactive Isotopes :
1H 1H 1H
(i) In agriculture : Certain elements such as boron,
1. Atomic number 1 1 1 cobalt, copper, manganese, zinc and molybdenum
2. No. of protons 1 1 1 are necessary in very minute quantities for plant
nutrition. By radioactive isotopes we can identify the
3. No. of electrons 1 1 1
presence and requirement of these element in the
4. Mass number 1 2 3 nutrition of plants.
5. No. of neutrons 0 1 2
(ii) In industry : Isotopes are used for coating on the
arm of clock to see in dark. To identify the cracks in
Isotopes of oxygen : metal casting.
(iii) In medicine :Thyroid, bone diseases, brain
16 17 18 tumors and cancer are controlled or destroyed with
8O 8O 8O
Oxygen isotopes
24 131 32
the help of radioactive isotope 11 Na, 53 Ι, 15P etc.
1. Atomic number 8 8 8
2. No. of protons 8 8 8 (iv) Determination of the mechanism of chemical
reaction by replacing an atom or molecule by its
3. No. of electrons 8 8 8
isotopes.
4. Mass number 16 17 18
5. No. of neutrons 8 9 10
(v) In carbon dating : Will and Libby (1960) developed
the technique of radiocarbon to determine the age of
plant, fossil and archeological sample.
 Note :  Note :
All the isotopes of an element have identical chemical Isotopes (Like Uranium-238) are used in Nuclear
properties. reactor to produce energy and power.

PAGE # 46
4. The magnitude of charge on an electron in coulombs
ISOBARS is -
The atoms of different elements with different atomic (A) 1.76×1011 (B) 1.6 × 10–19
–31
numbers, which have same mass number are called (C) 9.1 × 10 (D) none of these
isobars.
5. The mass of a proton -
e.g. 146 C and 147 N are isobars. (A) 1.0073 amu (B) 1.673 × 10–24 g
40
(C) 1.673 × 10–27 kg (D) all of these
18 Ar and 40
20 Ca are isobars.
6. W hich of the following has the same number of
Isobars 40 40
18 Ar 20 Ca protons, electrons & neutrons -
1. Atomic number 18 20 (A) 54
27 X (B) 55
27 X
1

2. Mass number 40 40
3. No. of electrons 18 20 (C) 54
26 X (D) 55
28 X

4. No. of protons 18 20
5. No. of neutrons 22 20 7. An element having atomic number 25 and atomic
6. Electronic configuration 2, 8, 8 2, 8, 8, 2 weight 55 will have –
(A) 25 protons and 30 neutrons
 Note (B) 25 neutrons and 30 protons
Isobars contain different number of electrons, (C) 55 protons
protons and neutrons. (D) 55 neutrons

8. Which of the following is isoelectronic with N2O :


ISOTONES
(A) NO (B) N2O5
Isotones may be defined as the atoms of different (C) CO2 (D) CO
elements containing same number of neutrons.
9. The fraction of volume occupied by the nucleus with
e.g. 136 C and 147 N (Both contain 7 neutrons) respect to the total volume of an atom is
(A) 10–15 (B) 10–5
30 31
14 Si , 15 P and 32
16 S (All three contain 16 neutrons) (C) 10–30 (D) 10–10

10. Charge on a positron is equal to that of :


ISOELECTRONIC (A) Proton (B) Electron
(C) Nucleon (D) Neutron
Ion or atom or molecule which have the same number
of electrons are called as isoelectronic species. 11. The e/m is not constant for :
e.g.   2 (A) Cathode rays (B) Positive rays
17Cl 18 Ar 19 K 20 Ca (C) -rays (D) -rays
No. of electrons 18 18 18 18
12. Atomic weight of an element is not a whole number
because :
(A) It contains electrons, protons & neutrons
EXERCISE-1 (B) It contains allotropic forms
(C) It contains isotopes
(D) None of the above
I. Electron proton & nuclear, models
13. Which one of the following pairs represents isobars
1. The numerical value of Planck’s constant is (A) 2He3 and 2He4 (B) 12Mg24 and 12Mg25
(A) 6.6 × 10–20 J sec (B) 6.6 × 10–34 J sec 40
(C) 19K and 19K 39
(D) 19K40 and 18Ar40
–25
(C) 6.6 × 10 J sec (D) 6.6 × 10–50 J sec
II. Atomic model & Energy
2. The ratio of e/m for a cathode ray -
(A) varies with a gas in a discharge tube. 14. Atoms may be regarded as comprising of protons,
(B) is fixed. neutrons and electrons. If the mass attributed by
(C) varies with different electrodes. electrons was doubled and that attributed by neutrons
(D) is maximum if hydrogen is taken. was halved, the atomic mass of 12C would be -
(A) approximately the same.
3. The highest value of e/m of anode rays has been (B) doubled.
(C) reduced approximately 25%.
observed, when the discharge tube is filled with -
(D) approximately halved.
(A) nitrogen (B) oxygen
(C) hydrogen (D) helium

PAGE # 47
15. The ionization energy of He+ is 19.6 × 10–18 J atom–1.
The energy of the first stationary state of Li+2 will be: (C)
(A) 84.2 × 10–18 J/atom
(B) 44.10 × 10–18 J/atom (D)
(C) 63.2 × 10–18 J/atom
(D) 21.2 × 10–18 J/atom
23. The outermost electronic configuration of the most
16. Energy required to pull out an electron from 1st orbit
electronegative element is -
of hydrogen atom to infinity is 100 units. The amount
(A) ns2 np2 (B) ns2 np4
of energy needed to pull out the electron from 2nd
(C) ns2 np5 (D) ns2 np6
orbit to infinity is :
(A) 50 units (B) 100 units
(C) 25 units (D) Zero 24. The electronic configuration of an element with atomic
number 29 is -
17. S1 : Potential energy of the two opposite charge system
(A) 1s2, 2s2, 2p6, 3s2, 3p6, 3d9, 4s2
increases with the decrease in distance.
(B) 1s2, 2s2, 2p6, 3s2, 3p6, 3d10, 4s1
S2 : W hen an electron make transition from higher
orbit to lower orbit it's kinetic energy increases. (C) 1s2, 2s2, 2p6, 3s2, 3p6, 3d8, 4s2,4p1
S3 : When an electron make transtition from lower (D) 1s2, 2s2, 2p6, 3s2, 3p6, 3d7, 4s2,4p2
energy to higher energy state its potential energy
increases. 25. The electronic configuration of Mn+2 is -
S4 : 11eV photon can free an electron from the 1st (A) [Ar] 3d4 4s1 (B) [Ar] 3d5 4s0
excited state of He+ -ion. 5
(C) [Kr] 4d 5s 0
(D) [Ar] 3d3 4s2
(A) T T T T (B) F T T F
(C) T F F T (D) F F F F 26. Electronic configuration of Si in ground state is -
(A) 1s2, 2s2 2p6,3s2 3p4
18. Bohr's model can explain :
(B) 1s2, 2s2 2p6, 3s2 3p3
(A) The spectrum of hydrogen atom only
(C) 1s2, 2s2 2p6, 3s2 3p2
(B) The spectrum of atom or ion containing one
(D) 1s2, 2s2 2p6, 3s2, 4s2
electron only
(C) The spectrum of hydrogen molecule only 27. The number of d-electrons in Fe+2 (Z = 26) is not equal
(D) The solar spectrum to that of the -
III. Atomic structure, Electric configuration, Filling (A) p- electron in Ne (Z = 10)
electron subshell (B) s-electron in Mg (Z = 12)
(C) d-electron in Fe (Z = 26)
19. Hund’s rule states that - (D) p-electron in Cl– (Z = 17)
(A) matter and radiation have a dual nature.
(B) in the orbitals of equivalent energy, pairing begins 28. When 3d orbital is completely filled, the next electron
only after all the orbitals have been singly will enter the -
occupied. (A) 3p- orbital (B) 4p - orbital
(C) the position and velocity of an electron cannot be (C) 4s - orbital (D) 4d - orbital
exactly measured at the same time.
(D) no two electrons in an atom can have the same 29. How many unpaired electrons are there in Ni2+ (Z = 28 ) ?
set of four quantum numbers. (A) 0 (B) 8
(C) 2 (D) 4
20. The electronic configuration of element of atomic 30. If the nitrogen atom had electronic configuration 1s7,
number 24 is - it would have energy lower than that of the normal
(A) 1s2,2s2 2p6,3s2 3p6 3d5,4s1 ground state configuration 1s2, 2s2 2p3, because the
(B) 1s2,2s2 2p6,3s2 3p6 3d6 electrons would be closer to the nucleus. Yet 1s7 is
(C) 1s2,2s2 2p6,3s1 3p3 3d10 not observed because it violates -
(A) Heisenberg uncertainty principle
(D) 1s2,2s2 2p6,3s2 3p6 3d4,4s2
(B) Hund Rule
(C) Pauli exclusion principle
21. W hich of the following has maximum unpaired d -
(D) Bohr postulate of stationary orbits
electrons ?
(A) Zn2+ (B) Fe2+ 31. In which of the following orbital diagrams are both
(C) Ni2+
(D) Cu+ Pauli’s exclusion principle and Hund’s rule violated ?

(A)
22. The orbital diagram in which the Hund’s rule is
violated is -
(B)
(A)
(C)

(B) (D)

PAGE # 48
IV. Isotope, Isobar, Isoelectronic, Quantum number 43. The p-orbitals have -
(A) dumb-bell shape
32. The set of quantum numbers not applicable to an
(B) spherical shape
electron in an atoms is -
(C) double dumb bell shape
(A) n = 1, = 1, m = 1, s = 1/2
(D) complex structure
(B) n = 1, = 0, m = 0, s = 1/2
(C) n = 1, = 0, m = 0, s = –1/2 44. Isotone of an element has -
(A) same number of electron
(D) n = 2, = 1, m = 1, s = 1/2
(B) same number of protons
(C) same number of neutrons
33. The correct set of quantum numbers (n, , m) for the
(D) same number of neutrons & protons
unpaired electron of chlorine atom is
(A) 2,1,0 (B) 2,1,1, 45. An isotone of 76
32 Ge is -
(C) 3,1,1 (D) 3,0,1
(A) 77
32 Ge (B) 77
33 As
34. The maximum no. of electrons in a subshell with
= 2 and n = 3 is -
(C) 77
34 Se (D) 79
34 Se
(A) 2 (B) 6
(C) 12 (D) 10 46. The triad of nuclei that is isotonic is -

35. Among the following groups which represents the (A) 14 15 17


6 C, 7 N, 9 F (B) 12 14 19
6 C, 7 N, 9 F
collection of isoelectronic species ?
(C) 14 14 17
6 C, 7 N, 9 F (D) 14 14 19
6 C, 7 N,9 F
(A) NO+ , C 22 – , O2–, CO (B) N2 , C 22 – , CO, NO
47. Pick out the isoelectronic structures from the following -
(C) CO , NO+, CN–, C 22 – (D) NO, CN–, N2, O2–  CH3 + H3O+
 NH3 V CH3–
36. For which of the following elements, outermost orbit’s (A)  and  (B)  and V
last electron has azimuthal quantum number  = 0 ? (C)  and  (D) , , V
(A) Na (B) O 48. The atomic orbital is -
(C) Cl (D) N (A) the circular path of the electron.
(B) elliptical shaped orbit.
37. x8 X atom is isotonic to 179 Y atom. The value of x is - (C) three dimensional field around nucleus.
(D) the region in which there is maximum probability
(A) 8 (B) 16
of finding an electron.
(C) 9 (D) 17
49. Any p-orbital can accommodate up to -
38. The electrons, identified by quantum numbers n and  (A) 4 electrons
(i) n = 4,  = 1 (ii) n = 4,  = 0 (B) 2 electrons with parallel spins
(iii) n = 3,  = 2, (iv) n = 3 ,  = 1 (C) 6 electrons
can be placed in order of increasing energy from the (D) 2 electrons with opposite spins.
lowest to highest as -
50. The total number of orbitals in a principal shell are -
(A) (iv) < (ii) < (iii) < (i)
(A) n (B) n2
(B) (ii) < (iv) < (i) < (iii) 2
(C) 2n (D) 3n2
(C) (i) < (iii) < (ii) < (iv)
(D) (iii) < (i) < (iv) < (ii) 51. The set of principal (n). azimuthal (l) and magnetic
quantum number not possible for electron in H-
39. If (n + ) = 4 , then maximum possible sub-shells will
be - atom is
(A) 1 (B) 2 (A) n = 3, l = 1, m  = -1
(C) 3 (D) 4 (B) n = 3, l = 0, m  = 0
(C) n = 2, l = 1, m = 0
40. If (n + ) = 4, then maximum possible orbitals will be -
(D) n = 2, l = 2, m  = -1
(A) 2 (B) 3
(C) 4 (D) 5
52. Which of the following set of quantum numbers are
41. If maximum value of  = 2, then the number of permitted
maximum possible electrons in the orbit will be - (A) n = 3, l = 2, m = – 2, s = +1/2
(A) 2 (B) 8 (B) n = 3, l = 2, m = – 1, s =– 1/2
(C) 18 (D) 32 (C) n = 2, l = 2, m = +1, s = 0
(D) n = 2, l = 1, m = +1, s = – 1/2
42. The maximum number of electrons in N shell is -
(A) 2 (B) 8
(C) 18 (D) 32

PAGE # 49
53. A given orbital is labeled by the magnetic quantum 2. Find the number of quanta of radiations of frequency
number m = –1. This could not be 7.55×1015 s–1 that must be absorbed in order to melt
(A) s - orbital (B) p-orbital
6 g of ice. The approximate energy required to melt
(C) d-orbital (D) f-orbital
1g of ice is 333 J. (h=6.62×10–34Js)
54. Which of the following represents the correct set of [IJSO-Stage-II/2014]
quantum numbers of a 4d electron ? (A) 0.90 × 10 22
(B) 0.111 × 1020
22
1 (C) 5.38×10 (D) 3.99 × 1020
(A) 4, 3, 2, + (B) 4, 2, 1, 0
2
3. How much energy in kJ is produced when 7.1 g of
1 1 gaseous chlorine atoms are converted to gaseous
(C) 4, 3, – 2, + (D) 4, 2, 1, –
2 2 chloride ions ? (Electron affinity of chlorine is -3.7eV.
55. A f-orbital can accommodate (1eV/atm = 96.49 kJ/moles) [IJSO-Stage-II/2014]
(A) 14 electrons with opposite spins
(B) 14 electrons with parallel spins
(A) 35.7 kJ (B) 26.2 kJ
(C) 12 electrons with parallel spins
(D) 12 electrons with opposite spins (C) 68.5 kJ (D) 71.4 kJ

56. An orbital containing electron having quantum number 4. All of these species have the same number of
valence electrons as nitrate ion, except
1 [IJSO-Stage-I/2016-17]
n = 4, l = 3, m = 0 and s = – is called
2 (A) Carbonate ion (B) Bicarbonate ion
(A) 3s orbital (B) 3p orbital (C) NF3 (D) SO3
(C) 4d orbital (D) 4f orbital

57. The maximum number of electrons in a subshell is


given by the expression
(A) 4l – 2 (B) 4l + 2
(C) 2l + 2 (D) 2n2

58. The maximum number of 3d-electrons having spin


quantum number s = +1/2 are -
(A) 10 (B) 14
(C) 5 (D) None of these

59. In which (n + ) rules not applicable -


(A) Cu, Cr (B) Cu, Zn
(C) Ag, Zn (D) All of these

60. The angular momentum of 3d orbital is :

3h h
(A) (B)
2 2

h 2h
(C) (D)
3 3

EXERCISE-2
1. The last electron of the element of atomic number 31
will have the following quantum numbers -
[IJSO-Stage-I/2012]
n  m s
(A) 3 0 0 –1/2
(B) 3 1 1 +1/2
(C) 4 1 –1 –1/2
(D) 4 0 0 +1/2

PAGE # 50
PERIODIC TABLE & PERIODICITY IN PROPERTIES

DEFINITION Atomic mass of sodium =


Atomic mass of lithium  Atomic mass of potassium
The Periodic Table is arguably the most important
2
concept in chemistry, both in principle and in practice.
It is the everyday support for students, it suggests 7  39
= = 23
new avenues of research to professionals, and it 2
provides a succinct organization of the whole of Some examples of triads are given in the table :
chemistry. It is a remarkable demonstration of the
fact that the chemical elements are not a random
cluster of entities but instead display trends and lie
together in families. An awareness of the Periodic
Table is essential to anyone who wishes to
disentangle the world and see how it is built up from
the fundamental building blocks of the chemistry, the
chemical elements

Definition: (ii) Limitations of Dobereiner’s Classification :


A periodic table may be defined as the table giving
(A) Atomic mass of the three elements of some triads
the arrangement of all the known elements according
to their properties so that elements with similar are almost same.
properties fall within the same vertical column and e.g. Fe, Co, Ni
elements with dissimilar properties are separated. (B) It was restricted to few elements, therefore
discarded.

(c) Newlands’ Law of Octaves :


EARLY ATTEMPTS TO CLASSIFY ELEMENTS
In 1866, an English chemist, John Newlands,
(a) Metals and Non-Metals :
proposed a new system of grouping elements with
Among the earlier classifications, Lavoisier classified similar properties. He tried to correlate the properties
the elements as metals and non-metals. However, of elements with their atomic masses. He arranged
this classification proved to be inadequate. In 1803, the known elements in the order of increasing atomic
John Dalton published a table of relative atomic masses. He started with the element having the
weights (now called atomic masses). This formed lowest atomic mass (hydrogen) and ended at thorium
an important basis of classification of elements. which was the 56th element. He observed that every
eighth element had properties similar to that of the
(b ) Dobere iner ’s T riad s: first.

(i) In 1817, J.W. Dobereiner a German Chemist gave Newlands called this relation as a law of octaves
this arrangement of elements. due to the similarity with the musical scale.

(A) He arranged elements with similar properties in (i) Newlands’ arrangement of elements into ‘Octaves’:
the groups of three called triads.
Notes of
Music sa (do) re (re) ga (mi) ma (fa) pa (so) dha (la) ni (ti)
(B) According to Dobereiner the atomic mass of the
H Li Be B C N O
central element was merely the arithmetic mean of
F Na Mg Al Si P S
atomic masses of the other two elements.
Elements Cl K Ca Cr Ti Mn Fe
e.g. Co and Ni Cu Zn Y In As Se
Br Rb Sr Ce and La Zr – –
Elements of
the triad Symbol Atomic mass (ii) Limitations of law of octaves : The law of octaves
Lithium Li 7 has the following limitations :

Sodium Na 23 (A) The law of octaves was found to be applicable


only upto calcium. It was not applicable to elements
Potassium K 39
of higher atomic masses.

PAGE # 51
(B) Position of hydrogen along with fluorine and
(e) Mendeleev’s Periodic Table :
chlorine was not justified on the basis of chemical
properties. The major credit for a systematic classification of
elements goes to Mendeleev. He tried to group the
(C) Newlands placed two elements in the same slot elements on the basis of some fundamental property
to fit elements in the table. He also placed some of the atoms. When Mendeleev started his work, only
unlike elements under the same slot. For example, 63 elements were known. He examined the
cobalt and nickel are placed in the same slot and in relationship between atomic masses of the elements
the column of fluorine, chlorine and bromine. But and their physical and chemical properties.
cobalt and nickel have properties quite different from Among chemical properties, Mendeleev concentrated
fluorine, chlorine and bromine. Similarly, iron which mainly on the compound formed by elements with
has resemblances with cobalt and nickel in its oxygen and hydrogen. He selected these two
elements because these are very reactive and formed
properties has been placed far away from these
compound with most of the elements known at that
elements.
time. The formulae of the compounds formed with
Thus, it was realized that Newlands’ law of octaves these elements (i.e. oxides and hydrides) were
worked well only with lighter elements. Therefore, this regarded as one of the basic properties of an element
classification was rejected. for its classification.

(i) Mendeleev’s periodic law : This law states that


(d) Lother Meyer’s Classification :
the physical and chemical properties of the elements
In 1869, Lother Meyer studied the physical properties
are the periodic function of their atomic masses.
like volume, melting point, boiling point etc. of different
elements. This means that when the elements are arranged in
He plotted a graph between atomic masses against the order of their increasing atomic masses, the
their respective atomic volumes for a number of elements with similar properties recur at regular
elements. He found the following observations - intervals. Such orderly recurring properties in a cyclic
(i) Elements with similar properties occupied
fashion are said to be occurring periodically. This is
similar positions on the curve.
responsible for the name periodic law or periodic
(ii) Alkali metals (Li, Na, K, Rb, Cs ) having larger atomic table.
volumes occupied the crests .
(ii) Merits of Mendeleev’s periodic table : Mendeleev’s
(iii) Transition elements (V, Fe, Co, Cu etc. ) occupied
periodic table was one of the greatest achievements in
the troughs.
the development of chemistry. Some of the important
(iv) The halogens (F, Cl, Br, ) occupied the ascending contributions of his periodic table are given below :
portions of the curve before the inert gases.
(A) Systematic study of elements : He arranged
(v) Alkaline earth metals (Mg, Ca, Sr, Ba ) occupied known elements in order of their increasing atomic
positions at about the mid points of descending masses considering the fact that elements with
portions of the curve.
similar properties should fall in the same vertical
column.

70 Cs (B) Correction of atomic masses : The Mendeleev’s



60 periodic table could predict errors in the atomic
Atomic Volume (cm per mole of atoms)

Rb
50
• masses of elements based on their positions in the
• table. Therefore atomic masses of certain elements
K
40 • • • Sr • Ba were corrected. For example, atomic mass of
Li
30 • Na
• Br • I• beryllium was corrected from 13.5 to 9. Similarly, with
3

• Cl • •
Ca
20 •
F • • the help of this table, atomic masses of indium, gold,
• •Be •• • •• VFe Co ••••Zn • • ••
Mg

••• •••• ••••Cu •• • platinum etc. were corrected.


10 •
••• ••• •••• •••
(C) Mendeleev predicted the properties of those
0
20 40 60 80 100 120 140 missing elements from the known properties of the
Atomic mass other elements in the same group. Eka-boron, eka -
aluminium and eka -silicon names were given for
Change of Atomic Volume with Atomic Mass.
scandium , gallium and germanium (not discovered
Drawback of Lother Meyer’s classification : This
at the time of Mendeleev ).
was a hypothetical classification and it was difficult to
remember the positions of different elements. (D) Position of noble gases : Noble gases like helium
(He), neon (Ne) and argon (Ar) were mentioned in
many studies. However, these gases were discovered

PAGE # 52
very late because they are very inert and are present (C) Anomalous pairs of elements : In certain pairs
in extremely low concentrations. One of the of elements, the increasing order of atomic masses
achievements of Mendeleev’s periodic table was that was not obeyed. In these, Mendeleev placed
when these gases were discovered, they could be elements according to similarities in their properties
and not in increasing order of their atomic masses.
placed in a new group without disturbing the existing
order.  For example :
• The atomic mass of argon is 39.9 and that of
(iii) Limitations of Mendeleev’s periodic table : potassium 39.1. But argon is placed before
Inspite of many advantages, the Mendeleev’s periodic potassium in the periodic table.
table has certain defects also. Some of these are • The positions of cobalt and nickel are not in proper
given below - order. Cobalt (at. mass = 58.9) is placed before nickel
( at. mass = 58.7).
(A) Position of hydrogen : Position of hydrogen in
the periodic table is uncertain. It has been placed in • Tellurium (at. mass = 127.6) is placed before
1A group with alkali metals, but certain properties of iodine (at. mass = 126.9).
hydrogen resemble those of halogens. So, it may be (D) Some similar elements are separated, in the
placed in the group of halogens as well. periodic table. For example copper (Cu) and mercury
(B) Position of isotopes : Isotopes are the atoms of (Hg). On the other hand, some dissimilar elements
the same element having different atomic masses. have been placed together in the same group.
Therefore, according to Mendeleev’s classification e.g. Copper (Cu), silver (Ag) and gold (Au) have been
these should be placed at different places depending placed in group 1 along with alkali metals. Similarly,
upon their atomic masses. For example, hydrogen
manganese (Mn) is placed in the group of halogens.
isotopes with atomic masses 1, 2 and 3 should be
placed at three places. However, isotopes have not (E) Cause of periodicity : Mendeleev could not
been given separate places in the periodic table explain the cause of periodicity among the elements.
because of their similar properties.

Groups I II III IV V VI VII VIII

Oxide : R2O RO R2O3 RO2 R2O5 RO3 R2O7


RO4
Hydride : RH RH2 RH3 RH4 RH3 RH2 RH

Periods Transition
A B A B A B A B A B A B A B
 series
H
1
1.008
Li Be B C N O F
2
6.939 9.012 10.81 12.011 14.007 15.999 18.998
Na Mg Al Si P S Cl
3
22.99 24.31 26.98 28.09 30.974 32.06 35.453
K Ca Sc Ti V Cr Mn Fe Co Ni
4 First 39.102 40.08 44.96 47.90 50.94 52.20 54.94 55.85 58.93 58.71
series Cu Zn As Se Br
Second 63.54 65.37 ––– ––– 74.92 78.96 79.909
series

Rb Sr ––– Zr Nb Mo Tc Ru Rh Pd
5 First 85.47 87.62 91.22 92.22 95.94 99 101.07 102.91 106.4
series Ag Cd In Sn Sb Te I
Second 107.87 112.40 114.82 118.69 121.75 127.60 126.90
series

Cs Ba La Hf Ta W Os Ir Pt
6 First 132.90 137.34 138.91 178.49 180.95 183.85 190.2 192.2 195.2
series Au Hg TI Pb Bi
Second 196.97 200.59 204.37 207.19 208.98
series

PAGE # 53
of uranium).
MODERN PERIODIC TABLE
(c) Periodicity :
(a) Introduction : The repetition of elements with similar properties after
W e must bear in mind that when Mendeleev certain regular intervals, when the elements are
developed his Periodic Table, chemists knew nothing arranged in order of increasing atomic number, is
about the internal structure of atom. However, the called periodicity.
beginning of the 20th century witnessed profound
(d) Cause of Periodicity :
developments in theories about sub-atomic particles.
The periodic repetition of the properties of the
(b) Modern Periodic Law (Moseley's Periodic Law) : elements is due to the recurrence of similar valence
Moseley did an experiment in which he bombarded shell (outermost shell) electronic configuration after
high speed electrons ondifferent Metal surfaces certain regular intervals.
and obtained X-rays(electromagnetic rays).
e.g. Alkali metals have similar electronic configuration
He observed regularities in the characteristic X-ray
(ns1) and therefore, have similar properties.
spectra of the elements and found that plot  vs. Z
Alkali Metals
(atomic number) is straight line while  vs. A (atomic
weight) is not, and Atomic Electronic
 = a (Z - b), where a and b are Element Symbol
configuration
number
constants that are same for all elements and is
frequency of X-rays. Thus he concluded that atomic 3 Lithium Li 2,1
number is more fundamental property than atomic
weight. 11 Sodium Na 2,8,1

19 Potassium K 2,8,8,1

37 Rubidium Rb 2,8,18,8,1

55 Caesium Cs 2,8,18,18,8,1

87 Francium Fr 2,8,18,32,18,8,1

(e) Long Form of Periodic Table :


(i) The long form of periodic table is based upon
Modern periodic law. Long form of periodic table is
the contribution of Range, Werner, Bohr and Bury.
(ii) This table is also referred to as Bohr’s table since
it follows Bohr’s scheme of the arrangement of
elements into four types based on electronic
configuration of elements.
(iii) Long form of periodic table consists of horizontal
rows (periods) and vertical columns (groups).
(f) Description of Periods :
(i) A horizontal row of periodic table is called a period.
(ii) There are seven periods numbered as 1, 2 , 3 , 4,
5, 6 and 7.
(iii ) Each period consists of a series of elements
The physical and chemical properties of the having the same outermost shell.
numbers. (iv) Each period starts with an alkali metal having
The Periodic Law revealed important analogies outermost shell electronic configuration ns1.
among the 94 naturally occurring elements (v) Each period ends with a noble gas with outermost
(neptunium and plutonium like actinium and shell electronic configuration ns2 np 6 except helium
protoactinium are also found in pitch blende - an ore having outermost electronic configuration 1s2.
(vi) Each period starts with the filling of a new energy
level.

PAGE # 54
(A) 1st period : This period is called very short period (v) The elements present in groups 1,2,13 to 17 are
because this period contains only 2 elements H and called normal or representative elements.
He. (vi) Elements of group 1 and 2 are called alkali metals
(B) 2nd and 3rd periods : These periods are called and alkaline earth metals respectively.
short periods because these periods contain 8
elements. 2nd period starts from 3Li to 10Ne and 3rd (vii) Elements present in group 17 are called
period starts from 11Na to 18Ar. halogens.

(C) 4th and 5 th periods : These periods are called  Note : Elements present in a period have different
long periods because these periods contain 18 properties, while elements present in a group have
elements. 4 th period starts from 19K to 36Kr and 5 th similar properties.
period starts from 37Rb to 54Xe.
 Note : Modern periodic table is based on atomic
(D) 6th period : This period is called very long period.
This period contains 32 elements. Out of the 32 number, not on atomic mass.
elements 14 elements belong to Lanthanoid series
(h) Merits of Modern Periodic Table :
(58Ce to 71Lu). 6th period starts from 55Cs to 86Rn.
(E) 7 th period : This period is called as very long (i) Anomalous pairs : The original periodic law based
period.. It contains 32 elements. Out of the 32 on atomic masses is violated in case of four pairs of
elements 14 elements belong to Actinoid series elements in order to give them positions on the basis
(90Th to 103Lr). of properties. The elements having higher atomic
 Note : Modern periodic table consists of seven periods masses have been assigned position before the
and eighteen groups. elements having lower atomic masses at four places
as shown below -
No. of
Periods Called as
Elements
st
Very short period (a) (b) (c) (d)
(1 ) n = 1 2
nd
(2 ) n = 2 8 Short period
rd
(3 ) n = 3 8 Short period Ar K Co Ni Te I Th Pa
(4th) n = 4 18 Long period
At. Mass 40 39 60 59 128 127 232 231
(5 th) n = 5 18 Long period
th
(6 ) n = 6 32 Very long period At. No. 18 19 27 28 52 53 90 91
th
(7 ) n = 7 25 Very long period
The discrepancy disappears, if the elements are
Different elements belonging to a particular period arranged in order of increasing atomic numbers.
have different electronic configurations and have (ii) Position of isotopes : Isotopes are atoms of the
same element having different atomic masses, but
different number of valence electrons. That is why
same atomic number. All the isotopes of an element
elements belonging to a particular period have
will be given different positions, if atomic mass is
different properties. taken as a basis. This shall disturb the symmetry of
(g) Description of Groups : the table. In modern table, one position is fixed for
one atomic number and since all the isotopes of an
(i) A vertical column of elements in the periodic table element have the same atomic number, these are
is called a group. assigned only one position.
(ii) There are eighteen groups numbered as 1, 2 , 3, (iii) Elements with similar properties were placed
4, 5, ------------ 13, 14, 15, 16,17,18. together and elements with dissimilar properties were
separated in modern periodic table.
(iii) A group consists of a series of elements having
similar valence shell electronic configuration and (iv) Cause of periodicity : Modern periodic table
explains the cause of periodicity among the elements.
hence exhibit similar properties.
(i) Demerits of Modern Periodic Table :
e.g. Li, Na, K belong to the same group and have 1
electron in their valence shell. Following are the demerits of modern periodic table -

(iv) The group 18 is also known as zero group because (i) Position of hydrogen : Position of hydrogen was
the valency of the elements of this group is zero. uncertain in the periodic table.

 Note : The elements of 18th or zero group are called (ii) Position of lanthanides and actinides : The
noble gases. positions of lanthanides and actinides were also
uncertain in the periodic table.

PAGE # 55
 Characteristics :
CLASSIFICATION OF THE ELEMENTS
(i) Elements of group 3 to 12 constitute the d - block.
It is based on the type of subshells which receives
(ii) General electronic configuration is (n – 1)d1–10 ns0- 2.
the differentiating electron (i.e. last electron).
(iii) d - block elements lie in the centre of the periodic
(a) s- Block Elements : table.
W hen last electron enters the s- orbital of the
(iv) All the d - block elements are metals and most of
outermost (n th) shell, the elements of this class are them form coloured complexes or ions.
called s- block elements.
Characteristics :  Note : The total number of elements in d-block is 40
.
(i) Group 1 & 2 elements constitute the s - block.
(ii) General electronic configuration is ns1–2 . (d) f - block elements :
When last electron enters into f - orbital of (n – 2)th
(iii) s - block elements lie on the extreme left of the
shell, elements of this class are called f - block
periodic table.
elements.
(iv) This block includes metals only, except H.
 Characteristics :
 Note : The total number of elements in s-block is 13
(i) All f - block elements belong to 3rd group.
(including hydrogen).
(ii) General electronic configuration is (n – 2)f 1 – 14
(b) p-Block Elements : (n– 1)d0-1 ns2.
(iii) f-block elements are present in two separate
When differentiating electron enters the p - orbital of
rows below the periodic table.
the nth orbit, elements of this class are called p - block
elements. (iv) All f-block elements are metals only.
The elements of f- block have been classified into
 Characteristics :
two series :
(i) Elements of group 13 to 18 constitute the p - block.
(A) Lanthanides : 14 elements present after element
2
(ii) General electronic configuration is ns np 1-6
. lanthanum ( 57La) are called lanthanides.1 st inner
transition series of metals or 4f - series, contains 14
(iii) p - block elements lie on the extreme right of the
elements i.e. 58Ce to 71Lu .
periodic table.
(B) Actinides : 14 elements present after element
(iv) This block includes some metals, all non-metals actinium (89Ac) are called actinides. 2nd inner transition
and metalloids. series of metals or 5f- series, also contains 14 elements
 Note : The total number of elements in p-block is 31. i.e. 90Th to 103Lr.
(c) d - Block Elements:  Note : The total number of elements in f-block is 28.
Division of the periodic table into various blocks :
When differentiating electron enters the (n–1)d orbital,
then elements of this class are called d - block
elements.

Different Types of Elements :


(i) Noble gases : The elements belonging to group 18 are called noble gases or aerogenous. They are also
known as inert gases because their outermost orbitals are completely filled. Helium (He) is an exception. It has only
two electrons which are present in s-orbital.

PAGE # 56
(ii) Representative elements : Elements in which atoms have all shells complete except outermost shell which is
incomplete. Except 18th group, all s - block and p - block elements are collectively called normal or representative
elements.

(iii) Transition elements : Those elements which have partially filled d - orbitals in neutral state or in any stable
oxidation state are called transition elements.

 Note : Zn, Cd and Hg are d-block elements , but not transition elements, because they do not contain partially filled
d-orbitals.
(iv) Inner transition elements :- They contain three incomplete outermost shell and were also referred to as rare
earth elements, since their oxides were rare in earlier days.
(v) Diagonal relationship : Some elements of 2nd and 3rd period show diagonal relationship among them. They
represent the same properties of two periods. This relation is known as diagonal relationship.

(vi) Transuranium elements : The elements with atomic number greater than 92 (Z > 92) are known as transuranium
elements.

PAGE # 57
PAGE # 58
(B) The formation of covalent bond involves
PERIODICITY IN PROPERTIES
overlapping of atomic orbitals, as a result of this, the
(a) Effective nuclear charge : internuclear distance between the covalently bonded
Between the outer most valence electrons and the
atoms is less than the internuclear distance between
nucleus of an atom, there exists number of shells
containing electrons. Due to the presence of these the non bonded atoms.
intervening electrons, the valence electrons are
e.g. Vander Waals radius of helium is 1.20 Å
unable to experience the attractive pull of the actual
number of protons in the nucleus. These intervening (iii) Metallic radius (Crystal radius) : Metallic radius
electrons act as shield between the valence
may be defined as half of the internuclear distance
electrons and protons in the nucleus. Thus, the
between two adjacent atoms in a metallic lattice.
presence of intervening (shielding) electrons reduces
the electrostatic attraction between the protons in the X X
nucleus and the valence electrons because
intervening electrons repel the valence electrons. The C D
concept of effective nuclear charge helps in
understanding the effects of shielding on periodic
properties.
The effective nuclear charge (Zeff) is the charge 1
actually felt by the valence electron Zeff is given by
CD = rcrystal
2
the formula Zeff = Z -, (where Z is the actual nuclear (of element X)
charge (atomic number of the element) and is the • The metallic radius of an atom is always larger
shielding (screening constant). The value of   i.e. than its covalent radius.
shielding effect can be determined using the Slater's
rule.  Note : The order of different radius is -

(b) Atomic Radius : r Vander Waals > rMetallic > rCovalent

(i) Covalent radius : It may be defined as one - half of


the distance between the centres of the nuclei of two (iv) Variation of atomic radii in a period : As we move
similar atoms bonded by a single covalent bond.
from left to right across a period, there is a regular
X X
decrease in atomic radii of the representative
A B elements. This is due to the fact that number of energy
shells remains the same in a period, but nuclear
charge increases gradually as the atomic number
1 increases. This increases the force of attraction
2 AB = rcovalent
(of element X) towards nucleus which brings contraction in size.
e.g. The internuclear distance between two hydrogen This can also be explained on the basis of effective
atoms in H 2 molecule is 74 pm. Therefore, the nuclear charge which increases gradually in a period
covalent radius of hydrogen atom is 37 pm.
i.e. electron cloud is attracted more strongly towards
 Note : Covalent radius is generally used for nucleus as the effective nuclear charge becomes
non - metals. more and more as we move in a period. The increased
(ii) Vander Waal’s radius : It may be defined as half force of attraction brings contraction in size.
of the internuclear distance between two adjacent (v) Variation of atomic radii in a group : Atomic radii
atoms of the same element belonging to two nearest
in a group increase as the atomic number increases.
neighbouring molecules of the same substance.
The increase in size is due to extra energy shells
X X X X which outweigh the effect of increased nuclear charge.
The following table illustrates the periodicity in atomic
E H
radii (covalent radii) of representative elements. The
radii given in the table are in angstrom (Å).
Atoms of zero group elements do not form chemical
1 EH = r
2 vander Waals bonds among themselves. Hence for them Vander
 Characteristics : W aals radii are considered.

(A) Covalent radius of the elements is shorter than


its Vander Waal’s radius.

PAGE # 59
Atomic radii Corresponding
Element He Ne Ar Kr Xe Atom (crystal, Å) cations Ionic radii (Å)
+
Li 1.52 Li 0.59
Vander Waals Na 1.86 Na
+
0.99
1.20 1.60 1.91 2.00 2.20
radii (in Aº ) K 2.31 K
+
1.33
2+
The sudden increase in atomic radii in comparison Mg 1.60 Mg 0.65
2+
to the halogens (the elements of 7th group) in case Ba 2.22 Ba 1.35
3+
of inert gases, is due to the fact that, Vander Waals Al 1.43 Al 0.50
2+
Pb 1.75 Pb 1.32
radii are considered which always possess higher
values than covalent radii. Conclusions
The decrease in the size of transition elements is
(i) The radius of cation (positive ion) is always
small since the differentiating electrons enter into smaller than that of the parent atom.
inner 'd' levels. The additional electrons into (n-1)d
(ii) The radius of anion (negative ion) is always larger
levels effectively screen much of increased nuclear than that of the parent atom.
charge on the outer ns electrons and therefore, size
(iii) The ionic radii in a particular group increases
remains almost constant. on moving from top to bottom.
Conclusions (iv) In a set of species having the same number of
(i) The alkali metals which are present at the extreme electrons (isoelectronic), the size decreases as the
charge on the nucleus increases.
left of the periodic table have the largest size in a
period. ( except nobel gases) (v) The size of the cations of the same element
decreases with the increase of positive charge.
(ii) The halogens which are present at the extreme
right of the periodic table have the smallest size. (d) Ionisation Energy (IE) :
(iii) The size of the atoms of inert gases are, however, Ionisation Energy (IE) of an element is defined as the
larger than those of preceding halogens because in amount of energy required to remove an electron from
inert gases van der W aals' radii are taken into an isolated gaseous atom of that element resulting
in the formation of a positive ion.
consideration.
(iv) In a group of transition elements, there is an Characteristics :

increase in size from first member to second member (A) The energy required to remove the outermost
electron from an atom is called first ionisation energy
as expected but from second member to third
(IE)1.
member, there is very small change in size and
After removal of one electron, the atom changes into
sometimes sizes are same. This is due to Lanthanide monovalent positive ion.
contraction. M(g) + IE1  M+ (g) + e–

(c) Ionic radius (B) The minimum amount of energy required to


It is the distance between the nucleus and outermost remove an electron from monovalent positive ion of
shell of an ion or it is the distance between the nucleus the element is known as second ionisation energy
(IE)2.
and the point where the nucleus exerts its influence
M+(g) + IE2  M2+(g) + e–
on the electron cloud.
(C) The first, second etc. ionisation energies are
(a) The radius of the cation is always smaller than
collectively known as successive ionisation energies.
the atomic radius of its parent atom. This is due to
M2+(g) + IE3  M3+(g) + e–
the fact that nuclear charge in the case of a cation is
acting on a lesser number of electrons and pulls them In general (IE)1 < (IE)2 < (IE)3 so on, because, as the
closer. number of electrons decreases, the attraction
between the nucleus and the remaining electrons
(b) The radius of the anion is always larger than the increases considerably and hence subsequent
atomic radius of its parent atom. In an anion as ionisation energies increase.
electron or electrons are added to the neutral atom,
the nuclear charge acts on more electrons so that (D) Units : Ionisation energy is expressed either in
each electron is held less tightly and thereby the terms of electron volts per atom (eV/atom) or Kilojoules
electron cloud expands. per mole of atoms (KJ mol – 1) or K cal mol – 1.
1 eV/atom = 96.49 KJ/mol = 23.06 Kcal/mol = 1.602 × 10–19
Comparative sizes of atoms and their cations
J/atom

PAGE # 60
Factors influencing ionisation energy :
1s 2 , 2s 2
(A) Size of the atom : Ionisation energy decreases with   
Be (Z = 4) Completely filled
increase in atomic size. As the distance between the orbital (more stable )
outermost electrons and the nucleus increases, the force
of attraction between the valence shell electrons and the
1s 2 , 2s 2 , 2 p1
nucleus decreases. As a result, outermost electrons are  
B (Z = 5) Partially filled
held less firmly and lesser amount of energy is required
orbital (less stable )
to knock them out.
For example, ionisation energy decreases in a group
from top to bottom with increase in atomic size. Because of penetration effect of s > p

(B) Nuclear charge : The ionisation energy Variation of ionisation energy in a period :
increases with increase in the nuclear charge. This
In general, the value of ionisation energy increases
is due to the fact that with increase in the nuclear
with increase in atomic number across a period. This
charge, the electrons of the outermost shell are more
firmly held by the nucleus and thus greater amount of can be explained on the basis of the fact that on
energy is required to pull out an electron from the atom. moving across the period from left to right-
For example, ionisation energy increases as we move (A) nuclear charge increases regularly.
from left to right along a period due to increase in nuclear
charge. (B) addition of electrons occurs in the same shell.
(C) Shielding effect : The electrons in the inner shells (C) atomic size decreases.
act as a screen or shield between the nucleus and
the electrons in the outermost shell. This is called Variation of ionisation energy in a group :
shielding effect or screening effect. Larger the number
In general, the value of ionisation energy decreases
of electrons in the inner shells, greater is the
screening effect and smaller the force of attraction while moving from top to bottom in a group.This is
and thus ionisation energy decreases. because -
(A) effective nuclear charge decreases regularly.

(B) addition of electrons occurs in a new shell.


(C) atomic size increases.
These electrons This electron does
shield the outer not feel the full inward pull Conclusions :
electrons from the of the positive charge of
nucleus the nucleus (i) In each period, alkali metals show lowest first
(D) Penetration effect of the electrons : The ionisation enthalpy. Caesium has the minimum value.
ionisation energy increases as the penetration effect (ii) In each period, noble gases show highest first
of the electrons increases. It is a well known fact that ionisation enthalpy. Helium has the maximum value
the electrons of the s-orbital have the maximum of first jonisation enthalpy.
probability of being found near the nucleus and this
probability goes on decreasing in case of p, d and f (iii) The representative elements show a large range
orbitals of the same energy level. of values of first ionisation enthalpies, metals having
low while non-metals have high values.
Greater the penetration effect of electrons more firmly
the electrons will be held by the nucleus and thus (iv) Generally. ionisation enthalpies of transition
higher will be the ionisation energy of the atom. metals increase slowly as we move from left to right
For example, ionisation energy of aluminium is in a period. The f-block elements also show only a
comparatively less than magnesium as outermost small variation in the values of first ionisation
electron is to be removed from p-orbital (having less enthalpies.
penetration effect) in aluminium, whereas in
magnesium it will be removed from s-orbital (having (e) Electron Affinity (EA) :
larger penetration effect) of the same energy level. Electron affinity is defined as the energy released in
 Note : With in the same energy level,the penetration the process of adding an electron to a neutral atom in
effect decreases in the order s > p > d > f the gaseous state to form a negative ion.
X(g) + e–  X–(g) + Energy (E.A.)
(E) Electronic Configuration : If an atom has exactly
Cl(g) + e–  Cl– (g) + 349 KJ/mol
half-filled or completely filled orbitals, then such an
The electron affinity of chlorine is 349 KJ/mol.
arrangement has extra stability.The removal of an
electron from such an atom requires more energy The addition of second electron to an anion is
than expected. For example, opposed by electrostatic repulsion and hence the
E1 of Be > E1 of B energy has to be supplied for the addition of second
electron.
PAGE # 61
(C) Be, Mg, N and P also have exceptionally low
O(g) + e–  O– (g) + Energy (EA -)
values of electron affinities due to their stable
O–(g) + e–  O2– (g) – Energy (EA-) electronic configurations.

(EA -) is exothermic whereas, (EA-) is endothermic. Be = 1s2, 2s2 N = 1s2, 2s2, 2p3
2 2 6 2
(i) Units : Kilo joules per mole (KJ/mol) of atoms or Mg = 1s , 2s , 2p , 3s P = 1s2, 2s2, 2p6, 3s2, 3p3
electron volts per atom (eV/atom).
 Conclusion
(ii) Factors affecting electron affinity: (i) The electron gain enthalpies, in general, become
(A) Nuclear charge : Greater the magnitude of less negative in going down from top to bottom in a
group. This is due to increase in size on moving down
nuclear charge greater will be the attraction for the
a group. This factor is predominant in comparison to
incoming electron and as a result, larger will be the
value of electron affinity. other factor, i.e., increase in nuclear charge.

Electron affinity  Nuclear charge.


Na K Rb Cs
–1
(B) Atomic size : Larger the size of an atom is, more –53 –48 –47 –46 KJ mol
will be the distance between the nucleus and the Cl Br I At
–1
incoming electron and smaller will be the value of –349 –325 –295 – 270KJ mol
electron affinity.
(ii) The electron gain enthalpies of oxygen and fluorine,
 1 
  the members of the second period, have less
E.A.  Atomic size 
  negative values than the elements sulphur and
chlorine of the third period. This is due to small size
(C) Electronic configuration : Stable the electronic
of the atoms of oxygen and fluorine. As a result, there
configuration of an atom lesser will be its tendency to
is a strong inter-electronic repulsion when extra
accept the electron and lower will be the value of its
electron is added to these atoms, i.e., electron density
electron affinity.
is high and the addition of electron is not easy. Thus,
(iii) Variation of electron affinity in a period : On the electron gain enthalpies of third period elements,
moving across the period the atomic size decreases sulphur and chlorine, have more negative values than
and nuclear charge increases. Both these factors corresponding elements oxygen and fluorine.
–1 –1
result into greater attraction for the incoming electron, O –141 kJ mol F –328 kJ mol
therefore electron affinity in general increases in a –1 –1
S –200 kJ mol Cl –349 kJ mol
period from left to right.
Similar trend is observed in nitrogen & phosphorous
(iv) Variation of electron affinity in a group : On
N +31 kJ mol–1
moving down a group, the atomic size as well as
P – 74 kJ mol–1
nuclear charge increase, but the effect of increase in
atomic size is much more pronounced than that of (iii) In general, electron gain enthalpy becomes more
nuclear charge and thus, the incoming electron feels and more negative from left to right in a period. This
less attraction consequently, electron affinity is due to decrease in size and increase in nuclear
decreases on going down the group. charge as the atomic number increases in a period.
Both these factors favour the addition of an extra
(v) Some irregularities observed in general trend:
electron due to higher force of attraction by the nucleus
(A) Halogens have the highest electron affinities in for the incoming electron.
their respective periods. This is due to the small size
and high effective nuclear charge of halogens. (iv) Electron gain enthalpies of some of the members
Halogens have seven electrons in their valence shell. of alkaline earth metals, noble gases and nitrogen
By accepting one more electron they can attain stable are positive.
electronic configuration of the nearest noble gas. Thus
This is because they have stable configurations.
they have maximum tendency to accept an additional
Alkaline earth metals have stable configurations due
electron.
to completely filled ns orbital while nitrogen has extra
(B) Due to stable electronic configuration of noble stability due to half filled p-orbitals (1s2, 2s2, 2p3) i.e.,
gases electron affinities are zero. symmetrical configuration. These atoms resist the
addition of extra electron as they do not want to disturb
their stability.
PAGE # 62
Noble gases have ns2np6 configuration, i.e., no place
Reducing nature
for incoming electron. In case the extra electron is to
be accommodated, it will occupy its position on a Reducing Decreases
nature ;
new principal shell, i.e., it will be weakly attracted
towards nucleus. Such anion will be extremely Increases
2
unstable. Helium has also stable 1s configuration
and cannot accommodate the incoming electron. Oxidising nature
(v) Halogens have highest negative electron gain
Oxidising Increases
enthalpies. nature
Following two factors are responsible for this: Decreases
(a) Small atomic size and high nuclear charge of (iii) Stability of metal increases while activity of the
halogens in a period. metal decreases in a period and in a group stability
decreases.
(b) Halogens have the general electronic
Stability of the metal
configuration of ns2 np 5, i.e., one electron less than
stable noble gas (ns2 np 6) configuration. Stability of the metal Increases
Thus, halogens have very strong tendency to accept
an additional electron and their electron gain Decreases
enthalpies are, therefore, high.
; Activity of the metal
• Importance of Electron Gain Enthalpy : Certain
properties of the elements can be predicted on the Activity of the metal Decreases
basis of values of electron gain enthalpies.
(i) The elements having high negative values of Increases
electron gain enthalpy are capable of accepting (f) Electronegativity :
electron easily. They form anions and thus form ionic
(electrovalent) compounds. These elements are Electronegativity is a measure of the tendency of an
electronegative in nature. element to attract electrons towards itself in a
(ii) The elements having high negative electron gain covalently bonded molecule .
enthalpies act as strong oxidising agents, for example,
F, CI, Br, O, S, etc. (i) Factors influencing electronegativity :
On the basis of the general trend of ionisation
(A) The magnitude of electronegativity of an element
enthalpy and electron gain enthalpy, the following
properties can be predicted: depends upon its ionisation potential & electron
(i) Metallic nature decreases in a period while non- affinity. Higher ionisation potential & electron affinity
metallic nature increases. Metallic nature increases values indicate higher electronegativity value.
in a group while non-metallic nature decreases. The
arrow () represents a group and () represents a (B) W ith increase in atomic size the distance
period. between nucleus and valence shell electrons
Metallic increases, therefore the force of attraction between
the nucleus and the valence shell electrons
Metallic Decreases decreases and hence the electronegativity values
(Electro + ve)
also decrease.
Increases (C) In higher oxidation state, the element has higher
Non-metallic magnitude of positive charge. Thus, due to more
positive charge on element, it has higher polarising
Non-metallic (Electro-ve) Increases power. Thus, with increase in the oxidation state of
Decreases element, its electronegativity also increases.
(ii) Reducing nature decreases in a period while  Note :
oxidising nature increases. The reducing nature
increases in a group while oxidising nature Polarising power is the power of an ion (cation) to
decreases.
distort the other ion.

PAGE # 63
(D) With increase in nuclear charge, force of attraction In a group, the strength of the oxy-acids of non-metals
between nucleus and the valence shell electrons decreases.
increases and, therefore electronegativity value V group strength decreases
increases.
VII group strength decreases

(ii) Variation of Electronegativity in a group : On If a non-metal forms a number of oxy-acids, the


moving down the group atomic number increases, strength increases with the increase of percentage
of oxygen.
so nuclear charge also increases. Number of shells
also increases, so atomic radius increases. Sulphur forms two oxy-acids H2SO3 and H2SO4.H2SO4
Therefore electronegativity decreases on moving is stronger acid than H2SO3.
down the group. Chlorine forms a number of oxy-acids:

(iii) Variation of Electronegativity in a period : While


moving across a period left to right atomic number,
nuclear charge increases & atomic radius decreases.
Therefore electronegativity increases along a period.

(g) Nature of Oxides :


Greater is the oxidation state of central atom more
In a period, the nature of the oxides varies from basic will be the acidic strength.
to acidic.
Na2O MgO AI2O3 SiO2 P2O5 Cl2O7 (i ) Nature of Hydrides :
Strongly Basic Amphoteric Weakly Strongly
The nature of the hydrides changes from basic to
basic acidic acidic
acidic in a period from left to right.
In a group, basic nature increases or acidic nature NH 3 H2O HF
decreases. Oxides of the metals are generally basic weak base netural weak acid
and oxides of the nonnmetals are acidic. The oxides
of the metalloids are amphoteric. The oxides of AI, PH 3 H 2S HCl
Zn, Sn, As and Sb are amphoteric. We can summarise very weak weak acid strong acid
that as the electronegativity of element increases, base
acidic character of oxides increases. W hen an
In a group, the acidic nature of the hydrides of non-
element forms a number of oxides, the acidic nature
metals increases. The reducing nature also increases
increases as the percentage of oxygen increases.
but stability decreases from top to bottom
Acidic nature

N2 O Neutral P2 O3 MnO Basic


increases

Acidic nature

NO oxide P2 O4 Mn2O3 Basic


increases

P2 O5 MnO2 Neutral
Acidic nature

N2O3 MnO3 Acidic


increases

NO2 Mn2O7 Acidic


N2O5
(h) Lattice Enthalpy :
CO, N2O, NO and H2O are neutral oxides. The oxides
The lattice enthalpy of an ionic solid is defined as the
CO2, N2O5, P2O3, P2O5, SO2, SO3, Cl2O7, etc., are called
energy required to completely separate one mole of
acid anhydrides as these combine with water to form
a solid ionic compound into gaseous constituent ions.
oxy-acids.
CO2  H2CO3 P2O3  H3PO3 The process involves both the attractive forces
SO3  H2SO4 N2O5  HNO3 between ions of opposite charges and the repulsive
P2O5  H3PO4 Cl2O7  HClO4 forces between ions of like charge. The solid crystal
N2O3  HNO2 SO2  H2SO3 being three dimensional; it is not possible to calculate
lattice enthalpy directly from the interaction of forces
(h) Nature of Oxy-Acids : of attraction and repulsion only. Factors associated
with the crystal geometry have to be included.
In a period, the strength of the oxy-acids formed by
non-metals increases from ieft to right. Factors affecting lattice energy of an ionic compound :

II Period strength increases


1
(i) Lattice energy  where (r+ + r- ) = Inter-ionic
r  r
III Period strength
Distance.
increases (ii) Lattice energy  Z+, Z-

PAGE # 64
Z+ ==> charge on cation in terms electronic charge. A secondary layer of water molecules further hydrates
the ions, though these are only held by week ion-
Z- ==> charge on anion in terms electronic charge.
dipole attractive forces. The strength of such forces
(iii) Lattice energy  coordination number (C.N.)
is inversely proportional to the distance, that is to the
Coordination number of cation is number of anions size of the metal ion. Thus the secondary hydration
surrounding the cation.
decrease from Lithium to caesium and accounts for
Coordination number of anion is number of cations
Li+ being the most heavily hydrated.
surrounding the anion.
• Anomalous behaviour of the elements of second
Coordinations number factor is important only in case
period :
of sulphates and carbonates of alkaline earth metals. It has been obesrved that in the case of representative
(i) Hydration : elements, the first element in each, i.e. lithium in the
All the simple salts dissolve in water, producing ions, first group, beryllium in the second group and boron
and consequently the solution conduct electricity. to fluorine in the group 13 to 17, differ in many respect
Since Li+ is very small, it is heavily hydrated. This from the other member of their respective group. The
anomalous behaviour of the first member of each
makes radius of hydrated Li+ ion large and hence it
group is attributed to following reasons :
moves only slowly. In contrast, Cs + is the least
(a) small atomic radius of the atom and ionic radius
hydrated because of its bigger size and thus the
of its ion.
radius of the Cs+ ion is smaller then the radius of
hydrated Li+, and hence hydrated Cs+ moves faster, (b) high electronegativity
and conducts electricity more readily. (c) non-availability of d-orbitals in their valence shell.
(d) tendency to form multiple bonds by carbon,
nitrogen and oxygen
Ionic Approx. (e) high charge/radius ratio
Ionic
mobility radius of
Ion radius
at infinite hydrated
(Å) EXERCISE-1
dilution ion (Å)
Li+ 0.76 33.5 3.4
+ I. Classification of elementary
Na 1.02 43.5 2.76
+
K 1.38 64.5 2.32 1. In Lother Meyer’s curve most electronegative
+
Rb 1.52 67 2.28 elements or halogens occupy -
+
Cs 1.67 68 2.28 (A) peaks
(B) ascending positions
(C) descending positions
(D) halogens were not shown
Some water molecules touch the metal ion and bond
to it, forming a complex. These water molecules 2. In Lother Meyer's plot, the peaks were occupied by -
constitute the primary shell of water. (A) alkali metals
Thus Li+ is tetrahedrally surrounded by four water (B) alkaline earth metals
molecules forming coordinate covalent bond between (C) halogens
metal ion and four water molecules using a lone pair of (D) noble gases
electrons on each oxygen atom.
3. The plot given by Lother Meyer, for then known
elements, was between their atomic volume and -
(A) atomic number (B) atomic mass
(C) density (D) ionisation energy

4. (X), (Y) and (Z) are elements in the third period. Oxide
of (X) is ionic, that of (Y) is amphoteric and of (Z) a giant
molecule. (X), (Y) and (Z) have atomic number in the
order :
(A) (X) < (Y) < (Z) (B) (Z) < (Y) < (X)
(C) (X) < (Z) < (Y) (D) (Y) < (X) < (Z)

II. Modern periodic table, long form

5. Metals are included in the long form of periodic table


in -
(A) s-block only (B) p-block only
(C) s & p blocks both (D) s,p,d and f blocks

PAGE # 65
6. One important merit of modern periodic table is - (A) 1s2, 2s2 2p6, 3s2 3p6 3d5, 4s2
(A) it explains why element in the same group have (B) 1s2, 2s2 2p6, 3s2 3p6 3d10, 4s2 4p5
the same chemical properties. (C) 1s2, 2s2 2p6, 3s2 3p6 3d6, 4s1
(B) hydrogen has been placed accurately. (D) 1s2, 2s2 2p6, 3s2 3p6 3d10, 4s2 4p6
(C) isobars have not been placed separately.
18. The number of periods in the long form of the periodic
(D) it is based on classifying elements according to
table is -
their atomic masses.
(A) 6 (B) 7
(C) 10 (D) 18
7. In the long form of periodic table, all the non-metals
are placed in- 19. W hich of the following pairs of elements does not
(A) s-block (B) p-block belong to same group ?
(C) d-block (D) f-block (A) Cl, Br (B) N, P
(C) Mg, Ca (D) Al, Si
8. Which of the following is not a representative element ?
(A) Fe (B) K 20. The group number of element in periodic table
(C) Ba (D) N indicates -
(A) valency with respect to hydrogen.
9. W hich of the following statements is not true for (B) the atomicity.
noble gases ? (C) the number of electrons in the outermost shell.
(A) They have stable configuration. (D) None of these
(B) All of them contain eight electrons in their
outermost shell. 21. Diagonal relationship is not shown by -
(C) They are the zero group elements. (A) Li and Mg (B) C and N
(D) They are colourless. (C) B and Si (D) Be and Al

10. The element with atomic number 35 belongs to - 22. Covalency is favoured in the following cases -
(A) s - block (B) p - block (A) a smaller cation
(C) d - block (D) f - block (B) a larger anion
(C) large charges on cation and anion
11. Elements in the same vertical column of the periodic (D) All of these
table have same -
(A) number of electrons 23. Which block of the periodic table contains maximum
(B) atomic number number of metals ?
(C) number of valence electrons (A) s-block (B) p-block
(D) electronic configurations (C) d-block (D) f-block

12. W hich pair of atomic numbers represents s-block III. Periodicity & properties
elements ?
24. In the third period of the periodic table, the element
(A) 7,15 (B) 6,12 having smallest size is -
(C) 9,17 (D) 3,12 (A) Na (B) Ar
(C) Cl (D) Si
13. W hich of the following represents the electronic
configuration of d-block elements ? 25. W hich of the following is the most non-metallic
(A) (n–1)s2 nd1–10 (B) (n–1)d1–10ns0-2 element ?
1–10 2 4
(C) (n–1)d ns p (D) (n–1)p 4ns2 (A) Br (B) Cl
(C) P (D) S
14. Elements of which group form anions most readily ?
(A) Oxygen family (B) Nitrogen family 26. With the increase in atomic number in a period -
(C) Halogens (D) Alkali metals (A) metallic character decreases
(B) non-metallic character decreases
15. Which of the following atoms has a valency equal to (C) ionisation energy decreases
zero ? (D) None of these
(A) Hydrogen (B) Lithium
(C) Neon (D) Oxygen 27. Largest in size out of Na+ , O2– and K+ is -
(A) Na+ (B) O 2–
16. W hich of the following electronic configurations (C) K +
(D) All are equal
represents most electropositive element ?
(A) [He]2s1 (B) [He]2s2 28. W hich of the following properties does not depend
(C) [Xe] 6s1 (D) [Xe]6s2 on periodicity ?
(A) Atomic weight (B) Electron affinity
17. The electronic configuration of the element which is
(C) Ionisation energy (D) Electronegativity
just above the element with atomic number 43 in the
same periodic group is -

PAGE # 66
29. Which of the following have isoelectronic structures ? (C) Oxygen has high electron affinity.
(i) CH3+ (ii) H3O+ (D) O– ion has comparatively larger size than oxygen
(iii) CH 3– (iv) NH3 atom.
(A) (i), (ii) and (iii) (B) (i) and (iii)
(C) (i) and (iv) (D) (ii),(iii) and (iv) 41. W hich of the following is the increasing order of
electron affinity of halogens ?
30. Which of the following ions has smallest radius ?
(A) Cl < Br <  < F (B)  < Br < F < Cl
(A) Cl– (B) S2–
+ (C) F < Cl < Br <  (D) Br < F <  < Cl
(C) K (D) Ca2+
– 2–
31. The radii of F, F , O and O are in the order -
42. Electron affinity of X would be equal to -
(A) O2– > F– > O > F (B) O2– > F– > F > O
(C) F– > O2– >F > O (D) O2– > O > F– > F (A) electron affinity of X 
(B) ionisation potential of X 
32. Correct increasing order of density is -
(C) ionisation potential of X
(A) Li < K < Na < Rb < Cs
(D) None of these
(B) Li < Na < K < Rb < Cs
(C) Cs < Rb < K < Na < Li 43. Among elements A,B,C and D having atomic numbers
(D) K < Li < Na < Rb < Cs 7,8,9 and 12 respectively, the element with smallest
33. Ionic radius of - size and highest I.E. is -
(i) Ti4+ < Mn7+ (ii) 35Cl– < 37Cl– (A) A (B) B
+
(iii) K > Cl–
(iv) P3+ > P5+ (C) C (D) D
Which of the above is/are correct ?
(A) Only (iv) (B) (i) and (ii) 44. W hich of the following transitions involve maximum
(C) (ii) and (iv) (D) (ii) and (iii) amount of energy ?
(A) M–(g)  M (g)
IV. Electron affinity, ionisation energy, Electronegativity
(B) M(g)  M–(g)
34. The ionisation energy of isotopes of an element will be - (C) M+(g)  M2+(g)
(A) same
(B) different (D) M2+(g)  M3+(g)
(C) dependent on atomic masses
(D) dependent on the number of neutrons present in 45. Ionisation energy of F– is 320 KJ mol–1. The electron
the nucleus affinity of fluorine would be -
(A) – 320 KJ mol–1 (B) – 160 KJ mol–1
35. The correct order of second I.E. of C,N,O and F is - –1
(C) 320 KJ mol (D) 160 KJ mol–1
(A) F > O > N > C (B) C > N > O > F
(C) O > N > F > C (D) O > F > N > C 46. A sudden large jump between the values of second
36. I.E increases with - and third ionisation energies of an element would be
(A) decrease in atomic size. associated with the electronic configuration -
(B) increase in nuclear charge. (A) 1s2, 2s2 2p6,3s1 (B) 1s2, 2s2 2p6,3s2 3p1
2 2 6 2 2
(C) increase in penetration effect of electrons. (C) 1s , 2s 2p ,3s 3p (D) 1s2, 2s2 2p6,3s2
(D) All of the above
47. In which of the following arrangements, the order is
37. Which of the following has zero electron affinity ? not according to the property indicated against it ?
(A) Radon (B) Nitrogen (A) Al3+ < Mg2+ < Na+ < F– - Increasing ionic size
(C) Oxygen (D) Radium (B) B < C < N < O - Increasing first ionisation energy
38. Which of the following has highest electron affinity ? (C)  < Br < F < Cl - Increasing electron affinity
(A) Br (B) Cl (D) Li < Na < K < Rb - Increasing metallic radius
(C)  (D) F
48. W hich of the following species has the highest
39. The amount of energy released on the addition of an electron affinity ?
electron in the outermost shell of an isolated gaseous (A) F– (B) O
atom is called -
(C) O– (D) Na+
(A) ionisation energy (B) hydration energy
(C) electron affinity (D) electronegativity 49. The first (IE 1) and second (IE 2) ionisation energies
2–
40. The formation of the oxide ion O (g) requires first an (KJmol–1) of a few elements are shown below :
exothermic and then an endothermic step as shown IE 1 IE 2
below - (i) 2372 5251
O(g) + e–  O–(g) ; Hº = –15.2 KJ/mol (ii) 520 7300
O–(g) + e–  O2–(g) ; Hº = 844 KJ/mol (iii) 900 1760
What does it show ? (iv) 1680 3380
(A) O– ion will tend to resist the addition of another Which of the above elements is likely to be a noble
electron. gas ?
(B) Oxygen is more electronegative.
(A) (i) (B) (ii)
(C) (iii) (D) (iv)

PAGE # 67
50. The electron affinities of halogens are - 10. Ionization Energy is defined as 'the energy required
F = –332 KJ mol–1 , Cl = –349 KJ mol–1, Br = –324 KJ for removing the most loosely bound electron from
an isolated gaseous atom or ion'.
mol–1 , I = –295 KJ mol–1
A(g) A+ + e– 1st Ionization Energy
The higher value of Cl as compared to that of F is due to - + 2+
A (gA (g)+ e –
2nd Ionization Energy
(A) higher atomic radius of F A2+ (g)A3+(g) + e- 3rd Ionization Energy
(B) smaller electronegativity of F and so on ....
(C) weaker electron repulsion in Cl
(D) more vacant p-subshell in Cl

EXERCISE-2 Identify element A. [IJSO-State-I/2015]


(A) Nitrogen (B) Oxygen
COMPETITIVE EXAM PREVIOUS YEARS’ QUESTIONS : (C) Carbon (D) Fluorine
11. Electronic configuration of Na+ is (2,8) and that of
1. Among Li, Be, N and F, the element having the largest sodium element is (2,8,1). Choose the correct
atomic radius, is : [KVPY-Part-I/2008] statements. [IJSO-State-I/2015]
(A) Li (B) Be i. Na+(g) is more stable than Na(g).
(C) N (D) F ii. Na+(g) is less stable than Na(g).
iii. Na+(aq) is more stable than Na(aq)
2. The atomic radii of the alkali metals follow the order
iv. Na+(aq) is less stable than Na(aq)
[KVPY-Part-I/2008]
(A) ii, iii (B) i, iii
(A) Li > Na > K > Cs (B) K > Cs > Li > Na (C) ii, iv (D) i, iv
(C) Na > K > Cs > Li (D) Cs > K > Na > Li
3. The most basic oxide among MnO, Mn2O3, MnO2 and 12. Which of the following series of elements have nearly
Mn2O7 is - [IJSO-State-I/2011] the same atomic radii ? [IJSO-State-I/2014-15]
(A) MnO (B) MnO2
(A) Fe, Co, Ni, Cu (B) Na, K, Rb, Cs
(C) Mn2O3 (D) Mn2O7
(C) Li, Be, B, C (D) F, Cl, Br, I
4. Which of the following group elements from the peri-
odic table form electron deficient molecules ?
13. W hich of the following has the maximum
[IJSO-State-I/2011]
number of unpaired electrons ?
(A) Group IV (B) Group V
(C) Group III (D) Group I [IJSO-State-I/2014-15]
(A) Ti3+ (B) V3+
5. The element with electronic configuration 1s 2 2s2
2p63s2 is a/an - [IJSO-State-I/2012] (C) Fe2+ (D) Fe3+
(A) Metal (B) Non- metal
(C) Metalloid (D) Inert gas 14. The following variation of propeties is generally seen
6. Atomic number decides chemical property of an in the periodic table. [IJSO-State-I/2014-15]
element. It also decides which group the element (A) Atomic radius and ionization energy both increase
belongs to. Identify which of the following elements across a period.
are from the same group in the periodic table. (B) Atomic radius increases and ionization energy
[IJSO-State-II/2013] decreases across a period
(A) 1, 3, 11, 19, 37 (B) 8, 24, 42, 74
(C) Atomic radius and ionization energy both de-
(C) 4, 12, 20, 58 (D) 5, 13, 27, 47
creases across a period
7. Which of the following set of elements have the stron- (D) Atomic radius decreases and ionization energy
gest tendency to form anions ? [IJSO-State-II/2013]
increases across a period
(A) N, O and P. (B) P, S and Cl.
(C) N, P and Cl. (D) N, P and S.
15. In which of the following series of transition metal
8. Arrange the following elements in the increasing or-
ions, all metal ions have 3d2 electronic configuration
der of their atomic radii. [IJSO-State-II/2013]
[IJSO-State-I/2014-15]
(A) Na < Li < Rb < C < K
(B) Cs < Rb < K < Na < Li (A) Ti+, V4+, Cr6+, Mn7+
(C) K < Rb < Cs < Na < Li (B) Ti3+, V2+, Cr3+, Mn4+
(D) Li < Na < K < Rb < Cs (C) Ti2+. V3+, Cr4+, Mn5+
(D) Ti4+, V3+, Cr2+, Mn3+
9. Which of these elements has the greatest electrone-
gativity ? [IJSO-State-I/2013] 16. Ionic radii of following species are
[IJSO-State-I/2015-16]
(A) Br (B) N
(A) Si4+ > P5+ > S6+ > C17+
(C) O (D) S (B) S6+ > p5+ > Si4+ > Al3+
(C) N3– >O2– >F– >Na1+
(D) Mg2+ > Na1+ >F– > O2–

PAGE # 68
17. An element with atomic number 44 is below which
element in the periodic table ?
[IJSO-State-I/2016-17]
(A) Calcium (B) Iron
(C) Argon (D) Magnesium

18. There are many elements in the periodic table that


are named after the country, where they were first
made or obtained. For example, the Latin name for
copper was coined by the Romans because their
chief source of copper was from the Island of Cyprus.
However, there is one country in the world which was
named after an element (the Latin name). A long time
ago, it was believed that this country had mountains
full of a valuable element, however all expeditions to
find these mountains failed. But the name stuck on.
The element in question is used for many
applications today, and many of its compounds are
used as catalysts. The ions of this metal have very
good anti-microbial property and finds application in
water purification. The element is
[IJSO-State-I/2016-17]
(A) Sodium (B) Gold
(C) Silver (D) Francium

PAGE # 69
MATTER

INTRODUCTION

There are a large number of things around us which


we see and feel. For example, we can see a book in
front of us. A book occupies some space. The space
occupied by the book is called its volume. If we pick
up the book, we can also feel its weight. So, we
conclude that the book has some mass. We cannot
see the air around us, yet if we fill a balloon with air
and then weigh it carefully, we will find that not only
does air occupy space (bounded by the balloon), but Experiment to show that matter is made of particles
it also has mass. Evidence - 2
Things like a book and air are examples of matter. Movement of pollen grains in water : The best
Other examples of matter are wood, cloth, paper, ice, evidence for the existence and movement of particles
in liquids was given by Robert Brown in 1827. Robert
steel, water, oil etc. Further, that matter offers
Brown suspended extremely small pollen grains in
resistance is borne out by the fact that we cannot
water. On looking through the microscope, it was
displace an object from one place to another without
found that the pollen grains were moving rapidly
applying some force. We have to apply force to pick throughout water in a very irregular way (or zig-zag
up a stone from the ground. Thus , matter can be way).
defined as follows - Conclusion : Water is made up of tiny particles which
Anything that occupies space, has mass and offers are moving very fast (the water molecules themselves
are invisible under the microscope because they are
resistance is called matter.
very, very small). The pollen grains move on the
surface of water because they are constantly being
PHYSICAL NATURE OF MATTER
hit by the fast moving particles of water. So, though
the water particles (or water molecules) are too small
(a) Matter is Made up of Particles :
to be seen, but their effect on the pollen grains can be
(i) Everything around us is made up of many tiny seen clearly. The random motion of visible particles
pieces or particles. (pollen grains) caused by the much smaller invisible
particles of water is an example of Brownian motion
(ii) Particles which make up the matter are constantly (after the name of the scientist Robert Brown who
moving. first observed this phenomenon.)
(iii) Particles which make up matter are atoms or
molecules.
(i) Evidences for the presence of particles in matter :
Most of the evidences for the existence of particles in
matter and their motion come from the experiments
on diffusion and Brownian motion.
Evidence - 1 Brownian motion : Zig-zag motion (in a very irregular
Dissolving a solid in a liquid : Take a beaker. Fill half way) of particles is known as brownian motion.
of it with water. Mark the level of water in the beaker. Brownian motion can also be observed in gases.
Add some sugar to the water and dissolve it with the Sometimes, when a beam of light enters in a room,
help of a glass rod. You will see that the sugar has we can see tiny dust particles suspended in air which
are moving rapidly in a very random way. This is an
disappeared, but there is no change in the level of
example of Brownian motion in gases. The tiny dust
water.
particles move here and there because they are
Conclusion : This can be explained by assuming that constantly hit by the fast moving particles of air.
matter is not continuous, rather it is made up of The existence of Brownian motion gives two
particles. Sugar contains a large number of separate conclusions.
particles. These particles when dissolved in water • Matter is made up of tiny particles.
• Particles of matter are constantly moving.
occupy the vacant spaces between the particles of
water. That is why, the water level in the beaker did  Note :
not rise. Had sugar been continuous, like a block of Brownian motion increases on increasing the
wood, the water level in the beaker would have risen. temperature.

PAGE # 70
Pre-foundation Career Care Programmes (PCCP) Division
(b) Characteristics of Particles of Matter :  Note :
The important characteristics of particles of matter The particles of matter possess kinetic energy and
are the following : so are constantly moving. As the temperature rises,
(i) The particles of matter are very, very small. particles move faster.
(iv) Particles of matter attract each other : There
(ii) The particles of matter have spaces between
are some forces of attraction between the particles of
them.
matter which bind them together.
(iii) The particles of matter are constantly moving : (A) Cohesive Force : The force of attraction between
This property can be explained by diffusion. the particles of same substances is called cohesive
(A) Diffusion :“Intermixing of particles of two different force.
types of matter on their own is called diffusion.” t is (B) Adhesive Force : The force of attraction between
the phenomenon in which the movement of the particles of different substances is called
molecules or particles occur from their higher adhesive force.
concentration towards their lower concentration.
e.g. : If we take a piece of chalk, a cube of ice and an
e.g. : When a perfume bottle is opened in one corner iron nail and beat them with a hammer, chalk will
of a room, its fragrance spreads in the whole room easily break into smaller pieces, but more force will
quickly. This happens because the particles of be required to break a cube of ice and iron nail will
perfume move rapidly in all directions and mix with not break.
the moving particles of air in the room.
Reason : The reason for this is, that the force of
(A) Experiment : We take a gas jar full of bromine attraction is quite weak in between the chalk particles,
vapours and invert another gas jar containinig air over but force of attraction in between the particles of ice
it, then after some time, the red-brown vapours of cube is a bit stronger, while force of attraction in
bromine spread out into the upper gas jar containing between the particles of iron is very-very strong.
air.
RIGID AND FLUID
(B) Conclusion : In this way, the upper gas jar which
contains colourless air in it, also turns red-brown. (i) Rigid : Rigid means unbending or inflexible. A solid
The mixing is due to the diffusion of bromine vapours is a rigid form of matter so that it maintains its shape
when subjected to outside force.
(or bromine gas) into air.
(ii) Fluids : Fluids are the substances which have
tendency to flow. A liquid is a fluid form of matter
which occupies the space of the container. Liquids
have a well defined surface. A gas is a fluid form of
matter which fills the whole container in which it is
kept.
 Note :
Liquids and gases are known as fluids.

CLASSIFICATION OF MATTER

On the basis of physical states, all matter can be


classified into three groups:-
Diffusion of bromine vapour into air (a) Solids (b) Liquids (c) Gases
COMPARISON OF THE CHARACTERISTICS OF THREE STATES OF MATTER

Property Solid state Liquid state Gaseous state


Comparatively large
Interparticle spaces Very small spaces Very large spaces
spaces than solids
Interparticle forces Very strong Weak Very weak

Nature Very hard and rigid Fluid Highly fluid

Compressibility Negligible Very small Highly compressible.


Definite shape and Indefinite shape, but Indefinite shape as
Shape and volume
volume definite volume well as volume
Density High Less than solid state Very low density
Kinetic Comparatively high
Low Very high
energy than solids
Diffusion Negligible Slow Very fast

PAGE # 71
Pre-foundation Career Care Programmes (PCCP) Division
Gases are Highly Compressible therefore :

(i) LPG (Liquefied Petroleum Gas) is used in our home


for cooking.

(ii) Oxygen cylinders supplied to hospitals contain


liquid oxygen.

(iii) These days C.N.G. (Compressed Natural Gas) is


used as fuel in vehicles.

 Note :
Gaseous particles move randomly at high speed and
hit each other and also walls of the container, so exert
pressure. Change of state from ice to water

(A) Melting or Fusion: The process due to which a


INTERCONVERSION OF STATES OF MATTER
solid changes into liquid state by absorbing heat
The phenomenon of change of matter from one state energy is called melting or fusion.
to another state and back to original state, by altering (B) Freezing or Solidification: The process due to
the conditions of temperature and pressure, is called which a liquid changes into solid state by giving out
interconversion of states of matter. heat energy is called freezing or solidification.

The various states of matter can be interchanged into (C) Melting Point: The constant temperature at which
one another by altering the conditions of - a solid changes into liquid state by absorbing heat
(a) Temperature (b) Pressure. energy at 1 atm pressure is called its melting point.

(D) Freezing Point: The constant temperature at which


(a) Altering the Temperature of Matter :
a liquid changes into solid state by giving out heat
(i) Interconversion of solid into liquid and vice versa : energy at 1 atm pressure is called freezing point.
Solids can be converted into liquids by heating them.  Note :
Similarly liquids can be cooled to form solids. The numerical value of freezing point and melting
point is same.
e.g. :ce at 00C changes into water at 00C, when heat
Melting point of ice = Freezing point of water = 0ºC
energy is supplied to it. The water at 0 0C changes
(273.16 K).
into ice at 00C on freezing.
Explanation: On increasing the temperature of solids,
Activity - the kinetic energy (K.E.) of particles increases. Due
To study the change of state from ice to water. to increase in K.E., the particles start vibrating with
greater speed. The energy supplied by heat
Materials required -
overcomes the force of attraction between the
A 100 cc beaker, a thermometer (Celsius), a glass
particles. Then, the particles leave their fixed positions
stirrer, a wire gauze, a tripod stand, a Bunsen burner, and start moving freely and thus solid melts.
an iron stand, ice cubes.
Latent Heat of Fusion : The amount of heat energy
Method - that is required to change 1 kg of solid into liquid at
Half fill the beaker with ice cubes and place it over a atmospheric pressure and at its melting point is
wire gauze and tripod stand. Suspend a Celsius known as the latent heat of fusion. (In Greek Latent
thermometer from the iron stand, such that its bulb is means Hidden) Latent heat of fusion of ice = 3.34 ×
105 J/kg.
touching the water level. Place a glass stirrer in the
ice.  Note :
Particles of water at 00C (273 K) have more energy as
Record the temperature of ice. You will find it is 00 C
compared to particles in ice at the same temperature.
(273 K). Now heat the beaker on a low bunsen flame
(ii) Interconversion of liquid into gaseous state and
and continuously stir the contents of beaker. Record
vice versa: Liquids can be converted into gases by
the temperature five to six times, till all the ice melts.
heating them. Similarly, gases can be converted into
You will observe that temperature throughout remains
liquids by cooling them.
00C (273 K), till all the ice melts.
e.g. : Water at 1 atm pressure changes into vapours
(steam) at 1000C by absorbing heat. Steam at 1000C
changes into water by giving out energy.

PAGE # 72
Pre-foundation Career Care Programmes (PCCP) Division
Activity - Explanation : When heat is supplied to water, particles
To study the change of state from water to steam. start moving faster. At a certain temperature, a point
is reached when the particles have enough energy to
Materials required - break the forces of attraction between the particles. At
A 100 cc beaker, a thermometer (Celsius), a glass this temperature the liquid starts changing into gas.
stirrer, a wire gauze, a tripod stand, a Bunsen burner,
Latent heat of vaporisation: The amount of heat which
an iron stand, tap water. is required to convert 1 kg of the liquid (at its boiling
Method - point) to vapour without any change in temperature.
Half fill the beaker with water and place it over a wire Latent heat of vaporisation of water = 22.5 × 105 J/kg.
gauze and tripod stand. Suspend a Celsius  Note :
thermometer from the iron stand, such that its bulb is Particles in steam, that is water vapour at 373 K have
touching the water level. Place a glass stirrer in the more energy than water at the same temperature.
water. Because steam has absorbed extra energy in the
Record the temperature of water. Heat the beaker on form of latent heat of vaporisation.
a low Bunsen flame and continuously stir the water
with glass stirrer. Go on recording the temperature till
water starts boiling. Allow the water to boil for few
minutes and record its temperature. We can show the change of temperature with time in
the form of a temperature-time graph drawn by using
You will notice that temperature of water rises till it the readings obtained in the above experiment. Such
starts boiling. The temperature of boiling water is 1000C a temperature-time graph is shown in figure.
(373 K). If we continue heating the water it changes
into steam, but the temperature remains constant, i.e.,
1000C (373 K).

Temperature Time Graph


W e can understand the above graph by taking an
example of water.In this graph at point A, we have all
ice. As we heat it, the ice starts melting to form water
but the temperature of ice and water mixture does not
Change of state from water to steam
rise. It remains constant at 0°C during the melting of
(A) Boiling or Vaporisation: The process due to which ice. At point B, all the ice has melted to form water.
a liquid changes into gaseous state by absorbing Thus, we have only water at point B. Now, on heating
heat energy is called boiling. beyond point B, the temperature of water (formed from
ice) starts rising as shown by the sloping line BC in
(B) Condensation or Liquefaction: The process due the graph. When the temperature of water reaches
to which a gas changes into liquid state by giving out its boiling point i.e; 100°C, water starts converting
heat energy is called condensation. into steam. But during the process of boiling, tem-
perature does not rise and thus constant tempera-
(C) Boiling Point: The constant temperature at which ture is observed (line CD). At point D all the water has
a liquid rapidly changes into gaseous state by boiled to form steam. Thus, we have only steam at
absorbing heat energy at atmospheric pressure is point D. Now on heating beyond point D, the tem-
called boiling point. perature of steam rises as shown by the sloping line
DE.
(D) Condensation Point:- The constant temperature
at which a gas changes into liquid state by giving out (iii) Direct interconversion of solid into gaseous state
heat energy at atmospheric pressure is called and vice versa: The changing of solid directly into
condensation point. vapours on heating and of vapours directly into solid
on cooling is known as sublimation.
 Note :
• The solid which undergoes sublimation to form
The numerical value of condensation point and
vapour is called ‘sublime’.
boiling point is same.
Condensation point of water vapour = Boiling point • The solid obtained by cooling the vapours of a solid
of water = 100ºC (373.16 K). is called ‘sublimate’.

PAGE # 73
Pre-foundation Career Care Programmes (PCCP) Division
e.g. : Ammonium Chloride (NH4Cl), iodine, camphor,
naphthalene (moth balls) and anthracene. EVAPORATION
Liquid The phenomenon of change of a liquid into vapours
at any temperature below its boiling point is called
n Fr Me
atio ion ee ltin evaporation.
t
oris sa zin g
a p en g W ater changes into vapours below 100 0C. The
V on d
particles of matter are always moving and are never
C
Sublimation at rest. At a given temperature in any gas, liquid or
Gas Solid solid, there are particles with different K.E.
Sublimation
In case of liquids, a small fraction of particles at the
Interconversion of states of matter surface, having higher K.E., is able to break the forces
Specific Heat of attraction of other particles and gets converted into
The specific heat of a substance is the amount of vapour.
heat which is required to raise the temperature of a  Note :
unit mass of the substance by 1º C. Now, if we The atmospheric pressure at sea level is 1 atm.
measure the heat in “joules” and mass in “kilograms”,
(a) Factors Affecting Evaporation:
then the definition of specific heat becomes.
The specific heat of a substance is the amount of (i) Temperature: With the increase in temperature the
heat in joules required to raise the temperature of rate of evaporation increases.
1 kilogram of the substance by 1ºC. Rate of evaporation  T
The specific heat of a substance is usually Reason : On increasing temperature more number
represented by the symbol C (Sometimes, however, of particles get enough K.E. to go into the vapour state.
the specific heat of a substance is also represented
by the letter ‘S’). The specific heat of a substance (ii) Surface Area : Rate of evaporation  Surface area
varies slightly with temperature. The change in the Since evaporation is a surface phenomena, if the
specific heat of a substance with temperature is due surface area is increased, the rate of evaporation
increases. So, while putting clothes for drying up we
to the changes which occur in the structure and
spread them out.
organization of the molecules in a substance with
change in temperature. 1
(iii) Humidity of Air : Rate of evaporation 
Units of Specific Heat Humidity
The unit of specific heat depends on the units in which Humidity is the amount of water vapour present in air.
“heat” and “mass” are measured. Now, the S.I. unit of When humidity of air is low, the rate of evaporation is
heat is “joule” and that of mass is “kilogram”, so, the high and water evaporates more readily. W hen
S.I. unit of specific heat is “joules per kilogram per humidity of air is high, the rate of evaporation is low
degree celcius”, which is written in short form as : and water evaporates very slowly.
J/kg°C or J kg–1 °C–1. (iv) Wind Speed : Rate of evaporation  Wind speed
With the increase in wind speed, the particles of water
vapour move away with the wind. So the amount of
water vapour decreases in the surroundings.

The difference in various states of matter is due to (v) Nature of substance : Substances with high
the different intermolecular spaces between their boiling points will evaporate slowly, while substances
particles. So when a gas is compressed the with low boiling points will evaporate quickly.
intermolecular space between its particles decreases Differences between evaporation and boiling
and ultimately it will be converted into liquid. Evaporation Boiling
Pressure and temperature determine the state of a It is a surface
substance. So, high pressure and low temperature It is a bulk phenomenon.
phenomenon.
can liquefy gases. It occurs at all
It occurs at B.P. only.
e.g. : Carbon dioxide (CO 2) is a gas under normal temperatures below B.P.
conditions of temperature and pressure. It can be The rate of evaporation The rate of boiling does
liquefied by compressing it to a pressure 70 times depends upon the surface not depend upon the
more than atmospheric pressure. area of the liquid, humidity surface area, wind speed,
Solid CO2 is known as ‘Dry ice’. Solid CO2 is extremely temperature & wind speed and humidity.
cold and used to ‘deep freeze’ food and to keep ice-
cream cold. (b) Cooling Cause d by Evaporation:
Unit of pressure : The cooling caused by evaporation is based on the
Atmosphere (atm) is a unit for measuring pressure fact that when a liquid evaporates, it draws (or takes)
exerted by a gas. the latent heat of vaporisation from ‘anything’ which
The S. unit of pressure is Pascal (Pa.) it touches.
1 atm = 1.01 × 105 Pa. For example :
 Note : • f we put a little of spirit, ether or petrol on the palm of
When pressure is lowered the boiling point of liquid is our hand then our hand feels very cold.
lowered. This helps in rapid change of liquid into gas. • Perspiration (or sweating) is our body’s method of
maintaining a constant temperature.

PAGE # 74
Pre-foundation Career Care Programmes (PCCP) Division
(c) We Wear Cotton Clothes in Summer :
BOSE-EINSTEIN CONDENSATE (B.E.C.)
During summer, we perspire more because of the
mechanism of our body which keeps us cool. During
The B.E.C. is formed by cooling a gas of extremely
evaporation, the particles at the surface of liquid gain
low density, about one-hundred-thousandth the
energy from the surroundings or body surface.
The heat energy equal to latent heat of vaporisation, density of normal air, to super low temperature.
is absorbed from the body, leaving the body cool. Substance : A substance is a kind of matter that cannot
Cotton, being a good absorber of water helps in
be separated into other kinds of matter by any physical
absorbing the sweat.
process. For example, sugar dissolved in water can
(d) Water droplets on the outer surface of
a glass containing ice cold water : be separated from water by simply evaporating the
water but it cannot be broken into its components by
If we keep some ice cold water in a glass then we will
observe water droplets on the outer surface of glass any physical process so here sugar is a substance.
after sometime.
PURE SUBSTANCE
Reason : The water vapour present in air on coming
in contact with glass of cold water, loses energy. So
A homogeneous material which contains particles of
water vapour gets converted to liquid state, which we
only one kind and has a definite set of properties is
see as water droplets.
called a pure substance.
PLASMA Examples : Iron, silver, oxygen, sulphur, carbon dioxide
etc., are pure substances because each of them has
This state consists of super energetic and super
only one kind of particles.
excited particles. These particles are in the form of
ionised gases. (a) Characteristics of A Pure Substance :
For eg: Neon sign bulb and fluorescent tube
(i) A pure substance is homogeneous in nature.
Neon sign bulb – Neon gas
Fluorescent tube – Helium gas (ii) A pure substance has a definite set of properties.
W hen electrical energy flows through gas, it gets These properties are different from the properties of
ionised and hence plasma is created. other substances.
Plasma glows with a special colour depending on
nature of gas. Sun and the stars glow because of the (iii) The composition of a pure substance cannot be
presence of plasma. altered by any physical means.
Matter

Pure substances Mixtures


Only one type of particles More than one type of
are present ( no impurities) particles are present

Elements Compounds Homogeneous


mixtures
(true solutions)

• Eleven elements exist as gases at room


(b) Elements :
temperature. They are hydrogen, nitrogen, oxygen,
A pure substance, which cannot be subdivided into
fluorine, chlorine, helium, neon, argon, krypton, xenon
two or more simpler substances by any physical or
and radon.
chemical means is called an element.
(i) Examples : Hydrogen, oxygen, nitrogen, copper, • Remaining elements are solids at room
zinc, tin, lead, mercury, etc. are all elements as they temperature.
cannot be subdivided into simpler parts by any
(B) Elements can be classified as metals and
physical or chemical means. A substance made up
non-metals. There are 22 non-metals and the rest
of the atoms with same atomic number is called an
aremetals.
element.
• Amongst the metals, only mercury is a liquid metal.
(ii) Classification of elements : All other metals are solids.
(A) On the basis of physical states, all elements can • Amongst the 22 non-metals : 10 non-metals are
be classified into three groups:- solids. They are boron, carbon, silicon, phosphorus,
(1) Solids (2) Liquids (3) Gases sulphur, selenium, arsenic, tellurium, iodine and
astatine. 1 non-metal, bromine, is a liquid. Five non-
It has been found that : metals, hydrogen, nitrogen, oxygen, fluorine and
• Two elements exist as liquids at room temperature. chlorine are chemically active gases. Six non-metals,
They are mercury and bromine. helium, neon, argon, krypton, xenon and radon are
chemically inactive gases. These are also called
noble gases, inert gases or rare gases.

PAGE # 75
Pre-foundation Career Care Programmes (PCCP) Division
METALLOIDS :
Their properties are intermediate between the
There are a few elements which show some
properties of metals and non-metals. Metalloids are
properties of metals and other properties of non-
also sometimes called semi-metals. The important
metals. For example they look like metals but they
examples of metalloids are : Boron (B), Silicon (Si),
are brittle like non-metals. They are neither
Germanium (Ge), Arsenic (As), Antimony (Sb),
conductors of electricity like metals nor insulators like
Tellurium (Te) Polonium (Po) and Astatine (At).
non-metals, they are semiconductors. The elements
which show some properties of metals and some  Note :
other properties of non-metals are called metalloids. Hydrogen is the lightest element.

(C) Elements can be classified as normal elements and radioactive elements. The elements which do not give out
harmful radiations are called normal elements. Elements from atomic number 1 to atomic number 82 are normal
elements. The elements which give out harmful radiations are called radioactive elements. Elements from atomic
number 83 to atomic number 112 and 114, 116 and 118 are radioactive in nature.

PAGE # 76
Pre-foundation Career Care Programmes (PCCP) Division
(c) Compounds : (ii) Bases : Compounds which give hydroxide ion in
A pure substance, which is composed of two or more aqueous solution for e.g.Sodium hydroxide,
different elements, combined chemically in a definite Potassium hydroxide etc.
ratio by mass, such that it can be broken into elements (iii) Salts : It is formed by the chemical reaction
only by chemical means is called compound. between acids and bases for e.g. ammonium
The two or more elements present in a compound chloride, zinc sulphate etc.
are called constituents or components of the
compound. For example, water is a compound of MIXTURES
hydrogen and oxygen, combined together in the ratio
of 1 : 8 by weight. W ater can be broken into its Most of the materials around us are not pure
constituents only by electro-chemical method, i.e., by substances, but contain more than one substances,
passing electric current through it. elements or compounds. Such materials are called
mixtures.
TYPES OF COMPOUNDS
(a) Definition :
(A) On the basis of constitutent elements : W hen two or more substances (elements,
(i) Inorganic compounds compounds or both) are mixed together in any
These compounds have been mostly obtained from proportion, such that they do not undergo any
non-living sources such as rocks and minerals. A chemical change, but retain their individual
few examples of inorganic compounds are : common characteristics, the resulting product is called a
salt, marble, washing soda, baking soda, carbon mixture.
dioxide, ammonia, sulphuric acid etc. (b) Types of Mixture :
(ii) Organic compounds (i) Homogeneous Mixture : A mixture in which different
The word ‘organ’ relates to different organs of living constituents are mixed uniformly is called a
beings. Therefore, organic compounds are the homogeneous mixture.
compounds which are obtained from living beings
i.e., plants and animals. It has been found that all the Examples : All solutions, such as solutions of
organic compounds contain carbon as their essential common salt, copper sulphate, sugar etc. are
constituent. Therefore, the organic compounds are examples of homogeneous mixtures. Similarly, alloys
quite often known as ‘carbon compounds’. A few such as brass, bronze etc. are homogeneous solid
common examples of organic compounds are : solutions of metals and air is homogenous mixture
methane, ethane, propane (all constituents of cooking of gases.
gas), alcohol, acetic acid, sugar, proteins, oils, fats (ii) Heterogeneous Mixture : A mixture in which
etc. different constituents are not mixed uniformly is called
(B) On the basis of their properties : a heterogeneous mixture.
(i) Acids : Compounds which give hydronium ion in
aqueous solution for e.g. hydrochloric acid, sulphuric Examples : A mixture of sand and salt, iron powder
acid, nitric acid, formic acid etc. and sulphur powder, soil etc. are examples of
heterogeneous mixtures.

PAGE # 77
Pre-foundation Career Care Programmes (PCCP) Division
(B) On the basis of solvent :
TRUE SOLUTIONS (i) Aqueous Solutions : The solutions obtained by
A homogeneous mixture of two or more substances dissolving various substances in water are called
is called a solution. Usually we think of a solution as aqueous solutions.
The common examples are :
a liquid that contains either a solid or a liquid or a gas
(i) Common salt dissolved in water.
dissolved in it. However, this is not true. We can also
(ii) Sugar dissolved in water.
have a solid solution and gaseous solution as in the
(iii) Acetic acid disssolved in water etc.
case of alloys and air respectively.
(ii) Non-Aqueous Solutions : The solutions obtained
(a) Components of a Solution : by dissolving the substances in liquids other than
The substances present in a homogeneous solution water are called non-aqueous solutions. The
are called components of the solution. A solution common non-aqueous solvents are alcohol, carbon
basically has two components, i.e., a solvent and a disulphide, carbon tetrachloride, acetone, benzene
solute. etc. Examples of non-aqueous solutions are :
(i) Iodine dissolved in carbon tetrachloride.
(i) Solvent : The component of a solution which is
(ii) Sulphur dissolved in carbon disulphide.
present in large proportion is called solvent.
(iii) Sugar dissolved in alcohol etc.
 Note : (C) On the basis of physical state of solute and
Usually, a solvent is the LARGER component of the solvent :
solution. (i) Solid-Solid solutions : All alloys are solid solutions
For example : In the solution of copper sulphate in of metals. Brass is a solid solution of approximately
30% of zinc and 70% of copper. In this solid solution,
water, water is the solvent. Similarly, in paints,
copper (larger component) is solvent and zinc
turpentine oil is the solvent.
(smaller component) is solute. Similarly, Bell Metal
(ii) Solute : The component of the solution which is is a solid solution of 80% of copper and 20% of tin, in
present in small proportion is called solute. which copper is the solvent and tin is the solute.

For example: In the solution of common salt in water, (ii) Solid-Liquid solutions : Sugar solution is an
the common salt is solute. Similarly, in carbonated example, in which sugar is the solute and water is
drinks (soda water), carbon dioxide gas is the solute. the solvent. Similarly, common salt solution is an
example, in which common salt is the solute and
 Note : water is the solvent. In case of tincture of iodine,
Usually, solute is the SMALLER component of the iodine is the solute and ethyl alcohol is the solvent.
solution.
(iii) Liquid-Liquid solutions : In case of an alcoholic
(b) Characteristics of a True Solution : drink, ethyl alcohol is solute and water is solvent.
Similarly, in case of vinegar, acetic acid is solute and
(i) A true solution is always clear and transparent, i.e., water is solvent.
light can easily pass through it without scattering.
(iv) Gas-Liquid solutions : In case of aerated drinks
(ii) The particles of a solute break down to almost
(soda water), carbon dioxide is the solute and water
molecular size and their diameter is of the order of 1
is the solvent.
nm (10–9 m) or less.
(v) Gas-Gas solutions : Air is a homogeneous mixture
(iii) A true solution can completely pass through a
of two main gases, i.e., 78% of nitrogen and 21% of
filter paper as particle size of solute is far smaller
oxygen. In this mixture, nitrogen is solvent and oxygen
than the size of pores of filter paper.
is solute. Similarly, the petrol fed into the engines of
(iv) A true solution is homogeneous in nature. automobiles is a mixture of petrol vapour and air.
(v) In a true solution, the particles of solute do not (d) Concentration of a Solution :
settle down, provided temperature is constant. It is defined as the amount of solute present in a
(vi) From a true solution, the solute can easily be given quantity of the solution. The most common
recovered by evaporation or crystallisation. method for expressing the concentration of a solution
is called percentage method. The concentration of
(c) Types of Solution: solution refers to the percentage of solute present in
(A) On the basis of concentration : the solution. Furthermore, the percentage of solute
(i) Saturated solution : A solution, which at a given can be expressed in terms of :
temperature dissolves as much solute as it is capable (i) mass of the solute (ii) volume of the solute.
of dissolving, is said to be a saturated solution. (i) Concentration of a solution in terms of mass
(ii) Unsaturated solution : When the amount of solute percentage of solute : If a solution is formed by
contained in a solution is less than the saturation level, dissolving a solid solute in a liquid solvent then the
the solution is said to be an unsaturated solution. concentration of solution is expressed in terms of
mass percentage of solute and is defined as under :
(iii) Super saturated solution : A solution, which The concentration of solution is the mass of the solute
contains more of the solute than required to make a in grams, which is present in 100 g of a solution.
saturated solution, is called a super saturated solution.

PAGE # 78
Pre-foundation Career Care Programmes (PCCP) Division
 Note : 3. A solution contains 50 mL of alcohol mixed with 150
It is very important to keep in mind that the percentage mL of water. Calculate concentration of this solution.
concentration of a solution refers to mass of solute in Sol. This solution contains a liquid solute (alcohol) mixed
100 g of solution and not 100 g of solvent, i.e., water. with a liquid solvent (water), so we have to calculate
the concentration of this solution in terms of volume
The concentration of a solution in terms of mass percentage of solute (alcohol). Now, we know that :
percentage of solute is calculated by the formula given Volume of solute
below : Concentration of solution = × 100
Volume of solution
Concentration of solution Here, Volume of solute (alcohol) = 50 mL
And. Volume of solvent (water) = 150 mL
Mass of solute (in grams) So, Volume of solution = Volume of solute + Volume
=  100
Mass of solution (in grams) of solvent
= 50 + 150 = 200 mL
Mass of solute (in grams) Now, putting these values of ‘volume of solute’ and
 100 ‘volume of solution’ in the above formula we get :
[Mass of solute  Mass of solvent ](in grams)
50 50
Concentration of solution = × 100 =
(ii) Concentration of a solution in terms of volume 200 2
= 25 percent (by volume)
percentage of solute : If a solution is formed by
Thus, the concentration of this alcohol solution is 25
dissolving a liquid solute in a liquid solvent, then the
percent.
concentration of the solution is expressed in terms of
volume percentage of solute. The concentration of a 4. How much water should be added to 16 ml acetone
solution is the volume of the solute in milliliters, which to make its concentration 48% ?
is present in 100 milliliters of a solution. Vol. of solute
Sol. Concentration of solution = × 100
Vol. of solution
 Note :
It is very important to keep in mind that the percentage 16 16
× 100 = 48 x = × 100 = 33.33 ml
concentration of solution refers to volume of solute in x 48
100 ml of solution and not 100 ml of solvent, i.e., Volume of solvent =33.33 – 16 = 17.33 ml.
water.
SUSPENSIONS
The concentration of a solution in terms of volume
percentage of the solute is calculated by the formula A heterogeneous mixture of insoluble particles of
given below : solute, spread throughout a solvent, is called a
suspension. The particle size (diameter) in a
Concentration of solution =
suspension is more than 10–5 cm. The particles have
Volume of solute (in ml) a tendency to settle down at the bottom of the vessel
 100 and can be filtered out, because their size is bigger
Volume of solution (in ml)
than the size of the pores of the filter paper.
Volume of solute (in ml) (a) Examples :
=  100
[Volume of solute  Volume of solvent] (in ml)
(i) Muddy water, in which particles of sand and clay
 Note : are suspended in water.
The concentration of a solution is a pure percentage (ii) Slaked lime suspension used for white-washing
number and has NO UNITS. has particles of slaked lime suspended in water.
(c) Examples : (iii) Paints in which the particles of dyes are suspended
in turpentine oil.
1. What is the meaning of 15% solution of NaCl ?
(b) Characteristic s of Suspensions :
Sol. 15% solution of NaCl is a solution 100 g of which
contains 15 g of NaCl and 85 g of water. (i) The size of particles is more than 10 –5 cm in
diameter.
2. Calculate the amount of glucose required to prepare
(ii) The particles of suspension can be separated
250 g of 5% solution of glucose by mass.
from solvent by the process of filtration.
Mass of solute (iii) The particles of suspension settle down, when
Sol. % of solute = × 100
Mass of solution the suspension is kept undisturbed.

Mass of solute (iv) A suspension is heterogeneous in nature.


5 = × 100
250 (v) More scattering takes place in suspensions,
because of bigger size of particles.
5  250 125
Mass of solute = = = 12.5 g  Note :
100 10
The process of settling of suspended particles under
the action of gravity is called sedimentation.

PAGE # 79
Pre-foundation Career Care Programmes (PCCP) Division
Tyndall effect is caused due to the scattering of light
by the colloidal particles. The true solutions do not
scatter light and hence do not show Tyndall effect.
A heterogeneous solution in which the particle size is
Tyndall effect can be seen when a fine beam of light
in between 10–7 cm to 10–5 cm, such that the solute
enters in a room through a small hole. This happens
particles neither dissolve nor settle down in a solvent
due to scattering of light particles of dust and smoke
is called colloidal solution.
in the air of the room.
In a colloidal solution, relatively large suspended
Tyndall effect can be observed when sunlight passes
particles are called dispersed phase and the solvent
through a dense forest. In the forest, fog contains tiny
in which the colloidal particles are suspended is
droplets of water which act as particles of colloid
called continuous phase or dispersing medium.
dispersed in air.
(a) Examples of Colloidal Solutions :
(viii) The particles of a colloidal solution are electrically
Few examples of colloidal solutions are as follows : charged.
• blood • Milk • Writing ink Electrophoresis
• Jelly • Starch solution • Gum solution The collodial solutions contain either positively or
• Tooth paste• Soap solution • Liquid detergents negatively charged particles and, therefore, when an
• Mist and fog. electric current is passed through them, the particles
(b) Characteristics of Colloidal Solutions : move towards either of the oppositely charged
(i) The size of colloidal particles is in between 10 –7 electrodes. Subsequently, they get discharged on the
cm to 10–5 cm. electrodes and precipitate out. For example, when a
negatively charged As2S 3 solution is taken in a U-
(ii) The particles of a colloidal solution are visible tube into which Platinum electrodes, connected to a
under a powerful microscope. source of E.M.F. are dipped, the colloidal particles
(iii) The particles of a colloidal solution do not settle move towards the positive electrode .
down with the passage of time. The migration of colloidal particles under the influence
of an electric field is known as electrophoresis.
(iv) The particles of a colloidal solution can easily
pass through filter paper.
(v) The colloidal solutions are heterogeneous in
nature.
(vi) Colloidal solutions are not transparent, but
translucent in nature.
(vii) The particles of a colloidal solution scatter light,
i.e., when strong beam of light is passed through the
colloidal solution, the path of beam becomes visible.
Scattering of Light (Tyndall Effect) Electrophoresis showing migration of colloidal
If a beam of light is passed through pure water or a particles
salt solution, the path of light is visible but when a (c) Classification of Colloids :
strong beam of light is passed through a colloidal The colloids are classified according to the state of
solution and viewed at right angles with the help of a dispersed phase (solid, liquid or gas) and the state
microscope, the path of light shows up a bright cone of dispersing medium. A few common examples are
of bluish light. shown in the table :

Tyndall effect shown by colloid in a beaker


This luminosity of path of beam is known as Tyndall
effect and the illuminated path is known as Tyndall
cone. (Tyndall being the name of the scientist who  Note :
studied this phenomenon first). Colloidal solutions can be separated by the process
of CENTRIFUGATION.

PAGE # 80
Pre-foundation Career Care Programmes (PCCP) Division
S.No. Property True solution Colloidal solution Suspension
1 Nature Homogeneous Heterogeneous Heterogeneous
2 Particle size Diameter less than 1 nm Diameter between Diameter more than
(or 10Å) or 10-7 cm 1-100 nm (or 10-1000Å) or 100 nm (or 1000Å)
10 -7 to 10 -5 cm or 10-5 cm.
3 Filtrability Passes through an Passes through Do not pass through
ordinary filter paper ordinary filter paper filter paper or animal
as well as animal or but not through animal or vegetable
vegetable membranes or vegetable membranes membranes
4 Visibility Particles are completely Particles themselves are Particles visible to
invisible invisible but their presence can the naked eye or
be detected by ultramicroscope under a microscope
since they scatter light.
5 Diffusion Diffuse rapidly Diffuse slowly Do not diffuse
6 Tyndall effect Not shown Shown May be shown
7 Appearance of Clear and transparent Generally clear and transparent Opaque
solution

SEPARATION OF Solid-Solid Mixture Sublimable Solid


HETEROGENEOUS MIXTURES
Common salt and Ammonium chloride
Heterogeneous mixtures can be separated into their ammonium chloride
respective components by simple physical methods
such as handpicking, sieving, filtration. Sand and iodine Iodine

Generally following physical properties are considered Common salt and iodine Iodine
in the separation of the constituents of a mixture.
Sodium sulphate and Benzoic acid
(i) Densities of the constituents of the mixture. benzoic acid
(ii) Melting points and boiling points of the constituents
Iron filings and naphthalene Naphthalene
of the mixture.
(iii) Property of volatility of one or more constituents of
(ii) Method :
the mixture.
• Place the mixture of common salt and ammonium
(iv) Solubility of the constituents of the mixture in
chloride in a china dish and heat it over a low Bunsen
different solvents.
flame.
(v) Ability of the constituents of the mixture to sublime.
• Place a clean glass funnel in an inverted position in
(vi) Ability of the constituents of the mixture to diffuse. the china dish and close the mouth of its stem with
 Note : cotton wool.
However, for separating homogeneous mixtures • The ammonium chloride in the mixture sublimes to
special techniques are employed depending upon form dense white fumes. These fumes condense on
the difference in one or more physical properties of the cooler sides of the funnel in the form of fine white
the constituents of the mixture. powder.
• When the mixture gives off no more white fumes, lift
TECHNIQUES USED FOR SEPARATING the funnel, scrap the fine white powder from its sides
THE COMPONENTS OF A MIXTURE on a piece of paper. This is pure ammonium chloride.
The residue left behind in the funnel is sodium
(A) Separation of mixture of two solids : chloride.
(a) By Sublimation:
The changing of solid directly into vapours on heating
and of vapours into directly solid on cooling is known
as sublimation.
(i) Separation of a mixture of common salt and
ammonium chloride :This method is used in the
separation of such solid-solid mixtures where one of
the components sublimes on heating. However, it is
useful only if the components of the mixture do not
react chemically on heating. The table shows the list
of mixtures which can be separated by the process
of sublimation.
Separation by sublimation

PAGE # 81
Pre-foundation Career Care Programmes (PCCP) Division
 Note :
To Separate a Mixture of Sugar and Sand
Dry ice (solid CO2), Naphthalene, Anthracene, Iodine
Sugar is soluble in water whereas sand is insoluble
etc. are sublimable solids.
in water. This difference in the solubilities of sugar
(b) By Using a Suitable Solvent and sand in water is used to separate them. This is
In some cases, one constituent of a mixture is soluble done as follows. The mixture of sugar and sand is
in a particular liquid solvent whereas the other taken in a beaker and water is added to it. The mixture
constituent is insoluble in it. This difference in the is stirred to dissolve the sugar . The sand remains
solubilities of the constituents of a mixture can be undissolved.
used to separate them. For example, sugar is soluble
in water whereas sand is insoluble in it, so a mixture
of sugar and sand can be separated by using water
as solvent. This will become more clear from the
following discussion.

Separation of sugar and sand mixture The sugar


solution containing sand is filtered by pouring over a
filter paper kept in a funnel. Sand remains as a residue
on the filter paper and sugar solution is obtained as
a filtrate in the beaker kept below the funnel. The sugar
solution is evaporated carefully to get the crystals of
sugar. In this way, a mixture of sugar and sand has
been separated by using water as the solvent.
(B) Separation of mixture of a solid and a liquid : (b) By Centrifugation
The method of separating finely suspended or
(a) By Evaporation : colloidal particles in a liquid, by whirling the liquid at
a very high speed is called centrifugation.
(i) Separation of coloured component (dye) from (i) Principle of centrifugation : It is based on the
blue ink : The process of evaporation is suitable for principle that when a very fine suspension or a
the separation of non-volatile soluble solid (dye) from colloidal solution is whirled rapidly, then the heavier
particles are forced towards the bottom of liquid and
its liquid solvent (water). the lighter stay at the top.
(ii) Method : (ii) Separation of cream from milk : The process of
centrifuging is employed in separating cream from
• Heat sand in an iron vessel by placing it over a tripod milk.This process is generally employed in separating
stand. This arrangement is called sand bath. colloidal solutions which easily pass through the filter
paper.
• Place a china dish on the sand bath. Pour about 5 cc
of the ink into the china dish.
• Heat gently evaporates water from the ink such that it
does not boil. In a few minutes the water evaporates
leaving behind dry blue ink. Method of evaporation is
spin
suitable for the following solid-liquid mixtures.

CENTRIFUGE

PAGE # 82
Pre-foundation Career Care Programmes (PCCP) Division
Method : • Suspend this filter paper in a wide and tall cylinder as
• Pour full cream milk in the test tube with a pivot in shown in Figure. Gradually, pour water into the
your laboratory centrifuge. cylinder till the lower end of filter paper slightly dips in
the water. Cover the cylinder with a glass lid to prevent
• Shut the lid of the centrifuge and switch on the current.
any evaporation and leave the apparatus undisturbed
W hen the centrifuge starts working, the tube
containing milk swings out in the horizontal position for an hour. The water rises up the filter paper and
and whirls around its axis at a high speed. reaches the ink mark. This water then dissolves
• The centrifugal force (in the outward direction) pushes various constituents of the ink, gets adsorbed by the
the heavier particles outward, i.e., towards the bottom filter paper in different amounts. More the constituent
of the mixture. Thus, the heavier particles of the gets adsorbed, the lesser it moves upward and vice
proteins, carbohydrates, etc. are pushed towards the versa.
bottom of the tube, but the lighter particles of the fat
stay near the top of the tube and hence separate. • When the solvent (water) reaches near the top of filter
paper, the filter paper is removed from water and dried.
(iii) Applications of centrifugation :
On the filter paper will be seen a band of colours, of
• It is employed in milk dairies to separate cream from various constituents.
the milk.
• A filter paper with separated bands of various
• It is employed in diagnostic laboratories in testing constituents of a coloured substance is called
urine samples.
chromatogram.
• It is employed in blood banks to separate different
constituents of blood.
• It is used in drying machines to squeeze out water
from the wet clothes.
(c) By Chromatography :
The process of separation of different dissolved
constituents of a mixture by adsorbing them over an
appropriate adsorbent material is called
chromatography.
The adsorbent medium is generally magnesium
oxide, alumina or filter paper. The solvent generally
used for dissolving a mixture of two or more
constituents is water or alcohol.
The different constituents of a mixture get adsorbed
differently on the same adsorbent material, because (iii) Advantages :
they have different rates of movement. The rate of
movement of each adsorbed material depends upon : • It can be carried out with a very small amount of
material.
• The relative solubility of the constituents of mixture in
a given solvent. • The substances under investigation do not get wasted
in chromatographic separation.
• The relative affinity of the constituents of mixture for
the adsorbent medium. (iv) Applications :
If a filter paper is used as an adsorbent material for • It is used to separate colours from dye.
the separation of various constituents of a mixture, • It is used in the separation of amino acids.
then this method of separation of mixture is called • It is used in the separation of sugar from urine.
paper chromatography. • It is used in the separation of drugs from the samples
Paper chromatography is very useful in separating of blood.
various constituents of coloured solutes present in a
mixture of lime, ink, dyes etc. (C) Separation of mixture of two liquids :

 Note : (a) By Distillation:


Kroma means colour in Greek language and
Distillation is the process of heating a liquid to form
technique of chromatography was first applied for the
vapour and then cooling the vapour to get the back
separation of colours, so this name was given.
(i) Separation of coloured constituents present in a liquid.
mixture of ink and water. Distillation can be represented as :

(ii) Method : Heating


Liquid Vapour
• Take a filter paper 22 cm long, 5 cm broad and stick Cooling
its smaller end to a glass rod with the help of gum.  Note :
On the other end, measure a distance of 2 cm from
lower end and mark a small point. On this point pour The liquid obtained by condensing the vapour in the
one or two drop of the ink. process of distillation is called DISTILLATE .

PAGE # 83
Pre-foundation Career Care Programmes (PCCP) Division
Simple distillation : Process of simple distillation is (A) Method :
used to recover both salt as well as water , from a • The process of fractional distillation is similar to the
salt-water mixture (or salt solution) and to separate process of distillation, except that a fractionating
of components of a mixture containing two miscible column is attached.
liquids that boil without decomposition and have
sufficient difference in their boiling points. • The design of a fractionating column is such that the
vapours of one liquid (with a higher boiling point) are
(i) Liebig condenser : Liebig condenser is a water
preferentially condensed as compared to the vapours
condenser. It is a long glass tube surrounded by a
of the other liquid (with lower boiling point).
wider glass tube (called water jacket) having an inlet
and outlet for water. During distillation, cold water from
tap is circulated through the outer tube of condenser.
This water takes away heat from the hot vapour
passing through the inner tube of condenser and
causes its condensation.

DIFFERENT TYPES OF FRACTIONATING COLUMNS


• Thus, the vapours of the liquid with low boiling point,
pass on to the Liebig’s condenser where they
condense. The liquid so formed is collected in
receiver.
• The thermometer shows a constant reading as long
as the vapour of one liquid are passing to Liebig’s
condenser. As soon as the temperature starts rising,
SIMPLE DISTILLATION the receiver is replaced by another receiver to collect
second liquid.
(ii) Fractional distillation : Separation of mixture of
two miscible liquids for which the difference in the
boiling points is less : In case of two liquids which
have very close boiling points, both the liquids tend to
distil over in different proportions. It means lesser
the boiling point of a liquid, more is the proportion of
it distilling over.
The above problem can be avoided by using a
fractionating column. It gives the effect of repeated
distillation by offering resistance to the passage of (b) By Separating Funnel :
vapour.
(i) Separation of a mixture of two immiscible liquids :
The process of separation of two miscible liquids by
The separation of two immiscible liquids is based
the process of distillation, making use of their on the difference in their densities. The apparatus
difference in boiling points, is called fractional used for separation is separating funnel. It is a long
distillation. glass tube provided with a tap at its bottom. The table
below shows different immiscible liquids which can
be separated by separating funnel.

Immiscible Heavier Lighter


Liquid-liquid Mixture Liquid Liquid

Benzene and water Water Benzene

Kerosene oil and water Water Kerosene oil

Turpentine oil and water Water Turpentine oil

Chloroform and water Chloroform Water

Mustard oil and water Water Mustard oil


 Note :
The process of fractional distillation is useful only, if (ii) Method :
the difference in the boiling points of the two miscible
• Close the tap of separating funnel and clamp it in a
liquids is less than 25ºC.
vertical position in an iron stand.

PAGE # 84
Pre-foundation Career Care Programmes (PCCP) Division
• Pour the immiscible liquid mixture (say benzene-water (a) Purification of Air :
mixture) in the separating funnel. Allow the mixture to
(i) Air generally contains carbon dioxide gas, hydrogen
stand for half an hour or more. sulphide gas and sulphur dioxide gas as impurities.
• The immiscible components of the mixture, i.e., In addition to it there are dust particles also .
benzene and water separate out into two distinct (ii) First of all air is washed by passing it through
layers. The benzene forms the lighter layer on the top water, where the dust particles are removed.
and the water forms the heavier layer at the bottom. (iii) The washed air is passed through dilute caustic
• Place a conical flask or a beaker under the nozzle of soda solution, where the gases like carbon dioxide,
the separating funnel. Turn the tap gently so that the sulphur dioxide and hydrogen sulphide are removed.
water trickles in the flask or the beaker drop by drop. (iv) The purified air, however, contains moisture. The
Once the water is drained out, close the tap. moist air is passed through pipes, maintained at a
temperature below – 20º C, where water vapour
• Now place another conical flask or a beaker under present in it freezes and hence, air becomes dry.
the nozzle of separating funnel. Open the tap to drain
(v) The air leaving the cooling pipes is free from all
out benzene.
impurities.
(b) Liquefaction of Air :
(i) The cool air, free from all impurities is compressed
to a pressure 200 times more than the atmospheric
pressure. The compression raises the temperature
of the air.
(ii) The hot compressed air is then passed through
cooling tank in which cold water enters from one end
and warm water leaves from the other end.
(iii) The compressed and cooled air is passed through
a spiral pipe, placed in a vacuum flask. The end of
Separation by separating funnel spiral pipe is provided with a fine jet.
(iii) Applications : (iv) W hen compressed air suddenly escapes from
• This method is used for separating any two the jet, its pressure suddenly falls. Thus, its molecules
move wide apart. W hen the molecules move wide
immiscible liquids.
apart, they need energy. This energy is taken by the
• This method is used in separation of slag (a waste molecules from themselves and hence, their
material) from the molten metals during their temperature drops.
extraction. For example, during the extraction of iron
(v) The air so cooled, is now at a pressure equal to
from its ore, the molten iron and slag collect at the that of atmosphere. This cooled air rises up and in
base of blast furnace. The slag being less dense the process further cools the incoming compressed
floats up the surface of molten iron. They are drained air in spiral tube. The air is then sucked again by the
out from two different outlets. compression pump and the cycle is repeated. With
every cycle, the temperature of air drops, till it liquefies.
SEPARATION OF GASES FROM AIR (c) Fractional Distillation of Air :
In order to separate the major components of air, it is (i) The liquid air mainly consists of nitrogen and oxygen,
first purified, then liquefied and finally fractionally and is at a temperature of – 200º C.
distilled. The steps involved in the process are as (ii) The boiling point of liquid nitrogen is – 195º C and
follows - that of liquid oxygen is – 183º C.

PAGE # 85
Pre-foundation Career Care Programmes (PCCP) Division
(iii) The liquid is gradually warmed to – 195º C, when Example : Ice melts to form water. In this example
nitrogen starts boiling off from the liquid air. The only the appearance (state) of matter has changed
nitrogen gas so formed, is compressed and filled in from solid to liquid. However, the composition of the
molecules of ice or water remains same, i.e., for every
steel cylinders.
1 g of hydrogen there is 8 g of oxygen required . Thus,
(iv) The liquefied oxygen left behind, is also changed only a physical change has occurred.
to gas and then filled in compressed state in steel (ii) The change is temporary and reversible : It
cylinders. means the change can be reversed by altering the
causes which produce the change.
Example : The water formed from ice can be changed
PHYSICAL AND CHEMICAL CHANGES
back to ice by placing it in a freezing mixture (a mixture
Some kind of change always takes place in the matter of ice and common salt).
when it is subjected to energy changes. Almost all  Note :
the changes (except nuclear changes) taking place On altering the experimental conditions, the change
in the matter can be classified under two headings, which gets reversed, is a physical change.
these are as follows - (iii) There is no net gain or loss of energy : The
(a) Physical Changes : amount of energy required to bring about a physical
change is generally equal to the amount of energy
Definition : A change which alters some specific required to reverse the change. Thus, there is no net
physical property of the matter, like its state, texture, energy change involved.
magnetic or electrical conditions or its colour, without Example : If 1 g of water at 100º C on changing into
causing any change in the composition of its steam at 100ºC needs 2260 J of heat energy, then 1
molecules, is called physical change, provided it gets g of steam at 100º C on changing into water at 100º
reversed, if the cause producing the change is C, gives out 2260 J of heat energy. Thus, the net energy
removed. change is zero.
(iv) There is no change in the weight of substance :
Following points need special consideration : During a physical change it is only the energy which
(i) No new or different product is formed : The is added or removed. No matter is added during a
composition of molecules of the substance remains physical change. Similarly, no matter is removed
during a physical change. Therefore, mass of the
unaltered.
substance remains same.
SOME EXAMPLES INVOLVING PHYSICAL CHANGES :

Physical Change Observation Change in Physical Property

1. Switching on an electric The bulb glows and gives The physical appearance of
bulb out heat and light energy. the bulb changes.

2. Rubbing a permanent The steel rod gets magnetised. The steel rod acquires the
magnet on a steel rod. If it is brought near iron nails, property of attracting pieces
they get attracted. of iron.

3. Action of heat on iodine The brownish grey crystals of Change in state and colour.
iodine change to form violet
vapours. On cooling the vapours
condense on cooler parts of the
test tube to form crystals.

4. Dissolving of common The white crystalline salt Change of state.


salt in water. disappears in water. However, the
water tastes exactly like common
salt. Moreover, common salt can
be recovered by evaporation.

Some Common Examples of Physical Changes : (b) Chemical Change :


• Formation of dew. Definition : A change which alters the specific
• Evaporation of water. properties of a material by bringing about a change
• Crystallisation of sugar from its solution. in its molecular composition, followed by a change in
state, is called a chemical change.
• Ringing of an electric bell.
• Breaking of a glass pane. Following points need special consideration :
• Freezing of ice cream. (i) A chemical change results in the formation of
• A rock rolling down a hill. one or more new products : The products formed
• Bending of a glass tube by heating. have different properties than the original substance.
• Melting of wax. Thus, the composition of the molecules of products
is different from the original substance.
• Sublimation of camphor.

PAGE # 86
Pre-foundation Career Care Programmes (PCCP) Division
Example : Heating of sugar and add to it the weight of carbon, then total weight
When sugar is gently heated in a test tube, it melts. It will be equal to the weight of sugar crystals. Thus,
gradually changes to brown colour, giving a large strictly speaking, total weight of substances taking
amount of steamy fumes. In the end a black mass is part in a chemical change remains constant.
left which consists of carbon. Thus, new substances,
(iii) The chemical change is permanent and
viz. carbon and water (steam), are formed. In this
irreversible : It means the change will not reverse by
change, the arrangement between the molecules of
altering the experimental conditions.
carbon, hydrogen and oxygen breaks. The hydrogen
and oxygen atoms separate from carbon atoms and Example : The sugar, which has decomposed on
join together to form water. The carbon atoms are set heating to form carbon and steam will not change to
free and are left as black residue. sugar on cooling.

Sugar heat
 Carbon  Steam (iv) During chemical change energy is either
absorbed or given out : The various atoms in a
(ii) The weight of the substance undergoing
chemical compound are joined by attractive forces
chemical change usually changes :
commonly called bonds. The making or breaking of
Example : During the heating of sugar, the weight of the bonds always requires exchange of energy. Thus,
the black residue is far less than the actual weight of some amount of heat is either absorbed or given out
the sugar. However, this is an apparent change in during a chemical change.
weight. If we take the weight of steam into account

SOME EXAMPLES INVOLVING CHEMICAL CHANGES :

Chemical Change Observation Equation

1. Burning of magnesium When a magnesium ribbon is Magnesium + Oxygen


in air heated in a flame of Bunsen Magnesium oxide
burner, it catches fire and burns
with a dazzling white flame to form
white ash.

2. Rusting of iron When iron (silver grey) is left


exposed to moist air for a few Iron + Oxygen+ Water vapour
days, reddish brown powdery Rust
mass (rust) is found on its
surface

3. Burning of LPG When LPG (Liquefied Petroleum Butane (LPG) + Oxygen


Gas) is burnt, it burns with a pale Carbon dioxide + Water
blue flame and liberates colourless
gas carbon dioxide along with steam.

SOME COMMON EXAMPLES OF CHEMICAL CHANGES


:• Burning of wood or charcoal • Burning of candle • Digestion of food • Curdling of milk

• Formation of biogas (Gobar gas) • Burning of petrol or diesel

• Smoking of cigarette • Drying of paint • Rusting of iron

• Ripening of fruit •Clotting of blood • Fading of the colour of a dyed cloth

• Baking of cake • Photosynthesis • Formation of wine • Butter turning rancid

• Electrolysis of water into hydrogen and oxygen • Formation of water from hydrogen and oxygen

PAGE # 87
Pre-foundation Career Care Programmes (PCCP) Division
(C) DIFFERENCE BETWEEN PHYSICAL AND CHEMICAL CHANGES

PAGE # 88
Pre-foundation Career Care Programmes (PCCP) Division
11. W hat sublimate will be obtained when a mixture of
EXERCISE-1 sand, sulphur, common salt and iodine is sublimed ?
(A) Sand (B) Iodine
P HYSI CA L NATU RE OF MATT ER , STAT ES OF (C) Sulphur (D) Common salt
MATT ER & TH EI R IN TE R CONVER SI ON 12. Purity of organic liquid can be checked by its
1. The quantity of matter present in an object is called its - characteristic -
(A) weight (B) volume (A) boiling point (B) volume
(C) mass (D) density (C) solubility in water (D) solubility in alcohol
2. Which of the following statements is/are correct ?
13. Which of the following statements is/are correct ?
(A) Interparticle spaces are maximum in the gaseous
(A) Intermolecular forces of attraction in solids are
state of a substance .
maximum.
(B) Particles which constitute gas follow a zig-zag
path. (B) Intermolecular forces of attraction in gases are
(C) Solid state is the most compact state of substance. minimum.
(D) All are correct (C) Intermolecular spaces in solids are minimum.
(D) All of the above
3. In sublimation process -
(A) solid changes into liquid. 14. A liquid disturbed by stirring comes to rest after
(B) liquid changes into vapour. sometime due to its property of -
(C) solid changes directly into vapour. (A) Compressibility (B) Diffusion
(D) None of these
(C) Viscosity (D) All of these
4. During evaporation of liquid -
15. W hich of the following statements regarding
(A) the temperature of the liquid falls.
melting point and freezing point of a substance is
(B) the temperature of the liquid rises.
(C) the temperature of the liquid remains unchanged. true ?
(D) all statements are wrong. (A) Melting point of a substance is more than its
5. When common salt is added is ice - freezing point.
(A) its melting point decreases. (B) Melting point of a substance is less than its freez-
(B) its melting point increases. ing point.
(C) its melting point does not change from 0ºC (C) Melting point and freezing point of a substance
(D) ice becomes harder.
are same numerically.
6. Which of the following statements is false ? (D) None of these.
(A) Melting and freezing point of a substance are the
same. 16. Which of the following conditions is most favourable
(B) Evaporation of liquid takes place only at its boiling for converting a gas into liquid ?
point. (A) High pressure, low temperature
(C) Pure water has no taste (B) Low pressure, low temperature
(D) Water allows sunlight to pass through it.
(C) Low pressure, high temperature
7. The water boils when : (D) High pressure, high temperature
(A) Saturated vapour pressure of water becomes
equal to the atmospheric pressure ELEMENTS, COMPOUNDS, MIXTURES &
(B) Boiling point of water becomes more than SOLUT IONS
atmospheric pressure
(C) Saturated vapour pressure of water is less than 17. W hich of the following statement is not true about
atmospheric pressure colloidal solution ?
(D) Vapour pressure of water becomes more than (A) These are visible under powerful microscope.
atmospheric pressure (B) Their particles do not settle down with passage
8. The temperature remain same during melting, while of time.
all the ice changes into water due to the : (C) Their particles are electrically charged.
(A) latent heat of fusion. (D) These are homogeneous in nature.
(B) latent heat of vapourisation. 18. Some matter and their groups are given in column P
(C) latent heat of evaporation.
and Q respectively.
(D) latent heat of sublimation.
(P) (Q)
9. Fusion is the process of conversion of - a. Air i. Element
(A) liquid into gas. (B) solid into gas. b. O2 ii. Mixture
(C) solid into liquid. (D) liquid into solid. c. Copper sulphate iii. Base
d. Sodium hydroxide iv. Salt
10. A thermometer is inserted into a beaker filled with ice
at 0ºC. The beaker is heated slowly. The temperature The correct option is -
does not rise for some time. This is because - (A) a-ii, b-iv, c-i, d-iii (B) a-iv, b-iii, c-ii, d-i
(A) ice is very cold (C) a-i, b-ii, c-iii, d-iv (D) a-ii , b-i, c-iv, d-iii
(B) heat was used for changing ice at 0ºC to water at 19. Solubility of a gas in a liquid increases on -
0ºC (A) increasing temperature.
(C) the density of water is more than ice (B) decreasing pressure.
(D) the density of water is less than the ice (C) increasing pressure.
(D) increasing temperature and pressure.

PAGE # 89
Pre-foundation Career Care Programmes (PCCP) Division
20. Carbon tetra chloride and benzene are - 29. Carbon burns in oxygen to form carbon dioxide. The
(A) immiscible liquid (B) miscible liquid properties of carbon dioxide are -
(C) both ( and ) (D) None of these (A) similar to carbon (B) similar to oxygen
(C) totally different from both carbon and oxygen
21. A pure substance can only be -
(D) much similar to both carbon and oxygen
(A) a compound (B) an element
(C) an element or a compound 30. Separation of cream from milk is done by :
(D) a heterogeneous mixture (A) filtration (B) centrifugation method
22. Which of the following statements is not true about (C) evaporation (D) boiling
suspension ?
(A) The particles of suspension can be separated EXERCISE-2
from solvent by the process of filtration.
(B) W hen the suspension is kept undisturbed then
1. Sudden decrease in the intermolecular forces of
the particles of suspension settle down.
(C) A suspension is homogeneous in nature. attraction occurs most efficiently in
(D) Scattering of particles take place in suspension. [IJSO-Stage-I/2011-12]
(A) evaporation (B) melting
23. In which of the following, dispersed phase is a liquid
and dispersion medium is a gas ? (C) condensation (D) sublimation
(A) Cloud (B) Smoke 2. Mixture of ethyl alcohol and water can be easily
(C) Gel (D) Soap bubble separated by using
[IJSO-Stage-I/2011-12]
SE PA RATI ON T ECHN IQUE S
(A) separating funnel
24. Which of the following method is used for separation (B) fractional distillation.
of different components of petroleum? (C) filter paper
(A) Fractional distillation (B) Sublimation
(D) None of the above
(C) Chromatography (D) Simple distillation
3. Charring of sugar in concentrated sulphuric acid
25. Which of the following is a chemical change ? is due to [IJSO-Stage-I/2012-13]
(A) Melting of Wax (A) Oxidation of sugar
(B) Dissolving sugar in water (B) Reduction of sugar
(C) Beating aluminium to make aluminium foil (C) Hydrolysis of sugar
(D) Burning of Coal
(D) Dehydration of sugar

26. The gas you use in kitchen is called liquefied 4. Cheese is a colloidal system of
petroleum gas (LPG). In the cylinder, it exists as a [IJSO-Stage-I/2012-13]
liquid. When it comes out of the cylinder, it becomes
(A) Gas in solid (B) Gas in liquid
a gas (process A), then it burns (process B). Choose
(C) Liquid in gas (D) Liquid in solid
the correct statement.
(A) Process A is a chemical change.
5. Green house effect is related to
(B)Process B is a chemical change
[IJSO-Stage-I/2012-13]
(C) Both processes A and B are chemical changes.
(A) Ozone layer depletion
(D) None of these processes is a chemical change. (B) Carbon dioxide emission and absorption (C)
27. Crystallization is considered better than evaporation Nitrogen radiation
for obtaining pure crystal of sugar because on (D) Oxygen radiation
heating- 6. The method that cannot be used for removing per-
(A) Sugar sublimes. manent hardness of water is
[IJSO-Stage-I/2013-14]
(B) Sugar particles will evaporate.
(A) adding sodium carbonate
(C) Sugar particles will decompose.
(B) distillation
(D) Sugar particles will melt. (C) adding caustic soda
28. The principle behind fractional distillation technique (D) boiling
in separation of two liquids is -
(A) difference in melting point
(B) difference in boiling point
(C) difference in concentration
(D) difference in solubility

PAGE # 90
Pre-foundation Career Care Programmes (PCCP) Division
7. W hich properties of plastics make their
disposal difficult : (I) PVC produces harmful com-
bustion products ; (II) polyalkenes are highly flam-
mable ; (III) polyalkenes are non-biodegradable
[IJSO-Stage-I/2013-14]

(A) I and II only (B) I and III only


(C) II and III only (D) I, II, III

8. Melting point of a substance is 10°C. What does


this mean? [IJSO-Stage-I/2016-17]
(A) The substance is a liquid at 10°C.
(B) The substance is a solid at 10°C.
(C) There is an equilibrium between solid phase
and liquid phase at 10°C.
(D) The substance is 50% solid and 50% liquid at
10°C.

PAGE # 91
Pre-foundation Career Care Programmes (PCCP) Division
ACIDS, BASES AND SALTS
Switch on the current and bulb will start glowing. This
ELECTROLYTE
shows that the electric current has passed through
An electrolyte is defined as a compound whose the base solution. As the current is carried by the
aqueous solution or melt conducts electricity. On the movement of ions, this shows that in solution NaOH
other hand, a compound whose aqueous solution or has ionised to give Na+ and OH – ions. Current will
melt does not conduct electricity is called a non- also be in a position to pass if the beaker contains in
electrolyte. it dilute H 2SO 4 (H + ions are released in aqueous
e.g. Aqueous solution of sugar, urea etc. do not conduct solution). Similarly, aqueous solutions containing HCI
electricity. Hence they are non- electrolytes whereas will also be conducting due to release of H+ and Cl–
solutions of inorganic acids , bases and salts conduct ions.
electricity. Hence they are electrolytes. Battery Bulb in
(a) Types of electrolyte : Electrolytes are of two types circuit

Switch
(i) Weak electrolyte : A weak electrolyte is defined as
a substance which dissociates to a small extent in
aqueous solution and hence, conducts electricity to
a small extent. Dilute NaOH
e.g. NH4OH,CH3COOH etc. Iron Nails
(ii) Strong electrolyte : A strong electrolyte is defined Rubber cork
as a substance which is completely ionized in its
aqueous solution and hence is a very good conductor Bulb will not glow if glucose (C6H12O6) or ethyl alcohol
of electricity. (C2H5OH) solution is kept in the beaker. This means
e.g. NaOH, KOH,HCl, H2SO4, NaCl, KNO3 etc. that both of them will not give any ions in solution.

In case of weak electrolyte, as they are partly ionised Activity : Strong & weak electrolyte
in the aqueous solution, an equilibrium is setup Set up the apparatus as shown in figure. Take 200 ml
between the ions and the unionised electrolyte. 0.1 M HCl and complete the circuit by putting the key
Hence, their equilibrium is represented by putting (K). Note down the deflection produced in the
double arrow () in between ionised and unionised ammeter. Now repeat the experiment by taking 200
form. ml of 0.1 M CH3COOH. You will observe that deflection
e.g. CH3COOH + H2O  CH3COO– (aq) + H3O+ shown by ammeter in case of 0.1 M HCl is more than
In general, the ionisation of a weak electrolyte, AB, is that shown in the case of 0.1 M CH 3COOH. Large
represented as follows : deflection means more flow of current and hence the
AB(s) + H2O  A+(aq) + B– (aq) presence of large number of hydrogen ions. In other
Such an equilibrium between the ions and the words, same concentration of hydrochloric acid
undissociated electrolyte is called ionic equilibrium produces more H +(aq) ions as compared to acetic
acid. This is because hydrochloric acid ionises
Conducting Nature of Acid and Base Solution
completely in water while acetic acid ionises partially
Acids are the substances which contain one or more
in solution.
hydrogen atoms in their molecules which they can
release in water as H+ ions. Similarly, bases are the
substances which contain one or more hydroxyl
groups in their molecules which they can release in
water as OH¯ ions . Since the ions are the carrier of
charge therefore, the aqueous solutions of both acids
and bases are conductors of electricity.

Experiment : In a glass beaker, take a dilute solution of


NaOH. Fix two small nails of iron in a rubber cork in
the beaker as shown in the figure. Connect the nails
to the terminals of a 6 volt battery through a bulb.

PAGE # 92
Acid solution in water conducts electricity (A) Organic acids
HCl (aq)  H+ (aq) + Cl–(aq) The acids which are usually obtained from organisms
CH3COOH (aq) CH3COO–(aq) + H+(aq) are known as organic acids. Oxalic acid [(COOH) 2 ],
It may be noted that single arrow () is used to acetic acid (CH 3 COOH) etc. are very common
represent complete ionization, while double half examples of organic acids. Some other organic acids
headed arrow is used to represent partial ionization. with their natural sources are given in the following
Table.
ACIDS
Some Organic Acids with Their Natural Sources
Substances with sour taste are regarded as acids.
Lemon juice, vinegar, grape fruit juice and spoilt milk
etc. taste sour since they are acidic. Many substances
S.No. Organic acid Natural sources
can be identified as acids based on their taste but 1 Acetic acid Micro-organism
some of the acids like sulphuric acid have very strong 2 Citric acid Citrus fruits (like
action on the skin which means that they are corrosive orange and lemon)
in nature. In such cases it would be according to
3 Butyric acid Rancid butter
modern definition -
An acid may be defined as a substance which releases 4 Formic acid Sting of bees and ants
one or more H+ ions in aqueous solution. 5 Lactic acid Sour milk
Acids are mostly obtained from natural sources.
6 Malic acid Apples
BASES 7 Oleic acid Olive oil
Substances with bitter taste and soapy touch are 8 Stearic acid Fats
regarded as bases. Since many bases like sodium 9 Amino acid Proteins
hydroxide and potassium hydroxide have corrosive 10 Uric acid Urine
action on the skin and can even harm the body, so 11 Tartaric acid Tamarind
according to the modern definition - 12 Oxalic acid Tomatoes
A base may be defined as a substance capable of
It may be noted that all organic acids contain carbon
releasing one or more OH¯ ions in aqueous solution. as one of their constituting elements. These are weak
acids and, therefore, do not ionise completely in their
(a) Characteristics of a Base :
aqueous solutions. Since these acids do not ionise
(i) A base changes red litmus to blue. completely in their aqueous solutions, therefore, their
(ii) A base reacts with an acid so that a salt and water solutions contains both ions as well as undissociated
are formed. molecules. For example, formic acid’s aqueous
solution contains H 3 O + , HCOO – as well as
Base + Acid  Salt + Water
undissociated HCOOH molecules.
(iii) A base combines with carbon dioxide so that a HCOOH + H2O H3O+ + HCOO–
carbonate is formed. Formic acid Hydronium ion Formate ion
(iv) A base is slippery like soap. It tastes unpleasant
(B) Inorganic Acids.
and bitter.
The acids which are usually obtained from minerals
Distinction Between an Alkali and a Base : are known as inorganic acids. Since the acids are
It may be kept in mind that a base which is soluble in obtained from minerals, therefore, these acids are also
called mineral acids. Some common examples of
water is called an alkali. On the other hand those bases
inorganic acids are : Hydrochloric acid (HCl), Sulphuric
which are not soluble in water are termed as bases
acid (H2SO4), Nitric acid (HNO3) etc.
only not alkalis. This means that all alkalis are bases It may be pointed out that except carbonic acid (H2CO3),
but all bases are not alkalis. For example, ferric these acids do not contain carbon. Acids like HCl,
hydroxide [Fe(OH)3] and cupric hydroxide [Cu(OH)2] are H 2 SO 4 and HNO 3 are strong acids which ionise
bases but not termed as alkalis because they are completely in their aqueous solutions and, therefore,
insoluble in water while NaOH and KOH are an their aqueous solutions do not contain any
undissociated molecules.
examples of alkalis since these are soluble in water.
(ii) Classification of acids on the basis of their Basicity :
(a) Classification of Acids :
The basicity of an acid is defined as the number of
(i) Classification of acids on the basis of their Source hydronium ions [H3O+ (aq.)] that can be produced by
On the basis of their source, acids can be classified in the complete ionisation of one molecule of that acid in
two categories : aqueous solution.
(A) Organic acids (B) Inorganic acids For example, basicity of HCl, H2SO4, H3PO4 is 1, 2 and
3 respectively because one molecule of these acids,

PAGE # 93
on ionisation, produces 1, 2 and 3 hydronium ions in (SO42–) in the second step.
aqueous solution respectively. H2SO4 + H2O H3O+ + HSO4–
It may be pointed out here that the basicity of an acid is Sulphuric acid Bisulphate ion

determined by number of hydronium ions produced HSO4 + H2O H3O + SO42–
+

per molecule of an acid on ionisation and not the Sulphate ion


number of hydrogen atoms present in one molecule \ (ii) Because of the presence of two replaceable
of an acid. For example, basicity of acetic acid hydrogen ions, a dibasic acid forms two series of salts
(CH3COOH) is 1 because one molecule of acetic acid, i.e., an acid salt and a normal salt. For example, H2SO4
on ionisation in aqueous solution, produces one reacts with NaOH to form NaHSO4 (an acid salt) and
hydronium ion although one molecule of acetic acid Na2SO 4 (a normal salt)
contains four hydrogen atoms. NaOH + H2SO4 NaHSO4 + H2O
Sodium hydrogen
+ – sulphate
CH3COOH + H2 O H3O + CH3COO
(An acid salt)
Acetic acid Hydronium ion Acetate ion
On the basis of basicity, the acids can be classified as 2NaOH + H2SO4 Na2SO4 + 2H2O
under : Sodium sulphate
(Normal salt)
(A) Monobasic Acids : (C) Tribasic Acids :
When one molecule of an acid on complete ionisation
produces one hydronium ion (H 3 O + ) in aqueous When one molecule of an acid on complete ionisation
solution, the acid is said to be a monobasic acid. produces three hydronium ions (H 3 O +) in aqueous
solution, the acid is said to be a tribasic acid.
Examples of Monobasic Acids. An example of tribasic acids is Phosphoric acid
Some examples of monobasic acids are : (H3PO4), citric acid.
(i) Hydrochloric acid (HCl) (D) Tetrabasic Acids :
(ii) Hydrobromic acid (HBr)
(iii) Nitric acid (HNO3) When one molecule of an acid on complete ionisation
(iv) Acetic acid (CH3COOH) produces four hydronium ions (H 3 O + ) in aqueous
(v) Formic acid (HCOOH) solution, the acid is said to be a tetrabasic acid.
An example of tetrabasic acids is silicic acid (H4SiO4),
Characteristics of Monobasic Acids. Pyrophosphoric acid ( H4P2O7)
Two important characteristics of monobasic acids
(iii) Classification of acids on the basis of their
are : strength :
(i) A monobasic acid ionises in one step in aqueous (A) Strong Acids :
solution. For example, The acids which undergo almost complete ionisation
HCl + H2O H3O+ + Cl– in a dilute aqueous solution, thereby producing a high
concentration of hydronium ions (H3O+) are known as
(Single step ionisation)
strong acids.
(ii) A monobasic acid forms only single salt or a normal
Examples of strong acids :
salt. For example,
Some examples of strong acids are :
HCl + NaOH  NaCl + H2O (i) Hydrochloric acid (HCl)
Sodium chloride (ii) Sulphuric acid (H2SO4)
(Normal salt) (iii) Nitric acid (HNO3)
(B) Dibasic Acids :
All these three mineral acids are considered to be
When one molecule of an acid on complete ionisation strong acids because they ionise almost completely
produces two hydronium ions (H 3 O + ) in aqueous in their dilute aqueous solutions.
solution, the acid is said to be a dibasic acid.
(B) Weak Acids :
Examples of Dibasic Acids : The acids which undergoes partial or incomplete
Some examples of dibasic acids are : ionisation in a dilute aqueous solution, thereby
(i) Sulphuric acid (H2SO4) producing a low concentration of hydronium ions
(ii) Sulphurous acid (H2SO3) (H3O+) are known as weak acids.
(iii) Carbonic acid (H2CO3)
Examples of weak acids :
(iv) Oxalic acid (COOH)2
(v) Hydrofluoric acid (HF) Some examples of weak acids are :
(i) Acetic acid (CH3COOH)
Characteristics of Dibasic Acids : (ii) Formic acid (HCOOH)
Two important characteristics of dibasic acids are : (iii) Oxalic acid [(COOH)2]
(i) A dibasic acid ionises in two steps in aqueous (iv) Carbonic acid (H2CO3)
solution. For example, sulphuric acid which is a dibasic (v) Sulphurous acid (H2SO3)
acid ionises to produce bisulphateion (HSO4– ) in the (vi) Hydrogen sulphide (H2S)
first step which further ionises to produce sulphate ion (vii) Hydrocyanic acid (HCN)
The aqueous solution of weak acids contain both ions
as well as undissociated molecules.
PAGE # 94
(iv) Classification on the basis of Concentration of It must be mentioned here that concentration of an
the Acid : acid simply tells the amount of water in the acid. It may
not be confused with strength of an acid, which is a
By the term concentration, we mean the amount of
measure of concentration of hydronium ion it produces
water present in the given sample of acid solution in
in aqueous solution.
water.
A concentrated acid may not necessarily be a strong
(A) Concentrated Acid :
acid while a dilute acid may not necessarily be a weak
The sample of an acid which contains very small or no
acid. A strong acid will remain strong even if it is dilute
amount of water is called a concentrated acid.
because it produces a large concentration of hydronium
(B) Dilute Acid : ions in aqueous solution. On the other hand, a weak
The sample of an acid which contains far more amount acid will remain weak even when concentrated
of water than its own mass is known as a dilute acid because it will produce lesser concentration of
hydronium ions in aqueous solution.

CHEMICAL FORMULAE, TYPES AND USES OF SOME COMMON ACIDS

Name Type Chemical Formula W here found or used

Carbonic acid Mineral acid H 2 CO 3 In soft drinks and lends fizz.

Used in the manufacture of explosives (TNT,


Nitric acid Mineral acid HNO 3 Nitroglycerine) and fertilizers (Amm onium nitrate,
Calcium nitrate, Purification of Au, Ag)

In purification of com mon salt, in textile industry


Hydrochloric acid Mineral acid HCl as bleaching agent, to m ake aqua regia, in
stomach as gastric juice, used in tanning industry

Comm only used in car batteries, in


the manufacture of fertilizers (Am monium
Sulphuric acid Mineral acid H 2SO 4 sulphate, super phosphate) detergents etc, in
paints, plastics, drugs, in m anufacture of artificial
silk, in petroleum refining.
Phosphoric acid Mineral acid H 3PO 4 Used in antirust paints and in fertilizers.
Found in the stings of ants and bees, used in
Formic acid Organic acid HCOOH
tanning leather, in m edicines for treating gout.
Found in vinegar, used as solvent in the
Acetic acid Organic acid CH 3 COOH
manufacture of dyes and perfumes.
Lactic acid Organic acid CH 3 CH(OH)COOH Responsible for souring of milk in curd.
Benzoic acid Organic acid C 6 H 5COOH Used as a food preservative.
Citric acid Organic acid C 6H 8O 7 Present in lemons, oranges and citrus fruits.
Tartaric acid Organic acid C 4H 6O 6 Present in tamarind.

(b) Classification of Bases or Alkalis :

Classification of bases or alkalis can be done in different ways as given below :

(i) Classification on the basis of their strength

(ii) Classification on the basis of their concentration

(iii) Classification on the basis of their acidity.

(i) Classification of the Bases or Alkalis on the Basis of their Strength

(A) Strong alkalis or bases :


The alkalis or bases which undergo almost complete ionisation in aqueous solution to produce high concentration
of hydroxyl (OH–) ions are known as strong alkalis or strong bases.
Example of strong alkalis or bases.
Some example of strong alkalis or bases are : Sodium hydroxide (NaOH), Potassium hydroxide (KOH) and Barium
hydroxide [Ba (OH)2] etc.

PAGE # 95
NaOH ( aq )  Na  ( aq )  OH  ( aq )  When one molecule of the base on complete ionisation

 
KOH ( aq )  K (aq )  OH (aq ) produces one hydroxyl (OH–) ion in aqueous solution,
 ( Almost completely
2   ionised ) the base or alkali is said to be monoacidic
Ba ( OH ) 2 ( aq )  Ba ( aq )  2 OH (aq )
 OR
A monoacidic base (or alkali) may be defined as one
(B) Weak alkalis or bases : whose one molecule reacts with one hydrogen (H +)
The alkalis or bases which undergo only partial ion completely to form salt and water as the only
ionisation in aqueous solution to produce a relatively products.
low concentration of hydroxyl (OH–) ions are known as
Examples of Monoacidic Bases (or alkalis) :
weak alkalis or weak bases.
Some examples of weak alkalis or bases are : Ammonium Sodium hydroxide NaOH, Potassium hydroxide KOH,
hydroxide NH 4 OH, Calcium hydroxide Ca (OH) 2 , Ammonium hydroxide NH 4 OH. All these substances
Magnesium hydroxide Mg (OH)2 etc. produce only one hydroxyl ion on complete ionisation
in aqueous solution.
NH4OH (aq) NH+4 (aq) + OH– (aq)
Ca (OH)2 (aq) Ca2+ (aq) + 2OH– (aq) NaOH(aq.)  Na+ (aq) + OH– (aq)
Mg (OH)2 (aq) Mg2+ (aq) + 2OH– (aq) KOH(aq.)  K+ (aq) + OH– (aq)

Since these alkalis are not ionising completely, The dissociation of monoacidic bases or alkalis takes
therefore, there is a dynamic equilibrium between the place in a single step.
undissolved alkali and the ions produced by it. (B) Diacidic Bases (or alkalis) :
(ii) Classification of Bases or Alkalis on the Basis of When one molecule of a base or alkali on complete
their Concentration : ionisation produces two hydroxyl (OH – ) ions in
By the term concentration, we mean the amount of aqueous solution, the base or alkali is said to be
water present in the given sample of alkali solution in diacidic.
water. On the basis of concentration, the alkalis can be OR
A diacidic base (or alkali) may be defined as one whose
classified as under :
one molecule reacts with two hydrogen (H +) ions
(A) Concentrated alkali : completely to form salt and water as the only products.
A solution of alkali having a relatively high percentage
Examples of Diacidic Bases
of alkali in its aqueous solution is known as
concentrated alkali. Calcium hydroxide Ca (OH)2 and magnesium hydroxide
Mg (OH)2
(B) Dilute alkali :
A solution of alkali having a relatively low percentage of Ca(OH)2(aq)  Ca2+ (aq) + 2OH– (aq)
alkali in its aqueous solution is known as a dilute Mg(OH)2(aq)  Mg2+ (aq) + 2OH– (aq)
alkali.
One molecule of both the bases are producing 2OH–
If the concentration of alkali in the solution is less than
ions in aqueous solution, therefore, these are termed
1 mole per litre, then it is considered to be a dilute
as diacidic bases .
alkali.
(C) Triacidic Bases :
(iii) Classification of Bases or Alkalis on the Basis of When one molecule of a base or alkali on complete
their Acidity : ionisation produces three hydroxyl (OH–) ions in aqueous
Before we discuss this classification, let us understand solution, the base or alkali is said to be triacidic base.
the meaning of the word ‘acidity’ of an alkali.
Examples of Triacidic Bases :
The number of hydroxyl (OH–) ions produced by one Aluminium hydroxide Al(OH)3, Ferric hydroxide Fe (OH)3
molecule of an alkali on complete dissociation in water
or the number of hydrogen ions (of an acid) with which Al (OH)3(aq)  Al3+ (aq) + 3OH– (aq)
a molecule of that alkali reacts to produce salt and Al (OH)3 + 3HCl (aq) AlCl3 + 3H2O
water only is known as acidity of an alkali.
In the above equations, one molecule of Al (OH)3 is
For water insoluble hydroxides, acidity of the base is producing three OH– ions and one molecule of Al (OH)3
equal to the number of OH– ions present in one molecule is reacting with three hydrogen (H+) ions to form salt
of that base. and water only, therefore, it is termed as a triacidic
base.
On the basis of acidity, the bases can be classified as
under :
(A) Monoacidic Bases (or alkalis) :

PAGE # 96
CHEMICAL FORMULAE, NAMES AND USES OF SOME react with acids to evolve CO2 gas and form salts.
COMMON BASES e.g.
CaCO3(s)+ 2HCl(aq)  CaCl2(aq) + H2O() + CO2(g)
Commerc Chemic Calcium Calcium
Name Uses
ial al carbonate chloride
In manufacture of soap, 2NaHCO3(s) + H2SO4(aq)  Na2SO4(aq) + 2H2O(aq) + 2CO2(g)
Sodium Caustic
NaOH paper, pulp, rayon, refining of Sodium Sodium
hydroxide soda
petroleum etc. bicarbonate sulphate
In alkaline storage batteries, (iv) Action with bases :
Potassium Caustic manufacture of soap, Acids react with bases to give salt and water.
KOH
hydroxide potash absorbing HCl (aq) + NaOH(aq)  NaCl + H2O
CO2 gas etc.
In manufacture of bleaching
Calcium Slaked BASES
Ca(OH)2 powder,
hydroxide lime
softening of hard water etc. (i) Action with metals :
As an antacid to remove Metals like zinc, tin and aluminium react with strong
Magnesium Milk of alkalies like NaOH (caustic soda), KOH (caustic
Mg(OH)2 acidity
hydroxide magnesia
from stomach. potash) to evolve hydrogen gas.

Aluminium As foaming agent in fire Zn(s) + 2NaOH(aq)  Na2ZnO2(aq) + H2(g)


– Al(OH)3 Sodium zincate
hydroxide extinguishers.

In removing grease stains from Sn(s) + 2NaOH(aq)  Na2SnO2(aq) + H2(g)


Ammonium Sodium stannite
– NH4OH clothes and in cleaning window
hydroxide
panes.
2Al(s)+ 2NaOH + 2H2O  2NaAlO2(aq) + 3H2(g)
Sodium meta
CHEMICAL PROPERTIES aluminate
Ac id s : (ii) Action with non-metallic oxides :
Acids react with metal oxides, but bases react with
(i) Action with metals :
oxides of non-metals to form salt and water.
Dilute acids like dilute HCl and dilute H2SO4 react with
e.g.
certain active metals to evolve hydrogen gas.
2NaOH(aq) + CO2(g)  Na2CO3(aq) + H2O()
2Na(s) + 2HCl (dilute)  2NaCl(aq) + H2(g) Ca(OH)2(s) + SO2(g)  CaSO3(s) + H2O()
Ca(OH)2(s) + CO2(g)  CaCO3(s) + H2O()
Mg(s) + H2SO4 (dilute)  MgSO4(aq) + H2(g)
Metals which can displace hydrogen from dilute acids
are known as active metals. e.g. Na, K, Zn, Fe, Ca, Mg
etc.
Zn(s) + H2SO4 (dilute)  ZnSO4(aq) + H2(g)
A c id s Ba s e s
The active metals which lie above hydrogen in the S o u r in ta s te . B itte r in ta s te .
activity series are electropositive and more reactive in
C h a n g e C o lo u rs o f
nature. Their atoms lose electrons to form positive ions
and these electrons are accepted by H + ions of the in d ic a to rs C h a n g e c o lo u rs o f
acid. As a result, H2 is evolved. e .g . litm u s tu rn s in d ic a to rs e .g . litm u s
e.g. fro m b lu e to tu rn s fro m re d to b lu e
Zn(s)  Zn2+ (aq) + 2e– re d , p h e n o lp h th a le in tu rn s
p h e n o lp h th a le in fro m c o lo u rle s s to p in k .
2H+(aq) + SO42– (aq) + 2e–  H2(g) + SO42–(aq)
re m a in s c o lo u rle s s .
Zn(s) + 2H+(aq)  Zn++(aq) + H2(g)
S h o w e le c tro lytic
S h o w e le c tro lytic
(ii) Action with metal oxides : c o n d u c tivity
c o n d u c tivity in
Acids react with metal oxides to form salt and water. in a q u e o u s
a q u e o u s s o lu tio n
These reactions are mostly carried out upon heating. s o lu tio n .
e.g.
Ac id ic p ro p e rtie s
ZnO(s) + 2HCl (aq)  ZnCl2(aq) + H2O() B a s ic p ro p e rtie s
d is a p p e a r
d is a p p e a r w h e n
MgO(s) + H2SO4(aq)  MgSO4(aq) + H2O() w h e n re a c t w ith
re a c t w ith a c id s
bas es
(N e u tra liza tio n )
CuO(s) + 2HCl(aq.)  CuCl2(aq) + H2O() (N e u tra liza tio n )
(Black) (Bluish green)
Ac id s d e c o m p o s e N o d e c o m p o s itio n o f
(iii) Action with metal carbonates and metal
c a rb o n a te c a rb o n a te
bicarbonates : Both metal carbonates and bicarbonates
s a lts . s a lts b y b a s e s

PAGE # 97
All the alkaline solution have a pH of more than 7.
PH & PH SCALE
So, whenever a solution has a pH of more than 7, it will
If an aqueous solution has equal concentrations of be alkaline in nature and it will turn red litmus blue and
hydrogen ions and hydroxide ions in it, it is neither colourless phenolphthalein solution pink.
acidic nor basic, it is said to be neutral. Now, if an Note : pH is a pure number. It has no unit.
aqueous solution has more number of hydrogen ions
(and less number of hydroxide ions), it will be an acidic THE pH SCALE HAVING pH VALUES FROM 0 TO 14
solution. On the other hand, if an aqueous solution
has more number of hydroxide ions (and less number
of hydrogen ions), it will be basic (or alkaline) in nature.
So, we usually describe the acidic nature or basic
nature (alkaline nature) of aqueous solution in terms
of hydrogen ion concentration or hydroxide ion
concentration in it. It should be noted that an acid solution having low pH
In 1909, Sorensen devised a scale known as pH scale value is stronger than another solution having higher
on which the acidic nature as well as the basic nature pH value. For example, a solution having pH of 2 is a
of solutions can be expressed only by considering the stronger acid than another solution having a pH of 5. It
hydrogen ion concentrations in them. Now, the is just opposite in the case of alkaline solutions. An
hydrogen ion concentrations of most of the common alkaline solution having higher pH value is a stronger
aqueous solutions are in negative powers of 10. By alkali than another solution having low pH value. For
using the Sorensen's pH scale, the hydrogen ion example, a solution having a pH of 12 is a stronger
concentrations of solution having complicated negative alkali than a solution of pH 10.
powers of 10 are converted into simple positive figures
of pH values. The pH values of some of the common substances
For example, in neutral water at 25°C (298K) the from our everyday life are given in the following table.
concentration of hydrogen ions is 10–7 M and that of the
hydroxide ions is also 10–7 M. It is not convenient to Solution pH Solution pH
1. 1M Hydrochloric acid 0 14. Urine 5.5 - 7.5
use such negative powers, therefore a scale has been 2. Battery acid 0.5 15. Saliva 6.5 - 7.5
3. Dilute hydrochloric acid 1.0 16. Blood 7.3 - 7.5
found on which these negative powers of concentration 4. Gastric juices 1.0 - 3.0 17. Eggs 7.8
can be written as positive figures. This is known as pH (Digestive juices in stomach)
5. Lemon juice 2.2 - 2.4 18. Baking soda solution 8.4
scale. 6. Vinegar 2.4 - 3.4 19. Sea-water 8.5
7. Soft drinks 3.0 20. Washing soda solution 9.0
8. Wine 2.8 - 3.8 21. Lime-water 10.5
The pH of a solution is the logarithm (to the base 10) of 9. Oranges 3.6 22. Milk of magnesia 10.5
[Mg (OH)2 solution]
the reciprocal of its hydrogen ion concentration in moles
10. Tomato juice 4.0 - 4.4 23. 1 M Ammonium hydroxide 11.6
per litre. (Household ammonia)
11. Beer 4.0 - 5.0 24. Dilute sodium hydroxide 13
1 12. Coffee 4.5 - 5.5 25. 1 M Sodium hydroxide 14
log 13. Milk 6.5
pH = [H ]
APPLICATION OF PH IN OUR DAILY LIFE
or pH = - log [H+]
(i) pH in our digestive system : Dilute hydrochloric
For calculating the pH of a solution, we have to use its acid produced in our stomach helps in the digestion of
hydrogen ion concentration [H+] in moles per litre. In food. However, excess of acid causes indigestion and
other words, we have to use the molar concentration leads to pain as well as irritation. The pH of the
(M) of the hydrogen ions. By using the above formula, digestive system in the stomach will decrease. The
we can calculate the pH of a solution from its hydrogen excessive acid can be neutralised with the help of
ion concentration. And if we know the pH value, we can antacids which are recommended by the doctors.
find out the hydrogen ion concentration. W e will now Actually, these are group of compounds (basic in
calculate the pH values of pure water, acidic solutions nature) and have hardly any side effects. A very popular
and alkaline solutions (or basic solutions) by using antacid is 'Milk of Magnesia' which is insoluble
magnesium hydroxide. Aluminium hydroxide and
this formula.
sodium hydrogen carbonate can also be used for the
same purpose. These antacids will bring the pH of the
All the acidic solutions have a pH less than 7.
system back to its normal value. The pH of human
So, whenever a solution has a pH of less than 7, it will
blood varies between 7.36 to 7.42 . It is maintained by
be acidic in nature and it will turn blue litmus red as
the soluble bicarbonates and carbonic acid present in
well as methyl orange indicator will be turned red. Acidic
the blood.
solution do not change the colour of phenolphthalein .
(ii) pH change leads to tooth decay : The white enamel

PAGE # 98
coating on our teeth is of insoluble calcium phosphate strip when dipped in a base solution becomes blue.
which is quite hard. It is not affected by water. However, (b) Phenolphthalein :
when the pH in the mouth falls below 5.5 the enamel It is also an organic dye and acidic in nature. In neutral
gets corroded. Water will have a direct access to the or acidic solution, it remains colourless while in the
roots and decay of teeth will occur. The bacteria present basic solution, the colour of indicator changes to pink.
in the mouth break down the sugar that we eat in one
(c) Methyl Orange :
form or the other to acids, Lactic acid is one of these.
Methyl orange is an orange or yellow coloured dye and
The formation of these acids causes decrease in pH. basic in nature. In the acidic medium the colour of
It is therefore advisable to avoid eating sugary foods indicator becomes red and in the basic or neutral
and also to keep the mouth clean so that sugar and medium, its colour remains unchanged.
food particles may not be present. The tooth pastes
(d) Red Cabbage Juice :
contain in them some basic ingredients and they help It is purple in colour in neutral medium and turns red
in neutralising the effect of the acids and also or pink in the acidic medium. In the basic or alkaline
increasing the pH in the mouth. medium, its colour changes to green.
(e) Turmeric Juice :
(iii) Role of pH in curing stings by insects : The stings
of bees and ants contain methanoic acid (or formic It is yellow in colour and remains as such in the neutral
acid). When stung, they cause lots of pain and irritation. and acidic medium. In the basic medium its colour
becomes reddish or deep brown.
The cure is in rubbing the affected area with soap.
Sodium hydroxide present in the soap neutralises acid  Note :
injected in the body and thus brings the pH back to its Litmus is obtained from LICHEN plant.
original level bringing relief to the person who has been
stung. Similarly, the effect of stings by wasps containing
THEORIES OF ACIDS AND BASES
alkali is neutralised by the application of vinegar which
is dilute solution of ethanoic acid (or acetic acid) (a) Arrhenius Theory :
This concept was given in 1884 .
(iv) Soil pH and plant growth : The growth of plants in
According to this theory all substances which give H+
a particular soil is also related to its pH. Actually, different
ions when dissolved in water are called acids, while
plants prefer different pH range for their growth. It is
therefore, quite important to provide the soil with proper those which ionise in water to give OH– ions are called
bases.
pH for their healthy growth. Soils with high iron minerals
The main points of this theory are -
or with vegetation tend to become acidic. The soil pH
can reach as low as 4.The acidic effect can be (i) An acid or base when dissolved in water, splits into
neutralised by 'liming the soil' which is carried by ions. This is known as ionisation.
adding calcium hydroxide. These are basic in nature (ii) Upon dilution, the ions get separated from each other.
and have neutralising effect. Similarly, the soil with This is known as dissociation of ions.
excess of lime stone or chalk is usually alkaline. (iii) The fraction of the acid or base which dissociates
Sometimes, its pH reaches as high as 8.3 and is quite into ions is called its degree of dissociation and is
harmful for the plant growth. In order to reduce the denoted by alpha  which can be calculated by the
alkaline effect, it is better to add some decaying organic following formula :
matter (compost or manure). No. of molecules dissociate d at equilibriu m
=
The soil pH is also affected by the acid rain and the total no. of molecules
use of fertilizers. Therefore soil treatment is quite
(iv) The degree of dissociation depends upon the nature
essential.
of acid or base. Strong acids and bases are highly
dissociated, while weak acids and bases are
INDICATORS dissociated to lesser extent.
, An indicator indicates the nature of a particular solution (v) The electric current is carried by the movement of
whether acidic, basic or neutral. Apart from this, indicator ions. Greater the ionic mobility more will be the
also represents the change in nature of the solution conductivity of the acid or base.
from acidic to basic and vice versa. Indicators are (vi) The H + ions do not exist as such and exist in
basically coloured organic substances extracted from combination with molecules of H 2O as H 3O + ions
different plants. A few common acid base indicators are- (known as hydronium ion).
(a) Litmus : H+ + H2O H3O+
Litmus is a purple dye which is extracted from ‘lichen’
a plant belonging to variety Thallophyta. It can also be HCl + H2O H3O+ + Cl–
applied on paper in the form of strips and is available e.g.
as blue and red strips. A blue litmus strip, when dipped HA + H2O H3O+ + A¯
in an acid solution acquires red colour. Similarly a red Acid

PAGE # 99
H2SO4 + 2H2O 2H3O+ + SO4–2 conjugate acid.
Acid
Water
BOH B+ + OH¯
Base
Water
NaOH Na+ + OH¯
Base
Water Here CH3COO– ion is conjugate base of CH3COOH,
NH4OH NH4+ + OH¯
while H3O+ ion is conjugate acid of H2O.
Base
(i) Merits :
Limitations of Arrhenius theory :
(A) Besides water any other solvent, which has the
• It is applicable only to aqueous solutions. For the tendency to accept or lose a proton may decide the
acidic or basic properties, the presence of water is acidic or basic behaviour of the dissolved substance.
absolutely necessary.
(B) This theory states that the terms acid and base are
• The concept does not explain the acidic or basic
properties of acids or bases in non - aqueous solvents. comparative. A substance may act as an acid in one
solvent, while as a base in another solvent.
• It fails to explain the basic nature of compounds like
e.g. Acetic acid acts as an acid in water while as a
NH 3, Na2CO 3 etc., which do not have OH– in their base in HF.
molecules to furnish OH– ions. (ii) Demerits :
• It fails to explain the acidic nature of non - protic
compounds like SO2, P2O5, CO2, NO2 etc., which do not (A) Many acid - base reactions proceed without H +
have hydrogen in their molecules to furnish H+ ions. transfer.
• It fails to explain the acidic nature of certain salts like e.g. SO2 + SO3 SO2+ + SO42-
AlCl3 etc., in aqueous solutions.
(C) Lewis concept (electronic concept) :
(b ) Ac id B as e Conc ep t of B rons te d and
Lowr y : An acid is a molecule/ion which can accept an
electron pair with the formation of a coordinate bond.
This theory was given by Bronsted, a Danish chemist
and Lowry, an English chemist independently in 1923. Acid  e– pair acceptor
According to it, an acid is a substance, molecule or ion e.g. Electron deficient molecules : BF3, AlCl3
which has a tendency to release the proton Cations : H+, Fe2+, Na+
(protogenic) and similarly a base has a tendency to Molecules with vacant orbitals : SF4, PF3
accept the proton (protophilic).
e.g.
A base is any molecule/ion which has a lone pair of
HCl + H2O H3O+ + Cl–
electrons which can be donated.
In this reaction, HCl acts as an acid because it donates
a proton to the water molecule. Water, on the other Base (One electron pair donate)
hand, behaves as a base by accepting a e.g. Molecules with lone pairs : NH3, PH3, H2O, CH3OH
proton. Examples
 Note : Ex. In which of the following reactions does NH3 act
Bronsted and Lowry theory is also known as proton as an acid ?
donor and proton acceptor theory.
Other examples : (1) NH3 + H+  NH4+
(i) CH3COOH + H2O H3O+ + CH3COO– 1
(2) NH3 + Na  NaNH2 + H2
2
(ii) NH4+ + H2O H3O+ + NH3
(3) NH3 + HCI  NH4CI
(iii) NH3 + H2O +
NH + OH
4
– (4) None of these
Sol. (B) In the following reaction, NH3 changes to
In the reactions (i) and (ii) water is acting as a base, NaNH2 which contains NH2– ion. This means that
while in reaction (iii) it is acting as an acid.Thus water
NH3 has donated a proton to Na and hence acts
can donate as well as accept H+ and hence can act as
both acid and base.
as an acid.
Ex. Ammonium ion is
 Note : (1) A Lewis acid (2) Lewis base
The species like H2O, NH3, CH3COOH which can act (3) Bronsted acid (4) Bronsted base
as both acid and base are called amphiprotic.
Moreover according to theory, an acid on losing a proton
Sol. Correct answer is (3).
becomes a base, called conjugate base, while the
base by accepting proton changes to acid called

PAGE # 100
Some Examples of Norm al Salts w ith their Parent Acids :
NEUTRALISATION
S.No. Parent Acid Norm al Salts

It may be defined as a reaction between acid and 1


Hydrochloric acid NaCl, KCl, MgCl 2, AlCl 3, ZnCl 2,
base present in aqueous solution to form salt and (HCl) CaCl 2 and NH 4Cl.
water. Nitric acid NaNO3, KNO 3, Mg(NO 3) 2,Al(NO 3) 3,
2
HCl(aq) + NaOH(aq)  NaCl(aq) + H2O() (HNO 3) Zn(NO 3) 2, Ca(NO 3) 2.
Basically neutralisation is the combination between Sulphuric acid Na 2SO4 , K 2SO4, MgSO 4, Al 2(SO 4) 3,
– 3
H+ ions of the acid with OH ions of the base to form H2O. (H 2SO4) ZnSO 4, CaSO 4.
e.g. Acetic acid CH 3COONa, CH 3COOK, (CH 3COO)2Ca,
– –
H+(aq) + Cl (aq) + Na+(aq) + OH (aq)  Na+(aq) + Cl–(aq) + H2O() 4
(CH 3COOH) (CH 3COO) 2 Pb, CH 3COONH 4.

H+(aq) + OH (aq)  H2O() Carbonic acid Na 2CO3, K 2CO3, MgCO3, ZnCO3,
5
(H 2CO3) CaCO3, (NH 4) 2CO3.
Neutralisation reaction involving an acid and base is
Sulphurous acid Na 2SO3, K 2SO3, MgSO3, ZnSO3,
of exothermic nature. Heat is evolved in all 6
(H 2SO3) CaSO 3, (NH 4) 2 SO3.
neutralisation reactions. If both acid and base are
Phos phoric acid Na 3PO4, K 3PO4, Mg 3(PO4) 2,
strong, the value of heat energy evolved remains same 7
(H 3PO4) Zn 3(PO 4) 2, Ca 3(PO4) 2, (NH 4) 3PO4.
irrespective of their nature.
e.g . (B) Acid Salts : The salts which are obtained by the
HCl (aq) + NaOH (aq)  NaCl (aq) + H2O () + 57.1 KJ partial replacement of ionisable hydrogen atoms of a
Strong Strong polybasic acid by a metal or an ammonium ion are
acid base known as acid salts.

HNO3 (aq) + KOH (aq)  KNO3 (aq) + H2O () + 57.1 KJ These are usually formed when insufficient amount of
Strong Strong the base is taken for the neutralisation of the acid. For
acid base example, when insufficient amount of NaOH is taken
Strong acids and strong bases are completely ionised to neutralise H2SO4, we get an acid salt NaHSO4.
of their own in the solution. No energy is needed for H2SO4 + NaOH  NaHSO4 + H2 O
their ionisation. Since the cation of base and anion of (Insufficient amount) Sodium hydrogensulphate
acid on both sides of the equation cancel out (Acid salt)
completely, the heat evolved is given by the following
In this case, only one hydrogen atom out of two has
reaction -
been replaced by sodium atom. Since there is one
H+ (aq) + OH– (aq)  H2O () + 57.1 KJ more hydrogen atom in NaHSO 4 which can be
replaced, therefore, it further reacts with another
SALTS molecule of NaOH to produce Na2 SO 4 which is a
A substance formed by neutralization of an acid normal salt.
with a base is called a salt. NaHSO 4 + NaOH  Na2SO4 + H2 O
e.g. Sodium Sodium
hydrogensulphate sulphate
Ca(OH)2(aq)+ H2SO4(aq)  CaSO4(aq) + 2H2O()
(Acid salt) (Normal salt)
Cu(OH)2(aq) + 2HNO3(aq)  Cu(NO3)2(aq) + 2H2O()
Acid salts ionise in aqueous solution to produce
NaOH(aq) + HCl(aq)  NaCl(aq) + H2O() hydronium ions (H3O+), therefore, they exhibit all the
(a) Classification of Salts : properties of acids.
It is based on their Mode of Formation :
Some other examples of acid salts are given in Table.
(A) Normal Salts :
SOME ACID SALTS WITH THEIR PARENT ACIDS
The salts which are obtained by complete replacement
of the ionisable hydrogen atoms of an acid by a metallic
or an ammonium ion are known as normal salts. For S.No. Parent Acid Acid Salts
example, normal salts NaCl and Na2 SO 4 are formed Sulphuric acid NaHSO4, KHSO4,
by the complete replacement of ionisable hydrogen 1 (H2SO4) Ca(HSO4)2
atoms of HCl and H2SO4 respectively Carbonic acid NaHCO3, KHCO3,
HCl + NaOH  NaCl + H2O 2 (H2CO3) Ca(HCO3)2, Mg(HCO3)2
Sodium chloride Sulphurous acid NaHSO3, KHSO3,
(Normal salt) 3 (H2SO3) Ca(HSO3)2, Mg(HSO3)2
H2SO4 + 2NaOH  Na2SO 4 + 2H2O Phosphoric acid NaH2PO4, Na2HPO4, KH2PO4,
Sodium sulphate 4 (H3PO4) K2HPO4, Ca(H2PO4)2, CaHPO4
(Normal salt)
(C) Basic Salts : The salts which are formed by partial
Some of the salts are given below in the table.
replacement of hydroxyl (–OH) groups of a di-or a
triacidic base by an acid radical are known as basic
salts.

PAGE # 101
to 7. They do not change the colour of litmus solution.
These are usually formed when an insufficient amount
e.g. NaCl, KCl, NaNO3, Na2SO4 etc.
of acid is taken for the neutralisation of the base. For
example, when insufficient amount of HCl is added to (B) Acidic salt solutions : Salt solutions of strong ac-
Lead hydroxide, Basic lead chloride [Pb(OH)Cl] is ids and weak bases are of acidic nature and have pH
formed less than 7. They change the colour of blue litmus
Pb(OH)2 + HCl  Pb (OH)Cl + H2O solution to red.
Lead hydroxide Basic Lead chloride e.g. (NH4)2SO4, NH4Cl etc.
(Diacidic base) (Basic salt) In both these salts, the base NH4OH is weak while the
Basic salts, for example, Pb(OH)Cl further reacts with acids H2SO4 and HCl are strong.
HCl to form normal salts (C) Basic salt solutions : Salt solutions of strong bases
Pb (OH) Cl + HCl  PbCl2 + H2O and weak acids are of basic nature and have pH more
Basic Lead Chloride Lead chloride than 7. They change the colour of red litmus solution to
(Basic salts) (Normal salt) blue.
e.g. Na2CO3, K3PO4 etc.
Some other important examples of basic salts are :
In both the salts, bases NaOH and KOH are strong
(i) Basic copper chloride, Cu(OH)Cl. while the acids H2CO3 and H3PO4 are weak.
(ii) Basic copper nitrate, Cu(OH)NO3 Some Important Chemical Compounds :
(iii) Basic lead nitrate, Pb(OH)NO3 .
(D) Double Salts : The salts which are obtained by the
crystallisation of two simple salts from a mixture of Common name : Table Salt
their saturated solutions are known as double salts. Chemical name : Sodium chloride
For example, a double salt potash alum [K 2 SO 4 . Chemical formula : NaCl
Al2 (SO 4 )3 . 24H 2 O] is prepared by mixing saturated
solutions of two simple salts, K2SO4 and Al2(SO4)3 and Sodium chloride (NaCl) also called common salt or
crystallization of the mixture. table salt is the most essential part of our diet.
Chemically it is formed by the reaction between
Crystallisation solutions of sodium hydroxide and hydrochloric acid.
K2SO4 + Al2(SO4)3+ 24H2O  K2SO4. Al2(SO4)3. 24 H2O
Sea water is the major source of sodium chloride
Potassium Potash alum
where it is present in dissolved form along with other
sulphate (Double salt)
soluble salts such as chlorides and sulphates of
calcium and magnesium. It is separated by some
Some other examples of double salts are :
suitable methods. Deposits of the salts are found in
(i) Mohr’s Salt, FeSO4 .(NH4)2SO4. 6H2O,
different parts of the world and is known as rock salt.
(ii) Dolomite, CaCO3. MgCO3,
When pure, it is a white crystalline solid, however, it is
(iii) Carnallite, KCl. MgCl2.6H2 O
often brown due to the presence of impurities.
(E) Mixed Salts : The salts which contain more than
Us es :
one type of acidic or basic radicals are called mixed
(i) Essential for life : Sodium chloride is quite essential
salts. For example, Sodium potassium carbonate
for life. Biologically, it has a number of functions to
(NaKCO3) is a mixed salt containing two basic radicals
perform such as in muscle contraction, in conduction
sodium and potassium. Similarly, calcium oxy chloride,
of nerve impulse in the nervous system and is also
Ca(OCI)Cl is also a mixed salt containing two acid
converted in hydrochloric acid which helps in the
radicals OCI– and Cl– .
digestion of food in the stomach. When we sweat, there
Some other important examples of mixed salts are : is loss of sodium chloride along with water. It leads to
Sodium potassium sulphate (NaKSO4) (containing two muscle cramps. Its loss has to be compensated
basic radicals), Disodium potassium phosphate suitably by giving certain salt preparations to the patient.
(Na2KPO4) (containing two basic radicals). Electrol powder is an important substitute of common
(ii) Classification of salt solutions based on pH salt.
values : (ii) Raw material for chemicals : Sodium chloride is
Salts are formed by the reaction between acids also a very useful raw material for different chemicals.
and bases. Depending upon the nature of the acids A few out of these are hydrochloric acid (HCl), washing
and bases or upon the pH values, the salt solutions soda (Na2CO 3.10H 2O), baking soda (NaHCO 3) etc.
are of three types. Upon electrolysis of a solution of the common salt
(A) Neutral salt solutions : Salt solutions of strong (brine), sodium hydroxide, chlorine and hydrogen are
acids and strong bases are neutral and have pH equal obtained.

PAGE # 102
Electrolysis of aqueous solution of NaCl :
Electrolysis
BAKING SODA
2NaCl(aq) + 2H2O()    2NaOH(aq) +
Cl2(g)+ H2 (g) Baking soda is sodium hydrogen carbonate or sodium
bicarbonate (NaHCO 3).
Reaction takes place as follows -
• NaCl  Na+ + Cl– (a) Preparation :

• 2Cl–  Cl2 (g) + 2e– (anode reaction) It is obtained as an intermediate product in the prepa-
– – ration of sodium carbonate by Solvay process. In this
• 2H2O + 2e  H2 + 2OH (cathode reaction)
process, a saturated solution of sodium chloride in
• Na+ + OH–  NaOH water is saturated with ammonia and then carbon di-
oxide gas is passed into the solution. Sodium chlo-
(iii) In leather industries : It is used in leather industry ride is converted into sodium bicarbonate which, be-
for the leather tanning. ing less soluble, separates out from the solution.
(iv) In severe cold, rock salt is spread on icy roads to 2NH3 (g) + H2O () + CO2 (g)  (NH4)2CO3(aq)
melt ice. (NH4)2CO3(aq)+2NaCl(aq) Na2CO3 (aq) + 2NH4Cl (aq)
(v) It is also used as a fertilizer for sugar beet. Na2CO3 (aq) + H2O () + CO2 (g)  2NaHCO3 (s)
(b) Properties :
CAUSTIC SODA (i) It is a white, crystalline substance that forms an
Chemical name : Sodium hydroxide alkaline solution with water. The aqueous solution of
Chemical formula : NaOH sodium bicarbonate does not change the colour of
methyl orange but gives pink colour with
Preparation : Sodium hydroxide is prepared by
phenolphthalein.
electrolysing a concentrated solution of sodium chloride.
This process is done in Castner - Kellner cell. (Phenolphthalein and methyl orange are dyes used
+ – as acid-base indicators.)
2NaCl  2Na + 2Cl (ii) W hen heated above 543 K, it is converted into

2Cl  Cl2 + 2e– sodium carbonate.

2Na+ + 2Hg + 2e  2NaHg 
2NaHCO3 (s)  Na2CO3 (s) + CO2 (g) + H2O ()
Sodium amalgam
2NaHg + 2H2O  2NaOH + H2 + 2Hg (c) Uses :

Us es : (i) It is used in the manufacture of baking powder. Bak-


(i) It is used in soaps, detergents, paper and silk ing powder is a mixture of potassium hydrogen
industries. tartarate and sodium bicarbonate. During the prepa-
(ii) It is used in refining of petroleum. ration of bread the evolution of carbon dioxide causes
(iii) It is used as a laboratory reagent. bread to rise (swell).
(iv) It is used in dye industry.
(v) It is used in concentration of bauxite ore.

WASHING SODA
Chemical name : Sodium carbonate decahydrate (ii) It is largely used in the treatment of acid spillage
Chemical formula : Na2CO3.10H2O and in medicine as soda bicarb, which acts as an
antacid.
Recrystallization of sodium carbonate :
(iii) It is an important chemical in the textile, tanning,
Sodium carbonate is recrystallized by dissolving in
paper and ceramic industries.
water to get washing soda.
(iv) It is also used in a particular type of fire extinguisher.
The following diagram shows a fire extinguisher that
uses NaHCO 3 and H 2SO 4 to produce CO 2 gas. The
extinguisher consists of a conical metallic container
(A) with a nozzle (Z) at one end. A strong solution of
(b) Uses : NaHCO3 is kept in the container. A glass ampoule (P)
(i) It is used as cleansing agent for domestic purposes. containing H2SO4 is attached to a knob (K) and placed
inside the NaHCO3 solution. The ampoule can be bro-
(ii) It is used in softening of hard water and controlling
the pH of water. ken by hitting the knob. As soon as the acid comes in
contact with the NaHCO3 solution, CO2 gas is formed.
(iii) It is used in the manufacture of glass.
When enough pressure in built up inside the container,
(iv) Due to its detergent properties, it is used as a con- CO2 gas rushes out through the nozzle (Z). Since CO2
stituent of several dry soap powders. does not support combustion, a small fire can be put
(v) It also finds use in photography, textile and paper out by pointing the nozzle towards the fire. The gas is
industries etc. produced according to the following reaction.
(vi) It is used in the manufacture of borax (Na2B4O7. 10H2O). 2NaHCO3 (aq) + H2SO 4 (aq)  Na2SO 4 (aq) +

PAGE # 103
CO2 2H2O() + 2CO2(g) (b) Properties :
(i) Action with water : When it is dissolved in water, it
(Z)
gets crystallized and forms gypsum.

(c) Uses :
NaHCO3
When finely powdered Plaster of Paris is mixed with
(A)
water and made into a paste, it quickly sets into a hard
mass. In the process, its volume also increases
• slightly. These properties find a number of uses. Addi-
••• H2SO4
(P) ••••••• tion of water turns Plaster of Paris back into gypsum.

•••••••• (i) It is used in the laboratories for sealing gaps where
••
air tight arrangement is required.
(ii) It is used for making toys, cosmetics and casts of
statues.
Knob (K) (iii) It is used as a cast for setting broken bones.
(iv) It also finds use in making moulds in pottery.
(v) It is used for making surfaces smooth and for mak-
Soda-Acid Fire Extinguisher
ing designs on walls and ceilings.

BLEACHING POWDER HYDRATED SALTS - SALTS CONTAINING


Bleaching powder is commercially called ‘chloride of WATER OF CRYSTALLISATION:
lime or ‘chlorinated lime’. It is principally calcium oxy-
chloride having the following formula : Certain salts contain definite amount of some water
molecules loosely attached to their own molecules.
These are known as hydrated salts and are of crystal-
line nature. The molecules of water present are known
Bleaching powder is prepared by passing chlorine
over slaked lime at 313 K. as ‘water of crystallisation’.
313 K In coloured crystalline and hydrated salts, the
Ca(OH)2 (aq) + Cl2 (g) Ca(OCl)Cl (s) + H2 O (g)
Slaked lime Bleaching molecules of water of crystallisation also account for
powder their characteristic colours. Thus, upon heating of hy-
Actually bleaching powder is not a compound but a
mixture of compounds : drated salt, its colour changes since molecules of
CaOCl2.4H2O, CaCl2.Ca(OH)2.H 2O water of crystallisation are removed and the salt be-
(a) Uses : comes anhydrous.
For example, take a few crystals of blue vitriol i.e. hy-
(i) It is commonly used as a bleaching agent in paper
and textile industries. drated copper sulphate in a dry test tube or boiling
tube. Heat the tube from below. The salt will change to
(ii) It is also used for disinfecting water to make it free
from germs. a white anhydrous powder and water droplet will ap-
pear on the walls of the tube. Cool the tube and add a
(iii) It is used to prepare chloroform.
few drops of water again. The white anhydrous pow-
(iv) It is also used to make wool shrink-proof.
der will again acquire blue colour.

PLASTER OF PARIS 
CuSO4. 5H2O  CuSO4 + 5H2O

(a) Preparation : Copper sulphate Copper sulphate


It is prepared by heating gypsum (CaSO 4.2H 2O) at (Hydrated) (Anhydrous)
about 373 K in large steel pots with mechanical stirrer,
or in a revolving furnace. APPLICATIONS OF NEUTRALISATION
373 K
2(CaSO 4.2H2O) (CaSO 4)2.H2O + 3H2O (i) People particularly of old age suffer from acidity
Gypsum Plaster of Paris problems in the stomach which is caused mainly due
or to release of excessive gastric juices containing HCl.
The acidity is neutralised by antacid tablets which
The temperature is carefully controlled, as at higher
temperature gypsum is fully dehydrated. The proper- contain sodium hydrogen carbonate (baking soda),
ties of dehydrated gypsum are completely different from magnesium hydroxide etc.
those of Plaster of Paris. (ii) The stings of bees and ants contain formic acid. Its
corrosive and poisonous effect can be neutralised by
rubbing soap which contains NaOH (an alkali).

PAGE # 104
(iii) The stings of wasps contain an alkali and its II. Theories of acid same bases
poisonous effect can be neutralised by an acid like
acetic acid (present in vinegar). 13. According to Arrhenius concept, base is a substance
that -
(iv) Farmers generally neutralize the effect of acidity in (A) gives H+ ions in aqueous solution.
the soil caused by acid rain by adding slaked lime (B) gives OH– ions in aqueous solution.
(Calcium hydroxide) to the soil. (C) accepts electrons.
(D) donates electrons.
14. According to Bronsted - Lowry concept an acid is a
EXERCISE-1 substance which -
(A) accepts proton.
I. Acids & bases (B) gives an electron pair.
(C) gives proton.
1. The acid used in making of vinegar is - (D) combines with H3O+ ions.
(A) Formic acid (B) Acetic acid 15. According to Lewis concept, a base is a substance
(C) Sulphuric acid (D) Nitric acid which -
2. Common name of H2SO4 is - (A) donates an electron pair.
(A) Oil of vitriol (B) Muriatic acid (B) accepts an electron pair.
(C) Blue vitriol (D) Green vitriol (C) produces hydronium ions.
(D) combines with OH– ions.
3. CuO + (X)  CuSO4 + H2O. Here (X) is -
(A) CuSO4 (B) HCl 16. Which among the following qualifies as a Lewis acid ?
(C) H2SO4 (D) HNO3 (A) NaF (B) NaCl
(C) BF3 (D) H3O+
4. Which of the following is the weakest base ?
(A) NaOH (B) NH4OH 17. Which of the following will qualify as a Lewis base ?
(C) KOH (D) Ca(OH)2 (A) BCl3 (B) CH4
(C) Cl2 (D) NH3
5. When CO2 is passed through lime water, it turns milky.
The milkiness is due to the formation of - 18. NH4+ ion in aqueous solution will behave as -
(A) CaCO3 (B) Ca(OH)2 (A) a base (B) an acid
(C) H2O (D) CO2 (C) both acid and base (D) neutral

6. Caustic soda is the common name for - 19. Which one of the following does not act as a Bronsted
(A) Mg(OH)2 (B) KOH acid ?
(A) NH4+ (B) HCO3–
(C) Ca(OH)2 (D) NaOH –
(C) HSO3 (D) CH3COO–

7. Calcium hydroxide (slaked lime) is used in - 20. Of the given anions, the strongest Bronsted base is -
(A) Plastics and dyes (B) Fertilizers (A) ClO– (B) ClO2–

(C) Antacids (D) White washing (C) ClO3 (D) ClO4–

8. Acids gives - 21. The compound that is not a Lewis acid is -


(A) H+ in water (B) OH– in water (A) BaCl2 (B) AlCl3
(C) Both (A) & (B) (D) None of these (C) BCl3 (D) SnCl4

9. H2CO3 is a - 22. Ionic dissociation of acetic acid is represented as -


(A) strong acid (B) weak acid CH3COOH + H2O CH3COO– + H3O+
(C) strong base (D) weak base According to Lowry and Bronsted, the reaction
possesses -
10. Which is a base and not an alkali ? (A) an acid and three bases
(A) NaOH (B) KOH
(B) two acids and two bases
(C) Fe(OH)3 (D) none is true
(C) an acid and a base
(D) three acid and a base
11. The strength of the acid depends on the -
(A) number of hydrogen atoms present in the molecule.
23. The concept of an acid as an acceptor of a pair of
(B) oxygen content.
electron was introduced by -
(C) density.
(D) concentration of hydrogen ions furnished by (A) Lowry (B) Bronsted
ionisation. (C) Arrhenius (D) Lewis

24. The conjugate base of HPO42- is -


12. Ammonia dissolves in water to give NH 4OH. In this
(A) PO43- (B) H2PO4-
reaction water acts as -
(C) H3PO4 (D) H4PO3
(A) an acid (B) a base
(C) a salt (D) conjugate base

PAGE # 105
– –
25. In the reaction, I2 + I I3 , the Lewis base is - (A) strength of the solution
(A) I2 (B) I
– (B) pH of the solution
– (C) degree of hydrolysis
(C) I3 (D) none
(D) solubility product of the electrolyte
26. Which of the following is the strongest base ?
– –
39. Which one of the following relationship is correct ?
(A) CH3 (B) NH2
– – 1
(C) OH (D) F (A) pH  (B) pH = log [H+]
[H ]
III. Indicators, Neutralisation 1
(C) log pH = [H+] (D) pH = log
[H ]
27. Reaction of an acid with a base is known as -
(A) decomposition (B) combination 40. When 10–6 mole of a monobasic strong acid is dissolved
(C) redox reaction (D) neutralization in one litre of solvent , the pH of the solution is -
(A) 6 (B) 7
28. Antacids contain - (C) less than 7 (D) more than 7
(A) Weak base (B) Weak acid
(C) Strong base (D) Strong acid 41. W hen pH of a solution is 2, the hydrogen ion
concentration in mol litre –1 is -
29. Nature of methyl orange is - (A) 1 × 10 –12 (B) 1 × 10–2
(A) acidic (B) basic (C) 1 × 10 –7
(D) 1 × 10–4
(C) neutral (D) none of these

30. Energy released in neutralisation reaction which 42. Fear and excitement generally cause one to breathe
occurs between strong acid and strong base is - rapidly and it results in the decrease of carbon dioxide
(A) 57.8 kJ (B) 57.1 kJ in the blood. In what way will it change the pH of the
(C) 57.9 kJ (D) 56.1 kJ blood ?
(A) pH will decrease (B) pH will increase
IV. pH scale (C) no change (D) pH will adjust to 7

31. A solution turns red litmus blue. Its pH is likely to be -


(A) 2 (B) 4
(C) 7 (D) 10
EXERCISE-2
32. If pH of any solution is equal to zero then solution will COMPETITIVE EXAM PREVIOUS YEARS’ QUESTIONS :
be -
1. Bromine can be liberated from KBr solution by the
(A) acidic (B) basic
action of [IJSO-Stage-I/2011]
(C) neutral (D) none of these
(A) iodine solution (B) chlorine water
33. pH of Blood is - (C) NaCl (D) Kl
(A) 6.4 (B) 7.4
2. The compound that has both ionic and covalent
(C) 4.7 (D) 6.4
bonds is - [IJSO-Stage-I/2011]
34. If pH of solution is 13, means that it is - (A) boric acid (H3BO3).
(A) weakly acidic (B) weakly basic (B) sodium chloride (NaCl).
(C) strongly acidic (D) strongly basic (C) ethyl alcohol (C2H5OH).
35. A solution has pH 2. It contains (D) sodium phenolate (C6H5ONa).
(A) CH3COOH (B) H2CO3
3. Hydrogen fluoride is a liquid at room temperature
(C) HNO3 (D) H2C2O4
due to [IJSO-Stage-I/2011]
36. A solution has pH 9. On dilution the pH value
(A) dimerisation
(A) decreases (B) increases
(B) dissociation followed by aggregation
(C) remain same (D) none of these
(C) association
37. A salt derived from strong acid and weak base will (D) polymerisation
dissolve in water to give a solution which is -
(A) acidic (B) basic 4. The compound used to remove carbon dioxide from
(C) neutral (D) none of these air is - [IJSO-Stage-I/2011]
(A) sodium carbonate (B) sodium hydroxide.
38. The numerical value of negative power to which 10 (C) sodium nitrate (D) sodium chloride
must be raised in order to express hydrogen ion
concentration is equal to -

PAGE # 106
5. Which of the following hydroxides is NOT an alkali ? 12. Which among the following salts will not change the
[IJSO-Stage-1/2011-12] pH of water on addition
(A) Ammonium hydroxide [IJSO-Stage-1/2014-15]
(B) Calcium hydroxide (A) Sodium cyanide
(C) Copper hydroxide (B) Sodium chloride
(D) Sodium hydroxide (C) Sodium bicarbonate

6. A white salt is readily soluble in water and gives (D) Sodium carbonate
colourless solution with pH of about 9. The salt would
be - [IJSO-Stage-II/2012] 13. An electron pair donor is a Lewis base and an electron
(A) NH4 NO3 (B) CH3 COONa pair acceptor is a Lewis acid. W hich among the
(C) CH3 COONH4 (D) CaCO 3 following statements, is correct ?
[IJSO-Stage-1/2016-17]
7.  W hich one is not an acid salt  ? (A) NH3 is a Lewis acid, because nitrogen has only 6
[IJSO-Stage-II/2012] electrons around it.
(A) NaH2PO 4 (B) NaH2PO 2 (B) BF 3 is a Lewis base, because fluorine has 8
(C) NaH 2PO 3 (D) Na3PO 3 electrons around it.
(C) NF3 is a Lewis base, because nitrogen has a lone
8. Esha performed a simple experiment to distinguish electron.
strong from weak acid. For this she performed qualita- (D) BCI3 is a Lewis acid because it has only 6 electrons
tive experiments with universal indicator, using tama- around it.
rind and the acid present in gastric juice and she re-
corded her observations. Which of the following find-
ings did she observe ? [IJSO-Stage-II/2013]

Acid present in gastric juice Acid present in tamarind


Colour of Colour of
Strength universal indicator Strength universal indicator
A Weak Red Strong Red
B Weak Yellow Weak Green
C Strong Light red Weak Yellow
D Strong Green Strong Blue

9. When the pH of the environment of a protein is changed,


it is said to be denatured. This is due to :
[IJSO-Stage-I/2013]
(A) breakage of peptide bond
(B) breakage of disulfide links
(C) loss of tertiary structure
(D) breakdown of R groups
10. There are 3 containers X, Y and Z. X contains 10ml of
water and Z contains 10 ml of milk. Y contains 5ml of
milk (same as in container Z) mixed with 5 ml of water.
All 3 containers have pH value of 6.5. P amount of
Acetic acid is added to container X, Q amount to Y and
R amount to Z. Such that the final pH value in each
container is 5.5. Then which of the following is true.
[IJSO-Stage-II/2014]
(A) P < Q < R (B) P < R = Q
(C) P = Q = R (D) P < R < Q

11. Bromine can be liberated from KBr solution by the


action of [IJSO-Stage-1/2011-12]
(A) iodine solution (B) chlorine water
(C) NaCl (D) Kl

PAGE # 107
METALS AND NON-METALS

INTRODUCTION GENERAL PROPERTIES OF METALS


AND NON-METALS
There are 118 chemical elements known at present.
(a) Electronic Configuration of Metals :
On the basis of their properties, all these elements
The atoms of metals have 1 to 3 electrons in their
can be broadly divided into two main groups: Metals
outermost shells. For example, all the alkali metals
and Non-Metals. A majority of the known elements have one electron in their outermost shells (lithium
are metals. All the metals are solids, except mercury, 2, 1 ; sodium 2, 8, 1; potassium 2, 8, 8, 1 etc.).
which is a liquid metal. There are 22 non-metals, out Sodium, magnesium and aluminium are metals
of which, 10 non-metals are solids, one non-metal having 1, 2 and 3 electrons respectively in their
(bromine) is a liquid and the remaining 11 non-metals valence shells. Similarly, other metals have 1 to 3
are gases. electrons in their outermost shells.
It may be noted that hydrogen and helium are
exception because hydrogen is a non-metal having
POSITION OF METALS AND NON-METALS only 1 electron in the outermost shell (K shell) of its
IN THE PERIODIC TABLE atom and helium is also a non-metal having 2
electrons in the outermost shell (K shell).
The metals are placed on the left hand side and in
the centre of the periodic table. On the other hand, the (b) Physical Properties of Metals :
non-metals are placed on the right hand side of the The important physical properties of metals are
periodic table. This has been shown in the figure. It discussed below :
may be noted that hydrogen (H) is an exception (i) Metals are solids at room temperature: All metals
because it is non-metal but is placed on the left hand (except mercury) are solids at room temperature.
side of the periodic table.  Note :
Metals and non-metals are separated from each other Mercury is a liquid at room temperature.
in the periodic table by a zig-zag line. The elements (ii) Metals are malleable :Metals are generally
close to zig-zag line show properties of both the malleable. Malleability means that the metals can be
metals and the non-metals. They show some beaten with a hammer into very thin sheets without
properties of metals and some properties of non- breaking. Gold and silver are among the best
metals. These are called metalloids. The common malleable metals. Aluminium and copper are also
highly malleable metals.
examples of metalloids are boron (B), silicon (Si),
germanium (Ge), arsenic (As), antimony (Sb), tellurium (iii) Metals are ductile : It means that metals can be
(Te) and polonium (Po). drawn (stretched) into thin wires. Gold and silver are
the most ductile metals. Copper and aluminium are
also very ductile, and therefore, these can be drawn
into thin wires which are used in electrical wiring.

(iv) Metals are good conductors of heat and


electricity : All metals are good conductors of heat.
The conduction of heat is called thermal conductivity.
Silver is the best conductor of heat. Copper and
aluminium are also good conductors of heat and
therefore, they are used for making household utensils.
Lead is the poorest conductor of heat. Mercury metal is
also a good conductor of heat.
Metals are also good conductors of electricity. The
In general, the metallic character decreases on going electrical and thermal conductivities of metals are
from left to right side in the periodic table. However, due to the presence of free electrons in them. Among
on going down the group, the metallic character all the metals, silver is the best conductor of electricity.
increases. Copper and aluminium are the next best conductors
of electricity. Since silver is expensive, therefore,
 Note :
copper and aluminium are commonly used for
The elements at the extreme left of the periodic table making electric wires.
are most metallic and those on the right are least
 Note :
metallic or non-metallic.
Silver is best conductor of heat and electricity.

PAGE # 108
(v) Metals are lustrous and can be polished : Most of
(e) Chemical Properties of Metals :
the metals have shine and they can be polished. The
shining appearance of metals is also known as metallic The atoms of the metals have usually 1, 2 or 3
lustre. For example, gold, silver and copper metals have electrons in their outermost shells. These outermost
metallic lustre. electrons are loosely held by their nuclei. Therefore,
the metal atoms can easily lose their outermost
(vi) Metals have high densities : Most of the metals
electrons to form positively charged ions. For example,
are heavy and have high densities. For example, the
density of mercury metal is very high (13.6 g cm –3). sodium metal can lose outermost one electron to form
However, there are some exceptions. Sodium, positively charged ion, Na+. After losing the outermost
potassium, magnesium and aluminium have low electron, it gets stable electronic configuration of the
densities. Densities of metals are generally noble gas (Ne : 2, 8). Similarly, magnesium can lose
proportional to their atomic masses. two outermost electrons to form Mg 2+ ion and
aluminium can lose its three outermost electrons to
(vii) Metals are hard : Most of the metals are hard. But
all metals are not equally hard. Metals like iron, copper, form Al3+ ion.
aluminium etc. are quite hard. They cannot be cut with Na Na+ + e–
a knife. Sodium and potassium are common (2, 8, 1) (2, 8)
exceptions which are soft and can be easily cut with a Mg Mg2+ + 2e–
knife. (2, 8, 2) (2, 8)
(viii) Metals have high melting and boiling points : Al Al3+ + 3e–
Most of the metals (except Na, K, Rb, Cs, Ga) have (2, 8, 3) (2, 8)
high melting and boiling points.  Note :
 Note : The metal atoms lose electrons and form positively
Tungsten has highest melting point (34100C) among charged ions, therefore, the metals are called
all the metals. electropositive elements.

(ix) Metals are rigid : Most of the metals are rigid and Some of the important chemical properties of metals are
they have high tensile strength. discussed below :

(x) Metals are sonorous : Most of the metals are (i) Reaction with oxygen : Metals react with oxygen to
sonorous i.e, they make sound when hit with an object. form oxides. These oxides are basic in nature. For
example, sodium metal reacts with oxygen of the air
(c) Electronic Configuration of Non-Metals :
and form sodium oxide.
The atoms of non-metals have usually 4 to 8 electrons 4 Na(s) + O2(g) 2 Na2O(s)
in their outermost shells. For example, Carbon (At. No. Sodium oxide
6), Nitrogen (At. No. 7), Oxygen (At. No. 8), Fluorine (At.
No. 9) and Neon (At. No. 10) have respectively 4,5,6,7,8 Sodium oxide reacts with water to form an alkali called
electrons in their outermost shells. sodium hydroxide. Therefore, sodium oxide is a basic
oxide.
(d) Physical Properties of Non-Metals :
Na2O(s) + H2O() 2NaOH(aq)
The important physical properties of non-metals are
Sodium hydroxide
listed below:
Due to the formation of sodium hydroxide (which is an
(i) Non-metals are brittle.
alkali), the solution of sodium oxide in water turns red
(ii) Non-metals are not ductile.
litmus blue (common property of all alkaline solutions).
(iii) Non-metals are bad conductor of heat and
 Note :
electricity. (Exception: Graphite is a good conductor
because of the presence of free electrons.) W hen metal oxides are dissolved in water, they give
(iv) Non-metals are not lustrous and cannot be alkaline solutions.
polished. (Exception: Graphite and Iodine are lustrous Similarly, magnesium is a metal and it reacts with oxygen
non-metals.) to form magnesium oxide. However, magnesium is less
(v) Non-metals may be solid, liquid, or gases at room reactive than sodium and therefore, heat is required
temperature. for the reaction.
(vi) Non-metals are generally soft. (Exception: 2Mg(s) + O2(g) 2 MgO(s)
Diamond, an allotropic form of non-metal Carbon, is
the hardest natural substance known). Thus, when a metal combines with oxygen, it loses its
valence electrons and forms positively charged metal
(vii) Non-metals have generally low melting and boiling ion. We can say that oxidation of metal takes place.
points. (Exception: Graphite, another allotropic form of
Carbon, has a melting point of about 3730°C). Reactivity of metals with oxygen :
(viii) Non-metals have low densities. (Exception: Iodine All metals do not react with oxygen with equal ease.
has high density). The reactivity of oxygen depends upon the nature of
 Note : the metal. Some metals react with oxygen even at room
temperature, some react on heating while still others
Graphite is a good conductor of electricity, lustrous and
react only on strong heating. For example :
has very high melting point.

PAGE # 109
(A) Metals like sodium, potassium and calcium react (D) Metals like zinc and aluminium react only with steam
with oxygen even at room temperature to form their to form their respective oxides and hydrogen.
oxides.
Zn (s) + H2O(g) ZnO (s) + H2(g)
4Na(s) + O2(g) 2Na2O(s) Zinc Steam Zinc oxide
Sodium Oxygen Sodium
oxide 2Al (s) + 3H2O (g) Al2O3(s) + 3H2(g)
4K(s) + O2(g) 2K2O(s) Aluminium Steam Aluminium
Potassium Oxygen Potassium oxide oxide
2Ca(s) + O2(g) 2 CaO(s)
(E) Iron metal does not react with water under ordinary
Calcium Oxygen Calcium oxide
conditions. The reaction occurs only when steam is
(B) Metals like magnesium do not react with oxygen at
passed over red hot iron and the products are iron
room temperature. They burn in air on heating to form
(II,III) oxide and hydrogen.
corresponding oxides.
3Fe(s) + 4H2O(g) Fe3O4(s) + 4H2(g)
2Mg(s) + O2(g) 2MgO(s)
Iron Steam Iron (II,III) Hydrogen
Magnesium Oxygen Magnesium oxide
(Red hot) oxide
(C) Metals like zinc do not react with oxygen at room (F) Metals like copper, silver and gold do not react with
temperature. They burn in air only on strong heating water even under strong conditions. The order of
to form corresponding oxides.
reactivities of different metals with water is :
2 Zn(s) + O2 (g) 2 ZnO(s) Na > Mg > Zn > Fe > Cu
Zinc Oxygen Zinc oxide Reactivity with water decreases
(D) Metals like iron and copper do not burn in air even
(iii) Reaction with dilute acids :
on strong heating. However, they react with oxygen only
on prolonged heating. Many metals react with dilute acids and liberate
3Fe(s) + 2O2(g) Fe3O4(s) hydrogen gas. Only less reactive metals such as
Iron Oxygen Iron (II, III) oxide copper, silver, gold etc. do not liberate hydrogen from
2Cu(s) + O2(g) 2CuO(s) dilute acids. The reactions of metals with dilute
Copper Oxygen Copper (II) oxide hydrochloric acid (HCl) and dilute sulphuric acid
(ii) Reaction with water : (H 2SO 4) are similar. W ith dil. HCl, they give metal
Metals react with water to form metal oxide or metal chlorides and hydrogen whereas with dil. H2SO4, they
hydroxide and hydrogen. The reactivity of metals give metal sulphates and hydrogen.
towards water depends upon the nature of the metals.  Note :
Some metals react even with cold water, some react
with water only on heating while there are some metals Dilute nitric acid (HNO 3) is an oxidising agent which
which do not react even with steam. For example, oxidises metals,but does not produce hydrogen.
(A) Sodium and potassium metals react vigorously But Mg & Mn produce hydrogen on reacting with
with cold water to form their respective hydroxides and dilute HNO3.
hydrogen gas is liberated.
2 Na(s) + 2H2O() 2NaOH (aq) + H2(g) The reactivity of different metals is different with the
Sodium Cold water Sodium Hydrogen same acid. For example :
hydroxide (A) Sodium, magnesium and calcium react violently
2K (s) + 2H2O() 2KOH (aq) + H2 (g) with dilute hydrochloric acid (HCl) or dilute sulphuric
Potassium Cold water Potassium Hydrogen acid (H 2 SO 4) liberating hydrogen gas and
hydroxide
corresponding metal salt.
 Note :
The reaction between sodium and water is so violent 2Na(s) + 2HCl (aq) 2NaCl(aq) + H2(g)
that the hydrogen evolved catches fire. Sodium Hydrochloric Sodium Hydrogen
(B) Calcium reacts with cold water to form calcium acid chloride
hydroxide and hydrogen gas. The reaction is less 2Na(s) + H2SO4(aq) Na2SO4 (aq) + H2(g)
violent.
Sodium Sulphuric Sodium Hydrogen
Ca(s) + 2H2O() Ca (OH)2 (aq) + H2(g)
acid sulphate
Calcium Cold water Calcium hydroxide
Similarly,
(C) Magnesium reacts very slowly with cold water but
reacts rapidly with hot boiling water forming Mg (s) + 2HCl (aq) MgCl2(aq) + H2(g)
Magnesium Hydrochloric Magnesium Hydrogen
magnesium oxide and hydrogen.
acid chloride
Mg (s) + H2O () MgO(s) + H2(g) Mg(s) + H2SO4 (aq) MgSO4 (aq) + H2(g)
Magnesium Boiling Magnesium Magnesium Sulphuric Magnesium Hydrogen
water oxide acid sulphate

PAGE # 110
(B) Aluminium and zinc react with dilute hydrochloric the solution. It is observed that the blue colour of copper
acid (HCl) or dilute sulphuric acid (H 2SO 4) liberating sulphate fades gradually and copper metal is deposited
hydrogen gas and corresponding metal salt. on the zinc strip. This means that the following reaction
2Al(s) + 6HCl (aq) 2AlCl3(aq) + 3H2(g) occurs :
Aluminium Hydrochloric Aluminium Hydrogen Zn(s) + CuSO4(aq) ZnSO4(aq) + Cu(s)
acid chloride Zinc Copper Zinc sulphate Copper
2Al(s) + 3H2SO4(aq) Al2(SO4)3(aq)+ 3H2(g) sulphate (Colourless solution)
Aluminium Sulphuric Aluminium Hydrogen (Blue solution)
acid sulphate
Zn(s) + 2HCl (aq) ZnCl2(aq) + H2(g) Here, zinc displaces copper from its salt solution.
Zinc Hydrochloric Zinc Hydrogen However, if we take zinc sulphate solution and put a
acid chloride strip of copper metal in this solution, no reaction occurs.
ZnSO4 (aq) + Cu(s) No reaction
Zn(s) + H2SO4(aq) ZnSO4 (aq) + H2(g) Zinc Copper
Zinc Sulphuric Zinc Hydrogen sulphate
acid sulphate
(C) Iron reacts slowly with dilute HCl or dil. H2SO4 and This means that copper cannot displace zinc metal
therefore, it is less reactive than zinc and aluminium. from its solution. Thus, we can conclude that zinc is
Fe(s) + 2HCl(aq) FeCl2(aq) + H 2(g) more reactive than copper. However, if we put gold or
Iron Hydrochloric Ferrous Hydrogen platinum strip in the copper sulphate solution, then
acid chloride copper is not displaced by gold or platinum. Thus, gold
Fe(s) + H2SO4(aq) FeSO4(aq) + H2(g) and platinum are less reactive than copper.
Iron Sulphuric Ferrous Hydrogen
acid sulphate REACTIVITY SERIES OF METALS
(D) Copper does not react with dil. HCl or dil H2SO4 .
Cu(s) + HCl (aq) No reaction (a) Introduction :
Cu(s) + H2SO4(aq) No reaction
Therefore copper is even less reactive than iron. We have learnt that some metals are chemically very
reactive while others are less reactive or do not react at all.
The order of reactivity of different metals with dilute
acid: On the basis of reactivity of different metals with oxygen,
water and acids as well as displacement reactions,
Na > Mg > Al > Zn > Fe > Cu the metals have been arranged in the decreasing order
Reactivity with dilute acids decreases from sodium to of their reactivities.
copper. The arrangement of metals in order of decreasing
reactivities is called reactivity series or activity series
(iv) Reactions of metals with salt solutions : of metals.
When a more reactive metal is placed in a salt solution The activity series of some common metals is given in
of less reactive metal, then the more reactive metal Table. In this table, the most reactive metal is placed at
displaces the less reactive metal from its salt solution. the top whereas the least reactive metal is placed at
For example, we will take a solution of copper sulphate the bottom. As we go down the series the chemical
(blue coloured solution) and put a strip of zinc metal in reactivity of metals decreases.
REACTIVITY SERIES OF METALS
Lithium Li Most reactive metal
Potassium K
Barium Ba
Metals
more reactive Sodium Na
than hydrogen Calcium Ca
Reactivity decreases
Reactivity increases

Magnesium Mg
Aluminium Al
Zinc Zn
Iron Fe
Nickel Ni
Tin Sn
Lead Pb
Hydrogen H
Copper Cu

Metals Mercury Hg
less reactive Silver Ag
than hydrogen
Gold Au
Platinum Pt Least reactive metal

PAGE # 111
(b) Reasons for Different Reactivities :
In the activity series of metals, the basis of reactivity is HOW METALS REACT WITH NON-METALS
the tendency of metals to lose electrons. If a metal can
lose electrons easily to form positive ions, it will react Octet Rule : Octet rule was given by G.N. Lewis and W.
readily with other substances. Therefore, it will be a Kossel in 1916.
reactive metal. On the other hand, if a metal loses
electrons less readily to form a positive ion, it will react According to octet rule “an atom whose outermost
slowly with the other substances. Therefore, such a shell contains 8 electrons (octet) is stable.”
metal will be less reactive. For example, alkali metals This rule, however, does not hold good in case of certain
such as sodium and potassium lose electrons very small atoms like helium (He) in which presence of 2
readily to form alkali metal ions, therefore, they are very electrons (duplet) in the outermost shell is considered
reactive. to be the condition of stability.

(c ) Di sp la ce me nt of Hy dr og en from Ac id s Examples of elements whose atoms have fully filled


by Metals : or 8 e– in their outermost shell are -
All metals above hydrogen in the reactivity series (i.e.
more active than hydrogen) like zinc, magnesium, nickel Atomic Electronic No. of valence
Element Symbol
etc can liberate hydrogen from acids like HCl and H2SO4. Number configuration electrons
These metals have greater tendency to lose electrons Neon Ne 10 2,8 8
than hydrogen. Therefore, the H+ ions in the acids will Argon Ar 18 2,8,8 8
accept electrons and give hydrogen gas as : Krypton Kr 36 2,8,18,8 8
M M+ (aq) + e–  Note :
Metal
All noble gases contain 8 valence electrons (except
1 He in which 2 valence electrons are present) and are
H+ (aq) + e– H 
2 2 stable. They do not usually form bonds with other
(From acid) elements.

The metals which are below hydrogen in the reactivity Atoms combine with one another to achieve the inert
series (i.e. less reactive than hydrogen) like copper, gas electron arrangement and become stable. Atoms
silver, gold cannot liberate hydrogen form acids like form chemical bonds to achieve stability by acquiring
HCl, H2SO4 etc. These metals have lesser tendency to the inert gas configuration or by completing their octet
lose electrons than hydrogen. Therefore, they cannot or duplet ( in case of small atoms) in outermost shell.
give electrons to H+ ions. An atom can achieve the inert gas electron
arrangement in three ways -
( d) R ea ct iv it y Se ri es a nd Dis pl ac em ent
Reacti ons : (i) by losing one or more electrons.
(ii) by gaining one or more electrons.
The reactivity series can also explain displacement
reactions. In general, a more reactive metal (placed (iii) by sharing one or more electrons.
higher in the activity series) can displace the less
 Note : Noble gases do not usually form bonds with other
reactive metal from its solution. For example, zinc,
elements, because they are stable. So, atoms of elements
displaces copper from its solution.
have the tendency to combine with one another to achieve
Zn (s) + CuSO4 (aq) ZnSO4(aq) + Cu(s)
the inert gas configuration.
(e) Usefulness of Activity Series :
The activity series is very useful and it gives the CONCEPT OF BOND
following informations :
(i) The metal which is higher in the activity series is Except the elements of group 18 of the periodic table
more reactive than the other. Lithium is the most reactive all the elements of the remaining group, at normal
and platinum is the least reactive metal. temperature and pressure, are not stable in
independent state. These elements form stable
(ii) The metals which have been placed above hydrogen compounds either by combining with the other atoms
are more reactive than hydrogen and these can or with their own atoms. W hen in gross electronic
displace hydrogen from its compounds like water and configuration of the elements there are 8 electrons
acids to liberate hydrogen gas. present then these elements do not take part in the
(iii) The metals which are placed below hydrogen are chemical reaction because atoms containing 8
less reactive than hydrogen and these cannot displace electrons in their outermost shell are associated with
hydrogen from its compounds like water and acids. extra stability and less energy.
(iv) A more reactive metal (placed higher in the activity Atoms with other electronic configuration, which do
series) can displace the less reactive metal from its not contain eight electrons in their outermost shell,
solution. are unstable and to achieve the stability they chemically
combine in such a manner that they achieve eight
(v) Metals at the top of the series are very reactive and, electrons in their outermost shell.
therefore, they do not occur free in nature. The metals
Two or more than two types of atoms mutually combine
at the bottom of the series are least reactive and,
with each other to achieve stable configuration of eight
therefore, they normally occur free in nature. For
example, gold, present in the reactivity series is found valence electrons. Attempt to achieve eight electrons
in free state in nature. in the outermost orbit of an element is the reason
behind its chemical reactivity or chemical bonding.

PAGE # 112
(ii) Physical nature : Ionic compounds are solid and
IONIC OR ELECTROVALENT BOND relatively hard due to strong electrostatic force of
attraction between the ions of ionic compound.
This bond is formed by the atoms of electropositive
(iii) Crystal structure : X-ray studies have shown that
and electronegative elements. Electropositive ionic compounds do not exist as simple single
elements lose electrons in chemical reaction and molecules as Na+ Cl–. This is due to the fact that the
electronegative elements gain electrons in chemical forces of attraction are not restricted to single unit such
reaction. When an atom of electropositive element come as Na+ and Cl– but due to uniform electric field around
in contact with that of an electronegative element then an ion, each ion is attracted to a large number of other
the electropositive atom loses electron & becomes ions. For example, one Na+ ion will not attract only one
positively charged, while the electronegative atom Cl– ion but it can attract as many negative charges as it
gains the electron to become negatively charged. can. Similarly, the Cl– ion will attract several Na+ ions.
Electrostatic force of attraction works between the As a result, there is a regular arrangement of these
positively and negatively charged ions due to which ions in three dimensions as shown in diagram. Such
both ions are bonded with each other. As a result, a a regular arrangements is called crystal lattice.
chemical bond is produced between the ions, forming
Ionic or Electrovalent compound.

 Note :
Number of electrons donated or accepted by any
element is called Electrovalency.
In an ionic compound every cation is surrounded by a
fixed number of anions and every anion is surrounded
by a fixed number of cations and they are bonded in a
fixed geometry in a three dimensional structure.
Example : Sodium chloride compound.
Sodium atom (Electropositive element) by losing an
electron from its outermost orbit, gets converted into a
cation and attains noble gas like stable configuration.
Energy required for this process is called “ionization
energy.” (iv) Melting point and boiling point : Strong electrostatic
force of attraction is present between ions of opposite
Na + IE  Na+ + e– charges. To break the crystal lattice more energy is
(2,8,1) (2,8) required so their melting points and boiling points are
high.
Chlorine atom (Electronegative element) accepts the
electron donated by sodium atom in its outermost (v) Solubility : Ionic compounds are generally soluble
orbit and forms chloride anion. in polar solvents like water and insoluble in non - polar
In this process energy is released which is known as solvents like carbon tetrachloride, benzene, ether,
“electron affinity.” alcohol etc.

(vi) Brittle nature : Ionic compounds on applying


+ e–  + EA external force or pressure are broken into small pieces,
such substances are known as brittle and this property
(2,8,7) (2,8,8)
is known as brittleness. When external force is applied
on the ionic compound, layers of ions slide over one
Due to the opposite charges on the Na+ and Cl¯ ions,
another and particles of the same charge come near
they are bonded by electrostatic force of attraction to
to each other as a result due to the strong repulsion
form NaCl compound. force, crystals of compounds are broken.
Na+ + Cl–  or NaCl
+ – + – + + – + – +
Here electrovalency of sodium and chlorine atom is – + – + – – + – + –
one. + – + – +
+ – + – +
For the formation of ionic bond, it is necessary that the – + – + –
– + – + –
ionization energy of electropositive element should be
less and the electron affinity of electronegative element Brittle nature of ionic compounds
should be high. (vii) Electrical conductivity : Electrical conductivity in
any substance is due to the movement of free electrons
(a) Properties of Ionic Compounds : or ions. In metals electrical conductivity is due to the
(i) Ionic compounds consist of ions : All ionic free movement of valency electrons. An ionic compound
compounds consist of positively and negatively exhibits electrical conductivity due to the movement of
charged ions and not molecules. For example, sodium ions either in the fused state or in the soluble state in
chloride consists of Na+ and Cl– ions, magnesium the polar solvent. But in the solid state due to strong
fluoride consists of Mg2+ and F– ions and so on. electrostatic force of attraction free ions are absent so
they are insulator in the solid state.

PAGE # 113
(C) Red brown rust is seen on the surface of iron nail
CORROSION OF METALS kept in the presence of both air and water in the third
test tube. This tells us that rusting of iron takes place
Surface of many metals is easily attacked when
in the presence of both air and water together.
exposed to atmosphere. They react with air or water
present in the environment and form undesirable (iii) Prevention of rusting :
compounds on their surfaces. These undesirable (A) Corrosion of metals can be prevented by coating
compounds are generally oxides. the metal surface with a thin layer of paint, varnish or
Thus, corrosion is a process of deterioration of metal grease.
as a result of its reaction with air or water (present in (B) Iron is protected from rusting by coating it with a
environment) surrounding it. thin layer of another metal which is more reactive than
iron. This prevents the loss of electrons from iron
(a) Corrosion of Iron : because the active metal loses electrons in preference
Iron corrodes readily when exposed to moisture and to iron. Zinc is commonly used for covering iron surface
gets covered with a brown flaky substance called rust. of iron. The process of covering iron with zinc is called
This is also called Rusting of Iron. Chemically, the rust galvanization. Iron is also coated with other metals
is hydrated iron (III) oxide, Fe2O3.xH2O. Rusting is an such as tin known as tin coating.
oxidation process in which iron metal is slowly oxidized (C) By alloying : Some metals when alloyed with other
by the action of air (in presence of water). Therefore, metals become more resistant to corrosion. For
rusting of iron takes place under the following example, when iron is alloyed with chromium and
conditions: nickel, it forms stainless steel. This is resistant to
• Presence of air (or oxygen) corrosion and does not rust at all.
• Presence of water (moisture) (D) To decrease rusting of iron, certain antirust
• More the reactivity of the metal, the more will be the solutions are used. For example, solutions of alkaline
phosphates are used as antirust solutions.
possibility of the metal getting corroded.
(i) Experiment to show that rusting of iron requires (b) Corrosion of Aluminium :
both air and water - Due to the formation of a dull layer of aluminium oxide
We take three test tubes and put one clean iron nail in when exposed to moist air, the aluminium metal loses
each of the three test tubes: its shine very soon after use. This aluminium oxide
(A) In the first test tube containing iron nail, we put layer is very tough and prevents the metal underneath
some anhydrous calcium chloride to absorb water (or from further corrosion (because moist air is not able to
moisture) from the damp air present in the test tube pass through this aluminium oxide layer). This means
and make it dry. sometimes corrosion is useful.
(B) In the second test tube containing iron nail, we put
boiled water because boiled water does not contain (c) Corrosion of Copper :
any dissolved air or oxygen in it. A layer of oil is put over W hen a copper object remains in damp air for a
boiled water in the test tube to prevent the outside air considerable time, then copper reacts slowly with
from mixing with boiled water. carbon dioxide and water of air to form a green coating
(C) In the third test tube containing an iron nail, we put of basic copper carbonate [CuCO 3.Cu(OH)2] on the
unboiled water so that about two-third of the nail is surface of the object. Since copper metal is low in the
immersed in water and the rest is above water exposed reactivity series, the corrosion of copper metal is very,
to damp air. very slow.
After one week, we observe the iron nails kept in all the
three test tubes. (d) Corrosion of Silver :
Silver is a highly unreactive metal, so it does not reacts
with oxygen of air easily. But, air usually contains a little
of sulphur compounds such as hydrogen sulphide gas
(H2S), which reacts slowly with silver to form a black
coating of silver sulphide (Ag 2S). Silver ornaments
gradually turn black due to the formation of a thin silver
sulphide layer on their surface and silver is said to be
tarnished.

ALLOY

An alloy is a homogenous mixture of two or more metals


or a metal and a non-metal.
(ii) We will obtain the following observations from
For example, iron is the most widely used metal. But it
the experiment :
is never used in the pure form. This is because iron is
(A) No rust is seen on the surface of iron nail kept in very soft and stretches easily when hot. But when it is
dry air in the first test tube. This tells us that rusting of mixed with a small amount of carbon (about 0.5 to
iron does not takes place in air alone.
1.5%), it becomes hard and strong. The new form of
(B) No rust is seen on the surface of iron nail kept in air iron is called steel.
free boiled water in the second test tube. This tells us
that rusting of iron does not take place in water alone.

PAGE # 114
(a) Objectives of Alloy Making : (C) German Silver : It is an alloy of copper, zinc and nickel
having the composition: Cu=60%, Zn=20%, Ni=20%. It is
Alloys are generally prepared to have certain specific used for making silverware, utensils and for
properties which are not possessed by the constituent electroplating.
metals. The main objectives of alloy-making are: (v) Alloying of Gold : Pure gold is very soft and cannot
(i) To increase resistance to corrosion : For example, be used as such for jewellery. Therefore, it is generally
alloyed with other metals commonly copper or silver to
stainless steel is prepared which has more resistance make it harder and modify its colour. The purity of gold
to corrosion than iron. is expressed as carats. Pure gold is of 24 carat. A 18
(ii) To modify chemical reactivity : The chemical carat gold means that it contains 18 parts of gold in 24
parts by weight of alloy. Most of the jewellery is made of
reactivity of sodium is decreased by making an alloy 22 carat gold.
with mercury which is known as sodium amalgam.
(iii) To increase the hardness : Steel, an alloy of iron AMALGAM
and carbon is harder than iron.
Amalgams are homogenous mixtures of a metal and
(iv) To increase tensile strength : Magnalium is an mercury. For example, sodium amalgam contains
alloy of magnesium and aluminium. It has greater sodium and mercury.
tensile strength as compared to magnesium and Different amalgams are prepared according to their
aluminium. uses. For example,
(v) To produce good casting : Type metal is an alloy of (i) Sodium amalgam is produced to decrease the
chemical reactivity of sodium metal. It is also used as
lead, tin and mercury. a good reducing agent.
(vi) To lower the melting point : For example, solder is (ii) Tin amalgam is used for silvering cheap mirrors.
an alloy of lead and tin (50% Pb and 50% Sn). It has a
(iii) The process of amalgamation is used for the
low melting point and is used for welding electrical extraction of metals like gold or silver from their native
wires together. ores.
(b) Some Important Alloys :
The approximate composition and uses of some OCCURRENCE OF METALS
important alloys are given below:
(i) Steel : Steel is an alloy of iron and carbon containing All metals are present in the earth’s crust either in the
0.5 to 1.5% carbon. Steel is very hard, tough and strong. free state or in the form of their compounds. Aluminium
It is used for making rails, screws, girders, bridges is the most abundant metal in the earth’s crust. The
etc. Steel can also be used for the construction of second most abundant metal is iron and third one is
buildings, vehicles, ships, etc.
calcium.
(ii) Alloy Steels : Steel obtained by the addition of some
other elements such as chromium, vanadium, titanium, (a) Native and Combined States of Metals :
molybdenum, manganese, cobalt or nickel to carbon Metals occur in the crust of earth in the following two
steel are called Alloy Steels.
states -
(iii) Alloys of Aluminium : The common alloys of
aluminium are: (i) Native state or free state : A metal is said to occur in
(A) Duralumin : It is an alloy containing aluminium, a free or a native state when it is found in the crust of
copper and traces of magnesium and manganese. Its the earth in the elementary or uncombined form.
percentage composition is - Al=95%, Cu=4%,
The metals which are very unreactive (lying at the
Mg=0.5%, Mn=0.5% It is stronger than pure aluminium.
Since duralumin is light and yet strong, it is used for bottom of activity series) are found in the free state.
making bodies of aircrafts, helicopters, jets and These have no tendency to react with oxygen and are
kitchenwares like pressure cookers etc. not attacked by moisture, carbon dioxide of air or other
(B) Magnalium : It is an alloy of aluminium and non-metals. Silver, copper, gold and platinum are
magnesium having the composition: Al=95%, Mg=5% It some examples of such metals.
is very light and hard. It is more hard than pure
aluminium. It is used for making light instruments, (ii) Combined state : A metal is said to occur in a
balance beams, pressure cookers etc. combined state if it is found in nature in the form of its
(C) Alnico : It is an alloy containing aluminium, iron, compounds. e.g. Sodium , magnesium etc.
nickel, and cobalt. It is highly magnetic in nature and
can be used for making powerful magnets.  Note :
Copper and silver are metals which occur in the free
(iv) Alloys of Copper : The important alloys of copper
are Brass and Bronze. state as well as in the combined state.
(A) Brass : It is an alloy of copper and zinc having the
composition - Cu=80%, Zn=20%. Brass is more MINERALS AND ORES
malleable and more strong than pure copper. It is used
for making cooking utensils, pipes, hardware, nuts, The natural substances in which metals or their
bolts, screws, springs etc. compounds occur either in native state or combined
(B) Bronze : It is an alloy of copper and tin having the state are called minerals.
composition: Cu=90%, Sn=10% Bronze is very tough The minerals are not pure and contain different types
and highly resistant to corrosion. It is used for making of other impurities.The impurities associated with
utensils, statues, coins, hardware etc.
minerals are collectively known as gangue or matrix.
The mineral from which the metal can be conveniently

PAGE # 115
and profitably extracted, is called an ore. (b) Concentration of the ore or enrichment of the ore.
For example, aluminium occurs in the earth’s crust in
the form of two minerals, bauxite (Al2O3.2H2O) and clay (c) Extraction of metal from the concentrated ore.
(Al2O3.2SiO2.2H2O). Out of these two, aluminium can (d) Refining or purification of the impure metal.
be conveniently and profitably extracted from bauxite . So,
These steps are briefly discussed below -
bauxite is an ore of aluminium.
Note : Oxygen is the most abundant element in earth’s (a) Crushing and Grinding of Ore :
crust. Most of the ores occur as big rocks in nature. They are
broken into small pieces with the help of crushers.
(a) Types of Ores :
These pieces are then reduced to fine powder with the
The most common ores of metals are oxides, help of a ball mill or a stamp mill.
sulphides, carbonates, sulphates, halides, etc. In
general, very unreactive metals (such as gold, silver,
platinum etc.) occur in elemental form or free state.

(i) Metals which are only slightly reactive occur as


sulphides (e.g., CuS, PbS etc.).
(ii) Reactive metals occur as oxides (e.g., MnO2, Al2O3
etc.).

(iii) Most reactive metals occur as salts as carbonates,


sulphates, halides etc. (e.g., Ca, Mg, K etc.).
Some common ores are listed in the table
Nature of
Metal Name of the ore Composition
ore
Aluminium Bauxite Al2O3.2H2O (a) Crushing in (B) Pulverisation in
Copper Cuprite Cu2O hammer mill stamp mill
Oxide
ores Magnetite Fe3O4 (b ) Concentra tion of Ore or Enr ichm ent of
Iron
Haematite Fe2O3 Ore :
Copper pyrites CuFeS2
Copper The process of removal of unwanted impurities
Copper glance Cu2S
Sulphide (gangue) from the ore is called ore concentration or
ores Zinc Zinc blende ZnS ore enrichment.
Lead Galena PbS
Mercury Cinnabar HgS (i) Hydraulic washing (washing with water) :
Carbonate Calcium Limestone CaCO3 Principle : This method is based upon the difference
ores Zinc Calamine ZnCO3 in the densities of the ore particles and the impurities
Sodium Rock salt NaCl (gangue).
Halide Magnesium Carnallite KCl.MgCl2.6H2O Ores of iron, tin and lead are very heavy and, therefore,
ores Calcium Fluorspar CaF2 they are concentrated by this method.
Silver Horn silver AgCl
Calcium Gypsum CaSO4.2H2O
Sulphate Magnesium Epsom salt MgSO4.7H2O
ores Barium Barytes BaSO4
Lead Anglesite PbSO4

METALLURGY
The process of extracting metals from their ores and
then refining them for use is called metallurgy.
The ores generally contain unwanted impurities such
as sand, stone, earthy particles, limestone, mica, etc.,
these are called gangue or matrix.
The process of metallurgy depends upon the nature of A hydraulic classifier
the ore, nature of the metal and the types of impurities
present. Therefore, there is not a single method for the (ii) Froth flotation process :
extraction of all metals. However, most of the metals Principle : This method is based on the principle of
can be extracted by a general procedure which involves difference in the wetting properties of the ore and
the following steps. gangue particles with oil and water respectively.
Various steps involved in metallurgical processes are -
This method is commonly used for sulphide ores.
(a) Crushing and grinding of the ore.

PAGE # 116
Al2O3.xH2O(s) 1470 K
(iii) Magnetic separation :   Al2O3(s) + xH2O(g)
Principle : This method depends upon the difference
(b) Other examples In the metallurgy of silver and that
in the magnetic properties of the ores and gangue.
of gold :
This method is used for the concentration of haematite,
The respective metal is leached with a dilute solution
an ore of iron.
of NaCN or KCN in the presence of air (for O2) from
which the metal is obtained later by replacement:
Ore 4M(s) + 8CN–(aq)+ 2H2O(aq) + O2(g)  4[M(CN)2]–(aq) +
4OH–(aq) (M = Ag or Au)
2[M(CN)2]–(aq.)+ Zn(s)  [Zn(CN)4]2-(aq) + 2M(s)
Elec trom agnet
(c) Extraction of Metal From The Concentrated Ore :
Metal is extracted from the concentrated ore by the
particles
M oving belt following steps :
M agnetic (i) Conversion of the concentrated ore into its oxide :
part ic les The production of metal from the concentrated ore
mainly involves reduction process. This can be usually
Magnetic separation method
done by two processes known as calcination and
 Note : roasting . The method depends upon the nature of the
The froth floatation process is commonly used for the ore.
sulphide ores of copper, zinc, lead etc.
(ii) Conversion of oxide to metal by reduction process
(iv) Leaching :
(i) Conversion of Ore into Metal Oxide :
Leaching is often used if the ore is soluble in some
suitable solvent. These are briefly discussed below :
The following examples illustrate the procedure:
(A) Calcination: It is the process of heating the concentrated
(a) Leaching of alumina from bauxite: ore in the absence of air.
The principal ore of aluminium, bauxite, usually The calcination process is used for the following
contains SiO2, iron oxides and titanium oxide (TiO2) as changes :
impurities. Concentration is carried out by digesting • to convert carbonate ores into metal oxide.
the powdered ore with a concentrated solution of
• to remove water from the hydrated ores.
NaOH at 473-523 K and 35-36 bar pressure. This way,
Al2O 3 is leached out as sodium aluminate (and SiO 2 • to remove volatile impurities from the ore.
too as sodium silicate) leaving the impurities behind: For example :
ZnCO3(s) Calcinatio n
Al2O3(s) + 2NaOH(aq) + 3H2O(l) 2Na[Al(OH)4](aq)
   ZnO(s) + CO2(g)
Calamine Zinc oxide Carbon dioxide
The aluminate in solution is neutralised by passing
Calcinatio n
CO2 gas and hydrated Al2O3 is precipitated. At this stage, FeCO3(s)      FeO(s) + CO2(g)
the solution is seeded with freshly prepared samples Siderite Iron (II) Carbon
of hydrated Al2O3 which induces the precipitation: oxide dioxide
2Na[Al(OH)4](aq) + CO2(g)  Al2O3.xH2O(s) (B) Roasting: It is the process of heating the
concentrated ore strongly in the presence of excess air.
+ 2NaHCO3(aq)
The sodium silicate remains in the solution and This process is used for converting sulphide ores to
hydrated alumina is filtered, dried and heated to give metal oxide. In this process, the following changes
back pure Al2O3: take place :

PAGE # 117
• the sulphide ores undergo oxidation to their oxides. electron gain or electronation).
ZnO(s) + C(s)  Zn (s) + CO(g)
• moisture is removed.
Zinc Carbon Zinc Carbon
• volatile impurities are removed. oxide (Reducing agent) metal monoxide
For example : Fe2O3(s) + 3C(s)  2Fe(s) + 3CO(g)
Ferric Carbon Iron Metal Carbon
Roasting
2ZnS(s) + 3O2 (g)    2ZnO(s) + 2SO2(g) oxide monoxide
Zinc Oxygen Zinc oxide Sulphur
sulphide (from air) dioxide PbO(s) + C(s)  Pb(s) + CO(g)
(Zinc blende) Lead oxide Carbon Lead metal Carbon
4FeS2(s) + 11O2(g)    2Fe2O3(s) + 8SO2(g)
Roasting monoxide
Iron Oxygen Ferric Sulphur One disadvantage of using carbon as reducing agent
pyrites oxide dioxide is that small traces of carbon are added to metal as
impurity. Therefore, it contaminates the metals.
 Note :
Note : Coke is very commonly used as a reducing agent
Calcination is used for hydrated and carbonate ores because it is cheap.
and roasting is used for sulphide ores.
Smelting :
(ii) Conversion of Metal Oxide into Metal :
Slag formation : In many extraction processes, an oxide
The metal oxide formed after calcination or roasting is is added deliberately to combine with other impurities
converted into metal by reduction. The method used and form a stable molten phase immiscible with molten
for reduction of metal oxide depends upon the nature metal called a slag. The process is termed smelting.
and chemical reactivity of metal. The principle of slag formation is essentially the
following :
The metals can be grouped into the following three
Non-metal oxide + Metal oxide  Fusible slag
categories on the basis of their reactivity :
(acidic oxide) (basic oxide) (easily melted)
• Metals of low reactivity. Removal of unwanted basic and acidic oxides: For
• Metals of medium reactivity. example, FeO is the impurity in extraction of Cu from
copper pyrite.
• Metals of high reactivity.
2CuFeS2 + 4O2  Cu2S + 2FeO + 3SO2
These different categories of metals are extracted by Cu2S + FeO + SiO2  FeSiO3 (Fusible slag)+Cu2S(matte)
different techniques. The different steps involved in 
separation are as follows : (roasted pyrite) (upper layer) (lower layer)
(a) Self reduction : Metals placed low in the reactivity series
are very less reactive. They can be obtained from their Matte also contains a very small amount of iron(II) sulphide.
oxides by simply heating in air. To remove unwanted acidic impurities like sand and P4O10,
2HgS(s) + 3O2(g)  2HgO(s) + 2SO2(g) smelting is done in the presence of limestone.
Mercuric Oxygen Mercuric Sulphur CaCO3  CaO + CO2
sulphide oxide dioxide CaO + SiO2  CaSiO3 (fusible slag)
6CaO + P4O10  2Ca3(PO4)2 (fusible slag - Thomas slag)
2HgO(s)  2Hg(l) + O2(g)
Mercuric Mercury metal Oxygen Properties of a slag :
oxide (i) Slag is a fusible mass.
(b) Chemical Reduction (For metals in the middle of the (ii) It has low melting point.
reactivity series) : (iii) It is lighter than and immiscible with the molten
The metals in the middle of the reactivity series, such metal. It is due to these impurities that the slag floats
as iron, zinc, lead, copper etc. are moderately reactive. as a separate layer on the molten metal and can thus
These are usually present as sulphides or carbonates. be easily separated from the metal. The layer of the
Therefore, before reduction the metal sulphides and slag on the molten metal prevents the metal from being
carbonates must be converted to oxides. This is done oxidised.
by roasting and calcination. The oxides of these metals
cannot be reduced by heating alone. Therefore, these Type of flux : Fluxes are of two types viz., acidic flux and
metal oxides are reduced to free metal by using basic flux.
chemical agents like carbon, aluminium, sodium or (a) Acidic flux : It is an acidic oxide (oxide of a non-
calcium. metal) like SiO2, P2O5, B2O3 (from borax). It is used to
(A) Carbon reduction: The oxides of moderately reactive remove the basic impurity like CaO, FeO, MgO etc. The
metals (occurring in the middle of reactivity series) like acidic flux combines with the basic impurity and forms
zinc, copper, nickel, tin, lead etc. can be reduced by a slag.
using carbon as reducing agent. (b) Basic flux : It is a basic oxide (i.e., oxide of a metal)
Reduction of the metal oxide usually involves heating like CaO (obtained from lime stone, CaCO3), MgO (from
it with some other substance acting as a reducing magnesite, MgCO3), haematite (Fe2O3) etc. It is used
agent (C or CO or even another metal). The reducing to remove the acidic impurity like SiO2, P2O 5 etc. The
agent (e.g., carbon) combines with the oxygen of the basic flux combines with the acidic impurity and forms
metal oxide. a slag.
MxOy + yC  xM + yCO Thus, slag can be defined as a fusible mass, which is
Some metal oxides get reduced easily while others obtained when a flux reacts with an infusible acidic or
are very difficult to be reduced (reduction means

PAGE # 118
basic impurity present in the oxide ore. Extraction of Magnesium :
Fe2O3(s) + 3CO(g)  2Fe(s) + 3CO2(g) (i) From Carnallite :
Ferric Carbon Iron Carbon The ore is dehydrated in a current of hydrogen chloride
oxide monoxide dioxide and the mixture of fused chlorides is electrolysed.
(C) Thermite Process (Reduction with aluminium):
Certain metal oxides are reduced by aluminium to (ii) From Sea water (Dow's process) :
metals. Sea water contains 0.13% magnesium as chloride
3MnO2 (s) + 4Al(s)  3Mn(l) + 2Al2O3 (s) and sulphate. It involves following steps.
Manganese Aluminium Manganese Aluminium
dioxide oxide (a) Precipitation of magnesium as magnesium
hydroxide by slaked lime :
Cr2O3 (s) + 2Al (s)  2Cr(l) + Al2O3 (s) MgCl2 + Ca(OH)2  Mg(OH)2 + CaCl2
Chromium Aluminium Chromium Aluminium
oxide oxide (b) Preparation of hexahydrated magnesium
chloride :
Fe2O3 (s) + 2Al (s)  2Fe(l) + Al2O3 (s) Mg(OH)2 + 2HCl(aq)  MgCl2  + 2H2O
Iron Aluminium Iron Aluminium The solution on concentration and crystallisation gives
oxide oxide the crystals of MgCl2.6H2O

Note : Reduction of metal oxides with aluminium is known (c) Preparation of anhydrous magnesium chloride :
as aluminothermy or thermite process.
 (calcination)
MgCl2. 6H2O 
Dry HCl(g)
 MgCl + 6H O
2 2
Electrolytic reduction:
The oxides of active metals (which are high up in the It is not made anhydrous by simple heating because it
activity series) are very stable and cannot be reduced gets hydrolysed
by carbon or aluminium. These metals are commonly MgCl2. 6H2O  MgO + 5H2O + 2HCl
extracted by the electrolysis of their fused salts using
suitable electrodes. This is also called electrolytic
reduction i.e. reduction by electrolysis.
In the metallurgy of aluminium, purified Al2O3 is mixed
with Na3AlF6 or CaF2 which lowers the melting point of
the mix and brings conductivity. The fused matrix is
electrolysed. Steel cathode and graphite anode are
used. The graphite anode is useful here for reduction
to the metal.The overall reaction may be taken as:
2Al2O3 + 3C  4Al + 3CO2
This process of electrolysis is widely known as Hall-
Heroult process.
The electrolysis of the molten mass is carried out in
an electrolytic cell using carbon electrodes. The oxygen
liberated at anode reacts with the carbon of anode
producing CO and CO2. This way for each 0.5 kg of
aluminium produced, about 0.5 kg of carbon anode
is burnt away. The electrolytic reactions are: (d) Electrolysis of fused anhydrous MgCl2 :
Magnesium chloride obtained by any of the above
Cathode: Al3+(melt) + 3e-  Al(l) methods is fused and mixed with sodium chloride and
Anode: C(s) + O2- (melt)  CO(g) + 2e- calcium chloride in the temperature range of 973 - 1023
C(s) + 2O2- (melt)  CO2 (g) + 4e- K. The molten mixture is electrolysed. Magnesium is
liberated at the cathode (iron pot) and chlorine is
evolved at graphite anode.
Note : The process of extraction of metals by electrolysis
process is called electrometallurgy.
MgCl2 Mg2+ + 2Cl–
At cathode : Mg + 2e  Mg(99% pure) ;
2+ –

At anode : 2Cl–  Cl2 + 2e–


A stream of coal gas is passed through the pot to
prevent oxidation of magnesium metal. The
magnesium obtained in liquid state is purified by
distillation under reduced pressure. (1 mm of Hg at
873 K).

(iii) From dolomite :


The concentrated ore is calcined at higher temperature

CaCO3 . MgCO3   CaO. MgO + 2CO2
It is then reduced by ferrosilicon at 1273 K under reduced
pressure.
2CaO. MgO + FeSi  2Mg + Fe + Ca2SiO4

PAGE # 119
(D) Refining of crude metal : Copper Copper (from impure anode)
The metal obtained by any of the above methods is ion
usually impure and is known as crude metal. The
process of purifying the crude metal is called refining. At cathode : Cu2+  + 2e– Cu
(a) Liquation : This is based on the principle that the Copper Copper ion
metal to be refined is easily fusible (melt easily) but (deposited at cathode)
the impurities do not fuse easily. The impure metal is Impurities from the blister copper deposit as anode
placed on the sloping hearth of a furnace and gently mud which contains antimony, selenium, tellurium,
heated. The metal melts and flows down leaving silver, gold and platinum; recovery of these elements
behind the impurities on the hearth. This method is may meet the cost of refining.
used for refining the metals having low melting points, Zinc may also be refined this way.
such as tin, lead, bismuth etc.
(b) Distillation : This method is used for the purification
of volatile metals (which form vapours readily). Impure
metal is heated and its vapours are separately
condensed in a receiver. The non-volatile impurities
are left behind. This is used for mercury, cadmium and
zinc.
(c) Zone refining : This method is based on the
principle that the impurities are more soluble in the
melt than in the solid state of the metal. A circular mobile
heater is fixed at one end of a rod of the impure metal
The molten zone moves along with the heater which is
moved forward. As the heater moves forward, the pure
metal crystallises out of the melt and the impurities
pass on into the adjacent molten zone. The process is
repeated several times and the heater is moved in the Note : In electrolytic refining impure metal is made anode
same direction. At one end, impurities get concentrated. and pure metal is made cathode.
This end is cut off. This method is very useful for
producing semiconductor and other metals of very high (a) Chemical refining (Vapour phase refining)
purity, e.g., germanium, silicon, boron, gallium and In this method, the metal is converted into its volatile
indium. compound and then collected. It is then heated so that
it gets decomposed to give pure metal. So, following
two requirements are essential for vapour phase
refining.
(i) The metal should form a volatile compound with an
available reagent.
(ii)The volatile compound should be easily
decomposable, so that the recovery is easy.

(i) Extraction of Nickel (Mond's process) :


Nickel forms a complex, the tetracarbonylnickel (0),
[Ni(CO)4] with carbon monoxide. This substance is
readily volatilized (boiling point 43ºC). This complex
(d) Electrolytic Refining: This is most general and widely on heating decomposes, giving pure metal and CO,
used method for the refining of impure metals. Many which is recycled.
metals such as copper, zinc, tin, nickel, silver, gold etc. The sequence of reaction is
are refined electrolytically. It is based upon the H2O(g) + C  CO(g) + H2
phenomenon of electrolysis. Ni(s) + 4 CO(g)  50 º C
 [Ni(CO4)] (g)
Anode : M  Mn+ + ne– [Ni (CO)4](g) 200º C
  Ni + 4CO(g)
Cathode : Mn+ + ne–  M
In this method, the crude metal is cast into thick rods (ii) Van Arkel-De Boer process : Small amounts of
and are made as anodes, while the thin sheets of very pure metals (Ti, Zr, or Bi) can be produced by this
pure metal are made as cathodes. An aqueous solution method. This process is based on the fact that iodides
of salt of the same metal is used as an electrolyte. On are the least stable of the halides. The impure element
passing current through the electrolyte, the pure metal is heated with iodine, producing a volatile iodide, TiI4,
from the anode dissolves into the electrolyte. An ZrI4, or BiI3. These are decomposed by passing the
equivalent amount of pure metal from the electrolyte is gas over an electrically heated filament of tungsten or
deposited on the cathode. The soluble impurities go tantalum that is white hot. The element is deposited
in the solution whereas the insoluble impurities settle on the filament and the iodine is recycled.
down at the bottom of the anode and are known as 1400 º C
Impure Ti + 2I2 50 – 250 º C TiI4    Ti + 2I2
anode mud. In this way, the pure metal from anode    Tungsten filament
goes into electrolyte and from electrolyte it goes to the The method is very expensive and is employed for the
cathode. preparation of very pure i.e. ultra pure metal for specific
use.
At anode : Cu  Cu+2 + 2e–

PAGE # 120
Ore
Gravity separation

Concentration By Physical
Leaching By Chemical method Electromagnetic separation
of ore method

By Conversion of By
Froath separation
Calcination Roasting
ore into oxide

Metal oxide
By Carbon
By Aluminium
Semlting and reduction
(of metal oxide) Self reduction
Electrolytic
Liquation By hydrogen
Impure metal Metal displacment
Distillation
Cupellation
Zone refining Refining
Electro refining
Vapour phase refining
Amalgamation Pure metal

PAGE # 121
10. W hich of the following metal reacts vigorously with
EXERCISE-1 oxygen ?
(A) Zinc (B) Magnesium
I. Physical & chemical properties & metals & non (C) Sodium (D) Copper
metals, Reactivity series II. Ionic bond

1. Which of the following properties is not a characteristic 11. Octet rule was given by -
of metals ? (A) Rutherford (B) Soddy
(A) Metallic lusture (C) Lewis & Kossel (D) None of these
(B) High density 12. Exception of octet rule is -
(C) Hardness (A) K (B) Ca
(D) Low melting and boiling point (C) N (D) He

2. Which of the following metals occur in liquid state at 13. Ionic bond is formed by -
room temperature ? (A) loss of electrons only.
(A) Mercury (B) Bromine (B) gain of electrons only.
(C) Gallium (D) A & C both (C) loss and gain of electrons both.
(D) sharing of electrons.
3. Reactivity of zinc is _________ than that of hydrogen.
(A) less 14. Ionic bond is formed between -
(A) two electropositive elements.
(B) more
(B) two electronegative elements.
(C) equal
(C) electropositive & electronegative elements.
(D) sometimes more sometimes less (D) None of these

4. Zn + xHCl  ZnCl2 + A, 15. During formation of ionic bond -


In above equation A & x are (A) there is force of repulsion between two negative
(A) H2, 2 (B) Cl2, 1 ions.
(C) H2, 3 (D) H2 , 4 (B) there is force of repulsion between two positive
ions.
5. When sodium reacts with cold water, then the product (C) there is a force of attraction between positive &
formed will be - negative ions.
(A) Na2O (B) NaOH (D) none of these
(C) Na2CO 3 (D) All of these
16. In the formation of ionic bond, cation is formed by -
6. What is the decreasing order of reactivity of following (A) gain of electron(s).
metals ? (B) loss of electron(s).
Na, Al, K, Cu, Ag, Fe (C) sharing of electron(s).
(A) Na > K > Al > Cu > Ag > Fe (D) None of these
(B) K > Na > Al > Cu > Fe > Ag 17. Ionic compounds have -
(C) K > Na > Al > Fe > Cu > Ag (A) low melting and high boiling points.
(D) K > Na > Al > Fe > Ag > Cu (B) high melting and low boiling points.
(C) low melting and low boiling points.
7. When a metal is added to dilute HCl solution, there is (D) high melting and high boiling points.
no evolution of gas. Metal is -
(A) K (B) Na 18. Ionic compounds conduct electricity in -
(C) Ag (D) Zn (A) solid state.
(B) fused state.
8. On addition of which metal, copper sulphate solution (C) gaseous state.
(blue colour) will be changed to colourless solution ? (D) Do not conduct electricity at all.
(A) Fe (B) Ag
(C) Zn (D) Hg 19. Ionic compounds are soluble in -
(A) water (B) benzene
9. Zn + H2O (Steam)  A + B (C) ether (D) alcohol
In the above equation (A) and (B) are
20. Physical nature of ionic compounds is -
(A) Zn & H2 (B) ZnH2 & O2 (A) solid (B) liquid
(C) ZnO2 & O2 (D) ZnO & H2 (C) gas (D) May exist in any state.

PAGE # 122
III. Metallurgy 32. Process of roasting and calcination takes place in -
(A) bessemer converter. (B) blast furnace.
21. Which of the following is/are oxide ore(s) ?
(C) reverberatory furnace. (D) electrolytic cell.
(A) Bauxite (B) Cuprite
(C) Haematite (D) All of these 33. Which reducing agent is used in chemical reduction ?
(A) C (B) CO
22. Horn silver is a/an -
(C) Al (D) All of these
(A) sulphate ore (B) halide ore
34. Which of the following is used in reduction of alumina ?
(C) sulphide ore (D) oxide ore
(A) Coke (B) Carbon monoxide
23. Carnallite is - (C) Aluminium (D) Electricity
(A) KCl. MgCl2 (B) KCl. MgCl2. 3H2O
(C) KCl. MgCl2.6H2O (D) KCl. MgCl2. H2O 35. For purification of which metal, liquation method is
used ?
24. Match column A with column B and select the correct
option - (A) Tin (B) Lead
Column A Column B (C) Bismuth (D) All of these
(Ore) (Nature of ore)
36. Which method is used in purification of mercury ?
(a) Copper glance (i) Sulphate ore
(A) Liquation
(b) Calamine (ii) Halide ore
(c) Rock salt (iii) Sulphide ore (B) Distillation
(d) Epsom salt (iv) Carbonate ore (C) Electrolytic refining
(A) a(i), b(ii), c(iii), d(iv) (B) a(iv), b(ii), c(iii), d(i) (D) Chemical reduction
(C) a(iii), b(iv), c(ii), d(i) (D) a(iv), b(i), c(ii), d(iii)
37. Which of the following methods is used for obtaining
25. Removal of impurities from ore is known as - metals of very high purity ?
(A) crushing and grinding (B) concentration of ore (A) Distillation (B) Zone refining
(C) minerals (D) gangue
(C) Liquation (D) Electrolytic refining
26. W hich of the following methods is used in the
38. W hich of the following methods is not used in
concentration of haematite ore ?
purification of metals ?
(A) Hydraulic washing
(B) Magnetic separation (A) Calcination (B) Liquation
(C) Froth floatation process (C) Distillation (D) None of these
(D) None of these
39. Anode mud is obtained in which process ?
27. Froth flotation method is used for the concentration of - (A) Roasting (B) Zone refining
(A) oxide ores (B) sulphide ores (C) Electrolytic refining (D) Calcination
(C) sulphate ores (D) halide ores
40. In thermite process reducing agent is -
28. W hich of the following methods is based on the
(A) C (B) CO
principle of the difference in the wetting properties of
(C) Al (D) none of these
the ore and gangue particles with water and oil ?
(A) Magnetic separation
IV. Corrosion, Alloy
(B) Froth flotation process
(C) Hydraulic washing
41. Pure gold is equal to -
(D) None of these
(A) 24 carat (B) 100 carat
29. Which of the following is most abundant metal on the (C) 22 carat (D) 1000 carat
earth’s crust ?
(A) Iron (B) Aluminium 42. Food cans are coated with tin and not with zinc because -
(C) Calcium (D) Oxygen (A) zinc is costlier than tin.
(B) zinc has higher melting point than tin.
30. Which of the following metal is found in native state ? (C) zinc is more reactive than tin.
(A) Sodium (B) Zinc (D) zinc is less reactive than tin.
(C) Gold (D) Iron
43. Chemically rust is -
31. Heating of concentrated ore in absence of air for
conversion in oxide ore is known as - (A) hydrated ferrous oxide.
(A) roasting (B) calcination (B) hydrated ferric oxide.
(C) reduction (D) none of these (C) only ferric oxide.
(D) None of these

PAGE # 123
44. W hich of the following methods is suitable for 5. Which of the following hydroxides on heating decom-
preventing an iron vessel from rusting ? poses to liberate oxygen as one of the products ?
(A) Applying grease [IJSO-Stage-I/2011]
(B) Applying paint
(A) Mercury hydroxide (B) Copper hydroxide
(C) Applying a coating of zinc (C) Sodium hydroxide (D) Zinc hydroxide
(D) All the above

6. Oxides are acidic , basic or amphoteric based on their


45. W hich of the following conditions are necessary for
metallic or non-metallic character. W hich one of the
rusting of iron ?
following oxides reacts with both HCl and NaOH?
(A) Presence of water only
[IJSO-Stage-II/2013]
(B) Presence of air only
(C) Presence of water and air both (A) CaO (B) ZnO
(D) None of these (C) SO2 (D) CO2

46. Silver metal becomes black on exposure to air by the


7. 1.84 g of Dolomite CaMg(CO3)2 ore was heated result-
coating of -
ing in a residue of constant weight 0.96 g. During heat-
(A) silver chloride (B) silver oxide
ing the metal of one of the product burnt with a dazzling
(C) silver sulphide (D) silver hydroxide white flame . The approximate percentage composi-
47. Alloys are a homogeneous mixture of - tion of the two products in the residue are respectively

(A) metals only [IJSO-Stage-II/2013]


(B) non -metals only (A) 54 and 46 (B) 46 and 54
(C) metals or a metal and non-metal
(C) 42 and 58 (D) 58 and 42
(D) None of these
8. Thermite reaction is one of the important reactions in
48. German silver is an alloy of -
the metallurgical industry. This reaction is best de-
(A) Cu and Ni (B) Cu, Sn and Ag
scribed as : [IJSO-Stage-II/2013]
(C) Cu, Zn and Ni (D) Cu, Ni, Fe and Mn
(A) iron is displacing aluminium from its ore where
49. An alloy which does not contain copper is - iron acts as reducing agent and aluminium as
(A) magnalium (B) bronze oxidizing agent
(C) brass (D) german silver (B) aluminium is displacing iron from its ore where
50. Which of the following is/are an alloy of aluminium ? iron acts as oxidizing agent and aluminium as
reducing agent
(A) Duralumin (B) Magnalium
(C) aluminium is displacing iron from its ore where
(C) Alnico (D) All of these
iron acts as a reducing agent and aluminium as
oxidizing agent.
EXERCISE-2 (D) iron is displacing aluminium from its ore where
iron acts as a oxidizing agent and aluminium as
COMPETITIVE EXAM PREVIOUS YEARS’ QUESTIONS : reducing agent.

1. The metal which cannot displace hydrogen from acid


9. Metals are arranged in reactivity series according to
is - [IJSO-Stage-I/2011]
their order of reactivity. Depending upon order of metal
(A) silver (B) sodium in the reactivity series it will be extracted from its ore.
(C) calcium (D) magnesium Which of the following methods will be used to extract
copper from its alloy cupferronickel ?
2. Correct formula of dolomite is - [IJSO-Stage-I/2011]
[IJSO-Stage-II/2013]
(A) CaCO3. MgCO3 (B) CaCO3. ZnCO3
I. Electrolysis.
(C) MgCO3. ZnCO3 (D) FeCO3. CaCO3
II. Reduction with Carbon
3. The element that is not present in the compound III. Calcination.
BARYTA is - [IJSO-Stage-I/2011] (A) I and II only (B) I, II and III.
(A) B (B) O (C) II and III only (D) I and III only
(C) H (D) Ba
10. The metal that does not give H2 on treatment with di-
4. Out of the following , the correct activity series of the
lute HCl is : [IJSO-Stage-I/2013]
metals is - [IJSO-Stage-I/2011]
(A) Zn (B) Fe
(A) K > Na > Ca > Mg (B) Na > K > Ca > Mg (C) Ag (D) Ca
(C) Mg > Ca > Na > K (D) Mg > Na > Ca > K

PAGE # 124
11. Choose the correct sets which represent the oxides 18. Elements A,B and C have atomic numbers x, x + 1 and
as acidic:basic:neutral:amphoteric respectively x + 2, respectively. ‘C’ is an alkali metal. ‘A’ reacts with
[IJSO-Stage-I/2015] another element ‘Y’ to form the compound ‘AY’ ‘A’ and
(i) CO2: MgO: N2O: H2O ‘Y’ belong to the same group. ‘AY’ possesses an
(ii) SO2: NO: CO: AI2O3 [IJSO-Stage-1/2016-17]
(iii) P2O5: ZnO: NO: AI2O3 (A) ionic bond (B) covalent bond
(iv) SO3: CaO : N2O : PbO (C) metallic bonding (D) coordinate bond
(A) i & ii (B) ii & iii
(C) iii & iv (D) iv& i 19. To prevent the formation of oxides, peroxides, and su-
peroxides, alkali metals are sometimes stored in an
12. Amongst the sixteen  and × marks, how many unreactive atmosphere. Which of the following gases
cases are incorrect ? [IJSO-Stage-I/2015] should not be used for lithium:
[IJSO-Stage-1/2016-17]
(A) Ne (B) Ar
(C) N2 (D) Kr

20. Choose that element which is most different from the


other three [IJSO-Stage-1/2016-17]
(A) Carbon (B) Nitrogen
(C) Silicon (D) Phosphorous
 means displacement reaction occurs
× means no displacement reaction occurs 21. Hennig Brand, one of the many alchemists was in
(A) 7 (B) 11 pursuit of the philosopher’s stone. Brand’s method is
(C) 10 (D) 9 believed to have consisted of evaporating large
quantities of urine to leave a black residue that was
13. A silver article turns black when kept open in air for few then left for a few months. The residue was then heated
days. The article when rubbed with toothpaste again with sand, driving off a variety of gases and oils. The
starts shining because [IJSO-Stage-I/2015] final substance to be driven off, was condensed as a
(A) Hydrogen peroxide present in the toothpaste reacts white solid, which he called as “cold fire” as it was
to give silver sulphate. luminous and glowed in the dark and also caught fire
(B) Hydrated silica present in the toothpaste reacts to on slight warming and producing a large quantity of
give silver oxide. light. It has also been called as the “Bearer of light”.
(C) Aluminium hydroxide present in the toothpaste Which element is “cold fire”. ?
reacts to give silver hydroxide. [IJSO-Stage-1/2016-17]
(D) Calcium carbonate present in the toothpaste reacts (A) Lithium (B) Tungsten
(C) Phosphorous (D) Cesium
to give silver carbonate.

22. Zinc and iron are two metals which have many uses
14. Arun is electrolyzing a mildly acidic aqueous solution
and they are part of the process of galvanizing. Zinc is
containing a mixture of CuCl2 and ZnSO 4 using Pt
24th most abundant element in earth’s crust. It has
electrodes. The products obtained at the cathode and
five stable isotopes. The most common ore of Zinc is
anode respectively are [IJSO-Stage-2/2016]
Zinc sulfide. Zinc is extracted from its ores by the
(A) H2 and Cl2 (B) Cu and Cl2 process of roasting and calcination. In roasting zinc
(C) Zn and O2 (D) H2 and O2 sulfide ore is converted into zinc oxide when heated in
excess of oxygen. Zinc is also extracted from its
15. Aluminium is extracted from its oxide by : carbonate ore by the process of calcination where it is
[IJSO-Stage-1/2014-15] heated at very high temperature and converted into
(A) Electrolysis zinc oxide. Both reactions are carried out in different
(B) Reduction reaction vessels. Zinc oxide is heated with carbon to
(C) Thermial decomposition extract the metal. [IJSO-Stage-2/2016]
(D) Caldination I. Write balanced chemical equation for the process of
roasting.
16. Rust is a mixture of : [IJSO-Stage-1/2014-15] II. Write the balanced chemical equation for the process
(A) FeO + Fe(OH)2 (B) FeO + Fe(OH)3 of calcination.
(C) FeO4 + Fe(OH)3 (D) Fe2O3 + Fe(OH)3 III. Write balanced chemical equation when zinc oxide
is heated with carbon.
17. If a firecracker burns with emission of red colour light,
which cation is it likely to contain ? 23. The iron pillar near Qutub Minar in Delhi was built more
[IJSO-Stage-1/2014-15] than 1600 years ago by metallurgists in India. They
(A) Lithium (B) Copper had developed the process which prevents iron from
(C) Iron (D) Sodium rusting. It has been examined by scientist from all over
the world. Iron can be extracted from its ore Hematite
by Thermite reaction. [IJSO-Stage-2/2016]

PAGE # 125
A. W hich of the following statement is correct about
thermite reaction ?
i. It is an endothermic reaction between Alumina and
iron where iron acts as reducing agent and Alumina
acts as oxidizing agent.
ii. It is an exothermic reaction between Alumina and
iron where iron acts as oxidizing agent and alumina
acts as reducing agent.
iii. It is an exothermic reaction between iron oxide
and aluminium where aluminium acts as oxidizing
agent and iron acts as reducing agent.
iv. It is an exothermic reaction between iron oxide and
aluminium where aluminium acts as reducing agent
and iron acts as oxidizing agent.
B. Write balanced equation for Thermite reaction ?

PAGE # 126
CARBON
approach each other, the single electron of both the
INTRODUCTION atoms form a shared pair.
This may be represented as :
INTRODUCTION & COVALENT BONDING
Organic compounds : The compounds like urea, • x
H + H H •x H or H2 H •x H
sugars, fats, oils, dyes, proteins vitamins etc., which Hydrogen Hydrogen
were isolated directly or indirectly from living atoms molecule
organisms such as animals and plants were called
organic compounds.The branch of chemistry which Shared electron
or H–H pair
deals with the study of these compounds is called
ORGANIC CHEMISTRY. According to Lewis notation, the electrons in the valence
Most carbon compounds are poor conductors of shell are represented by dots and crosses. This
electricity. The boiling and melting points of the carbon method was proposed by G.N. Lewis and is known
compounds are low. Forces of attraction between as Lewis representation or Lewis structure. The
these molecules of organic compounds are not very shared pair of electron (shown x/o) is said to constitute
strong. As these compound are largely non a single bond between the two hydrogen atoms and
conductors of electricity hence the bonding in these is represented by a line between the two atoms.
compound does not give rise to any ions. Pictorially, the molecule can be represented by
drawing two overlapping circles around the symbols
The reactivity of elements is explained as their of the atoms and showing the shared pair of electrons
tendency to attain a completely filled outer shell, that in the overlapping part.
is, attain noble gas configuration. Element forming
ionic compounds achieve this by either gaining or (ii) Chlorine molecule : Each chlorine atom has seven
losing electrons from the outermost shell. In the case electrons in its outermost shell. W hen the two
of carbon, it has four electrons in its outermost shell chlorine atoms come close together, an electron of
and needs to gain or lose four electrons to attain both the atoms is shared between them.
noble gas configuration. If it were to gain or lose
•• •• • ••
xx xx
electrons - x x
x
••Cl •x Cl xx
•• xx •• x x
(i) it could gain four electrons forming C4- anion. But Chlorine Chlorine
it would be difficult for the nucleus with six protons to atoms molecule
hold onto ten electrons, that is, four extra electrons.
xx
• • •x or Cl –Cl
(ii) it could lose four electrons forming C4+ cation. or Cl 2 ••Cl Cl xx
•• xx
But it would require a large amount of energy to
remove four electrons leaving behind a carbon cation
with six protons in its nucleus holding onto just two Shared electron
pair
electrons.
(iii) Hydrogen chloride molecule : It may be noted that a
covalent bond is not only formed between two similar
Carbon overcomes this problem by sharing its
atoms, but it may be formed between dissimilar atoms
valence electrons with other atoms of carbon or with
also. For example, hydrogen and chlorine form a
atoms of other elements.The shared electrons belong
covalent bond between their atoms. In HCl, hydrogen
to the outer shell of both the atoms and lead to both
atom (1) has only one electron in its valence shell
atoms attaining the noble gas configuration.
and chlorine atom (2,8,7) has seven electrons in its
valence shell. Therefore, by mutual sharing of electron
(a) Some Simple Molecules Formed by the Sharing of
pair between a hydrogen and a chlorine atom, both
Valence Electrons are as Follows :
the atoms acquire nearest noble gas configuration.
(i) Hydrogen molecule : This is the simplest molecule
formed by sharing of electrons. The atomic number
of hydrogen is 1 and it has only one electron in its xx
• xx xx
H •x x
outermost K shell. It requires only one more electron H + xCl x
x H •x Cl
xx
x
x Cl
xx
x
xx
to complete the K shell. So, when two hydrogen atoms Hydrogen Chlorine
Hydrogen chloride
atom atom molecule
Shared electron
or H – Cl pair

PAGE # 127
(b) Different Kinds of Covalent Bonds : (iii) Crystal structure - Covalent compounds exhibit
both crystalline and non crystalline structure.
Electron pair shared between two atoms results in
the formation of a covalent bond. This shared pair is (iv) Melting point and boiling point : Energy required
also called bonding pair of electron. to break the crystal is less due to the presence of
weak Vanderwaal's force, so their melting and boiling
o If two atoms share one electron pair, bond is known
points are less.
as single covalent bond and is represented by one
dash (-). (v) Electrical conductivity - Covalent compounds are
bad conductors of electricity due to the absence of
o If two atoms share two electron pairs, bond is
free electrons or free ions.
known as double covalent bond and is represented
by two dashes (=). (vi) Solubility : Due to the non - polar nature of covalent
compounds they are soluble in non - polar solvents
o If two atoms share three electron pairs, bond is
like benzene, carbon tetrachloride etc. and insoluble
known as triple covalent bond and is represented by
in polar solvents like water etc.
three dashes ( ).
(i) Formation of double bond (oxygen molecule) :
VERSATILE NATURE OF CARBON
Two oxygen atoms combine to form oxygen molecule
by sharing two electron pairs. Each oxygen atom (2,6) About 3 million organic compounds are known today.
has six electrons in the valence shell. It requires two The main reasons for this huge number of organic
electrons to acquire nearest noble gas configuration. compounds are -
Therefore, both the atoms contribute two electrons
(i) Catenation : The property of self linking of carbon
each for sharing to form oxygen molecule. In the
atoms through covalent bonds to form long straight
molecule, two electron pairs are shared and hence
or branched chains and rings of different sizes is
there is a double bond between the oxygen atoms.
called catenation. Carbon shows maximum
(ii) Formation of triple bond (Nitrogen molecule) :
catenation in the periodic table due to its small size,
Nitrogen atom has five electrons in its valence shell.
electronic configuration and unique strength of carbon-
In the formation of a nitrogen molecule, each of the
carbon bonds.
nitrogen atoms provide three electrons to form three
electron pairs for sharing. Thus, a triple bond is formed (ii) Tetravalency of carbon : Carbon belongs to group
between two nitrogen atoms. 14 of the periodic table. Since the atomic number of
carbon is 6, the electronic configuration of carbon
• x
•• N •• xx N xx atom is 2,4. It has four electrons in the outermost
•• N• + Nx xx •• xx

x
•x •• N •x N xx
shell. Therefore, its valency is four. Thus, carbon forms
Nitrogen Nitrogen
four covalent bonds in its compounds. A methane
atoms molecule
molecule (CH4) is formed when four electrons of
3 Shared electron carbon are shared with four hydrogen atoms as
or N N pairs
shown below.
Nitrogen
molecule x H
x•

(c) Characteristic Properties of Covalent Compounds : xC x + 4H H x• C
x•
x• H
x
The important characteristic properties of covalent H
compounds are : Carbon Hydrogen Methane
atom atoms
(i) Covalent compounds consist of molecules : The
or CH4
covalent compounds consist of molecules. They do
(iii) Tendency to form multiple bonds : Due to small
not have ions. For example - water, hydrogen chloride,
size of carbon it has a strong tendency to form multiple
methane consist of H2O, HCl, CH4 molecules
bonds (double & triple bonds) by sharing more than
respectively.
one electron pair. As a result, it can form a variety of
(ii) Physical state : Weak Vanderwaal's forces are compounds. For example -
present between the molecules of covalent
compounds. So, covalent compounds are in solid, H H
gaseous or liquid state at normal temperature and H H
C C H
pressure. C C H
C C H
H Ethyne
For example : Hydrogen chloride , methane are gases H
Ethene
Ethane
while carbon tetrachloride, ethyl alcohol, ether etc.
are liquids. Glucose, sugar, urea etc. are some solid (iv) Electronegativity and strength of bonds : The
covalent compounds.
electronegativity of carbon (2.5) is close to a number
of other elements like H (2.1) , N(3.0) , P (2.1), Cl (3.0)
and O (3.5). So carbon forms strong covalent bonds

PAGE # 128
with these elements.
Organic compounds
(v) Isomerism : It is a phenomenon by the virtue of
which two compounds have same molecular formula
but different physical and chemical properties. Open chain Closed chain
compounds compounds
VITAL FORCE THEORY OR
BERZELIUS HYPOTHESIS Aromatic
Alicyclic
compounds compounds
Organic compounds cannot be synthesized in the
laboratory because they require the presence of a (a) Open Chain Compounds :
mysterious force (called vital force) which exists only
These compounds contain an open chain of carbon
in living organisms.
atoms which may be either straight chain or branched
chain in nature. Apart from that, they may also be
WOHLER’S SYNTHESIS saturated or unsaturated based upon the nature of
bonding in the carbon atoms. For example.
In 1828, Friedrich Wohler synthesized urea (a well
known organic compound) in the laboratory by heating
ammonium cyanate.

(NH4)2 SO 4 + 2 KCNO 2NH 4CNO + K 2SO 4 , ,


Ammonium Potassium Ammonium Potassium
sulphate cyanate cyanate sulphate

,
 Note :
Urea is the first organic compound synthesized in
the laboratory.

MODERN DEFINITION OF ORGANIC CHEMISTRY


Organic compounds may be defined as hydrocarbons
and their derivatives and the branch of chemistry
which deals with the study of hydrocarbons and their
derivatives is called ORGANIC CHEMISTRY.
Organic chemistry is treated as a separate branch
because of following reasons- n-Butane is a straight chain alkane while
(i) Organic compounds are large in number. 2-Methylpropane is branched alkane.

(ii) Organic compounds generally contain covalent (b) Closed Chain or Cyclic Compounds :
bond.
Apart from the open chains, the organic compounds
(iii) Organic compounds are soluble in non polar solvents. can have cyclic or ring structures. A minimum of three
(iv) Organic compounds have low melting and boiling atoms are needed to form a ring. These compounds
points. have been further classified into following types.
(v) Organic compounds show isomerism . (i) Alicyclic compounds : Those carbocyclic
(vi) Organic compounds exhibit homology. compounds which resemble to aliphatic compounds
in their properties are called alicyclic compounds .
CLASSIFICATION OF ORGANIC COMPOUNDS

The organic compounds are very large in number on


account of the self -linking property of carbon called e.g. or Cyclopropane
catenation. These compounds have been further
classified as open chain and cyclic compounds.

or Cyclobutane

PAGE # 129
e.g. CH4 ( Methane)
C2H6 (Ethane)

(ii) Unsaturated hydrocarbons :


or Cyclopentane
(A) Alkenes : These are unsaturated hydrocarbons
which contain carbon - carbon double bond. They
contain two hydrogen less than the corresponding
alkanes.

General formula - C nH 2n
e.g. C 2H 4 (Ethene)
or Cyclohexane C 3H 6 (Propene)

(B) Alkynes : They are also unsaturated hydrocarbons


which contain carbon-carbon triple bond. They contain
four hydrogen atoms less than the corresponding
(ii) Aromatic compounds : Organic compounds which alkanes.
contain one or more fused or isolated benzene rings
General formula - C nH 2n–2
are called aromatic compounds. e.g. C 2H 2 (Ethyne)
C 3H 4 (Propyne)

HOMOLOGOUS SERIES
e.g. Homologous series may be defined as a series of
similarly constituted compounds in which the
Benzene Toluene Ethyl benzene members possess similar chemical characteristics
and the two consecutive members differ in their
molecular formula by - CH2.

(a) Characteristics of Homologous Series :

(i) All the members of a series can be represented


by the same general formula.
Phenol Aniline e.g. General formula for alkane series is CnH2n+2 .
 Note : (ii) Any two consecutive members differ in their formula
Benzene is the parent compound of majority of
by a common difference of - CH2 and differ in molecular
aromatic organic compounds.
mass by 14.

HYDROCARBONS (iii) Different members in a series have a common


functional group.
The organic compounds containing only carbon and e.g. All the members of alcohol family have -OH group
hydrogen are called hydrocarbons. These are the
simplest organic compounds and are regarded as (iv) The members in any particular family have almost
parent organic compounds. All other compounds are identical chemical properties. Their physical
considered to be derived from them by the properties such as melting point, boiling point, density
replacement of one or more hydrogen atoms by other etc. show a regular gradation with the increase in the
atoms or groups of atoms. The major source of molecular mass.
hydrocarbons is petroleum. (v) The members of a particular series can be
prepared almost by the identical methods.
Types of Hydrocarbons :
(b) Homologues :
The hydrocarbons can be classified as :
The different members of a homologous series are
(i) Saturated hydrocarbons : known as homologues .
(A) Alkanes : Alkanes are saturated hydrocarbons For example :
containing only carbon - carbon and carbon - hydrogen (i) Homologous series of alkanes
single covalent bonds. General formula : CnH2n+2
Value of n Molecular formula IUPAC name
General formula- CnH2n + 2(n is the number of carbon atoms)
n=1 CH 4 Methane
n=2 C 2H 6 Ethane
n=3 C 3H 8 Propane

PAGE # 130
(ii) Homologous series of alkenes (B) Primary suffix :
General formula : CnH2n
Value Molecular IUPAC name Common
of n formula name
n=2 C 2H 4 Ethene Ethylene
n=3 C 3H 6 Propene Propylene
n=4 C 4H 8 But-1-ene - Butylene

(iii) Homologous series of alkynes


General formula : CnH2n-2
Value Molecular IUPAC name Common
of n formula name
n=2 C2H2 Ethyne acetylene Examples :
n=3 C3H4 Propyne methyl acetylene
n=4 C4H6 But -1-yne ethyl acetylene a
M
Eth - E
NOMENCLATURE OF ORGANIC COMPOUNDS P
B
Nomenclature means the assignment of names to E
organic compounds . There are two main systems of Propene
nomenclature of organic compounds -
(1) Trivial system
(2) IUPAC system (International Union of Pure and  Note :
Applied Chemistry) The name of the compound, in general , is written in
(a) Basi c rul es of IUPAC nomencl ature of the following sequence-
orga nic compound s : (Position of substituents )-(prefixes ) (word root)-(p -
For naming simple aliphatic compounds, the normal suffix).
saturated hydrocarbons have been considered as
the parent compounds and the other compounds as (iii) Names of branched chain hydrocarbon : The
their derivatives obtained by the replacement of one carbon atoms in branched chain hydrocarbons are
or more hydrogen atoms with various functional
present as side chain . These side chain carbon atoms
groups.
constitute the alkyl group or alkyl radicals. An alkyl group
(i) Each systematic name has two or three of the
is obtained from an alkane by removal of a hydrogen.
following parts-
(A) Word root : The basic unit of a series is word root General formula of alkyl group = CnH2n+1
which indicate linear or continuous number of carbon An alkyl group is represented by R.
atoms. e.g.
(B) Primary suffix : Primary suffixes are added to the
word root to show saturation or unsaturation in a H
carbon chain.
–H
(A) H C
(C) Secondary suffix : Suffixes added after the
primary suffix to indicate the presence of a particular H
functional group in the carbon chain are known as Methyl
secondary suffixes.
(ii) Names of straight chain hydrocarbons : The
name of straight chain hydrocarbon may be divided H H
into two parts- –H
(B) H C C
(A) Word root (B) Primary suffix
H H
(A) Word roots for carbon chain lengths : Ethyl

Chain Word Chain Word


length root length root

C1 Meth- C6 Hex-
C2 Eth - C7 Hept-
C3 Prop - C8 Oct-
C4 But - C9 Non-
C5 Pent- C10 Dec-

PAGE # 131
(ii) If some multiple bond is present in the chain, the
carbon atoms involved in the multiple bond should
get lowest possible numbers.

e.g.

(C)
2–Methylbutane 3–Methylbutane
(Correct) (Wrong)

e.g.
A branched chain hydrocarbon is named using the
following general IUPAC rules : 2–Methylpentane 4–Methylpentane
(Correct) (W rong)
Rule1: Longest chain rule : Select the longest possible
continuous chain of carbon atoms. If some multiple
e.g.
bond is present , the chain selected must contain
the multiple bond.
3–Methylbut–1– ene 2–Methylbut – 3 – ene
(i) The number of carbon atoms in the selected chain (Correct) (W rong)
determines the word root .
(ii) Saturation or unsaturation determines the primary CH3 CH3
4 3
| 2 1 1 2
| 3 4
suffix (P. suffix). e.g.
3-Methylbut-1-yne 2-Methylbut-3-yne
(iii) Alkyl substituents are indicated by prefixes. (Wrong)
(Correct)
Prefix : Methyl
e.g. CH3 – CH2 – CH – CH2 – CH3 Rule 3 : Use of prefixes di, tri etc. : If the compound
Word root : pent-
P. Suffix: - ane contains more than one similar alkyl groups,their
CH3
positions are indicated separately and an appropriate
numerical prefix di, tri etc. , is attached to the name
of the substituents. The positions of the substituents
e.g. CH3 – CH – CH2 – CH – CH3 Prefix : Methyl are separated by commas.
Word root : Hept-
CH3 CH2 – CH2 – CH3 CH3
P. Suffix : -ane
5 4 3 2 1
e.g. CH3 – CH2– C – CH – CH3

CH3 CH3
e.g. CH3 – CH2 – C – CH3 Prefix : Methyl 2,3 - Dimethylpentane 2,3,3 - Trimethylpentane
Word root : But-
CH2 P. Suffix : –ene
e.g.

e.g. CH3 – CH 2– CH – CH2– CH 3 Prefixes : Ethyl, Methyl


Word root : Pent- 2,3,5 -Trimethylhexane 2,2,4 - Trimethylpentane
P. Suffix : -ane
CH – CH3
Rule 4 : Alphabetical arrangement of prefixes: If there
CH3 are different alkyl substituents present in the
compound their names are written in the
alphabetical order. However, the numerical
Rule 2 : Lowest number rule: The chain selected is
prefixes such as di, tri etc. , are not considered for
numbered in terms of arabic numerals and the the alphabetical order.
position of the alkyl groups are indicated by the number
of the carbon atom to which alkyl group is attached .
e.g.
(i) The numbering is done in such a way that the
substituent carbon atom has the lowest possible
number. 3-Ethyl - 2,3-dimethylpentane

PAGE # 132
Rule 5 : Naming of different alkyl substituents at the Substituent : one methyl group
equivalent positions : IUPAC name : 4 - Methylhex - 2 - yne
Numbering of the chain is done in such a way that the
FUNCTIONAL GROUP
alkyl group which comes first in alphabetical order
gets the lower position. An atom or group of atoms in an organic compound
or molecule that is responsible for the compound’s
characteristic reactions and determines its properties
e.g. is known as functional group. An organic compound
generally consists of two parts -
3-Ethyl-4-methylhexane (i) Hydrocarbon radical (ii) Functional group
(e) Some More Examples :

e.g.

(i)
Hydrocarbon radical Functional group
• Functional group is the most reactive part of the
Word root : Hex molecule.
Primary suffix : ane • Functional group mainly determines the chemical
Substituents : two methyl & one ethyl groups properties of an organic compound.
IUPAC name : 4-Ethyl-2,4-dimethylhexane • Hydrocarbon radical mainly determines the physical
properties of the organic compound.

(a) Main Functional Groups :

(i) Hydroxyl group (– OH) : All organic compounds


containing - OH group are known as alcohols .
Word root : Prop e.g. Methanol (CH3OH) , Ethanol (CH3 – CH2 – OH) etc .
P. Suffix : ane (ii) Aldehyde group (–CHO) : All organic compounds
Substituents : two methyl groups containing –CHO group are known as aldehydes.
IUPAC name : 2,2-Dimethylpropane e.g. Methanal (HCHO), Ethanal (CH3CHO) etc.
(iii) Ketone group (–CO–) : All organic compounds
Nam ing of Unsatur ated hydr ocarb ons
containing –CO– group are known as ketones.
e.g. Propanone (CH3COCH3), Butanone (CH3COCH2CH3) etc.
(iv) Carboxyl group ( – COOH) : All organic compounds
(i) containing carboxyl group are called carboxylic acids.
e.g. CH3COOH (Ethanoic acid)
CH 3CH 2COOH(Propanoic acid)
Word root : Prop - (v) Halogen group (X = F, Cl, Br, I) : All organic
P. Suffix : -ane compounds containing –X (F, Cl, Br or I) group are
Substituent : two methyl groups known as halides.
IUPAC name : 2, 2 - Dimethylpropane e.g. Chloromethane (CH3Cl), Bromomethane (CH3Br) etc

(b) Nomenclature of Compounds Containing


Functional Group :
(ii)
In case functional group (other than C = C and C  C)
is present, it is indicated by adding secondary suffix
after the primary suffix. The terminal ‘e’ of the primary
Word root : But - suffix is removed if it is followed by a suffix beginning
P. Suffix : - ene with ‘a’, ‘e’, ‘i’, ‘o’, ‘u’. Some groups like
Substituent : two methyl groups –F, – Cl, – Br and –  are considered as substituents
IUPAC name : 2, 3 - Dimethylbut - 1 - ene and are indicated by the prefixes.
O
(iii)
Some groups like – CHO, – C – , – COOH, and – OH
are considered as functional groups and are
Word root : Hex - indicated by suffixes.
P. Suffix : - yne

PAGE # 133
Functional General
Class Group Formula Prefix Suffix IUPAC Name

Carboxylic
Carboxy - oic acid Alkanoic acid
acid
(R = CnH2n+1)

Carbalkoxy or
Ester alkyl (R’) - oate Alkyl alkanoate
alkoxy carbonyl

Formyl
Aldehyde – CHO R – CHO - al Alkanal
or oxo

Ketone oxo - one Alkanone

Alcohol – OH R – OH Hydroxy - ol Alkanol

Alkenes CnH2n – - ene Alkene

Alkynes –C C– CnH2n–2 – - yne Alkyne

–X
Halides R–X Halo – Haloalkane
(X = F,Cl,Br,I)
Steps of naming of an organic compound containing functional group :

Step 1:

Select the longest continuous chain of the carbon atoms as parent chain. The selected chain must include the

carbon atoms involved in the functional groups like – COOH, – CHO, – CN etc, or those which carry the functional

groups like – OH, – NH2,– Cl, – NO2 etc.

The number of carbon atoms in the parent chain decides the word root.

Step 2 :

The presence of carbon - carbon multiple bond decides the primary suffix.

Step 3 :

The secondary suffix is decided by the functional group.

Step 4 :

The carbon atoms of the parent chain are numbered in such a way so that the carbon atom of the functional group

gets the lowest possible number . In case the functional group does not have the carbon atom, then the carbon atom

of the parent chain attached to the functional group should get the lowest possible number.

Step 5 :

The name of the compound is written as -

Prefixes - word root - primary suffix - secondary suffix

 Note :

The number of carbon atoms in the parent chain decides the word root.

PAGE # 134
S.No. Compound Common name Derived name IUPAC Name Structure

Methyl alcohol
1 CH3 – OH Carbinol Methanol
or Wood spirit

2 CH3 – CH2 – OH Ethyl alcohol Methyl carbinol Ethanol

3 CH3 – CH2 – CH2 – OH n-Propyl alcohol Ethyl carbinol 1- Propanol

H H
4 Isopropyl alcohol Dimethyl carbinol 2 - Propanol H – C – C – O– H
H C H3

5 CH3 – CH2 – CH2 – CH2 – OH n-Butyl alcohol n-Propyl carbinol 1- Butanol

6 HCOOH Formic acid – Methanoic acid

7 CH3COOH Acetic acid – Ethanoic acid

methyl acetic
8 CH3 – CH2 – COOH Propionic acid Propanoic acid
acid

ethyl acetic
9 CH3 – CH2 – CH2 – COOH Butyric acid Butanoic acid
acid

O
n-Propyl acetic
10 CH3 – CH2 – CH2 – CH2 – COOH Valeric acid Pentanoic acid
acid

Some more examples : IUPAC name : Pent-2-en-1-oic acid/Pent-2-enoic acid


(iii) CH3 – CH2 – CH2 – NH2
(i) Word root : Prop -
Primary suffix : - ane
Secondary suffix : - amine
Word root : Hept- IUPAC name : Propan - 1 - amine
Primary suffix : – ane
Functional group : – OH
(iv)
Secondary suffix : – ol
Word root : Prop-
IUPAC Name : 2, 5-Dimethylheptan–1– ol Primary suffix : - ane
Substituent : nitro(prefix)
IUPAC name : 1 - Nitropropane
(ii)
Word root : Pent - (v)
Primary suffix : – ene
Secondary suffix : – oic acid
Word root : But -
Primary suffix : – ane
Prefix : – chloro
PAGE # 135
IUPAC name : 2 - Chlorobutane or functional group.
e.g.
(i) C4H8
(vi)
CH3 – CH2 – CH = CH2 , CH3 – CH = CH – CH3
But -1 - ene But -2 - ene
Word root : But -
Primary suffix : – ane (ii) C3H8O
Secondary suffix : – one
CH3 – CH2 – CH2 – OH , CH3 – CH – CH3
Prefix : Methyl
Propan-1-ol
IUPAC name : 3 - Methylbutan - 2- one
OH
Propan-2-ol
ISOMERISM (c) Functional Group Isomerism :
Such compounds which have same molecular In this type of isomerism, isomers differ in the
formula but different physical and chemical properties structure due to the presence of different functional
are known as isomers and the phenomenon is groups.
known as isomerism. e.g.

(i) C3H8O

CH3 – CH2 – O – CH3 CH3 – CH2 – CH2 – OH

Methoxy ethane Propan-1-ol

(ii) C4H6
CH3 – CH2 – C  CH CH2 = CH – CH = CH2
But - 1- yne Buta - 1, 3 - diene
(a) Chain Isomerism : [or 1, 3 - Butadiene ]
The isomerism in which the isomers differ from each
other due to the presence of different carbon chain CHEMICAL PROPERTIES OF CARBON COMPOUNDS
skeletons is known as chain isomerism.
The important chemical properties of organic
e.g.
compounds are discussed below :
(i) C4H10 (a) Combustion Reactions :

, Carbon in all its allotropic forms burns in air or oxygen


to give carbon dioxide and releases energy in the
form of heat and light.
Most carbon compound also release a large amount
2 - Methylpropane of heat and light on burning.
(Isobutane)
C + O2  CO2 + Heat and light
(ii) C5H12 Carbon Oxygen Carbon
dioxide
CH 4 + 2O2  CO2 + 2H2O + Heat and light
2 - Methylbutane Methane Oxygen Carbon Water
(Isopentane) dioxide
C2H4 + 3O2  2CO2 + 2H2O + Heat and light
Ethene Oxygen Carbon Water
dioxide
2C2H2 + 5O2  4CO2 + 2H 2O + Heat and
light
Ethyne Oxygen Carbon Water
2, 2 -Dimethylpropane dioxide
(neo - pentane) CH3CH2OH + 3O2  2CO2 + 3H 2O+ Heat and
(iii) C4H8 light
Ethanol Oxygen Carbon Water
dioxide
CH3 – CH2 – CH = CH2 , Saturated hydrocarbons will generally give a clean
But - 1 - ene Methylpropene flame while unsaturated carbon compounds will give
a yellow flame with lots of black smoke. This results
(b) Position Isomerism : in a sooty deposit on the surface. However, limiting
the supply of air results in incomplete combustion of
In this type of isomerism, isomers differ in the structure even saturated hydrocarbons giving a sooty flame.
due to difference in the position of the multiple bond

PAGE # 136
(b) Oxidation Reactions : (d) Addition Reactions :
Oxidation is a process in which oxygen is added to a
substance. The substances which add oxygen to other The reactions in which a reactant molecule is added
substances are called oxidising agents. There are to another reactant molecule to form a product are
many oxidising agents such as alkaline potassium
permanganate (alk. KMnO 4), acidified potassium known as addition reactions. Unsaturated
dichromate (K 2Cr2O 7), nitric acid (HNO 3) etc. which hydrocarbons, such as alkenes and alkynes undergo
are commonly used in organic chemistry. Some
addition reactions. For example, unsaturated
common reactions of oxidation are -
hydrocarbons add hydrogen, in the presence of
Alkaline KMnO
4  CH – CH
(i) CH2 = CH2 + H2O + (O)     
 2 2 catalysts, such as nickel or palladium, to give saturated
Ethene OH OH hydrocarbons as given below :
Ethylene glycol
Ni
CH2 = CH2 + H2  CH3 – CH3
Alkaline KMnO4 Heat
(ii) CH  CH + 4(O)       
Ethyne Ethene Hydrogen Ethane
Alkaline KMnO Or
4 Ni
(iii) CH3CH2OH        CH3COOH CH  CH + 2H2  CH3 – CH3
Acidified K Cr O Heat
2 2 7
Ethanol Ethanoic acid
Ethyne Hydrogen Ethane
Alkaline KMnO 4 Or
(iv) CH3CHO       
 CH3COOH This proces of addition of hydrogen across double or
Acidified K Cr O
2 2 7
triple bond is known as hydrogenation and is very
Ethanal Ethanoic acid
useful in daily life. This process is especially useful
(c ) Substitution Reactions : for the preparation of vanaspati ghee from vegetable
oils, in presence of nickel as discussed below :
The reaction in which an atom or group of atoms in a
molecule is replaced or substituted by different atoms Hydrogenation of oils :
or group of atoms are called substitution reactions. Certain vegetable oils such as groundnut oil, cotton
Saturated hydrocarbons are fairly unreactive. For seed oil and mustard oil, contain double bonds (C =
example, chlorine does not react with methane at C) and are liquids at room temperature. Because of
room temperature. However, in the presence of the presence of double bonds, the vegetable oils un-
dergo hydrogenation, like alkenes, to form saturated
sunlight the reaction of chlorine and hydrocarbons is
products called vanaspati ghee, which is a solid or a
fairly fast. It gives a variety of products.
semi-solid at the room termperature.

Diffused Nickel catalyst


Vegetable oils + Hydrogen     Vanaspati ghee
CH 4 + Cl2   CH3Cl + HCl 473 K
Sunlight

Methane Chlorine Chloromethane (Unsaturated and (Saturated and


Hydrogen in liquid state) in solid state)
chloride You must have seen advertisements stating that
In this reaction H – atom of methane has been some of the vegetable oils are ‘healthy’. But it is worth
mentioning that animal fats, such as ‘Ghee’ and but-
replaced by a –Cl atom converting CH 4 to CH 3Cl.
ter contain saturated fatty acids which are not consid-
However, if Cl2 is used in excess, all the hydrogen
ered good for health and even doctors discourage
atoms are replaced by chlorine atom one by one.
their use. On the other hand, oils containing unsatur-
Diffused
CH3Cl + Cl 2   CH2Cl2 + HCl ated fatty acids are generally used for cooking pur-
Sunlight
Chloromethane Chlorine Dichloromethane Hydrogen poses.
chloride
In addition to hydrogenation, some other typical ex-
Diffused
CH2Cl2 + Cl 2   CHCl3 + HCl amples of addition reactions are given below :
Sunlight
Dichloromethane Chlorine T richloromethane
(i) Addition of chlorine
Hydrogen
Ethene reacts with one molecule of chlorine to give
(Chloroform) chloride
1,2-dichloroethane (or ethylene dichloride) which is
Diffused a saturated compound.
CHCl3 + Cl 2   CCl4 + HCl
Sunlight
4 CCl
Trichloromethane Chlorine Tetrachloromethane Hydrogen CH2=CH2 + Cl2   Cl–CH2–CH2–Cl
(Carbon tetrachloride) chloride
Ethene Chlorine 1, 2-Dichloroethane

PAGE # 137
Similarly, ethyne reacts with one molecule of chlorine (ii) Baeyer’s test: It decolourises the purple colour of
to form 1, 2-dichloroethene, which is still unsatur- Baeyer’s reagent.
ated and, therefore, adds one more molecule of chlo-
rine to form a saturated compound, i.e., 1, 1, 2, 2-
Tetrachloroethane as given below :

(iii) Silver nitrate Test: No reaction


(ii) Addition of bromine : (c) Alkynes :
When bromine water (which is a solution of bromine
(i) Bromine water test : It decolourises the Br2 water.
in water) is added to ethene, it forms 1, 2
dibromoethane and the solution becomes colourless, H H Br2
H H
H – C  C – H + Br2 C=C Br – C – C – Br
i.e., the reddish-brown colour of bromine is discharged Ethyne Bromine
Br Br
Br Br
water 1,2-Dibromoethene
CH2 = CH2 + Br2 CCl 4 Br – CH2 – CH2 – Br 1,1,2,2-Tetrabromoethane
 (Colourless)

Ethene Bromine water 1, 2-Dibromoethane (ii) Baeyer’s test : It also decolourises the purple
(Reddish brown) (Colourless) colour of alkaline KMnO4 .
This reaction is used as a test for unsaturation.

Similarly, bromine reacts with ethyne to give, 1, 1, 2,


2-Tetrabromoethane which is a saturated compound
(iii) Silver nitrate Test : It gives white precipitate
Br Br
CCl4
H– C–
– C – H + 2Br2 H–C–C–H
Ethyne Bromine Br Br ETHANOL
(Unsaturated) (Reddish 1,1,2,2-Tetrabromoethane
Brown) (A) Properties of Ethanol :
(Acetylene tetrabromide)
(Saturated and colourless) (1) Physical properties
•Ethanol is a colourless liquid having a pleasant
Test for unsaturation. The addition of bromine water smell.
to alkenes or alkynes is also used to test the
• Ethanol boils at 351 K.
unsaturation in the given organic compound. If the
reddish - brown colour of bromine water gets dis-
• It is miscible with water in all proportions.
charged on adding it to the organic compound, the • It is a non-conductor of electricity (it does not contain
compound must be unsaturated, i.e., it must be con- ions).
taining a double or triple bond. Similarly, the • It is neutral to litmus.
unsaturation in the organic compound can also be
(2) Chemical properties :
tested with the help of Bayer’s test.
• Combustion : Ethanol burns in air with a blue flame
to form CO2 & H2O.
TEST FOR ALKANES, ALKENES AND ALKYNES
C2H5OH + 3O2  2CO2 + 3H2O + heat & light
(a) Alkanes :
(i) Bromine water test: It does not decolourise the
• Oxidation :
bromine water.
 By mild oxidizing agent CrO 3 (Chromic
(ii) Baeyer’s test: It does not, react with Baeyer’s anhydride).
reagent (alkaline solution of KMnO4).
(iii) Silver nitrate Test: No reaction CH3CH2OH   CrO  3   CH3CHO + H2O
In glacial acetic acid
(b ) Al kene s: Ethanol Ethanal
(i) Bromine water test: It decolourises the orange  By strong oxidizing agent (K2Cr2O7 + H 2SO 4 or
colour of Bromine water.
alkaline KMnO4).
H H H H
CCl
C=C + Br2 4
C–C
H H Bromine water H Br Br H
Ethene (red-brown colour)
1,2-Dibromoethane
(Colourless) • Reaction with sodium : Ethanol reacts with sodium
to produce hydrogen gas and sodium ethoxide.

PAGE # 138
with petrol , therefore, a third solvent such as benzene,
ether etc. is added as a co-solvent.

(C) Uses of Ethanol :


•Reaction with carboxylic acids : [ESTERIFICATION]
• Ethanol is a constituent of beverages like beer,
The process of formation of an ester by the wine, whisky and other liquors.
combination of an alcohol with carboxylic acid is Beer = 3 – 6% Ethanol
known as esterification. Whisky = 50% Ethanol
W ine = 10 – 20% Ethanol

• Ethanol is used to sterilize wounds and syringes.


• Antifreeze : It is a mixture of ethanol and water
which has a much lower freezing point than that of
water. It is used in radiators of vehicles in cold
When ethanol reacts with ethanoic acid in presence countries.
of concentrated sulphuric acid, ethyl ethanoate and
water are formed .
• It is used in manufacture of paints, dyes, medicines,
soaps and synthetic rubber. Solution of ethanol
prepared in pharmaceutical industry are known as
tinctures.
(D) Harmful effects of drinking alcohol :
 Esters are sweet smelling substances and thus • If ethanol is mixed with CH3OH and consumed, it
are used in making perfumes. may cause serious poisoning and loss of eyesight.

• Action with concentrated sulphuric acid : Ethanol • It causes addiction (habit forming) and mixes with
blood. It damages liver if taken regularly.
reacts with concentrated sulphuric acid at 443 K to
produce ethylene. This reaction is known as acidic • Higher amount of consumption of ethanol leads to
dehydration of ethanol because in this reaction, water loss of body control & consciousness. It may even
cause death.
molecule is removed from ethanol.
(E) Test of alcohol :
Conc. H2SO4
CH3CH2OH      CH2 = CH2 + H2O The presence of ethanol or alcoholic group in an
443 K organic compound can be tested by the following
Ethanol Ethene tests:
The concentrated sulphuric acid may be regarded as
• Sodium metal test :
a dehydrating agent because it removes water from
Add a small piece of sodium metal to the organic
ethanol.
compound (to be tested) in a dry test tube. If the
(B) Some Important Terms : bubbles of hydrogen gas are produced, it indicates
the presence of alcohol.
• Denatured alcohol : To prevent the misuse for
• Ester formation test :
drinking purpose, the alcohol supplied for industrial Warm the organic compound with acetic acid and a
purpose is rendered unfit by mixing it with some few drops of conc. H2SO4. A sweet smell (due to the
poisonous substances like methanol, pyridine, copper formation of ester) indicates the presence of
sulphate etc. It is known as denatured alcohol. alcohol.

• Rectified spirit : Ethanol containing 5 percent


ETHANOIC ACID OR ACETIC ACID
water is known as rectified spirit.
(i) Molecular formula : CH3COOH
• Absolute alcohol : Rectified spirit is heated under
reflux over quicklime for about 5 to 6 hours and then
allowed to stand for 12 hours. On distillation, pure (ii) Structural formula :
alcohol (C2H5OH = 100%) is obtained. This is called
absolute alcohol. (iii) The IUPAC name of acetic acid is ethanoic acid .
• Power alcohol : Alcohol, which is used for (iv) Occurrence : Ethanoic acid is known as vinegar,
generating power is called power alcohol. It consists from ancient times. Vinegar is essentially a dilute
of a mixture of absolute alcohol and petrol roughly in solution of ethanoic acid in water. The acid is also
the ratio 20 : 80. Since alcohol itself, does not mix present in some fruit juice . In the combined form, it is
also present in many perfumed oils. Ethanoic acid was
first prepared in the pure state by Stahl in 1720 .

PAGE # 139
(v) Physical properties :
(vii) Uses :
(A) Ethanoic acid is a colourless, viscous liquid but
(A) Dilute aqueous solution (5–8%) of ethanoic acid
has a pungent and irritating smell of vinegar.
is called vinegar, which is used to preserve food
(B) Its boiling point is 391 K. (sausage, pickles, etc).
(C) It dissolves in water, alcohol and ether. Its (B) Pure ethanoic acid is used as a solvent and
dissolution in water takes place with the evolution of chemical reagent.
heat and decrease in volume of the solution.
(C) As cellulose ethanoate, it is used in making
(D) The melting point of ethanoic acid is 290 K and photographic films and rayon.
hence it often freezes during winter in cold climates.
Therefore, it is named as glacial acetic acid. (D) Ethanoic acid also finds application in the
preparation of propanone, chloroethanoic acid,
(vi) Chemical properties : ethanoates of metals etc.
(A) Acidic character : Ethanoic acid is a monobasic (E) It is widely used in the manufacture of textiles.
acid. It has a replaceable hydrogen atom in its –
COOH group. Therefore, it neutralizes alkalies. (F) It is used in the preparation of white lead.

(1) It reacts with a solution of sodium hydroxide to (viii) Tests for Carboxylic Acid :
form sodium ethanoate and water.
(A) Litmus test : Add small amount of blue litmus
CH3COOH+ NaOH CH3COONa + H2O solution to the given compound. If the blue litmus
Sodium solution turns red, it indicates that the organic
ethanoate compound is acidic in nature.
Sodium ethanoate is an ionic compound which
dissolves in polar solvents such as water, but does (B) Sodium bicarbonate test : Take a small portion
not dissolves in non polar solvents such as alcohol, of the organic compound in a test tube and add a
propanone etc. The aqueous solution of sodium pinch of solid sodium bicarbonate. Evolution of carbon
ethanoate is alkaline due to hydrolysis . dioxide with brisk effervescence shows that the
organic compound is acidic in nature.
(C) Ester formation: W hen a mixture of given
compound and ethanol is heated in the presence of
(2) It reacts with sodium carbonate and sodium concentrated sulphuric acid, a fruity smelling ester,
bicarbonate with the evolution of CO2 gas. ethyl ethanoate, is produced which shows that the
organic compound is acidic in nature.
2CH3COOH + Na2CO3  2CH3COONa + H2O + CO2
 SOAPS AND DETERGENTS
CH3COOH + NaHCO3  CH3COONa + H2O + CO2 
The word ‘detergent’ means’ cleansing agent and
(3) It reacts with metals like sodium, zinc and
magnesium to liberate hydrogen gas. so the detergents are substances which remove dirt
and have cleansing action in water. According to this
2CH3COOH + 2Na  2CH3COONa + H2 
definition of detergents, soap is also a detergent and
2CH3COOH + Zn  (CH3COO)2Zn + H2  has been used for more than two thousand years.
(B) Ester formation : When ethanoic acid is heated There are two types of detergents :
with ethanol in presence of small quantity of conc. (a) Soapy detergents or soaps
H 2SO 4 ethyl ethanoate, a sweet smelling ester, is
formed. (b) Non - soapy detergents or soapless soaps.

CH3COOH + C2H5OH CH3COOC2H5 + H2O (a) Soap :


Ethyl ethanoate A soap is a sodium or potassium salt of some long
This process of ester formation is called esterification. chain carboxylic acids (fatty acid). Sodium salts of
(C) Decarboxylation : fatty acids are known as hard soaps and potassium
salts of fatty acid are known as soft soaps. A soap
W hen sodium ethanoate is heated with soda lime,
methane is formed. has a large non-ionic hydrocarbon group and an ionic
CaO COO – Na + group.The structure of soap can be
CH3COONa + NaOH 
 CH4 + Na2CO3
represented as :
Sodium Methane
+
ethanoate – Na
 The term‘ decarboxylation ‘ is used when the , where represents the hydrocarbon group
elements of carbon dioxide are removed from a and – represents negatively charged carboxyl group.
molecule. Some examples of soaps are sodium stearate,

PAGE # 140
C17H35COO– Na+, sodium palmitate, C15H31COO– Na+ Examples :
and sodium oleate, C17H33COO–Na+ .
 Hard water, which contains salts of magnesium and – + –+
calcium, reacts with soap to form magnesium and C12H25 SO3Na , C12H25 – O – SO2 – ONa
calcium salts of fatty acid. Sodium p-dodecyl Sodium lauryl
benzenesulphonate sulphate
(i) Preparation of soap : Soap is prepared by heating
oil or fat of vegetable or animal origin with
concentrated sodium hydroxide solution (caustic  W ashing powders available in the market contain
soda solution). Hydrolysis of fat takes place and a
about 15 to 30 percent detergents by weight.
mixture of sodium salt of fatty acids and glycerol is
formed. Since the salt of fatty acids thus formed are
used as soap so alkaline hydrolysis of oils and fats  Alkaline hydrolysis of oils and fats is commonly known
is commonly known as saponification.
as saponification.
(c) Comparison Between Properties of
CH2O. COC15H31 CH2OH
Soaps and Detergents :
CHO. COC15H31 + 3NaOH CHOH + 3C15H31COONa
Soap
(sodium palmitate) S.N. Soaps Synthetic detergents
CH2O. COC15H31 CH2OH
Fat or oil Glycerol Synthetic detergents are
(Tripalmitin)
sodium alkyl sulphates
Soaps are sodium
(ii) Limitation of soaps : Soap is not suitable for or sodium alkyl benzene
washing clothes with hard water because of the 1 salts of higher
sulphonates with alkyl
following reasons. fatty acids
group having more than ten
(A) Hard water contains salts of calcium and
carbon atoms.
magnesium. W hen soap is added to hard water,
calcium and magnesium ions of hard water react
with soap forming insoluble calcium and magnesium Soaps are prepared Synthetic detergents are
salts of fatty acids. 2 from prepared from the
2C17H35COONa + MgCl2  (C17H35COO)2 Mg + 2NaCl natural oils and fats. hydrocarbons of petroleum.
Soap (White ppt)
Soaps form
2C17H35COONa + CaCl2  (C17H35COO)2 Ca + 2NaCl insoluble salts Calcium and magnesium
(White ppt) (curdy white ppt.) salts of detergents are
Therefore, a lot of soap is wasted if water is hard. with calcium and soluble in water and,
(B) W hen hard water is used, soap forms insoluble 3 magnesium which therefore, no curdy white
precipitates of calcium and magnesium salts, which are present in hard precipitates are obtained in
stick to the cloth being washed . Therefore, it interferes water and hence, hard water and hence, can
with the cleaning ability of the soap and makes the cannot be used in be used even in hard water.
cleaning process difficult. hard water.
 These calcium and magnesium salts of fatty acid are
Soaps cannot be
insoluble in water and separate as curdy white They can be used in acidic
used in acidic
precipitate. medium as they are the salt
medium as they are
4 of strong acids and are not
(b) Detergents : decomposed into
decomposed in acidic
carboxylic acids in
medium.
These are also called synthetic detergents or acidic medium.

soapless soaps. A synthetic detergent is the sodium Some of the synthetic


Soaps are
salt of a long chain benzene sulphonic acid or the 5 detergents are not
biodegradable.
sodium salt of a long chain alkyl hydrogen sulphate. biodegradable.

(i) Preparation of Synthetic Detergents : Synthetic ( d) A dv anta ge s of S ynthet ic Det er ge nt s


Ove r Soap s :
detergents are prepared by reacting hydrocarbons
from petroleum with conc. sulphuric acid and Synthetic detergents are widely used as cleaning
agents these days. Some of their advantages over
converting the product into its sodium salt.
soaps are :
(i) Synthetic detergents can be used for washing even
in hard water. On the other hand, soaps are not
suitable for use with hard water.

PAGE # 141
(ii) Synthetic detergents can be used even in acidic  In a soap molecule hydrophilic polar end is water
solutions because they are not readily decomposed
soluble and hydrophobic hydrocarbon part is
in acidic medium. On the other hand, soaps cannot
insoluble in water.
be used in acidic medium because they are
decomposed into carboxylic acids in acidic medium. Soap or detergent helps in cleansing in another way.
(iii) Synthetic detergents are more soluble in water Not only it emulsifies oil or grease but it also lowers
than soaps. the surface tension of water. As a result of this water
wets things more effectively.
(iv) Synthetic detergents have a stronger cleaning
action than soaps. When water is added on to the surface of the cloth
then water molecules tend to stay as close to each
(e ) Cl ea ns ing Ac ti on of Soap s and
other as possible because of the strong forces of
De terg ents :
attraction (hydrogen bonding ) for each other and do
A molecule of soap is made up of two parts : a non not wet the cloth properly. If some soap solution is
polar part consisting of a long chain of 12 - 18 carbon added to this water then polar end of soap dissolves
atoms and a polar part, –COO–Na+. The polar end is in water and non polar hydrocarbon end remains
water soluble and is thus hydrophilic whereas away from the water. Thus, soap molecules arrange
hydrocarbon part is insoluble in water and is thus themselves between the water molecules on the
hydrophobic. surface of water and decrease the forces of attraction
In a soap solution, the hydrocarbon portions of several
between the water molecules. Water can now spread
soap molecules huddle together to form aggregates
on the surface of cloth and can make it wet effectively.
of molecules (or ions) called micelles. The soap
micelles are negatively charged due to the presence
of carboxylate ions at the surface. Repulsion between (f) Synthetic Detergents : A Serious Problem
similarly charged micelles keeps them dispersed in :
the solution.
 The hydrocarbon part is however soluble in non-polar It may be noted that in the past, the widespread use
solvents and is sometimes called lipophilic. of detergents caused pollution of rivers and other water
bodies. Earlier the synthetic detergents were made
Cleansing action of soap : Mostly the dirt is held to
from long chain of hydrocarbons having a lot of
any surface such as cloth by the oil or grease which
branched chains in them. These branched chain
is present there. Now since the oil and grease are detergent molecules were degraded very slowly by
not soluble in water, the dirt particles cannot be the micro organisms present in water bodies like
removed by simply washing the cloth with water. lakes or rivers.
However, when soap is applied, the non polar
Therefore, they tend to remain in water bodies for a
hydrocarbon part of the soap molecules dissolves in long time and make water unfit for aquatic life. For
oil droplets while the polar – COO – Na + groups example, detergents containing phosphates can
remain attached to water molecules. In this way, each cause rapid growth of algae and therefore, deplete
oil droplet gets surrounded by negative charge. the dissolved oxygen present in the water of lakes
Water and rivers. As a result of lack of oxygen, fish and other
Na+ Na+
Na+ – –
+ aquatic animals may die.
– – Na
To solve these problems, now-a-days, the detergents
Na+ – Oil – Na+ are prepared from hydrocarbons which have
droplet
Micelle minimum branching. These are degraded more easily
Na+

– Na+ than branched chain detergents. Therefore, these are
– – Na+
Na+ biodegradable and create less problems.

These negatively charged oil droplets cannot


coalesce and continue breaking into small droplets EXERCISE-1
.These oil droplets (containing dirt particles) can be
washed away with water along with dirt particles. So, I. Organic compound defination & classification,
the action of soap or detergents is to emulsify oil or Hydrocarbon, Homologous series
grease, this loosens the solid particles of dirt and
they are removed. 1. Which of the following is not an example of aromatic
compound ?
(A) Benzene (B) Naphthalene
(C) Cyclobutane (D) Phenol

PAGE # 142
2. Which of the following is an alkyne ? 14. The general formula of saturated hydrocarbons is -
(A) C4H8 (B) C5H8 (A) CnH2n (B) CnH2n+2
(C) C7H19 (D) None of these (C) CnH2n–2 (D) CnH2n+1

3. The first organic compound synthesized in the 15. The third member of methyl ketone homologous
laboratory was - series is -
(A) urea (B) glucose (A) Acetone (B) 2–Butanone
(C) alcohol (D) None of these (C) 2–Pentanone (D) 3–Pentanone.

4. Propane is an - 16. The general molecular formula of alkynes is -


(A) acyclic compound (A) CnH2n (B) CnH2n–4
(B) open chain compound (C) CnH2n–2 (D) CnH2n+2
(C) alipthatic compound
17. Which of the following is not a pair of homologues ?
(D) All of these
(A) Ethylacetylene and Dimethylacetylene
5. The scientist who gave vital force theory was - (B) Methylacetylene and Ethylacetylene
(A) Berzelius (B) Avogadro (C) 1-Butyne and 2-Pentyne
(C) Wohler (D) Lavoisier (D) 1-Pentyne and 3-Hexyne

6. Which one of the following is not an organic compound ? II. Nomeclature & organic compounds
(A) Hexane (B) Urea
18. The IUPAC name of the compound having the formula
(C) Ammonia (D)Ethyl alcohol
(CH3)3 CCH = CH2 is -
7. Vast number of carbon compounds is due to the fact
that carbon has - (A) 3,3,3-Trimethyl -1-propane
(A) variable valency
(B) 1,1,1-Trimethyl-1-butene
(B) property of catenation
(C) 3,3-Dimethyl-1-butene
(C) great chemical affinity
(D) None of these (D) 1,1–Dimethyl- 3 -butene

8. Any two consecutive members in a homologous 19. The IUPAC name of the following compound is -
series differ in molecular mass by- CH2 = CH – CH (CH3)2
(A) 8 (B) 14 (A) 1,1-Dimethyl -2-propene
(C) 24 (D) 12 (B) 3-Methyl-1-butene
(C) 2-Vinylpropane
9. The nature of linkage in organic compounds is
(D) 1-Isopropylethylene
generally -
(A) ionic bond 20. The IUPAC name of the following compound is -
(B) covalent bond
(C) co-ordinate covalent C2H5
(D) metallic bond CH3 – CH – C = CH2
10. Which of the following statements is incorrect ? CH3
The members of the homologous series of alkanes
(A) 3-Ethyl-2-methylbut-3-ene
(A) are all straight chain compounds.
(B) 2-Ethyl-3- methylbut -1-ene
(B) have the general formula CnH2n+2.
(C) 2-Methyl-3-ethylbut-3-ene
(C) have similar chemical properties .
(D) 3-Methyl-2-ethylbut-1-ene
(D) show a regular gradation of physical properties.
O
11. Which of the following forms a homologous series ? ||
21. The IUPAC name for CH3 – C – H is -
(A) Ethane, Ethylene, Acetylene
(B) Ethane, Propane, Butanol (A) Acetal (B) Methanal
(C) Methanal, Ethanol, Propanoic acid (C) Ethanal (D) Acetaldehyde
(D) Butane, 2-Methylbutane, 2,3-Dimethylbutane
22. The IUPAC name of compound
12. Homologous have the same -
(A) empirical formulae H
(B) molecular formulae CH3 – C – CH2 – CH3 is -
(C) chemical properties
COOH
(D) physical properties
(A) Butan -3- oic acid
13. Write down the general formula of homologous series (B) Butan -2- oic acid
whose third homologue is C4H6 ?
(C) 3-Methylbutanoic acid
(A) CnH2n – 2 (B) Cn H2n + 2
(D) 2-Methylbutanoic acid
(C) Cn + 1 H2n – 2 (D) Cn H2n

PAGE # 143
23. The functional group, present in CH3COOC2H5 is - IV. Alkane, Alkene, Alkyne
(A) ketonic (B) aldehydic 32. Which of the following properties is not true regarding
(C) ester (D) carboxylic organic compounds ?
(A) They are generally covalent compounds.
24. The IUPAC name of-
(B) They have high melting and boiling points.
CH3 – C(CH3) (OH) CH2 – CH(CH3) CH3 is -
(C) They are generally insoluble in water.
(A) 2,4-Dimethylpentan -2-ol
(D) They generally show isomerism.
(B) 2,4- Dimethylpentan-4-ol
(C) 2,2-Dimethylbutane 33. The values of bond energies of single, double and
(D) Isopentanol triple bonds are in the order -
(A) C – C > C = C > C  C
25. The IUPAC name of (CH3)2 CHCH2 CH2 Br is-
(B) C = C > C – C > C  C
(A) 1-Bromopentane
(C) C  C > C = C > C – C
(B) 2-Methyl-4-bromopentane
(D)C = C > C  C > C – C
(C) 1-Bromo -3- methylbutane
(D) 2-Methyl-3-bromopentane 34. Ethyne on passing through a red hot iron tube gives -
(A) mesitylene (B) benzene
26. Which of the following does not belong to homologous (C) butenyne (D) None
series of alkanes ?
(A) C2H6 (B) C3H4 35. Ethyne is obtained by dehydrobromination of -
(C) C4H10 (D) C5H12 (A) CHBr = CHBr (B) CH3CHBr2
(C) CH3–CH2Br (D) None of these
III. Isomerism
36. Ethyne is isoelectronic with-
27. How many chain isomers (non- cyclic aliphatic) could (A) chlorine (B) oxygen
be obtained from the alkane C6H14 ? (C) nitrogen gas (D) CO2
(A) 6 (B) 5 37. C  C bond length is -
(C) 4 (D) 3 (A) 1.54 Å (B) 1.20 Å
(C) 1.34 Å (D) 1.39 Å
28. The isomerism exhibited by n-propyl alcohol and
isopropyl alcohol is - 38. Which of the following gives silver nitrate test ?
(A) chain isomerism (A) Methane (B) Ethene
(B) position isomerism (C) Ethyne (D) All
(C) functional isomerism 39. Which of the following does not decolourise bromine
(D) None of these water ?
(A) Alkanes only (B) Alkenes only
29. Isomers have - (C) Alkynes only (D) (B) and (C) both
(A) same molecular formula & same structure.
(B) different molecular formula & different structure.
EXERCISE-2
(C) same molecular formula & different structure.
(D) different molecular formula & same structure.
COMPETITIVE EXAM PREVIOUS YEARS’ QUESTIONS :
30. Which of the following pairs is an example of chain
isomer ? 1. Unsaturated fatty acids contain -
[IJSO-State-I/2011]
(A) CH3 – CH2 – OH & CH3OCH3
(A) atleast one double bond
(B) CH3 – CH2 – CHO & CH3 – CO-CH3
(B) two double bonds
(C)CH3–CH2–CH2–CH2–CH3 & (C) more than two double bonds
(D) no double bond
(D) All of the above
2. A compound used for cleaning purpose having hy-
31. W hich of the following is a functional isomer of drophobic and hydrophilic ends is :
CH3 – COOH ? [IJSO-Stage-II/2013]
(A) Sodium or potassium salt of saturated or
unsaturated fatty acids
(A) CH3 – CH2 – OH (B)
(B) Triglycerides of saturated or unsaturated fatty
acids
(C) Monoesters of saturated or unsaturated fatty
(C) (D) All of these
acids
(D) Triglycerides of unsaturated fatty acids

PAGE # 144
3. Hydrogenation is a reaction in which hydrogen is
added to a compound. One of the applications of 6. The maximum number of isomers for an alkene with
hydrogenation reaction is to convert unsaturated molecular formula C4H8 is : [IJSO-State-I/2013]
compounds into saturated ones. This reaction is
(A) 5 (B) 4
applicable to which of the following compounds ?
[IJSO-Stage-II/2013] (C) 2 (D) 3
(A) Ethyl alcohol (B) Chloroethane
(C) Vegetable oil (D) Animal fat 7. Methylcyclobutane is a saturated alkane. What is its
molecular formula? [IJSO-Stage-I/2015]
4. One of the test used to distinguish a saturated from
(A) C5H10 (B) C5H8
an unsaturated compound is bromine water test.
(C) C5H12 (D) C5H6
Ethene and ethane are reacted with bromine water
and the results are displayed on the table given
8. W hich among the following organic compounds is
below. From the following table choose the correct
likely to have more than one possible structure ?
observation. [IJSO-Stage-II/2013]
[IJSO-Stage-I/2014-15]
Ethene Ethane
(A) C3H6 (B) C3H8
Br Br Br H (C) C2H4 (D) CH4

(A) H–C–C–H Br–C–C–H 9. Given below are the structures of the famous mol-
H H ecules called Aspirin and Paracetamol. Which among
H H
the listed functional groups do the two molecules put
with decolourisation with decolourisation together NOT contain ?

Br H Br Br

(B) Br–C–C–H H–C–C–H

H H H H

with no decolourisation with no decolourisation

Br Br
(C) H–C–C–H no reaction

H H [IJSO-Stage-I/2014-15]
(A) Ester (B) Ketone
with decolourisation (C) Alcohol (D) Carboxylic acid

Br Br 10. From the following pool of molecules


(D) no reaction Br–C–C–Br
H H

with decolourisation

5. The isomerism which exists between CH3CHCl2 and


CH2CICH 2Cl is : [IJSO-State-I/2013]
(A) chain isomerism
(B) functional group isomerism
(C) positional isomerism
[IJSO-Stage-I/2014-15]
(D) matamerism
How many isomers are represented above
(A) 2 (B) 3 (C) 4 (D) 5

PAGE # 145
11. Which of the following Lewis dot structures best (A) oxidation, oxidation, reduction, reduction
describes the structure of peroxide ion of sodium (B) hydration , oxidation, reduction, dehydration
peroxide? [IJSO-Stage-I/2014-15] (C) reduction , dehydration, hydration, oxidation
X- electrons from oxygen . (D) reduction, reduction, oxidation, oxidation
•- electrons from sodium
14. How many different compounds can have the formula,
C 3H 4? [IJSO-Stage-I/2016-17]
(A) One (B) Two
(C) Three (D) Four
(A)

(B)

(C)

(D)

12. The following substances have approximately same


molecular mass. Which is likely to have the highest
boiling point ? [IJSO-Stage-I/2016-17]
(A) n-butane (B) isobutane
(C) n-butanol (D) isobutanol

13. Identify the overall change in the following set of reac-


tions :
1. Carbon dioxide  carbonic acid (H2CO3)
2. Ethanol (alcohol)  Ethanal (aldehyde)
3. Ethanal (aldehyde)  Ethanol (alcohol)
4. Sulphuric acid  Sulphur trioxide (SO3)
Choose the correct option which best describes these
conversions [IJSO-Stage-I/2016-17]

PAGE # 146
STUDY OF GAS LAWS

INTRODUCTION This behaviour was generalised and named as


Boyle's law as stated below -
Gas laws are the rules which the gases obey when
"Temperature of an enclosed mass of dry gas
subjected to changes in volume, temperature or
remaining constant, its volume is inversely
pressure. Any change in one of the aforesaid variables proportional to pressure.
affects the other two variables. " Let ‘V’ be the volume of an enclosed dry gas and ‘P’
For example, if the pressure of a gas undergoes some is its pressure, such that temperature is constant.
significant change, its volume and temperature also
1
change. These variables are discussed below - So, V  (at constant T)
P
(a) Volume : 1
V=K [ K is constant of proportionality]
Gases always occupy the complete volume of the P
container on account of their high expansion. Thus, PV = K
the volume of a gas is always equal to the volume of If the temperature of a gas is kept constant, such that
container. its pressure changes to P 1 and then P 2, when
Units of Volume : corresponding volumes are V 1 and V 2 respectively,
then according to Boyle's law -
The volume of gases is measured in the following
P1V1 = K ...(i)
units :
P2V2 = K ...(ii)
(i) 1 millilitre (1 ml) = 1 cm3 (1 cc)
Comparing (i) and (ii), we have
(ii) 1 litre (1  ) = 1 cubic decimeter (dm3) P1V1 = P2V2
The above relation is called Boyle's law equation.
(iii) 1  = 1 dm3 = 1000 cc = 1000 ml. From the above equation, the Boyle's law can be
(b) Temperature : defined as under -
The temperature of a gas is the average kinetic energy "Temperature of an enclosed mass of dry gas
of its molecules. If the average kinetic energy of the remaining constant, the product of its pressure and
molecules of a gas increases, its temperature rises volume is a constant quantity."
and vice versa. The table given below shows the experimental data
for the validity of Boyle's law. The data are also plotted
Units of temperature : on graph to illustrate change in volume with the
The temperature is measured in the following units - change in pressure at constant temperature.
(i) Celsius temperature is measured in degrees
Pressure (P) Volume (V)
Celsius = °C (in cm of Hg) (in litres)
P×V

(ii) Kelvin temperature is measured in kelvin = K. 10 2.0 20

(iii) Temperature in kelvin = 273 + temperature in °C 20 1.0 20


K = 273 + °C 30 0.67 20 (approx)

(c) Pressure : 40 0.5 20


One of the fundamental properties of a gas is
pressure. Formally, pressure is defined as the force 10 cm 20 cm 30 cm 40 cm
of Hg of Hg of Hg of Hg
per unit area.
Units of pressure :
2.0
The pressure is measured in the following units -
(1) 1 atmosphere (atm) = 760 mm Hg
(2) N/m2 or pascals (Pa)
(3) 1 atm = 760 mm Hg = 760 torr = 1.01325 × 105 Pa
Volume in Litres

1.0
BOYLE'S LAW
0.67
Sir Robert Boyle (1662) studied the relationship 0.50
between the volume of a fixed mass of an enclosed
gas at a constant temperature by increasing or
decreasing pressure on it. He found that on 0
10 20 30 40
increasing pressure the volume of the gas decreases Pressure in cm of Hg

and vice versa.

PAGE # 147
Conclusion : The product of pressure and volume (P Ex.4 At constant temperature the volume of a certain gas
x V) is a constant quantity provided that the was found 800 cm3, when pressure was 760 mm of
temperature is constant.
mercury. If the pressure increases by 25% , find the
T3
T2 new volume of gas.
Sol. Initial volume of gas (V1) = 800 cm3

Pressure (p)
T1
Final volume of gas (V2) = ?
Isotherms
Initial pressure of gas (P1) = 760 mm of Hg
T3 > T2 > T1
0 Volume(1/V)
25
Increase in pressure = 760 × = 190 mm of Hg
 Note : 100
A curve plotted between P and V at constant
 Final pressure of gas (P2) = 760 +190 = 950 mm of Hg
temperature is known as isotherm.
By Boyle’s law : P1V1 = P2V2
The size of weather balloon keeps on becoming
larger as it rises to higher altitude because at higher
P1V1 760  800
altitude the external pressure (i.e., atmospheric  V2 = P = = 640 cm3.
pressure) on balloon goes on decreasing and thus, 2 950
size of balloon increases.
Ex.5. A vessel of capacity 12 dm3, contains nitrogen gas at
Ex.1 A gas occupies 1500 cm3 at pressure of 720 mm of
mercury. Find at what pressure its volume is 1000 a pressure of 152 cm of Hg. If this vessel is connected
cm 3. Assume temperature remains constant to another evacuated vessel of 6 dm3 capacity, what
throughout the experiment. will be the pressure of nitrogen in both vessels.
Sol. Initial volume of gas (V1) = 1500 cm3
(Assume that temperature remains constant).
Initial pressure of gas (P1) = 720 mm of Hg
Final volume of gas (V2) = 1000 cm3 Sol. Given that - V1 = 12 dm3 , V2 = 12 + 6 = 18 dm3,
Final Pressure of gas (P2) = ? P1 = 152 cm of Hg , P2 = ?
By Boyle's law : P1V1 = P2V2
By Boyle’s law : P1V1 = P2V2
P1V1 720  1500 P1V1 152  12
 P2 = V = = 1080 mm of Hg.  P2 = = = 101.33 cm of Hg.
2 1000 V2 18
Ex.2. A gas occupies a volume of 800 cm3 at a pressure P.
If the pressure is altered to 2.5 atm, the volume of CHARLES' LAW
gas found 900 cm3. Calculate the value of P.
Sol. Given that - P1 = P , P2 = 2.5 atm , V1 = 800 cm3 , In 1787, Jacques Charles experimentally studied the
V2 = 900 cm3 relationship between the volume of a fixed mass of
By Boyle's law : P1V1 = P2V2
an enclosed dry gas and the temperature, when the
2.5  900
 P × 800 = 2.5 x 900 P = = 2.81 atm pressure of the gas was kept constant, throughout
800
the experiment. He found out -
Ex.3. At constant temperature, a gas is at a pressure of
1080 mm of mercury. At what pressure its volume will ''The volume of the fixed mass of an enclosed dry gas
decrease by 40% ? increases by 1 / 273th part of its initial volume at 0 ºC
Sol. Let initial volume of gas (V1) =V
for every 1 ºC rise in temperature and vice versa,
40 V
 40% of initial volume = = 0.4 V provided the pressure remains same throughout the
100
 Final volume of gas (V2) = V - 0.4 V = 0.6 V experiment.”
Initial pressure of gas (P1) = 1080 mm of Hg For example, if the initial volume of an enclosed dry
Final pressure of gas (P2) = ? gas is 273 cc, at 0 ºC, then its volume for 1 ºC rise in

By Boyle's law : P1V1 = P2V2 temperature will be -


1
P1V1 1080  V Volume at 1 °C = 273 + × 273 = 274 cc
 P2 = = = 1800 mm of Hg.
273
V2 0 .6 V 1
Conversely, volume at -1°C = 273 – x 273 = 272 cc.
273

PAGE # 148
(a) Concept of Absolute Zero (Kelvin Zero) (e ) De fi ni ti on of Char le s’ Law B as ed on
Te mp erature : Kelvin Scale :
Lord Kelvin, by applying Charles' experimental Pressure of an enclosed mass of dry gas remaining
deductions, theoretically tried to calculate the volume constant, the volume of the gas is directly proportional
of fixed mass of an enclosed gas at constant to its kelvin temperature (absolute temperature).
pressure. According to him, if initial volume of dry Thus, if V is the volume of an enclosed mass of dry
enclosed gas is 273 cc at 0 °C, then : gas and T is the kelvin temperature, then
1 V T [at constant pressure]
Volume of gas at –1 °C = 273 + x 273 x –1 = 272 cc. V
273 or = K [K is the constant of proportionality]
1 T
Volume of gas at –100 °C = 273 + x 273 x – 100 = 173 cc. Now, if we consider V1 as the volume of gas at T1 (K)
273 and V 2 as the volume of gas at T 2 (K), such that
1 pressure of the given mass of dry gas remains
Volume of gas at – 273 °C = 273 + x 273 x – 273 = 0 cc.
273 constant, then -
Thus, according to Lord Kelvin, if the temperature of V1  T1 ; V2  T2
an enclosed gas at 0 °C is lowered to –273°C, its V1 V2
or
volume becomes zero. However, this is not possible. T1 = K ;
T2 = K
It is because gas is one of the states of matter and
hence, must have some definite mass and volume. V1 V2
 T = T
The other alternative can be that Charles' 1 2
experimental deductions were wrong. However, this The above equation is called Charle’s law equation.
is not possible, as the experimental observations can A graph is shown between the volume and kelvin
be verified independently. temperature, when a fixed mass of dry gas is heated
Lord Kelvin offered the solution to the above riddle. at a constant pressure.
He suggested that in all probability
(i) If the gas is initially at 100 K, its volume increases
- 273°C is the last limit of temperature, which cannot
to four times when heated to 400 K.
be reached and hence, an enclosed gas will never
(ii) If the temperature is lowered below 100 K, the
have zero volume.
gas liquefies. Thus, the experimental points cannot
 Note : be plotted as shown by a dotted line.
The last limit of temperature was named absolute
zero by Lord Kelvin. However, in order to honour Lord
Kelvin, the absolute zero was renamed as Kelvin zero.
(b) Definition of Absolute Zero (Kelvin Zero) :
It is the last limit of the lowest temperature, where the
volume of a given mass of dry enclosed gas at
constant pressure becomes zero. Its theoretical value
is -273 °C.
or it is defined as theoretical temperature, when the
molecules of an enclosed dry gas at constant
pressure have zero kinetic energy, i.e., they stop
vibrating.
(c ) Conc ep t of A bs ol ut e Sc al e (Kel vi n
Scale) of Temperature :
The new temperature scale with its zero at -273°C,
such that each degree on it is equal to 1 degree on
the celsius scale is called absolute scale or kelvin
scale.
(d ) Char ac te ri st ic s of A bs ol ut e Sc al e
(Kelvin Scale)  Note :
A curve plotted between V and T at constant pressure
(i) The temperature scale, with kelvin zero as starting
is known as isobar.
point, is called kelvin scale.
(ii) All temperature on kelvin scale are positive.
(iii) Temperature on kelvin scale = 273 + temperature
in ºC.
K = 273 + oC
(iv) Temperature on kelvin scale is not expressed in
degrees. For example, 273 K is the correct
temperature and not 273 oK.

PAGE # 149
Ex.8. A gas is enclosed in a vessel, at standard
CONCEPT OF STANDARD temperature. At what temperature the volume of
TEMPERATURE AND PRESSURE
enclosed gas is 1/8 of its initial volume, pressure
From the Boyle's law and Charles' law it is very clear remaining constant ?
that the volume of a given mass of dry enclosed gas Sol. Let initial volume of gas (V1) = x
depends upon the Initial temperature of gas (T1) = 0 ºC = 273 K
(i) pressure of the gas x
Final volume (V2) =
(ii) temperature of the gas in kelvin. 8
Final temperature of gas (T2) = ?
Thus, we cannot correctly express the volume of a
given mass of dry enclosed gas, unless we specify V1 V2
By Charle’s law : 
or standardise the temperature and pressure. Thus, T1 T2
to compare the mass or the density of two or more
x x
gases having same volume, we must standardise  =
the temperature and pressure at which the volume of 273 8  T2
the gases is measured.
This standard temperature and standard pressure
273  x
T2 = = 34.125 K
for all gases is called standard temperature and 8x
pressure whose short form is written as S.T.P.
= 34.125 – 273 = – 238.875 ºC.
Standard temperature is taken as 0 o C or 273 K.
Standard pressure is taken as 76 cm or 760 mm of
CONCEPT OF GAS EQUATION
Hg or one atmosphere.
It has been found that in all practical situations , the
PRESSURE-TEMPERATURE LAW volume, the pressure and the temperature of an
OR GAY-LUSSAC'S LAW
enclosed gas change simultaneously, when any of
According to this law the pressure of a given mass of the above variables are altered. Thus, there is a need
gas is directly proportional to absolute temperature to have a mathematical equation which connects
at constant volume. these variables.
P  T (at constant V and for a fixed amount of gas)
P P1 P2 Definition :
or = constant or T  T
T 1 2 “A mathematical equation used in calculating the
Ex.6. 279 cm 3 of gas at 87°C is cooled to standard change in volume when the initial temperature and
temperature, at constant pressure. pressure of an enclosed gas simultaneously change
Calculate the volume of gas at standard temperature. is called gas equation.”
Sol. Initial volume of gas (V1) = 279 cm3
The perfect gas equation can be derived by combining
Initial temperature of gas (T 1) = 87°C = (273 + 87)
K = 360 K Boyle’s law and Charles’ law .
Final temperature of gas (T2) = 0 °C = (0 + 273) = 273 K Consider an enclosed dry gas of volume V at pressure
Final volume of gas (V2) = ? P and temperature T kelvin.
V1 V2 1
By Charle’s law : Applying Boyle’s law - V 
T1 = T2 P
(At constant temperature) -- (i)

279 V2 279  273 Applying Charles’ law - V  T (At constant pressure)--- (ii)
= V2 = = 211.57 cm3
360 273 360 Combining (i) and (ii)
Ex.7. A dry gas occupies a volume of 1054 dm 3, at a 1
temperature of –73 ºC . At what temperature its V ×T
P
volume is 4216 dm 3 when the pressure remains T
constant throughout experiment ? or V = K × (K = constant)
P
3
Sol. Initial volume of gas (V1) = 1054 dm PV
Initial temperature of gas ( T1) = – 73º C = (–73 + 273) or =K
K = 200 K T
Final volume of gas (V2) = 4216 dm3 If the initial volume of an enclosed dry gas is V1, when
Final temperature of gas (T2) = ? its pressure is P1 and temperature is T1(K), such that,
V1 V2 its volume changes to V2, when its pressure is P2 and
By Charle’s law : = temperature is T2(K), then -
T1 T2
1054 4216 4216  200 P1V1
 =
T2
 T2 = = 800 K T1 = K (constant) ------ (iii)
200 1054
 Temperature in ºC = (800 – 273) = 527 ºC. P2 V2
= K (constant) ------ (iv)
T2

PAGE # 150
Combining (iii) and (iv)
AVOGADRO’S LAW
P1V1 P2 V2
T1 = T2 According to this law equal volumes of gases under
[Gas equation] similar conditions of pressure and temperature
Initial pressure Initial volume possess equal number of moles or molecules.
or V  NA (at constant P and T)
Initial temperature in kelvin
Vn
Final pressure  Final volume where, NA and n are number of molecules and moles
= Final temperatur e in kelvin of gas taken respectively.
Ex.9 1.57 dm3 of dry hydrogen gas is at a pressure of 750
mm of mercury when the temperature is 37.5 ºC. Find IDEAL GAS EQUATION
the volume of gas at S.T.P.
Sol. P1 = 750 mm of Hg According to Boyle’s law : V  (1/P)
P2 = 760 mm of Hg According to Charle’s law: V  T
V1 = 1.57 dm3 According to Avogadro’s law : V  n
V2 = ? Thus, V  (nT/P)
T1 = (273 + 37.5) K = 310.5 K T2 = 273 K PV  nT or PV = nRT ----- (i)
P1V1 P2 V2 Equation (i) is known as ideal gas equation.
By gas equation : = R is known as universal gas constant or molar gas
T1 T2 constant. The term ideal gas refers to one which
P1V1 T2 750  1.57  273 obeys equation (i) in all temperature and pressure
V2 = × = = 1.36 dm3
T1 P2 310 .5  760 ranges. However, since none of the gases present in
universe obeys this equation rigidly and thus, gases
Ex.10. Sulphur dioxide occupies a volume of 512 cm3 at are named as real or non-ideal gases.
S.T.P. Find its volume at 27 ºC and at a pressure of
720 mm of mercury.
Sol. P1 = 760 mm of Hg
P2 = 720 mm of Hg Gas equation for one gram mole,
V1 = 512 cm3 PV = RT
V2 = ?
P  V Pr essure  Volume
T1 = 273 K or R =
T Temperature
T2 = 273 + 27 = 300 K
P1V1 P2 V2 Force Force
By gas equation : = Pressure = = 2
T1 T2 Area (Length )
P1V1 T2 760  512 300 Volume = (Length)3
 V2 = T × P =  = 593.89 cm3.
1 2 273 720
Force
Ex.11. A gas occupies a volume of 100 cm3 at 0 ºC and  (Length )3
760 mm Hg pressure. If the kelvin temperature of the
(Length )2
So, R=
gas is increased by one fifth and its pressure is Temperatur e
increased one half times, calculate the final volume
of gas. Force  Length
=
 760  Temperature
Sol. P1 = 760 mm of Hg P2 =  760   = 1140 mm of Hg
 2  Work
V1 = 100 cm3 =
Temperature
V2 = ?
Thus, the value of R should always be expressed in
 273  units of work per degree kelvin per mole.
T1 = 0 ºC = 273 K T2 =  273   K = 327.6 K
 5 
Numerical values of R :
P1V1 P2 V2 (a) In litre atmosphere : R = 0.0821 litre atm K–1 mol 1
By gas equation : =
T1 T2
(b) In C.G.S. System : R = 8.314 × 107 erg K–1 mol–1
(c) In S.I. system : R = 8.314 JK–1 mol–1
P1V1 T2 760  100  327.6
 V2 = T × P = = 80 cm3. (d) In calories : R = 1.987 calorie K–1 mol–1
1 2 273  1140
 Note :
Real gases behave almost ideally at low P and high T.

PAGE # 151
where d1, d2 are distances travelled by molecules in
narrow tube in time t1 and t2
It is commonly observed that whenever two gases w
Also, we have n=  
are placed in contact with each other, they readily in- M
termix with each other without the aid of any external n1 t 2 M2
agency.  By equation  
t1 n 2 M1
Such a phenomenon of intermixing of two or more
gases, irrespective of density relationship and with- w 1 M2 t 2 M2
 =
out the help of external agency is called diffusion. M1t 1 w 2 M1
Effusion is a process in which a gas escapes from w1 t 2 M1
high pressure region to low pressure region through or  
t1 w 2 M2
a porous portion, with microscopic holes or through
a single pinhole. where w1, w2 are weights diffused in time t1 and t2.
Ex.12. 20 cm3 of SO2 diffuses through a porous plug in 60
GRAHAM’S LAW OF DIFFUSION seconds. What volume of O2 will diffuse under similar
conditions in 30 seconds ?
According to Graham, the rate of diffusion of a gas at Sol. V1 = 20 cm3 V2 = V
constant P and T is inversely proportional to square t1 = 60 sec. t2 = 30 sec.
root of its molecular weight. M1 = 64 M2 = 32
1 V1 t 2 M2
r (at constant P and T)  
M t 1 V2 M1
r1 M2 20 30 32
  (at constant P and T) 
r2 M1  =
60 V 64
Since, molecular wt. of a gas = 2 × vapour density ()  V = 14.14 cm3
r1 2 Ex.13. One mole of nitrogen gas at 0.8 atm takes 38
  (at consant P and T )
r2 1 seconds to diffuse through a pinhole, whereas one
mole of an unknown compound of xenon with fluorine
The rate of diffusion r for two gases under different
at 1.6 atm takes 57 seconds to diffuse through the
pressure can be given by - same hole. Calculate molecular mass of the
r1   M 2  P1 

compound.
=    (at constant T) r1 M2 P1
r2   M1  P2   
Sol.
r2 M1 P2
Further rate of diffusion (r) can be expressed in terms
of - n1 t 2 M2 P1
or   
t1 n 2 M1 P2
Volume diffused ( V ) Moles diffused (n)
r = Time taken in diffusion = Time taken in diffusion
1 57 M 0.8
or   
38 1 28 1.6
Distance travelled in a narrow tube (d)
r=  M = 252
Time taken
Therefore, according to Graham’s law of diffusion at
constant P and T -
A mixture of gases that do not react with one another
V1 t 2  M2 behaves like a single pure gas. For example, we can
  2 
t1 V2 1 M1 treat air as a single gas when we want to use the
ideal gas laws to predict its properties. The total
where V1, V2 are volumes diffused in time t1 and t2
pressure exerted by a gaseous mixture is equal to
n1 t 2  M2 the sum of partial pressures of each component (gas)
  2  present in mixture.
t1 n2 1 M1
Thus, PM = P1 + P2 + P3 + ............ (i)
where n1, n2 are moles diffused in time t1 and t2. Partial pressures P1, P2 , ............ are defined as the
d1 t 2  M2 pressure exerted by that component if same amount
  2 
t1 d2 1 M1 of gas is filled in the same container at the same
temperature.

PAGE # 152
Let n1, n2, n3 , ......... are moles of gases 1,2,3, ....... Postulates of Kinetic Molecular Theory
which are filled in a container of volume V at 1. A gas consists of a large number of minute particles,
called atoms or molecules. The atoms or molecules
temperature T, then -
are so small that their actual volume is negligible as
n1RT compared to empty space between them. They are
P1 = ........ (ii) (R is gas constant)
V considered as point masses. This explains the great
n 2RT compressibility of the gases.
P2 = ......... (iii)
V 2. The molecules are supposed to be moving
n1RT n 2RT continuously in different directions with different
 PM = P1 + P2 + P3 + ......... =  + ......... speeds. If the particles were at rest and occupied
V V
RT fixed positions then gas should have fixed shape
or PM = (n1 + n2 + ...... ) ---------- (iv) geometry which is not there.
V
By Equations (ii) and (iv) 3. They keep on colliding with one another and with
the walls of the vessel. The pressure exerted by the
P1 n1
 gas is due to the bombardment of its molecules on
PM (n1  n 2  ....) the walls of the vessel.

 n1  4. The distance between the gas molecules being


 
 n  n  ....  mole fraction large, there is no effective force of attraction or
 1 2  repulsion between them.
 P1= PM × mole fraction of 1
5. The force of gravitation on the molecules is
i.e. Partial pressure of any constituent gas negligible.
= Total pressure × its mole fraction 6. The molecular collisions are perfectly elastic so
that there is no net loss of energy when gas molecules
Ex.14 Two gases A and B having molecular weights 60
collide with one another or against the walls of the
and 45 respectively are enclosed in a vessel. The
vessel.
weight of A is 0.50g and that of B is 0.20 g . The total
7. The average kinetic energy of the gas molecules is
pressure of the mixture is 750 mm. Calculate partial
directly proportional to the absolute temperature of
pressure of the two gases.
the gas.
Sol. Given that weight of gas A = 0.50 g and weight of gas The above postulates are meant for an ideal gas only.
B = 0.2 g. These are only approximately true for a real gas.
Molecular weight of A = 60 and mol. weight of B = 45
Pmixture = 750 mm, pA = Partial pressure of A and pB =
Partial pressure of B
From postulates of kinetic theory of gas, it is possible
From pA = PM × mole fraction of A to derive a mathematical expression, known as kinetic
gas equation which can be written as
0.5 / 60 PV = 1/3 mNu2,
pA = 750 × = 489.13 mm
 .5 0.2 
0 Where m is the mass of the gas molecule, N is the
  
 60 45  number of molecules in given volume V of the gas, u
is the root mean square speed of gas molecules
 pB = PM – pA = 750 – 489.13 and P is the pressure exerted by the gas molecules.
= 260.87 mm

The molecules of a gas moves very fast but their


The gas laws discussed earlier were merely based speed constantly changes due to collisions. For
on experimental observations. In order to explain the quantitative description of the behaviour of gases,
observed behaviour of gases, a theoretical model three different types of molecular speeds are
described for the gas molecules which are:
was proposed which takes into account the molecular
(a) Most Probable Speed
concept as we as the kinetic concept of gas
(b) Average Speed
molecules. This theoretical model which was put (c) Root Mean Square Speed
forward by Bernoulli and was further developed by
(a) Most Probable Speed :
Clausius, Maxwell, Kroning, Boltzmann and others is
Most probable speed of a gas at a given temperature
based upon microscopic model of a gas and is known is defined as the speed possessed by the maximum
as kinetic molecular theory of gases. fraction of the total number of molecules at that
temperature.
2RT
It is equal to
M
PAGE # 153
(b) Average Speed :
It is defined as the arithmetic mean of the speeds of
different molecules of the gas at a given temperature.
If all the n molecules of the gas are considered to
The three different kind of molecular speeds are
possess different speeds, say 1, 2, 3,......... n cm/
sec. then, related to each other as :
1  2  3  ........  n Most probable speed : Average speed : RMS speed
=
n 2RT 8RT 3RT
On the other hand, if n1 molecules of the gas have : :
M M M
speed 1cm sec–1, n 2 molecules have speed 2 cm/
sec, and n3 molecules have speed 3 cm/sec and so
on, then,
8
= 2 : : 3

n11  n 2 2  n 3  3  .......
= n1  n 2  n 3  ....... = 1 : 1.128 : 1.224
 is related to temperature as :
Ex. 15. Calculate the root mean square velocity of a
8RT molecule of sulphur dioxide at STP. What is its most
=
M probable velocity ?
(c) Root Mean Square Speed : Sol.Volume of 1 mole of SO2 at STP = 22400 ml
It is defined as the square root of the mean of the Molecular mass of SO2 = 64 amu
squares of the different speeds. P = 76  13.6  981 dynes cm–2
If all the molecules are assumed to have different Root mean square velocity,
velocities viz., 1, 2, 3,......, n cm/sec then
3PV
u=
2 2 2 M
1  2  3
u=
n
3  76  13.6  981 22400 NA
On the other hand, if n1 molecules have velocity 1 = 64  NA
cm/sec, n 2 molecules have velocity  2 cm/sec, n 3
= 3.263  104 cm sec–1
molecules have velocity 3 cm/sec and so on, then
Most probable velocity =0.82 × u
2 2 2 = 0.82 × 3.263 × 104
n11  n 2 2  n 3 3  .......
u= = 2.676 × 104 cm sec–1
n1  n2  n 3  .......
Ex.16. Oxygen has a density of 1.429 g per litre at STP.
u is related to temperature as :
Calculate the RMS and average velocities of the oxy-
3RT gen molecules.
u= Sol. (a) Calculation of RMS velocity
M
3P
u=
D
Here, P = 1 atmosphere
= 76 × 13.6 × 981 dynes cm–2
D = 1.429 g/litre

1.429
= g/ml = 0.001429 g/ml
1000
Putting the values in the above formula, we get Root
mean square velocity,

3  76  13.6  981
Distribution of molecular speed of Cl2 and N2 at 300K u=
0 .001429
It is also observed from the above relationships that
= 4.613×104 cm sec–1.
speed of a gas molecule at a given temperature also
(b) Calculation of Average velocity
depends upon its molecular mass. Lighter the
Average velocity = 0.9213 × u
molecules, more is their speed. Thus, lighter value
of most probable velocity than chlorine molecules. = 0.9213 ×4.61 ×104
= 4.2461 cm sec–1

PAGE # 154
(C) 40 litres (D) 20 litres

Real gases do not obey the gas laws or the ideal gas 3. If the volume of a given mass of a gas at constant
equation under all conditions of temperature and temperature becomes three times, the pressure will
pressure. This means that some of the postulates of be-
kinetic theory do not hold good under all conditions.
Two main postulates which need critical examina- (A) 3p (B) p/3
tion are discussed below : (C) 9p (D) p
(i) According to one of the postulates of kinetic theory.
4. If the pressure of a given mass of a gas is reduced to
“The volume occupied by gaseous molecules them-
half and temperature is doubled simultaneously , the
selves is negligibly small as compared to the total
volume occupied by the gas". This postulate holds volume will be -
good only under specific conditions of temperature (A) same as before
and pressure (i.e., when pressure is low and the (B) twice as before
temperature is high). But if the pressure becomes (C) four times as before
very high, (D) one fourth as before
then the volume occupied by the gaseous molecules
will no longer be negligible as compared to the total 5. 10 g of a gas at STP occupies a volume of 2 litres. At
volume occupied by the gas. The same thing is also what temperature will the volume be double, pressure
observed when the temperature is lowered appre- and amount of the gas remaining same ?
ciably.
(A) 273 K (B) 546 K
Thus we conclude that the above postulate of kinetic
(C) –273 ºC (D) 546 ºC
theory of gases does not hold good at high pres-
sures and low temperatures. 6. Remaining the pressure constant temperature must a
(ii) According to another postulate of kinetic theory, gas at 127 ºC be cooled, so that its volume is reduced to
'The forces of attraction between gaseous molecules 1/5 of the initial volume ?
are negligible'. This postulate also holds good tem- (A) –193 ºC (B) – 93 ºC
perature.
(C) 293 ºC (D) None of these
But as the pressure becomes high or the tempera-
ture becomes low, the volume also becomes very 7. At constant pressure, a gas at –33 ºC is heated to 127 ºC.
small and the molecules come closer to each other The percentage increase in the volume of gas will be -
so that the intermolecular forces of attraction become
(A) 6.67 (B) 16.67
appreciable and cannot be ignored.
(C) 66.67 (D) 43.67
Here we again conclude that this postulate of kinetic
theory of gases also does not hold good at high pres- 8. 5 dm3 of dry hydrogen is collected at a temperature of
sure and at low temperature.
37 ºC and 720 mm pressure. Find the volume of gas
This means that suitable corrections must be ap-
at S.T.P. ?
plied to ideal gas equation to make it applicable to
real gases. (A) 8.34 dm3 (B) 1.32 dm3
(C) 2.65 dm3 (D) 4.17 dm3

9. The pressure of one mole of a gas at S.T.P. is doubled


EXERCISE-1 and the temperature is raised to 546 K. What is the
final volume of gas, when one mole of gas occupies
I. Laws, Ideal gas equation, Diffusion & effusion of 22.4 dm3 at S.T.P. ?
gases (A) 11.2 dm3 (B) 22.4 dm3
(C) 5.6 dm3 (D) 44.8 dm3
1. Volume occupied by 4.045 × 1023 molecules of oxygen
at 27º C and 700 torr pressure is 10. A fixed mass of gas has volume 750 cm3 at –23º C
and 800 mm pressure. What will be the pressure for
(A) 17.95 L (B) 8.97 L
which its volume will be 720 cm 3, the temperature
(C) 27.50 L (D) 6.72 L being –3 ºC.
(A) 700 mm of Hg (B) 800 mm of Hg
2. The temperature of 20 litres of nitrogen was increased (C) 900 mm of Hg (D) 600 mm of Hg
from 100 K to 300 K at a constant pressure. Change 11. Containers A and B have same gases. Pressure,
in volume will be - volume and temperature of A are all, twice that of B,
(A) 80 litres (B) 60 litres then the ratio of number of molecules of A and B are:
(A) 2 : 1 (B) 1 : 3
(C) 1 : 4 (D) 4 : 1
PAGE # 155
12. W hat will be the minimum pressure required to 19. If P, V, M, T and R are pressure, volume, molar mass,
3
compress 500 dm of air at 1 bar to 200 dm at 30ºC? 3 temperature and gas constant respectively, then for
an ideal gas, the density is given by
(A) 4 bar (B) 12 bar
(C) 2.5 bar (D) 1.5 bar RT P
(A) (B)
PM RT
13. For an ideal gas. Boyle’s law is best described by
P P M PM
(C) (D)
V RT
(A) (B)
II. Graham’s law of diffution, Dalton law of partial
T V pressure, KTG, Molecular speed

P P
20. 180 mL of a hydrocarbon diffuses in 15 minutes.
Under same conditions 120 mL of SO2 diffuses in 20
(C) (D) minutes. If molecular weight of SO 2 is 64 then,
molecular weight of hydrocarbon would be -
V T
(A) 8 (B) 16
14. At constant temperature, if pressure increases by 1 (C) 32 (D) 64
%, the percentage decrease of volume is % :
(A) 1 % (B) 100/101% 21. If ratio of rates of diffusion of two gases is 4 : 3, then
(C) 1/101% (D) 1/100% ratio of their molecular weights would be -
(A) 3 : 4 (B) 9 : 16
15. If the pressure and absolute temperature of 2 litres of
CO2 are doubled, the volume of CO2 would become (C) 4 : 3 (D) 9 : 8
(A) 2 litres (B) 4 litres
(C) 5 litres (D) 7 litres 22. In a mixture of N2 and O2 , total pressure of mixture is
90 atm. If partial pressure of O2 is 63 atm, then mole
16. A sample of gas occupies 100 ml at 27°C and 740 fraction of N2 will be -
mm pressure. When its volume is changed to 80 ml
at 740 mm pressure, the temperature of the gas will (A) 0.7 (B) 0.6
be (C) 0.3 (D) 0.2
(A) 21.6 °C (B) 240 °C (C )
– 33°C (D) 89.5 °C 23. Total pressure of a mixture of two gases ‘A’ and ‘B’ is
762 mm of Hg. If mixture contains 0.015 g molecule
17. A gas at 298K is shifted from a vessel of 250 cm 3 of gas ‘A’ and 0.008 g molecule of gas ‘B’ , the partial
capacity to that of 1 L capacity. The pressure of the
pressures of gases ‘A’ and ‘B” would be -
gas will:
(A) become double (A) 260 mm, 470 mm (B) 375 mm, 870 mm
(B) becomes four times (C) 974 mm, 562 mm (D) 497 mm, 265mm
(C) decrease to half of the original value
(D) decrease to one-fourth of the original value 24. W hich of the following gases will have the highest
rate of diffusion?
18. The correct representation of Charles' law is given by :
(A) O2 (B) CO2
(C) NH3 (D) N2

25. In the equation of state of an ideal gas PV = nRT, the


value of the universal gas constant would depend
(A) (B)
only on:
(A) the nature of the gas
(B) the pressure of the gas
(C) the units of measurement
(D) none of these
26. The rate of diffusion of methane at a given
(C) (D) temperature is twice that of X. The molar mass of X in
is:
(A) 64 (B) 32
(C) 40 (D) 80
PAGE # 156
27. At 298 K, assuming ideal behaviour, the average 34. A gaseous mixture contains 56 g of N2, 44 g CO2 and
16 g of CH4. The total pressure of the mixture is 720
kinetic energy of a deuterium molecule is :
mm Hg. The partial pressure of CH4 is
(A) two times that of a hydrogen molecule (A) 180 mm (B) 360 mm
(B) four times that of a hydrogen molecule (C) 540 mm (D) 720 mm
(C) half of that of a hydrogen molecule
35. Equal weights of two gases of molecular weight 4
(D) same as that of a hydrogen molecule and 40 are mixed. The pressure of the mixture is 1.1
atm. The partial pressure of the light gas in this mixture
28. Two balloons A and B containing 0.2 mole and 0.1
is
mole of helium at room temperature and 2.0 atm. (A) 0.55 atm (B) 0.11 atm
respectively, are connected. W hen equilibrium is (C) 1 atm (D) 0.11 atm
established, the final pressure of He in the system 36. W hat will be the partial pressure of H 2 in a flask
is - containing 2 g of H2, 14 g of N2 and 16 g of O2 :
(A) 0.1 atm (B)1.5 atm (A) 1/2 the total pressure
(C) 0.5 atm (D) 2.0 atm (B) 1/3 the total pressure
(C) 1/4 the total pressure
29. C o n si d e r  a  g a s  en c l o s e d   in  a  c yl in d e r  w i th   (D) 1/16 the total pressure
frictionless piston.W hen the gas is compressed at 37. Equal weights of ethane and hydrogen are mixed in
c o n s ta n t  te m p er at u re  b y  th e  p is to n ,  t h e an empty container at 25°C. The fraction of the total
pressure of the gas increases. Consider the   follow- pressure exerted by hydrogen is :
(A) 1/2 (B) 1/1
ing statements :
(C) 1/16 (D) 15/16
i. The average speed of the molecules increases.
ii. The rate at which the molecules collide with 38. A sample of O2 gas is collected over water at 23°C at
the piston increases. a barometric pressure of 751 mm Hg (vapour
iii. The molecules collide with each other pressure of water at 23°C is 21 mm Hg). The partial
more often. pressure of O2 gas in the sample collected is
(A) 21 mm Hg (B) 751 mm Hg
W hich of the above statement(s) is/are correct?
(C) 0.96 atm (D) 1.02 atm
(A) i only
(B) iii only 39. Same mass of CH 4 and H 2 is taken in a container.
(C) ii & iii only The partial pressure caused by H 2 (where total
(D) all three are correct pressure is P) is

8 1
30. Kinetic energy of molecules is highest in : (A) P (B) P
9 9
(A) Gases (B) Solids
(C) Liquids (D) Solution 1
(C) P (D) P
2
31. A cylinder is filled with a gaseous mixture containing
equal masses of CO and N 2. The partial pressure 40. If 4g of oxygen diffuse through a very narrow hole,
ratio is : how much hydrogen would have diffused under
(A) PN2 = PCO (B) PCO = 0.875 PN2 identical conditions ?
(A) 16 g (B) 1 g
(C) PCO = 2 PN2 (D) PCO = 1/2 PN2 (C) 1/4 g (D) 64 g
41. Two gram of hydrogen diffuse from a container in 10
32. The ratio of partial pressure of a gaseous component
minutes. How many grams of oxygen would diffuse
to the total vapour pressure of the mixture is equal to
through the same container in the same time under
(A) mass of the component (B)
similar conditions ?
mole fraction of the component
(A) 0.5 g (B) 4 g
(C) mass % of the component
(C) 6 g (D) 8 g
(D) molecular mass of the component
42. If some moles of O 2 diffuse in 18 sec. and same
33. Equal volumes of two gases which do not react
moles of other gas diffuse in 45 sec. then what is the
together are enclosed in separate vessel. Their
molecular weight of the unknown gas ?
pressure at 100 mm and 400 mm respectively. If the
two vessel are joined together, then what will be the 45 2 18 2
pressure of the resulting mixture (temperature (A)  32 (B)  32
remaining constant) ? 18 2 45 2
(A) 125 mm (B) 500 mm
(C) 1000 mm (D) 250 mm 18 2 45 2
(C) (D)
45 2  32 18 2  32

PAGE # 157
43. The ratio of the rate of diffusion of a given element to 55. Kinetic energy of a gas depends upon its
that of helium is 1 : 4. The molecular weight of the (A) Molecules mass (B) Atomic mass
element is (C) Equivalent mass (D) None of these
(A) 32 (B) 64
(C) 16 (D) None of these 56. If a gas is expanded at constant tempertaure
(A) the pressure increase
44. The molecular weight of a gas which diffuse through (B) the kinetic energy of the molecules remains the
a porous plug at 1/6th of the speed of hydrogen under
sam e
identical conditions is
(C) the kinetic energy of the molecules decrease
(A) 27 (B) 72
(D) the number of molecules of the gas increases
(C) 36 (D) 48

45. The time taken for a certain volume of a gas 'X' to 57. The ratio among most probable velocity, mean velocity
diffuse through a small hole is 2 minutes. It takes and root mean square velocity is given by
5.65 minutes for oxygen to diffuse under the similar (A) 1 : 2 : 3
conditions. The molecular weight of 'X' is (B) 1 :
(A) 8 (B) 4
2 : 3
(C) 16 (D) 32 (C) 2 : 3 : 8/
46. The rate of diffusion of SO2 and O2 are in the ratio : (D) 2 : 8/ : 3
(A) 1 : 2 (B) 1 : 32
(C) 1 : 2 (D) 1 : 4
58. The root mean square speeds at STP for the gases
H2, N2, O2 and HBr are in the order :
47. The ratio of rates of diffusion of SO2, O2 and CH4 is :
(A) H2 < N2 < O2 < HBr
(A) 1 : 2 : 2 (B) 1 : 2 : 4 (B) HBr < O2 < N2 < H2
(C) 1 : 2 :1 (D) 1 : 2 : 2 (C) H2 < N2 = O2 < HBr
(D) HBr < O2 < H2 < N2
48. X ml of H2 gas effuses through a hole in a container in
5 sec. The time taken for the effusion of the same 59. The kinetic energy of N molecules of O2 is x joule at
volume of the gas specified below under identical  123ºC. Another sample of O2 at 27ºC has a kinetic
conditions is : energy of 2 x. The latter sample contains _______
(A) 10 sec. He (B) 20 sec. O2 molecules of O2
(C) 25 sec. CO2 (D) 55 sec. CO2 (A) N (B) N/2
(C) 2 N (D) 3 N
49. The densities of hydrogen and oxygen are 0.09 and
1.44 g L–1. If the rate of diffusion of hydrogen is 1 then
60. The average kinetic energy in joules of molecules in
that of oxygen in the same units will be :
8.0 g of methane at 27º C is :
(A) 4 (B) 1/4
(A) 6.21 × 10-20 J/molecule
(C) 16 (D) 1/16
(B) 6.21 × 10-21 J/molecule
50. Molecular weight of a gas that diffuses twice as rapidly (C) 6.21 × 10-22 J/molecule
as the gas with molecular weight 64 is (D) 3.1 × 10-22 J/molecule
(A) 16 (B) 8
(C) 64 (D) 6.4 61. The kinetic energy for 14 grams of nitrogen gas at
127°C is nearly (mol. mass of nitrogen = 28 and gas
51. If rate of diffusion of A is 5 times that of B, what will be constant = 8.31 JK–1 mol–1)
the density ratio of A and B ? (A) 1.0 J (B) 4.15 J
(A) 1:25 (B) 1:5 (C) 2493 J (D) 3.3 J
(C) 25:1 (D) 4:1
62. The temperature at which RMS velocity of SO 2
52. 50 ml of hydrogen diffuses out through a small hole molecules is half that of He molecules at 300 K is :
from a vessel in 20 minutes. The time needed for 40 (A) 150 K (B) 600 K
ml of oxygen to diffuse out is (C) 900 K (D) 1200 K
(A) 12 min (B) 64 min
(C) 8 min (D) 32 min 63. The rms velocity of an ideal gas at 27°C is 0.3 ms–1.
53. The densities of two gases are in the ratio of 1 : 16. Its rms velocity at 927°C (in m–1) is :
The ratio of their rates of diffusion is (A) 3.0 (B) 2.4
(A) 16 : 1 (B) 4 : 1 (C) 0.9 (D) 0.6
(C) 1 : 4 (D) 1 : 16
64. At 27°C, the ratio of rms velocities of ozone to oxygen
54. The ratio of root mean square velocity to average is :
velocity of gas molecules at a particular temperature
is (A) 3/5 (B) 4/3
(A) 1.086 : 1 (B) 1 : 1.086
(C) 2 : 1.086 (D) 1.086 : 2 (C) 2/3 (D) 0.25

PAGE # 158
65. The average kinetic energy of an ideal gas per
molecule in SI units at 25°C will be :
(A) 6.17  10–21 kJ (B) 6.17  10–21 J PV PV
–20
(C) 6.17  10 J (D) 7.16  10–20 J (A) RT (B) RT

1/T V
EXERCISE-2
COMPETITIVE EXAM PREVIOUS YEARS’ QUESTIONS : PV PV
(C) RT (D) RT
1. Two liters of oxygen gas diffused through a mem-
brane in 600 seconds. 0.6 liter of an unknown, gas P 1/P
diffused through the same membrane in half the time
required for oxygen to diffuse. The molecular weight 7. Pressure of a certain volume V of an ideal gas is
of the unknown gas is - [IJSO/Stage - I/2012] increased by four times its initial pressure whereas
(A) 16 (B) 44 the temperature is reduced to 50% of its initial tem-
(C) 89 (D) 64 perature. The resulting volume of the gas is
[IJSO/Stage -1/2011-12]
2. The rms speed of hydrogen is 7 times the rms (A) 10V. (B) 0.1V
speed of nitrogen. If T is the temperature of the gas, (C) 0.5V (D) 0.25V
then - [IJSO/Stage - I/2012]
8. If equal weights of oxygen and nitrogen are kept in
(A) TH2  TN2 (B) TH2  TN2 seperate containers at the same temperature then.
[IJSO/Stage -1/2012-13]
(A) Both the containers have the same number of
TH2  7TN2
(C) TH2  TN2 (D) moelcules.
(B) More mlolecules are present In the oxygen con-
tainer
3. A balloon initially contains 7 g of nitrogen and then 14 (C) The pressure of the nitrogen container is greater
g of nitrogen is added to the ballon to expand its vol- than that of the oxygen container.
ume to 12 litre at the same temperature and pres- (D) The pressure of the oxygen container is greater
than that of nitrogen container
sure. Find the initial volume of the ballon -
[IJSO/Stage - II/2012] 9. Two liters of oxygen gas diffused through a mem-
(A) 8 litre (B)7 litre brane in 600 seconds. 0.6 liter of an unknown, gas
(C) 5.6 litre (D) 4 litre diffused through the same membrane in half the time
required for oxygen to diffuse. The molecular weight
4. What occurs when H20(l) evaporates of the unknown gas is [IJSO/Stage -1/2012-13]
(i) Covalent bonds are broken. (A) 16 (B) 44
(ii) Only dipole-dipole forces are overcome.
(C) 89 (D) 64
(iii) Heat is absorbed by water from the
surroundings.
(iv)lt becomes oxygen and hydrogen gas. 10. Assuming ideal gas behavior, which among the fol-
[IJSO/Stage -I/2015] lowing gases will have the least density under room
(A) Only (i) and (iii) occurs temperature and pressure.
(B) Only (ii) and (iii) occurs [IJSO/Stage -1/2014-15]
(C) (i), (iii) and (iv) occurs (A) Oxygen (B) Nitrogen
(C) Ozone (D) Fluorine
(D) (i), (ii), (iii) and (iv) take place

5. 5g of a gas at 27°C occupied the same volume as 11. 5g of a gas at 27°C occupied the same volume as
0.250 g of dihydrogen at 17°C at the same pressure. 0.250 g of dihydrogen at 17°C at the same pressure.
Find out molecular weight ofthe gas. Find out molecular weight ofthe gas.
[IJSO/Stage -I/2015] [IJSO/Stage -1/2015-16]
(A) 41.37 (B) 20.68 (A) 41.37 (B) 20.68
(C) 25.80 (D) 39.87 (C) 25.80 (D) 39.87

6. For one mole of an ideal gas, which of the following 12. An iron cylinder contains helium at a pressure of
graphical respresentations holds true. 250kPa at 300K. The cylinder can with stand a
[IJSO/Stage -2/2016] pressure of 1 × 106 Pa. If the room in which the cylinder
is placed catches fire. Predict whether the cylinder
would blow up before it melts (melting point of cylinder
is 1800 K). [IJSO/Stage -2/2016]

PAGE # 159
CHEMICAL AND IONIC EQUILIBRIUM
vessel.Most of them condense and return to the liquid.
REVERSIBLE & IRREVERSIBLE REACTIONS A state is soon reached when the number of
molecules leaving the surface of the liquid equals
(a) Reversible Reactions : the number of molecules returning to the liquid phase
The chemical reactions which can proceed both in in per unit time. At this equilibrium state, the number
of molecules present in the vapour phase and liquid
the forward and reverse directions under similar
phase become fixed.
conditions are called reversible reactions.
Closed vessel
(i) They are indicated by a double half headed arrow.
( ).
(ii) Change in temperature, pressure or concentration Water vapour

changes the direction of reaction.


(iii) Such reactions are carried out in a closed
container. Vaporisation Condensation

(iv) Such reactions never go to completion.


e.g. Water
2HI(g) H2(g) + 2(g) (thermal dissociation )

C2H5OH () + HOOC-CH3 () C2H5-OOCCH3 (aq) +


H2O()
(esterification) CHEMICAL EQUILIBRIUM
FeCl3(s) + 3H2O() Fe(OH)3(aq) + 3HCl (aq)
A chemical equilibrium may be defined as a state of a
(hydrolysis)
reversible reaction when the observable properties
(b) Irreversible Reactions : such as colour, concentration etc., do not show any
The reactions in which complete conversion of the noticeable change.
reactants into products occur are called irreversible or
reactions. The state of a reversible reaction when the rates of
both forward & backward reaction become equal.
(i) They are indicated by a single headed arrow. (  )
(a) Dynamic Equilibrium :
(ii) Change in concentration, temperature and
All the observable properties of the system become
pressure doesn’t change the direction of reaction.
constant at dynamic equilibrium. It may lead us to
(iii) They can proceed only in the forward direction. think that the reaction stops altogether at equilibrium
and that equilibrium is static in nature. But this is not
Heat
e.g. 2KClO3(s)  2KCl(s) + 3O2(g) true. Actually, at equilibrium, the rate of forward
(Thermal decomposition) reaction becomes equal to the rate of backward
reaction so that there is no net change in the
NaOH(aq) + HCl(aq)  NaCl(aq) + H2O () concentrations of various species. In other words,
(neutralization) we can say that the equilibrium state is a dynamic
balance between the forward and the backward
BaCl2(aq) + H2SO 4 (dil)  BaSO4(s) + 2HCl(aq) reaction.
(precipitation) This can be illustrated by considering the reaction
between hydrogen and iodine to form hydrogen iodide.
2FeCl3 + SnCl2  2FeCl2 + SnCl4 (redox ) W hen hydrogen and iodine are taken in a closed
vessel maintained at 444 ºC, hydrogen molecules
PHYSICAL EQUILIBRIUM combine with iodine molecules to form hydrogen
iodide.
Equilibrium is also attained in physical processes H2(g) + I2 (g)   2HI(g)
such as vaporization and condensation. The Hydrogen Iodine Hydrogen Iodide
conversion of a liquid into vapour is known as
vaporization, while the opposite process, i.e. the Since the reaction is reversible in nature, the molecules
conversion of a vapour into liquid is called of HI formed also begin to dissociate to form H2 and I2.
condensation. When the vessel containing a liquid is 2HI(g)  H2(g) + 2(g)
tightly closed, the liquid does not evaporate The variation of rates of forward and backward
completely. This is because the molecules reaction have been shown in the figure . After the
constituting the vapour can not go out of the attainment of equilibrium , change in concentrations
occurs provided the temperature of the reaction

PAGE # 160
mixture is kept constant. Thus, at equilibrium, the between two opposing tendencies.
reaction does not stop but the system acquires  Molecules try to minimise energy.
constant observable properties because of the equal  Molecules try to maximise entropy.
rates of forward and backward reactions. Thus, the
In a reversible reaction like–
equilibrium is dynamic in nature.
Initially only reactants are present. R1 and R2 combine
to form P1 and P2. As soon as P1 and P2 are
formed, they start the backward reaction. As
H
2 +
concentrations of R1 and R2 decrease rate of
Fo
rwa
2
2HI
forward reaction decreases and rate of backward
Rate

rd
r ea c
tion Equilibrium state reaction increases. Ultimately a stage is reached
n when both the rates become equal. Such a state
ti o
ac is known as “ Chemical Equilibrium” or “state of
re
rd + 2
wa H2
Equilibrium”.
ck
Ba I
2H
At equilibrium :
Progress of the reaction
(i) Rate of forward reaction (rf) = rate of backward
Attainment of equilibrium state
reaction (rb)
(ii) Concentration (mole/litre) of reactant and
Characteristics of Dynamic Equilibrium :
product remains constant with respect to time.

(i) At equilibrium, rate of forward reaction is equal to R1  R 2 P1  P2


  
that of backward reaction. Re ac tan ts Pr oducts

(ii) At equilibrium observable properties (Macroscopic)


such as concentration, density, colour etc. are
constant under the given conditions of temperature
and pressure.
(iii) At equilibrium, microscopic properties involving
processes continue but they are in balance.
(iv) Equilibrium can be attained from either direction
of reaction beginning with reactants or products.
(v) Equilibrium can be attained in a closed system only.
(vi) A catalyst leads to early attainment of a chemical
equilibrium.As it accelerates both the forward and
the backward reactions to the same extent, thus it
does not disturb the equilibrium point.
(b) Static Equilibrium:
A system is in static equilibrium when various forces
acting on it produce no resultant force and therefore,
system remains motionless.
e .g. Wooden block floating on water

Wooden
block

Water

STATES OF CHEMICAL EQUILIBRIUM


State of equilibrium means the balance of driving
forces i.e. the factors taking the reaction in forward
direction and the backword direction are balancing
each other.
The equilibrium state represents a compromise

PAGE # 161
(ii) The value of KC is different for different reaction at
LAW OF MASS ACTION equilibrium. It is also different for the same reaction
The law was introduced by Guldberg & W aage in carried out at different temperatures.
1864. (iii) The change in either pressure or concentration at
According to the law of mass action at a given equilibrium may change the state of equilibrium but it
temperature, the rate of a chemical reaction is cannot change the equilibrium constant which
proportional to the product of the active masses of depends only on temperature.
the reactants.
(iv) The value of KC indicates the extent to which a
Consider a reaction - chemical reaction can occur.
A+B C+D
Larger the value of KC for a particular reaction, higher
Rate of forward reaction  [A][B]
will be the concentration (or amount) of the product
rf = Kf [A][B] {Kf = Rate constant for forward reaction} formed. On the other hand, a small value of K C
Similarly, indicates that the concentration of the product at
rb = Kb [C][D] { K b = Rate constant for backward equilibrium will be low.
reaction}
e.g. PCl3(g) + Cl2(g) PCl5(g)
At equilibrium rf = rb
So, Kf [A][B] = Kb [C][D] The value of KC for the above reaction is 1.9 at 298 K.
This low value indicates that at equilibrium the
Kf
= [C][D] concentration of PCl5 will be low.
Kb
[A][B] Similarly, the value of KC for the reaction -
[C][D]  K 
 KC = KC  f  N2(g) + 3H2(g) 2NH3(g), is 3.6 × 108 at 298 K,
  K b 
[ A ][B] 
The large value of KC indicates that a major portion of
Here K C is called the equilibrium constant of the
N2 and H2 will be combined to form NH3.
reaction.
Therefore, at constant temperature, the ratio between
the rate constant of forward reaction and the rate
constant of backward reaction is called equilibrium
constant.
Let us consider the following reaction at equilibrium
For a general reaction : 2SO2 + O2 2SO3
The equilibrium constant for the reaction (say K1) is
aA + bB cC + dD
given by -
KC may be written as :
[ SO 3 ] 2
[C]c [D]d
KC = K1 = 2 ----------------- (i)
[ A]a [B]b [ SO 2 ] [ O 2 ]
If reactants & products are gaseous then, If we reverse the representation of above reaction
2SO3 2SO2 + O2
(PC )c  (PD )d then the equilibrium constant for the reaction (say K2)
KP 
(PA )a  (PB )b is given by -
Where PA , PB , PC , PD are partial pressures of gas [ SO 2 ] 2 [ O 2 ]
A,B,C,D respectively. K2 =
[ SO 3 ] 2
Partial Pressure =
Number of moles of reactant
 Total Pressure 1 [SO3 ]2
Total moles in the reaction mixture
or  ------------------(ii)
K2 [SO2 ]2 [O2 ]
Relation between Kp & Kc:
Kp = Kc (RT ) n Comparing (i) and (ii) we get - K1 =
1
(a) If n= 0, then Kp = Kc K2
(b) n = + ve, then Kp > Kc  Note:
(c) n = – ve, then Kp < Kc Thus, the equilibrium constants of the reactions are
 Note : Rate constant of a chemical reaction is equal reciprocal of each other.
to the rate of the chemical reaction while the
concentration of reactants are unity. APPLICATION OF EQUILIBRIUM
CONSTANT
SIGNIFICANCE OF EQUILIBRIUM CONSTANT (KC)  Predicting the direction of the reaction
(i) The equilibrium constant (Kc) is a constant for any Reaction Quotient (Q)
given chemical reaction at equilibrium at a definite
temperture.
At each point in a reaction, we can write a ratio of

PAGE # 162
concentration terms having the same form as the [Product] << [Reactant]
equilibrium constant expression. This ratio is called Hence concentration of Product can be
the reaction quotient denoted by symbol Q. neglected as compared to the reactant.
It helps in predicting the direction of a reaction. In this case, the reaction is reactant
favourable.

[C]c [D] d CHARACTERISTICS OF EQULIBRIUM


The expressionQ = a b at any time during CONSTANT & FACTORS AFFECTING IT
[ A ] [B]
 Equilibrium constant does not depend upon
reaction is called reaction quotient.The concentration of various reactants, presence of
concentrations [C] , [D] ,[A], [B] are not necessarily catalyst, direction from which equilibrium is
at equilibrium. reached
 The equilibrium constant does not give any idea
about time taken to attain equilibrium.
 K depends on the stoichiometry of the
reaction.
 If two chemical reactions at equilibrium having
equilibrium constants K1 and K2 are added
then the resulting equation has equilibrium
constant K = K1 . K2

 The reaction quotient is a variable quantity with Equation constant


time. A(g) B(g) K1
 It helps in predicting the direction of a reaction. B(g) C(g) K2
 if Q > Kc reaction will proceed in backward On adding A(g) C(g) K = K1 . K 2
direction until equilibrium in reached.  If the reaction having eq. constant K1 is reversed
 if Q < Kc reaction will proceed in forward 1
direction until equilibrium is established. then resulting equation has eq. constant K
1
 if Q = Kc Reaction is at equilibrium.
A(g)+B(g) C(g) + D(g)
eg. 2A(g) + B(g) C(g) + D(g) 1
Q C = Reaction quotient in terms of concentration On reversing, C(g)+D(g) A(g) + B(g)K = K
1
 If a chemical reaction having equilibrium constant
[C][D]
QC = K1 is multiplied by a factor n then the resulting
[ A ]2 [B] equation has equilibrium constant K = (K1)n , n
can be fraction
[C] eq [D] eq
KC = [ A ] 2 [B] [Here all the conc. are at  1
eq eq eg. D2(g) 2A(g) K1 Multiplying by   ,
2
equilibrium]
1
 Predicting the extent of the reaction D (g) A(g) K = (K1)1/2 = K1
2 2
[Pr oduct]  Equilibrium constant is dependent only
K=
[Re ac tan t] on the temperature.
It means kp and kc will remain constant at constant
temperature no matter how much changes are
made in pressure, concentration, volume or
catalyst.
 However if temperature is changed,
Case-I If K is large (k > 103) then product concentration k2 H 1 1
is very very larger than the reactant ([Product] log k = 2.303 R  T  T  ; H = Enthalpy of
1  1 2
>>[Reactant]) Hence concentration of reactant
can be neglected with respect to the product. In reaction
this case, the reaction is product favourable and If T2 > T1 then
K2 > K1 provided H = +ve (endothermic reaction)
equilibrium will be more in forward direction than
K2 < K1 if H = –ve (exothermic reaction)
in backward direction. In the above equation, the unit of R and H/T should
Case-II If K is very small (K < 10–3) be same.

PAGE # 163
 Relation between equilibrium constant & density CaCO 3
standard free energy change. = MCaCO3 = constant
Gº = – 2.303 RT log K
Where G° = standard free energy change [CaO(s)][CO 2 (g)]
T = Absolute temperature, K=
R = universal gas constant. [CaCO 3 (s)]
K.[CaCO 3 (s)]
Homogeneous liquid system : Formation = [CO 2(g)]
[CaO(s)]
of ethyl acetate : KC = [CO 2(g)]
The reaction between alcohol and acid to form ester
 H2O(l) H2O(g)
is an example of homogeneous equilibrium in liquid
system. KP = PH2O (g) , KC = [H2O (g)]
CH3COOH() + C2H5OH()
[For pure solid and pure liquid active mass
CH3COOC2H5() + H2O()
is taken as unity i.e. = 1]
[CH3 COOC2H5 ][H2O] Ex.12 In a reaction C(s) + CO2 (g) 2CO(g),
KC = [CH COOH][C H OH]
3 2 5 the equilibrium pressure is 12 atm. If 50% of CO2
Ex.11 In an experiment starting with 1 mole of ethyl reacts. Calculate KP.
alcohol, 1 mole of acetic acid and 1 mole of water Sol. C(s) + CO2 (g) 2CO(g)
at T0C, the equilibrium mixture on analysis shows t=0 a 0
that 54.3% of the acid is esterfied. Calculate the
a a
equilibrium constant of this reaction. t = teq a– 2
2 2
Sol.
Peq = 12 atm
CH3COOH() + C2H5OH() CH3COOC2H5() + H2O()
Initial 1 1 0 1 1 2
X CO2 = , XCO =
At equilibrium 1 – x 1–x x 1+x 3 3
1–0.543 1– 0.543 0.543 1 + 0.543
1
1 x 54.3 PCO2 = × 12 = 4
(54.3% of 1 mole = = 0.543 mole) 3
100
Hence given x = 0.543 mole 2
PCO = × 12 = 8
Applying law of mass action : 3

[ester ][ water ] 0.543  1.543 88


KC = = = 4.0  KP = = 16
[acid][alcohol] 0.457  0.457 4

 EQUATION INVOLVING IONS :


Equilibrium involving ions always take place in DEGREE OF DISSOCIATION
aquous medium . In case of expression of KC It is the fraction of one mole dissociated into the
concentration of ion is taken. products.
1 (Defined for one mole of substance)
Ex. Ag+(aq.) + Cl–(aq.) AgCl(s) Kc =
[ Ag ][Cl – ]

So,  = no. of moles dissociated / initial no. of
Heterogenous Equilibrium : moles taken
= fraction of moles dissociated out of 1 mole.
For pure solid and pure liquid, active mass is
taken to be unity i.e. 1 as they remain constant Note : % dissociation =  x 100
throughout the reaction Suppose 5 moles of PCl5 is taken and if 2 moles
 CaCO3 (s) CaO (s) + CO2 (g) 2
of PCl5 dissociated then  = = 0.4
KP = PCO2 , KC = [CO2 (g)] 5
Let a gas An dissociates to give n moles of A as
WCaCO3 follows-
moles
[CaCO3(s)]= = MCaCO3
volume
V

PAGE # 164
An (g) n A (g) where Mth = theoritical molecular weight (n =
t=0 a 0 atomicity)
t = teq a–x n.x MAn
x Mmixture = , MAn = Molar mass of gas
[1  (n  1) ]
=  x = a.
a An
a – a  = a(1-) na
Vapour density (V.D). : Density of the gas divided
Total no. of moles = a – a  + n a 
by density of hydrogen under same temp &
= [1 + (n – 1)] a
pressure is called vapour density.
Significance of n MA n
 D = vapour density without dissociation =
sum of stoichiometric coefficient of product 2
n=
sum of coefficient of reactants Mmix
d = vapour density of mixture = observed v.d. =
2
(i) for PCl5(g) PCl3(g) + Cl2(g) (n = 2)
D
3 1 = 1 + (n – 1)
(ii) for 2NH3(g) N2(g) + 3H2(g) (n = + = 2) d
2 2
Dd M  Mo
(iii) for 2HI(g) H2(g) + 2(g) (n = 1)    T
(n  1)  d (n  1)M0
Ex.13 Calculate the degree of dissociation and K p
for the following reaction. where MT = Theoritical molecular wt. M0 = observed
PCl5(g) PCl3(g) + Cl2(g) molecular wt. or molecular wt. of the mixture at
t=0 a 0 0 equilibrium.
t=t a –x x x Note : It is not applicable for n = 1 [eg. Dissociation
Since for a mole, x moles are dissociated of HI & NO].
x Ex.14 The vapour density of a mixture containing
Sol.  For 1 mole, moles = are dissociated
a NO 2 and N2O 4 is 38.3 at 33°C calculate the no.
 x=a of moles of NO 2 if 100g of N2O 4 were taken
PCl5(g) PCl3(g) + Cl 2(g) initially.
t = t eq a – a a a Sol. N2O 4(g) 2NO2(g)
Total no. of moles at equilibrium = a + a = a (1 +  ) Mmix = 2 x 38.3 = 76.6
a(1   ) P a  .P M th 92
PPCl5 = , PPCl3 = a 1    , Mmix = =   = 0.2
a(1  ) 1  1 

a N2O4 2NO2
PCl2 = .P t=0 a 0
a (1   )
t=t a – a 2a
2
 P  2  100  0.2
 
no. of moles of NO 2 = 2a= = 0.435
 1    2 . P 92
KP =  1    KP =
 P 1 2
 1   External factors affecting equilibrium :
Observed molecular weight and Observed Vapour Le Chatelier's Principle:
Density of the mixture If a change is appli ed to the syst em at
Observed molecular weight of An (g) equilibrium, then equilibrium will be shifted in
that direction in which it can minimise the effect
molecular weight of A n (g)
= total no. of moles at equilibrium of change applied and the equilibrium is
established again under new conditions.
 Effect of concentration : If the concentration of
a.Mth
= a component is increased, reaction shifts in a
a(1  (n  1) )
direction which tends to decrease its concentration.
Mth e.g. In the following example.
 Mobs =
[1  (n  1) ]

PAGE # 165
N2 (g) + 3H2(g) 2NH3(g)  P  no. of moles
[reactant]  Forward shift (i) For ng = 0  No. effects
(ii) For ng > 0, PCl 5(g) PCl 3(g) + Cl 2(g)
[Product]  Backward shift
( XPCl 3 P ). ( X Cl2 P )
 If concentration of reactant is increased at
Qp = ( XPCl 5 . P)  Q P  P [X = mole
equilibrium then reaction shifts in the forward
direction .
fraction]
 If concentration of product is increased then
P  ; Q P  ; (Forward shift)
reaction shifts in the backward direction
[P = Total pressure at equilibrium]
Note : The addition of any solid component does P  ; Q P  ; (Backward shift)
not affect the equilibrium. (iii) For n < 0, eg. N2(g) + 3H2(g) 2NH3(g)
 Effect of volume :
( X NH3 
)P) 2 1
 If volume is increased, pressure decreases QP = 3  QP 
[( XN2 ).P][( X H2 ) P] P2
hence reaction will shift in the direction in which
pressure increases that is in the direction in which P ; Q P ; (Forward shift) ; P ; Q P ; (Backward
number of moles of gases increases and vice versa. shift)
 If volume is increased then, for ng > 0  Effect of catalyst :
reaction will shift in the forward direction Due to catalyst, the state of equilibrium is not
ng < 0 reaction will shift in the backward direction affected i.e. no shift will occur as catalyst lowers
ng = 0 reaction will not shift. eg. H2(g) + I2(g) the activation energy of both the forward & reverse
2HI(g) (No effect) reaction by same amount, thus altering the forward
Explanation : & reverse rate equally and hence, the equilibrium
will be attained faster i.e time taken to reach the
(i) ng > 0, eg. PCl 5(g) PCl 3(g) + Cl 2 (g) equilibrium is less.

(nCl2 ) (nPCl3 )  Effect of inert gas addition :



V V 1 (i ) At constant volume : Inert gas addition
QC = Q C  for ng > 0
(nPCl5 ) V has no effect at constant volume
V (ii) At constant pressure : If inert gas is added
then to maintain the pressure constant, volume is
On increasing V, Q C, decreases.
increased. Hence equilibrium will shift in the
Now, for Q C < KC reaction will shift in forward
direction in which larger no. of moles of gas is
direction.
formed
Thus, if, Volume  Q C  (Forward shift)
t) (i) ng > 0, reaction will shift in the forward direction
Volume  Q C (Backward shift) t) (ii) ng < 0, reaction will shift in the backward direction

(iii) ng = 0, no effect
(ii) ng < 0, eg. N2(g) + 3H2(g) 2NH3(g)
 Effect of temperature :
2
 ( nNH3 )  (i) Exothermic reaction : The reaction in which
 
 V  heat is evolved
3
QC =  (nN2 )   (nH2 )  A(g) + B(g) C(g) + D(g) + Heat H = – ve
  
 V  V  eg. N2(g) + 3H2(g) 2NH3(g) + Heat

 Q C  V2 for ng < 0 T   K' will decrease (from vant’ hoff equation)
V  Q C  (Backward shift) ; K
1 Hº  1 1
log K = 2.303 R  T  T 
V  Q C  (Forward shift) 2  2 1

K1
 Effect of pressure : log K < 0 log K1 – log K2 > 0
2
On increasing pressure, equilibrium will shift in the
direction in which pressure decreases i.e. no. of  log K1 > log K2 K1 > K2
moles in the reaction decreases and vice versa. Reaction will shift in backward direction.
T   K will increases.

PAGE # 166
Reaction will shift in forward direction. (H2O) may be supposed to remain constant [ Water
is ionised to a small extent.]
(ii) Endothermic reaction : energy consumed.
 K[H2O]2 = [H3O+][OH–]
A(g) + B(g) C(g) + D(g) – Heat
Kw = [H3O+][OH–]
 H = + ve
 Kw = Constant (It is known as Ionic product of water).
T   K   Forward ; T Therefore the product of concentration of H+ ions and
 K   Backward OH– ions in water at a particular temperature is known
as ionic product of water.
The value of K w increases with the increase of
ELECTROLYTES AND NON-ELECTROLYTES
temperature, i.e. the concentration of H+ and OH– ions
An electrolyte is defined as a compound whose increase with increase in temperature.
aqueous solution or melt conducts electricity. On the Temperature Value of Kw
other hand, a compound whose aqueous solution or at 0ºC 0.11 × 10–14
melt does not conduct electricity is called a non- at 10ºC 0.31 × 10–14
electrolyte. at 25ºC 1.00 × 10–14
e.g. Aqueous solution of sugar, urea etc. do not conduct
at 100ºC 7.50 × 10–14
electricity. Hence they are non- electrolytes whereas
solutions of inorganic acids , bases and salts conduct (a) For neutral solution [H3O+] = 10–7 M
electricity. Hence they are electrolytes. (b) For acidic solution [H3O+] > 10–7 M
(a) Types of electrolyte : (c) For basic solution [H3O+] < 10–7 M
Electrolytes are of two types

(i) Weak electrolyte : A weak electrolyte is defined as OSTWALD DILUTION LAW


a substance which dissociates to a small extent in (for weak electrolyte’s)
aqueous solution and hence, conducts electricity to  For a weak electrolyte A+B– dissolved is water,
a small extent. if  is the degree of dissociation then
e.g. NH4OH,CH3COOH etc. AB A+ + B–
(ii) Strong electrolyte : A strong electrolyte is defined initial conc C 0 0
as a substance which is completely ionized in its conc-at eq. C(1 – ) C C
aqueous solution and hence is a very good conductor
of electricity. Then according to law of mass action,
e.g. NaOH, KOH,HCl, H2SO4, NaCl, KNO3 etc.
[ A  ][B – ] C . C C 2
Keq =   = dissociation
[ AB ] C(1   ) (1   )
IONIC EQUILIBRIUM
constant of the weak electrolyte
In case of weak electrolyte, as they are partly ionised
in the aqueous solution, an equilibrium is setup 1
[C = , then V = 1/C(volume of Solution in which
between the ions and the unionised electrolyte. V
Hence, their equilibrium is represented by putting 1 mole is present) is called dilution,
double arrow ( ) in between ionised and 2
so keq = ]
unionised form. (1   )V
If  is negligible in comparison to unity then, 1 – 
e.g. CH3COOH + H2O CH3COO– (aq) + H3O+
In general, the ionisation of a weak electrolyte, AB, is k eq
represented as follows : – 1. So keq = 2 C
~ =  k eq . V .
c
+ –
AB(s) + H2O A (aq) + B (aq)
1
Such an equilibrium between the ions and the   { Thumb rule }
concentrat ion
undissociated electrolyte is called ionic equilibrium .
 as concentration increases   decreases
 at infinite dilution reaches its maximum
IONISATION OF WATER value, unity.
Pure water is a very weak electrolyte and ionised
according to the following equation.
H2O () + H2O () H3O+ (aq) + OH–(aq) 1

Applying the law of mass action
[H3O  ][OH – ] 0.1
K=
[H2O]2
C

PAGE # 167
(f) pH of a weak acid (monoprotic) Solution : so we have to do the exact calculation,
 Weak acid does not dissociate 100% therefore 2
2×10–5 = 10–6 1      0.95 or 95%
we have to calculate the percentage
dissociation using Ka, dissociation constant [H+] = 0.95 ×10–6 = 9.5 × 10–7
of the acid.  pH = 7– log (9.5) = 6.022 Ans.
 We have to use Ostwald’s Dilution law (as At very low concentration (at infinite dilution)
have been derived earlier) weak electrolyte will be almost 100%
HA H+ + OH– dissociated, so behave as strong electrolyte
t=0 C 0 0 (pH) of 10–6 M HCl ~
 pH of 10–6 M CH3COOH ~  6)
t = teq C(1–) C C
SALT HYDROLYSIS
[H ] [OH ] C 2
Ka = = (Reverse of neutralisation)
[HA ] 1 
If <<1 1 –  1  Ka  C2 Salt + Water acid +base
W hen acids and bases are mixed in equal
Ka
 = ( is valid if  < 0.1 or 10%) amounts(equivalents), then we will be having salt
C Solutions only in the water and we have to calculate
pH of salt Solutions.
Ka
[H+] = C = C = Ka  C When a salt is added to water, the solid salt first
C dissolves and breaks into ions completely(unless
1
otherwise specified). The ions of the salt may or
So pH = pK a  log C may not react with water, the cations when react
2
with water will always produce H3O+ ions and the
on increasing the dilution
anions on reaction with water will produce OH–
C   and [H+]  pH  ions. Depending on the extent of hydrolysis and
Ex.13. Calculate pH of on the amounts of H3O+ and OH– ions the Solution
(a) 10–1 M CH3COOH can be acidic, basic or neutral. If salt is BA, then
(b) 10–3 M CH3COOH
BA(s)  BA(aq)  B+(aq) + A–(aq)
(c) 10–6 M CH3COOH A–(aq) + H2O(l) HA(aq) + OH–(aq)
Take Ka = 2 ×10–5. (anionic hydrolysis)
Sol. (a) CH3COOH CH3COO– + H+ B+ (aq) + 2H2O(l) BOH (aq) + H3O+ (aq)
C 0 0 (cationic hydrolysis)
C(1–) C C
ANIONIC HYDROLYSIS
C 2 Ka 2  105 Anions can function as a base on reaction with
Ka =   = = =
1  C 101 water and hydrolyse as follows :
A–(aq) + H2O (l) HA (aq) + OH (aq)
2104 (<< 0.1) The extent of hydrolysis of a given anion depends
on its basic strength
So, [H+] = 10 –1 × 2 ×10 –2  pH = 3 –
CATIONIC HYDROLYSIS
1
log 2 = 2.85 Ans. Cations can function as acid on reaction with water
2
and hydrolyse as follows.
B+(aq) + 2H2O(l) BOH(aq) H3O+(aq)
Ka 2  105
(b) = = = 2 102 ( > 0.1) The extent of hydrolysis of a given cation depends
C 103 on its acidic strength.
So we have to do the exact calculations
There are four types of salt.
C 2 10 3   2 (i) Salt of strong acid and strong base
Ka =  2×10–5 = (ii) Salt of strong acid and weak base
1  1 
  = 13.14% (iii) Salt of weak acid and strong base
[H+] = 10–3 × 0.1314 =1.314 ×10–4 (iv) Salt of weak acid and weak base
pH = 4 – log (1.314)  3.8 Ans. Salts of first type does not undergo hydrolysis and
(c) If approximation is used then, rest three types undergo hydrolysis.
(i) Salt of strong acid and strong base :
2  105 Neither of the ions will undergo hydrolysis so the
= = 20 > 1,
10 6 Solution contain only the equilibrium of ionization
of water.

PAGE # 168
2H2O(l) H3O+ + OH– (iv) Salt of weak acid and weak base :
Thus, the pH of Solution will be 7(neutral Examples can include CH3COONH4, NH4CN,
Solution). Ca(CH3COO)2, MgC2O4
(ii) Salt of strong acid and weak base : CH3COO– + NH4+ + H2O CH3COOH + NH4OH
The Examples can be NH4Cl,(NH4)2 SO4, AlCl3 t=0 c c 0 0
Only the cation will undergo hydrolysis and the at eq. c – ch c – ch ch ch
Solution will be acidic in nature. for Example in
[CH3COOH] [NH4OH]
the Solution of NH4Cl of concentration c, we will Kh = 
have [CH3COO ] [NH4 ]
.....(i)
NH4+ + H2O NH4OH + H+
CH3COOH CH3COO– + H+,
t= 0 c 0 0 0
at eq. c(1– h) ch ch [CH3COO ][H ]
(h - degree of hydrolysis) Ka = .....(ii)
[CH3COOH]
[NH4OH][H ] NH4OH NH4+ + OH– ,
Kh= = called hydrolysis constant of the
[NH4  ]
salt [NH4 ][OH ]
Kb = ......(iii)
[NH4OH]
 
[NH4 ][OH ]
NH4OH NH4+ + OH– , Kb= H2O H+ + OH–,
[NH4OH]
Kw = [H ] [OH–]
+ ......(iv)
H2O H+ + OH–, Kw = [H+] [OH–] So, Kh× Ka × Kb = Kw ,
From above equations we can get ch. ch  h 
2

Kh × Kb = Kw  kh =  
c(1  h). c(1  h)  1  h 
ch .ch ch2
Kh = = (genarally 1-h 1) so we  h 
c(1  h) (1  h)   kh
 
 1 h 
Kh from (ii) equation
get, h =
c
[CH3COOH] ch h
Kw [H+] =ka = Ka = Ka ×
c [CH 3 COO ] c(1  h) 1 h
 [H+] = ch = K h  c = Kb
Kw K w  Ka
1 = Ka × K h = Ka × Ka  Kb = Kb
 pH = – log [H+] = – [log Kw – log Kb + log c]
2
1
1 pH = – log [H+] = [pKw + pKa – pKb ]
= [pKw – pKb – log c] 2
2
(iii) Salt of weak acid and strong base :
COMMON ION EFFECT
The Examples can be CH 3COONa, KCN,
Na2C2O4, K3PO4 Let AB be a weak electrolyte. Considering its
Similar to above analysis we will get dissoication,
AB A+ + B– .
ch .ch ch2 and applying law of mass action we get
kh = = (genarally 1-h 1) so we
c(1  h) (1  h)
[A ][B– ]
K
kh [AB]
get, h =
c The equilibrium constant 'K' has a definite value at
any given temperature. If now another electrolyte
kh × k a = k w
furnishing the A + or B – ions is added to the above
kw solution, it will increase the concentration of either A+
[OH–] = ch = k h  c = c ions or B– ion (whichever has been added ) and in
ka
order to keep K constant, the concentration of AB
1 must increase, i.e, the equilibrium will shift to the left
pH = – log [H+] = – [ log k w  log k a  log c ] hand side.
2
In other words, the degree of dissociation of an
1 electrolyte (weak) is suppressed by the addition of
= [pKw + pKa +log c] another electrolyte (strong) containing a common ion.
2
This is termed as common ion effect.

PAGE # 169
e.g. When sodium acetate (CH3COONa) is added to molar concentration of F– will be twice of Ca2+
the solution of acetic acid (CH 3COOH), then the i.e. [Ca2+] = 2 × 10–4
degree of dissociation of acetic acid (weak acid) is [F–] = 2 × 2 × 10–4
suppressed due to common ion effect. KSP = [Ca2+] [F–]2
CH3COOH CH3COO– + H+ ... (i) = (2 × 10–4) × ( 2 × 2 × 10–4)2
Weak electrolyte = 32 × 10–12
CH3COONa  CH3COO– + Na+ ... (ii) = 3.2 × 10–11
Strong electrolyte Common Ex.3 Solubility of AgCl at 20ºC is 1.435 × 10–3 g/L. Calculate
acetate ion the solubility product of AgCl at the same temperature.
Applying law of mass action on equilibrium.
(143.5 is m.w. of AgCl)
[CH3 COO– ][H ] Sol. Solubility of AgCl(s) = 1.435 × 10–3 g/L
Ka  ... (iii)
[CH3 COOH] 1.435  10 –3
= mol/L = 1 × 10–5 mol/L
K a [CH 3 COOH ] 143 .5
[H  ] 
[ CH 3 COO – ]
... (iv) AgCl Ag+ + Cl–
Above relation shows, that on increasing the (S) (S)
concentration of acetate ion (CH3COO–), hydrogen ion KSP = [Ag+] [Cl–]
concentration decreases which shows that, in = (1 × 10–5) × (1 × 10–5)
presence of sodium acetate degree of ionization of = 1 × 10 –10
acetic acid is decreased.
Ex.4. Solubility product of AgBr is 4 × 10 –13 . If Ag +
concentration in solution is 1 × 10–6 mol/L, then find
SOLUBILITY PRODUCT
out the Br– ion concentration in the solution.
In a saturated solution of an electrolyte two equilibrium
Sol. AgBr Ag+ + Br–
exist and can be represented as :
Ksp = [Ag ] [Br –]
+

AB AB A+ + B– 4 × 10–13 = [1 ×10–6][Br–]
Solid Unionised Ions 4  10 –13
(dissolved) [Br–] =
Applying the law of mass action to the ionic
1 10 – 6
[Br ] = 4 × 10–7 mol/L

equilibrium,
Ex.5. Find out the solubility of BaSO4 in g/L, if the solubility
[ A  ][B – ] product of BaSO 4 at 25º C is 1 × 10 –10 (molecular
=K
[ AB ] mass of BaSO4 = 233.3)
Since the solution is saturated, the concentration of Sol. BaSO4 Ba2+ + SO42–
unionised molecules of the electrolyte is constant at Suppose solubility is S mol/litre
a particular temperature. then KSP = [Ba2+] [SO42 –]
i.e. [AB] = K’ = constant. KSP = (S) × (S)
[A+] [B–] = K × K’ = KSP
KSP = S2
W here K SP is termed as the solubility product. It is
1 10 –10 = 1 × 10 mol/L
–5
defined as the product of the concentration of ions in S= K SP =
a saturated solution of an electrolyte at a given
1 × 10–5 × 233.3 g/L = 2.333 × 10–3 g/L
temperature.
Ex.1 If solubility of CaCO3 in water is 0.0305 g/L. What will
be the solubility product of CaCO3.
So lub ility in g / L EXERCISE-1
Sol. Solubility in mole/litre (S) =
Molecular weight
I. Reversible & irreversible reaction, Chemical
0.0305
= = 3.05 × 10–4 equilibrium
100
Since dissolved CaCO3 is completely ionized - 1. W hich of the following reactions is an endothermic
CaCO3 Ca2+ + CO32 – reaction ?
(A) CaCO3 (s)  CaO (s) + CO2 (g) – Heat
KSP = [Ca 2 ][CO 3 2 – ] Electric current
(B) 2H2O      2H2 (g) + O2(g)
KSP = (3.05 × 10–4) × (3.05 × 10– 4) = 9.3 × 10–8 (C) N2 (g) + O2(g)  2NO(g) – Heat
(D) All are endothermic reactions
Ex.2 Solubility of CaF2 in water is 0.0002 mol/litre. Calculate
its solubility product.
2. Which of the following does not disturb the equilibrium
Sol. Solubility of CaF2 = 0.0002 mol/litre = 2 × 10–4 mol/ point ?
litre. (A) Concentration (B) Temperature
(C) Catalyst (D) Pressure
CaF2 Ca2+ + 2F–
(S) (2S)

PAGE # 170
3. Which of the following statements is not correct ? 11. In a container equilibrium N2O4 (g) 2NO2 (g)
(A) Combustion reactions are exothermic.
(B) Most of the reactions are endothermic. is attained at 25°C. The total equilibrium pressure in
(C) Decomposition reactions are generally endothermic. container is 380 torr. If equilibrium constant of above
(D) In exothermic reactions potential energy of product equilibrium is 0.667 atm, then degree of dissociation
is less than that of reactants. of N2O4 at this temperature will be

1 1
4. A reversible reaction is the one in which - (A) (B)
(A) the reactants are completely converted into 3 2
products.
(B) half of the reactants are used up. 2 1
(C) (D)
(C) the rates of both the forward & the reverse 3 4
reactions are unequal.
(D) the concentrations of reactants & products 12. The extent of dissociation of PCl 5 at a certain
become fixed at equilibrium.
temperature is 20 % at one atm pressure . Calculate
the pressure at which this substance is half
5. The reaction CaCO 3(s)  CaO(s) + CO 2(g) is dissociated at the same temperature.
irreversible only when -
(A) 0.123 (B) 0.246
(A) the reaction is carried out in a closed vessel.
(C) 0.826 (D) 0.111
(B) the reaction is carried out in an open vessel.
(C) the reaction takes place at a definite temperature
13. At 248º C , the kP for the reaction , SbCl5 (g)  SbCl3
& pressure.
(g) + Cl2 (g) is 1.07 atm at a total pressure of 1 atm .
(D) none of these.
Calculate the degree of dissociation of SbCl5 .
(A) 0.516 (B) 0.718
6. Which of the following reactions is an endothermic one ?
(C) 0.321 (D) None of these
(A) N2(g) + 3H2(g) 2NH3(g)
(B) C (s) + O2(g) CO2(g) 14. 4 moles of A are mixed with 4 moles of B, when 2
(C) N2(g) + O2(g) 2NO(g) moles of C are formed at equilibrium, according to
(D) 2SO2(g) + O2(g) 2SO3(g) the reaction, A + B C + D. The equilibrium
constant is :
7. In which of the following reaction rate increases with (A) 4 (B) 1
the increase of temperature ? (C) (D)
2 4
(A) Endothermic reactions
(B) Exothermic reactions
(C) Any reaction 15. 4 moles of PCl5 are heated at constant temperature in
(D) None closed container. If degree of dissociation for PCl5 is
0.5 calculate total number of moles at equilibrium :
8. In the reaction, (A) 4.5 (B) 6
(C) 3 (D) 4
A(s) + B(g) + heat 2 C(s) + 2D (g) at equilibrium,
pressure of B is doubled. To re-establish the equilibrium, 16. In the reaction 2P(g) + Q(g) 3R(g) + S(g). If 2
the factor by which conc. of D is changed is -
moles each of P and Q taken initially in a 1 litre flask.
(A) 2 (B) 2 At equilibrium which is true :
(C) 3 (D) 3 (A) [P] < [Q] (B) [P] = [Q]
(C) [Q] = [R] (D) None of these
9. When H2 and 2are mixed and equilibrium is attained,
then - 17. In a 0.25 litre tube dissociation of 4 moles of NO is
(A) amount of H formed is equal to the amount of H2 take place. If its degree of dissociation is 10%. The
dissociated. value of Kp for reaction 2NO N2 + O2 is :
(B) H dissociation stops.
(C) the reaction stops completely. 1 1
(A) 2 (B)
(D) None of these 18  8 2
10. At 1000 K, the value of Kp for the reaction : 1 1
A(g) + 2B(g) 3C(g) + D(g) is 0.05 atmosphere. (C) (D)
16 32
The value of KC in terms of R would be :
(A) 20000 R (B) 0.02 R
18. The vapour density of N2O4 at a certain temperature
(C) 5  10–5 R (D) 5  10–5  R–1
is 30. What is the percentage dissociation of N2O4 at
this temperature ?
(A) 53.3 % (B) 106.6%
(C) 26.7% (D) none

PAGE # 171
19. A reaction in equilibrium is represented by the 27. The correct expression for the equilibrium constant
following equation – 1 3
2A(s) + 3B(g) 3C(g) + D(g) + O2 if the pressure on for the reaction - NH3(g) N (g) + H2 (g) is
2 2 2
the system is reduced to half of its original value [N2 ]1/ 2 [H2 ] 3 / 2 [N2 ] [H2 ]
(A) The amounts of C and D decreases (A) (B)
[NH3 ] [NH3 ]
(B) The amounts of C and D increases
(C) The amount of B and D decreases [N2 ]1/ 2 [H2 ] 3 / 2
(D) All the amounts remain constant (C) [N2] [H2] – [NH3] (D)
[NH3 ] 2
28. The value of ionic product of water at 25ºC is -
20. In which of the following equilibrium reactions, the
(A) 1 × 10–1 mol2 L–2 (B) 1 × 10–14 mol2L–2
equilibrium would shift to right side, if total pressure
(C) 1 × 1014 mol2L–2 (D) 1 × 107 mol2L–2
is decreased :
(A) N2 + 3H2 2NH3 (B) H2 + I2 2HI (C) 29. The value of equilibrium constant for an unfavourable
N2O4 2NO2 (D) H2 + Cl2 2HCl forward reaction is -
(A) less than one (B) equal to one
21. The effect of increasing the pressure on the following (C) more than one (D) None of these
equilibrium 2A + 3B 3A + 2B is -
(A) Forward reaction is favoured 30. Consider the following reaction in a closed container
(B) Backward reaction is favoured
N2O4(g) 2NO2(g).
(C) No effect
(D) None of these At a fixed temperature, the volume of the reaction
container is halved. For this change, which of the
II. Laws of mass action, Effect on equilibrium constant following statements holds true regarding the
equilibrium constant (Kp) and degree of dissociation ()
22. The equilibrium constant for the reaction (A) neither Kp nor  changes
mA + nB pC + qD may be written as - (B) both Kp and  change
(C) Kp changes, but  does not change
[ A ]m [B]n [ A B]m  n
(A) KC = (B) KC = (D) Kp does not change, but  changes
[C]p [D] q [ C D] p  q
[C]p [D] q [C]1/ m [D]1 / n 31. At constant temperature, the equilibrium constant (Kp)
(C) KC = m n
(D) KC = for the decomposition reaction
[ A ] [B] [ A ]1/ p [B]1 / q
N2O4 2NO2 is expressed by Kp = 4x2 P/(1 – x2),
23. In the reaction, H2(g) + Cl2 (g) 2HCl (g) : where P is pressure, x is extent of decomposition.
Which of the following statement is true ?
(A) KP = KC (B) KP  KC
(A) Kp increases with increase of P
(C) KP > KC (D) KP < KC
(B) Kp increases with increase of x
(C) Kp increases with decrease of x
24. For the system NH4Cl (s) NH3 (g) + HCl (g) if
(D) Kp remains constant with change in P or x
the concentration of NH3 is doubled, the equilibrium
constant will -
(A) be doubled 32. For the reversible reaction N 2(g) + 3H 2(g)
(B) increase, but by less than a factor of 2 2NH3(g) at 500°C, the value of Kp is 1·44 × 10–5, when
(C) be halved partial pressure is measured in atmospheres. The
(D) remains the same corresponding value of Kc with concentration in mol
L–1 is
25. For which of the following reactions, the numerical
values of KP & KC are the same ? (A) 1·44 × 10–5/(0·082 × 500)–2
(B) 1·44 × 10–5/(8·314 × 773)–2
(A) 2NOCl(g) 2NO(g) + Cl2 (g)
(C) 1·44 × 10–5/(0·082 × 500)2
(B) N2(g) + 3H2 (g) 2NH3(g) (D) 1·44 × 10–5/(0·082 × 773)–2.
(C) H2(g) + Cl2 (g) 2HCl (g)
(D) PCl3 (g) + Cl2(g) PCl5(g) 33. The equilibrium constant for the following reaction
will be
26. In which of the following reactions, the product
3A+2B C
formation is most favoured ?
(A) N2O4 (g) 2NO2 (g) ; KC = 4.66 × 10–3 3 A 2B C
(A) (B)
(B) 2NH3 (g) N2 (g) + 3H2 (g) ; KC = 3 × 10 –9 C 3 A 2B
(C) N2(g) + O2 (g) 2NO (g) ; KC = 4.8 × 10–3 C C
(C) (D)
(D) PCl3 (g) + Cl2 (g) PCl5 (g) ; KC = 1.9 × 10–1 A 2 B2 A 3 B2

PAGE # 172
34. The rate constant of forward reaction is two times III. Electrolytes & non electrolytes, Ionic equilibrium
that of the reverse reaction at a given temperature, 41. In a chemical system at equilibrium, the addition of
the value of equilibrium constant is - catalyst would result in -
(A) Increase in rate of forward reaction only.
(A) 0·5 (B) 1·5
(B) Increase in rate of backward reaction only.
(C) 2·5 (D) 2·0.
(C) No effect on rate of reaction.
(D) Provide a new path having low activation energy
35. If K 1 and K 2 are the equilibrium constants of the
of the reaction.
equilibria (a) and (b) respectively, what is the
relationship between the two constants ?
42. For the equilibrium N2(g) +3H2(g) 2NH3(g) + 92 kJ
1
(a) SO2(g) + O (g) SO3(g) ; K1 The ammonia formation is favoured by
2 2 (A) increasing the pressure
(b) 2SO3(g) 2SO2(g) + O2(g) ; K2 (B) Adding more nitrogen
(C) Decreasing the temperature
1
(A) (K1)2 = (B) K2 = (K1)2 (D) All
K2 43. In the given reaction :
1 2SO2 (g) + O2 2SO3 (g) + heat
(C) K1 = (D) K1 = K2. The formation of sulphur trioxide is favoured by -
K2
(A) increasing the pressure
(B) decreasing the temperature
36. At a given temperature, the equilibrium constant for (C) adding more amount of oxygen
the reactions (D) all of the above
NO(g) + 1/2O2(g) NO2(g) and 2NO2(g)
2NO(g) + O2(g) are K1 and K2 respectively. If K1 is 4 × 44. For the chemical reaction 3X(g) + Y(g) X3Y(g)
10–3 then K2 will be
the amount of X3Y at equilibrium is affected by
(A) 8 × 10–3 (B) 16 × 10–3 (A) temperature and pressure
(C) 6·25 × 104 (D) 6·25 × 106. (B) temperature only
(C) pressure only
37. If Kp for a reaction (D) temperature, pressure and catalyst.
A(g) + 2B(g) 3C(g) + D(g)
45. The oxidation of SO2 by O2 to SO3 is an exothermic
is 0·05 atm at 1000 K. Its Kc in terms of R will be reaction. The yield of SO3 will be maximum if
(A) 20000 R (B) 0·02 R (A) temperature is increased and pressure is kept
5  10 –5 constant
(C) 5 × 10–5 R (D) . (B) temperature is reduced and pressure is increased
R (C) both temperature and pressure are increased
(D) both temperature and pressure are decreased
38. Ammonia dissociates into N2 and H2 such that degree
of dissociation  is very less than 1 and equilibrium
46. In which of the following equilibrium, change in the
pressure is P0 then the value of  is [if Kp for 2NH3(g)
volume of the system does not alter the number of
N2 (g) + 3H2(g) is 27 × 10–8 P02: moles.
(1) 10–4 (2) 4 × 10–4
(A) N2(g) + O2(g) 2NO(g)
(3) 0.02 (4) can’t be calculated.
(B) PCl5 (g) PCl3 (g) + Cl2(g)
39. At a temp, T, a compound AB 4(g) dissociates as
(C) N2(g) + 3H2 (g) 2NH3 (g)
2AB 4 (g) A 2(g) + 4B 2 (g) with a degree of
dissociation x, which is small compared with unity. (D) SO2Cl2(g) SO2(g) + Cl2(g).
The expression of Kp in terms of x and total pressure
P is ; 47. Consider the reaction
(1) 8P3x5 (2) 256P3x5
CaCO3(s) CaO(s) + CO2(g)
(3) 4Px2 (4) None of these
in closed container at equilibrium. What would be the
40. The equilibrium constant, Kp for the reaction effect of addition of CaCO 3 on the equilibrium
concentration of CO2 ?
2SO2(g) + O2(g) 2SO3(g)
–1 (A) Increases
is 4.0 atm at 1000 K. W hat would be the partial
(B) Decreases
pressure of O 2 if at equilibrium the amount of SO 2
(C) Data is not sufficient to predict it
and SO3 is the same ? (D) Remains unaffected.
(1) 16.0 atm (2) 0.25 atm
(3) 1 atm (4) 0.75 atm

PAGE # 173
48. When pressure is applied to the equilibrium system, 59. Which pair will show common ion effect ?
ice water,, (A) BaCl2 + Ba(NO3)2 (B) NaBr+ HCl
which of the following phenomenon will occur ? (C) NH4OH + NH4Cl (D) AgCl + KCN
(A) More ice will be formed
(B) Water will be evaporated 60. The solubility of AgCl in a solution of common salt is
(C) More water will be formed lower than in water. This is due to -
(D) Equilibrium will not be disturbed (A) salt effect
(B) lowering of solubility product
49. Aniline is a very weak base. Which of the given aniline (C) common ion effect
solution will have highest degree of dissociation. (D) complex formation
(A) 1M aniline (B) 0.1 M aniline
(C) 0.01 M aniline (D) 0.02 M aniline 61. On passing a current of HCl gas in saturated solution
of NaCl, the solubility of NaCl–
50. The pH of a 0.1 M solution of weak base is 11. On (A) increases
diluting the solution : (B) decreases
(A) pH increases (B) [OH–] increases (C) remains unchanged
(C)  decreases (D) pH decreases (D) NaCl decomposes

51. A monoprotic acid in a 0.1 M solution ionize to 0.001%. 62. On addition of ammonium chloride to a solution of
Its ionization constant is – NH4OH -
(A) 1.0  10–3 (B) 1.0  10–6 (A) dissociation of NH4OH increases
(C) 1.0  10–8 (D) 1.0  10–11 (B) concentration of OH– decreases
(C) concentration of OH– increases
52. Ka for a weak acid HA is 1.44 × 10– 5. What is the
(D) concentration of both NH4+ and OH– increases
concentration of A– when 0.01 mole of HA is dissolved
in 1L of 0.01M HCl solution ?
(A) 0.01 M (B) 1.2 × 10–3 M (C) 63. The following equilibrium exists in aqueous solution -
1.44 × 10–5M (D) 0.012 M CH3COOH H+ + CH3COO–
If dilute HCl is added –
53. 100 ml of 1M HCl is mixed with 900 ml of 0.1 M NaOH.
(A) the equilibrium constant will increase
In the final solution :
(B) the equilibrium constant will decrease
(A) [H+] = 10–1 M (B) [Cl–] = 10–1M (C) [Na +] =
[Cl–] (D) [OH–] = 10–2 M (C) unionised acetic acid concentration will increase
(D) acetate ion concentration will not change
IV. Common ion effect, Solubility product
64. W hich of the following salts undergo anionic
54. Solubility of AgCl in water is 0.976 × 10 –10
g/L. Its hydrolysis ?
solubility product is - (A) Na3PO4 (B) NaCl
(A) 4.625 × 10–15 (B) 4.624 × 10–25 (C) NH4Cl (D) FeSO4
–14
(C) 4.624 × 10 (D) None of these
65. Which of the following compound forms an aqueous
solution which is acidic when compared with water –
55. Solubility product of AgBr is 4.9 × 10–13. Its solubility in
(A) NaOH (B) K2CO3
mol/litre will be -
(C) BaCl2 (D) Al2(SO4)3
(A) 7 × 10–7 (B) 7 × 10–4
(C) 8 × 10–5 (D) None of these
66. The salt of which of the following four weak acids will
–17
be most hydrolysed
56. Solubility product of AgI at 25º C is 8.13 × 10 . (A) HA ; Ka = 1  10–6
Calculate solubility of AgI in g/L. (B) HB ; Ka = 2  10–6
(A) 2.12 × 10–6 g/L (B) 3.12 × 10–4 g/L (C) HC ; Ka = 3  10–8
–3
(C) 2.14 × 10 g/L (D) 3.13 × 10–4 (D) HD ; Ka = 4  10–10

57. W hen NH 4 Cl is added to NH 4OH solution, the 67. The reverse process of neutralisation is:
dissociation of ammonium hydroxide is reduced. It is (A) Hydrolysis (B) Decomposition
due to - (C) Dehydration (D) Synthesis
(A) common ion effect (B) hydrolysis
(C) oxidation (D) reduction 68. The pH of 0.01 M ammonium sulphate solution is
[KbNH3] = 2  10–5
58. The addition of HCl will not suppress the ionisation of - (A) 5.5 (B) 6.8
(A) acetic acid (B) sulphuric acid (C) 8.6 (D) 7
(C) H2S (D) benzoic acid

PAGE # 174
69. The pH of 0.01 M sodium acetate solution is 4. W hich of the following will change the value of the
[Ka(CH3COOH] = 2  10–5 equilibrium constant for the reaction?
(A) 7.25 (B) 6.5 [IJSO Stage-II 2014]
(C) 8.05 (D) 8.35 N2(g) + O2(g)  2NO (g)
(A) Add more N2
70. A certain weak acid has a dissociation contant 1.0 ×
(B) Increase of pressure
10–4 . The equilibrium constant for its reaction with a
(C) Use a smaller reaction vessel
strong base is :
(A) 1.0 × 10–4 (B) 1.0 × 10–10 (D) Increase the temperature
–10
(C) 1 × 10 (D) 1.0 × 10–14
5. For the reaction
71. The solubility of CaF2 (Ksp = 3.4 × 10–11 ) in 0.005 M aE + bF cG + dH
solution of BaF2 is : The expression of equilibrium constant can be writ-
(A) 3.4 × 10–19 (B) 1.36 × 10–6 ten as [IJSO Stage-II 2014]
(C) 3.4 × 10 –7 (D) 1.36 × 10–7 c d a b
Kc= [G] [H] / [E] [F]
Where all the concentrations are equilibrium concen-
72. Ksp of AgBr is 5 × 10–13 . Precipitation of AgBr will take
trations. Before approaching equilibrium, the same
place in a solution having :
concentration ratio is called reaction quotient Qc.
(A) 0.1 M AgNO3 and 5 × 10–12 M NaBr (B)
5 × 10–12 M AgNO3 and 0.1 M NaBr For the reaction system to reach equilibrium
(C) 2 × 10–6 M AgNO3 and 4 × 10–8 M NaBr (A) Qc must increase in the reaction
(D) 2 × 10–6 M AgNO3 and 4 × 10–6 M NaBr (B) Qc must decrease in the reaction
73. The solubility of CaF2 (Ksp, = 3.4 × 10–11) in 0.1 M (C) Qc= Kc
solution of NaF would be (D) Qc = zero
(A) 3.4 × 10–12 M (B) 3.4 × 10–10 M
–9
(C) 3.4 × 10 M (D) 3.4 × 10–13 M. 6. In Haber’s process 0.240 mole of Nitrogen, 3.9 moles
of hydrogen are taken which lead to the formation of
7.8 moles product in a 3.00 litres of reaction vessel at
74. In a saturated solution of Ag 2 CO 3 , silver ion
375oC. Considering that equilibrium constant at this
concentration is 2 × 10–4 M. Its solubility product is temperature is 41.2 Calculate the value of reaction
(A) 4 × 10–12 (B) 3.2 × 10–11 quotient (Q) and predict whether the reaction is in
–12
(C) 8 × 10 (D) 10–12 equilibrium or it will proceed in either direction.
[IJSO Stage-II 2014]
(A) Q = 38.62 and reaction will be in equilibrium.
EXERCISE-2 (B) Q = 19.31 and reaction will proceed in forward
direction.
COMPETITIVE EXAM PREVIOUS YEARS’ QUESTIONS : (C) Q = 38.62 and reaction will proceed in forward
direction.
(D) Q = 19.31 and reaction will proceed in backward
1. The equilibrium constant for the gaseous reaction
direction.
N2 + O2  2NO is k. The equilibrium constant for the
formation of one mole of NO will be
7. Variation of the concentration of the reactant (X) and
[IJSO_Stage-I/ 2012]
the product (Y) are shown in the figure. Select the
(A) k/2 (B) k
correct statement. [IJSO Stage-II 2014]
(C) 2k (D) k

2. Consider the following reaction :


4 PCl3 (g)  P4 (g) + 6Cl2(g). If the initial concentra-
tion of PCl3(g) is 1.0 M, and “x” is the equilibrium con-
centration of P4(g), what is the correct equilibrium re-
lation ? [IJSO_Stage-I 2013]
(A) Kc= 6x7 (B) Kc= 6x7 / (1.0 – x)4
(C) KC = (x) (6x)6 / (1.0 – 4x)4 (D) Kc = x7 (1.0 – x)4

3. Consider the following reaction : 4 PCl3 (g)  P4 (g) (A) I and II both are kinetic regions.
+ 6Cl2(g). If the initial concentration of PCl3(g) is 1.0 (B) I and II both are equilibrium regions.
M, and “x” is the equilibrium concentration of P4(g), (C) I is equilibrium and II is kinetic region.
what is the correct equilibrium relation ? (D) I is kinetic and II is equilibrium region.
[IJSO_Stage-I/ 2013-14]
8. At 5 atm pressure PCl5 gas dissociates by 10%. What
a) Kc= 6x7 b) Kc= 6x7 / (1.0 – x)4
will be the value of Kp at same temperature ?
c) KC = (x)(6x)6 /(1.0–4x)4 d) Kc = x7 (1.0 – x)4 [IJSO Stage-II 2014]
(A) 0.045 atm (B) 0.050 atm
(C) 0.9 atm (D) 0.5 atm

PAGE # 175
9. Assertion (A) : If the volume of the vessel is doubled A certain volume 'V1' of unknown concentration 'M1' of
then for the following reaction. HCI is taken in a conical flask, to which a few drops of
A (g)  B (g) + C (g) [IJSO Stage-II 2014] phenolphthalein indicator solution is added. The
Equilibrium constant is decreased. solution remains colourless. From a burette (a
Reason (R) : Equilibrium constant Kc = X2/ (1 – X ) V graduated dropper) a solution of NaOH, whose
(A) Both (A) and (R) are true and (R) is the correct concentration is known, 'M2', is added dropwise into
explanation of (A). the conical flask until a pale pink colour is obtained
(B) Both (A) and (R) are true and (R) is not the correct and is termed as the end point. The amount of
explanation of (A). solution dispensed from the burette to obtain the end
(C) (A) is true but (R) is false. point is noted as 'V 2'. Phenolphthalein indicator
(D) (A) is false but (R) is true. changes its colour to pink only when the pH of the
solution is above 9.5. Similarly, another indicator,
10. Given below is the pH vs volume curve for titration of methyl orange, is red in colour below pH 3.7 and
Na2CO3 (in conical flask) with HCl. yellow above. Given below is a graph of pH of the
IJSO Stage-1/2015 solution in the conical flask and the reading of the
burette in the course of the titration. The equivalent
point is theoretically defined as the point in the graph
where the number of moles HCI in the conical flask
becomes equal to the number of moles of NaOH run
down the burette. Note the difference between end
point and equivalence point. IJSO Stage-1/2015

Which is the correct graph that represents the


titration of NH4OH (from burette) with HCI?

Choose the correct statement based on the above


graph
(A)
(A) 1st equivalence point represents NaHCO3 getting
protonated to H2CO3.
(B) 2nd equivalence point represents NaHCO3 getting
protonated to H2CO3.
(C) 1st equivalence point represents deprotonation
of HCI by NaHCO3 alone.
(D) 2nd equivalence point represents deprotonation
of HCI by Na2CO3 alone.

11. An acid-base titration is a technique where a solution


of known concentration of acid/base is used to
determine the concentration of an unknown solution
of acid/base. These titrations typically use a pH (B)
indicator solution to denote the end point of the
reaction. A pH indicator is a compound added in small
quantities to a solution to indicate the pH visually
(generally by appearance/disappearance or change
in colour). A typical procedure is as follows:

(C)

PAGE # 176
(D)

12. When solid KOH is mixed with solid NH4Cl, a gas is


produced. Which gas is it ?
[IJSO_Stage-I/ 2016-17]
(A) Chlorine (B) Hydrogen
(C) Hydrogen chloride (D) Ammonia

13. The heat of neutralisation of CH 3COOH, HCOOH,


HCN and H2S are - 55. 2, - 56.07,- 2.8 and - 3.34 kJ
per equivalent respectively. The increasing order of
strength of these acids is. [IJSO-Stage-1/2016-17]
(A) HCOOH < CH3COOH < H2s < HCN
(B) H2S < HCN < HCOOH < CH3COOH
(C) HCN < H2S < CH3COOH < HCOOH
(D) CH3COOH < HCOOH < HCN < H2S

PAGE # 177
CHEMICAL KINETICS

INTRODUCTION
In the thermodynamics, we have studied whether a
reaction will take place or not and if it does then upto
what extent (chemical equiibrium), In this chapter we
will study about how fast a chemical reaction takes
place and what are the different factors affecting this
rate of chemical reaction. How to optimise the
conditions as to maximse the output in optimum time.
The last part of chapter will be dealing with the
mechanism of a chemical reaction and catalysis. Relation between reaction rates of different
species involved in a reaction :
Rate/Velocity of chemical reaction For the reaction : N2 + 3H2  2NH3

The rate of change of concentration with time of d [N2 ]


Rate of reaction of N2 = 
different chemical species taking part in a chemical dt
reaction is known as rate of reaction of that
d [H2 ]
species. Rate of reaction of H2 = 
dt
c mol / lit. d [NH3 ]
Rate= = =mol lit–1 time–1 = mol dm–3 time–1 Rate of reaction of NH3 =
t sec dt

Rate is always defined in such a manner so that it is These rates are not all equal. Therefore by convention
always a positive quantity. the rate of a reaction is defined as
d [N2 ] 1d [H2 ] 1 d [NH3 ]
Types of Rates of chemical reaction Rate of reaction =  =  3 dt =
dt 2 dt
Note : Rate of reaction value is dependent on the
For a reaction R  P
stoichiometric coefficients used in the reaction while
rate of any species will be fixed value under given
Total change in concentrat ion conditions.
Average rate =
Total time taken
Factor affecting rate of chemical reaction

c  [R]  [P]
= =– = 1. Concentration
t t t 2. Temperature
3. Nature of reactants & products
Instantaneous rate : rate of reaction at a particular
4. Catalyst
instant.
 Concentration : We known from law of mass action
 c  dc d [R] d [P] that Rate is proportional to concentration of reactants.
Rinstantaneous = tlim
0   = =– =
 t  dt dt dt “ So, generally rate of reaction decreases with
passage of time, since concentration of reactants
Instantaneous rate can be determined by drawing a decreases.
tangent at time t on curve drawn for concentration versus
time.
Initial Rate : Instantaneous rate at ‘t = 0’ is called
initial rate [slope of tangent at t = 0].

PAGE # 178
these may differ for the same reaction on conditions
 Nature of reactants & Products :
under which the reaction is being carried out.
(a) Physical state of reactants :
But for large number of reactions starting with pure
Gaseous state > Liquid state > Solid state
reactants we can obtain simple rate laws.
Decreasing order of rate of reaction.
For these reactions :
(b) Physical size of reactants : As we decreases Rate  (conc.)order
the particle size rate of reaction increases since Rate = K (conc.)order (differential rate equation or
surface area increases. rate expression)
Where K = Rate constant = specific reaction rate =
(c) Chemical nature of reactants :
rate of reaction when concentration is unity
If more bonds are to be broken, the rate of reaction unit of K = (conc)1– order time–1
will be slow.
Note : Value of K is a constant for a given reaction,
Similarly bond strength is more, rate of reaction depends only on temperature
will be slow.
Order of reaction
 Catalyst :
 Presence of positive catalyst lower down the Let there be a reaction
activation energy hence increases the rate of reaction. m1A + m2B  products.
Now, if on the basis of experiment, we find that
presence of negative catalyst increases activation R  [A]P [B]q Where p may or may not be equal to
energy hence decreases the rate of reaction. m1 & similarly q may or may not be equal to m2.
p is order of reaction with respect to reactant A and q
is order of reaction with respect to reactant B and (p
Rate Law (Dependence of rate on
concentration of reactants
+ q) is overall order of the reaction.
Note : Order of a reaction can be ‘zero’ or any whole
The representation of rate of reaction in terms of the
number, can be a fractional number and it can even
concentration of the reactants is called the rate law.
be negative with respect to a particular reactant. But
It can only be established by experiments.
oveall order is not found to be negative for any reaction
Generally rate law expressions are not simple and
till observed.

Examples showing different values of order of reactions :

Reaction Rate law Order

2N2O5 (g)  4NO2 (g) + O2 (g) R = K [N2O5]1 1

5Br–(aq)+BrO3– (aq)+6H+(aq)3Br2()+3H2O() R= K [Br–] [BrO3–] [H+]2 1+1+2=4

H2 (Para)  H2 (ortho) R = K [H2 (Para) ]3/2 3/2

NO2 (g) + CO (g)  NO (g) + CO2 (g) R = K [NO2]2 [CO]º 2+0=2

2O3 (g)  3O2 (g) R = K [O3]2 [O2]–1 2–1=1

h
H2 + Cl2   2 HCl R = K [H2]0[Cl2]0 0+0=0

The reaction (B) does not take place in one single step. It is almost impossible for all the 12 molecules of the
reactants to be in a state of encounter simultaneously. Such a reaction is called complex reaction and takes
places in a sequence of a number of elementary reactions. For an elementary reaction the sum of stoichiometric
coefficients = order of the reactions. But for complex reactions order is to be experimentally calculated.

PAGE # 179
Examples C0
Ex.2 The reaction 2NO(g) + Cl2(g)  2NOCl(g) is t1/2 (half life period) at t1/2 , Ct = ,
2
second order in NO and first order in Cl2. In a volume
of 2 dm3, 5 mole of nitric oxide and 2 mol of Cl2 were C0 C
so kt1/2 =  t1/2 = 0
brought together, and the initial rate was 2.4 × 10 3 2 2k
m o l e
dm3 s. What will be the rate when half of the chlorine  t 1 / 2 C 0
has reacted ?
Ans. 4.32 × 10–4 M sec–1 Examples of zero order reactions :
Generally decomposition of gases on metal surfaces
Sol. 2NO + Cl2  2NaCl at high concentrations follow zero order kinetics.
5 2 Ni
2PH3 (g)  2 P + 3 H2 Rate = K [PH3]º
5-2 1 2
2H (g) Au
2  H2 + 2

5
R=k   N
2NH3(g)  N2 + 3H2
2
hv
H2 + Cl2 
 15   2 HCl Rate = R [H2]º [Cl2]º
2.4 × 10–3 = k  4 
 
(b) First Order Reactions :
4  2.4  10 3
(i) Let a 1st order reaction is
R=  Products
25 A
2 conc. ‘a’ 0 t =0
4  2.4  10 3  3   1  ‘a–x’ ........ t = ‘t’
R= 2 2
25    
dx
3 Let be the rate of reaction at time ‘t’
4  2.4  10 9 1 dt
R=  = 4.32 × 10–4 M sec–1
25 8
dx dx
 = k (a–x)1 or = kdt.
dt ax
Integrated rate law
2.303 a
(a) Zero order reactions : On solving t = log
k ax
For a zero order reaction
2.303 C0
General rate law is, Rate = k [conc.]º = constant k= log C
If C0 is the initial concentration of a reactant and Ct is t t
the
concentration at time ‘t’ then
C0  C t
Rate = k = or kt = C0 – Ct or Ct = C0 – kt
' t'

Unit of K = same as that of Rate = mol lit–1 sec–1. If any substance is growing/increasing following
first order kinetics then :
C0
Time for completion = 2.303 ax
k k= log  
t  a 
where a is initial concentration of the substance and
x is the increment in its concentration after time t.

PAGE # 180
2.303 2C 0 t1 / 4 log 4 2 log 2
Half life time (t1/2) k = log  = = =2
t1 / 2 C0 t 1/ 2 log 2 log 2

2.303 log 2  Ex.4 At least how many half-lives should elapse for a
 t1/2 = = n 2 = 0.693
k k k 1st order reaction A  products so that the
 Half life period for a 1st order reaction is a constant reaction is at least 95% completed ? (log 2 = 0.3)
quantity.
(A) 4 (B) 5
Graphical Representation : (C) 6 (D) 7
2.303 2.303
t=  log Ct + log C0 t1 / 2 t 1/ 2 t1 / 2
k R Sol.(B) 100   50   25  
0% 50% 75%

t1 / 2 t 1/ 2
12.5   6.25   3.125
tan= 2.303 tan= 2.303 96.875%
't' k 87.5% 93.75%
k
 't'

log C0/Ct
or log a/a-x log Ct
(c) Psuedo first order reaction :
The order of a reaction is sometimes altered by conditions.
Examples of 1st order reactions : Consider a chemical reaction between two substances
1. Decomposition of azoisopropane when one reactant is present in large excess. During the
 hydrolysis of 0.01 mol of ethyl acetate with 10 mol of
= N– (g) N2(g)+ C6H14 (g)
water, amounts of the various constituents at the beginning
2. Conv ersion of N–chloro acetanilide into p– (t = 0) and completion (t) of the reaction are given as
chloroacetanilide under.

H
CH3COOC2H5 + H2O 
 CH3COOH + C2H5OH
t=0 0.01 mol 10 mol 0 mol 0 mol
HCl


t 0 mol 9.9 mol 0.01 mol 0.01 mol
The concentration of water does not get altered much
during the course of the reaction. So, in the rate equation
1 Rate = k' [CH3COOC2H5] [H2O]
3. H2O2  H2O + O
2 2
the term [H2O] can be taken as constant. The equation,
4. NH4 NO2  2H2O + N2
thus, becomes
5. Radiactive decay Rate = k [CH3COOC2H5]
 All radioactive decays are always first order kinetics. where k = k’ [H2O] and the reaction behaves as first order
reaction. Such reactions are called pseudo first order
 + 2He4 reactions.
Inversion of cane sugar is another pseudo first order
Examples reaction.

t 0.75 H
C12H22O11 + H2O 

Ex.3 Calculate for a 1st order reaction  C6H12O6 + C6H12O6


t 0.50
Cane sugar Glucose Fructose
Rate = k [C12H22O11]
2.303 C0 2.303 C0
Sol. k= log 1  log C 0
t 1/ 4 t1 / 2
C0
4 2

PAGE # 181
Table : Characterstics of First-and Second-Order Reactions of the Type A  Products
––––––––––––––––––––––––––––––––––––––––––––––––––––––––––––––––––––––––––––––––––––––––––––––
Zero Order First-Order
––––––––––––––––––––––––––––––––––––––––––––––––––––––––––––––––––––––––––––––––––––––––––––––
 A [ A ]
Differential Rate law = k[A]°  = k[A]
t t
(Integrated Rate law)
[A]t = [A]0 – kt In [A]t = –kt + In [A]0
Linear graph [A]t versus t In [A] versus t
[ A ]0 0.693
Half-life t1/2 = t1/2 =
2k k
(depends on [A]0) (independent of [A]0)

[A]0 [A]0

[A] [A]
Time Time

––––––––––––––––––––––––––––––––––––––––––––––––––––––––––––––––––––––––––––––––––––––––––––––
Graphical comparison of different orders

Methods to determine order of reaction


(A) Initial rate method :
 By comparision of different initial rates of a reaction by varying the concentration
of one of the reactants while others are kept constant

PAGE # 182
r = k [A]a [B]b [C]c if [B] = constant Examples
[C] = constant Ex.7 In the reduction of nitric gas with hydrogen, the
then for two different initial concentrations of A we reaction was found to be 50% complete in 210
have seconds when the initial pressure of the mixture
was 200 mm. In a second experiment the time of
x
r01  [A ]  half reaction was 140 seconds when the initial
r01 = k [A0]1a r02 =k [A0]2a    0 1 
r0 2 pressure was 300 mm. Calculate the total order of
 [A 0 ]2 
the reaction.


log r01 r02  Sol. For a nth order reaction (n  1), t 1/2 
1
or in log form we have a =
log [ A 0 ]1 [ A 0 ]2  c n01

n1
210  300 
Examples =   n=2
140  200 
Ex.5 The pressure of a gas decomposing at the surface
of a solid catalyst has been measured at different
times and the results are given below :
t/s 0 100 200 300
Arhenius theory of reaction rate
p/Pa 4.00 × 103 3.50 × 103 3.00 × 103 2.5 × 103 Most of the chemical reactions are accelerated by
Determine the order of reaction, its rate constant and increase in temperature.
half-life period. For example, in decomposition of N2O5, the time taken
Sol. It Can be seen that rate of reaction between different for half of the
time intervals is : original amount of material to decompose is 12 min at
[3.50  4.00]  10 3 Pa 50oC, 5 h at
0–100 s, rate = = 5 Pa/s 25oC and 10 days at 0oC. You also know that in a mixture
100
of potassium
[3.00  3.50]  10 3 Pa permanganate (KMnO4) and oxalic acid (H2C2O4),
100–200s, rate = – = 5 Pa/s potassium
100s
permanganate gets decolourised faster at a higher
[2.50  3.00]  10 3 Pa temperature than
200–300s, rate = – = 5 Pa/s that at a lower temperature.
100
It has been found that for a chemical reaction with rise in
We notice that the rate remains constant and temperature by 10°, the rate constant is nearly doubled.
therefore, reaction is of zero order. Alternatively, if we The temperature dependence of the rate of a chemical
plot a p against t, it is a straight line agaion indicating reaction can
it is a zero order reaction. be accurately explained by Arrhenius equation (4.18). It
k = rate = 5 Pa/s was first
4.00  10 3 Pa proposed by Dutch chemist, J.H. van’t Hoff but Swedish
initial concentrat ion or pressure
t1/2= = chemist,
2k 2  5 Pa s 1
Arrhenius provided its physical justif ication and
= 400s interpretation.
k = A e -Ea /RT (4.18)
where A is the Arrhenius factor or the frequency factor. It
(B) Method of half lives : is also called
 The half lives of each order is unique so by comparing pre-exponential factor. It is a constant specific to a
half lives we can determine order particular reaction.
R is gas constant and Ea is activation energy measured
1 in joules/mole
for nth order reaction t1/2 
[R 0 ]n1 (J mol –1).

t 1/ 2 (R '0 )n 1

t 1' / 2 (R 0 )n 1

PAGE # 183
Fig. Formation of HI through the intermediate Examples of catalysis
Intermediate (a) Thermal decomposition of KCIO3 is found to be
accelerated by the presence of MnO2. Here MnO2
It can be understood clearly using the following simple (foreign substance) acts as a catalyst.
reaction
2KClO3 + [MnO2] 2KCl + 3O2  + [MnO2]
MnO2 can be received in the same composition
According to Arrhenius, this reaction can take place only and mass at the end of the reaction.
when a molecule of hydrogen and a molecule of iodine (b) In the permanganate titration of oxalic acid intially
collide to form an unstable intermediate (Fig. 4.6). It exists there is slow discharge of the colour of permanganate
for a very short time and then breaks up to form two solution but afterwards the discharge of the colour
molecules of hydrogen iodide. become faster. This is due to the formation of MnSO4
during the reaction which acts as a catalyst for the
same reaction. Thus, MnSO4 is an “auto catalyst”
for this reaction. This is an example of auto catalyst.
2KMnO4 + 3H2SO4 + 5H2C2O2  K2SO4 + 8H2O + 10CO2

General characteristics of catalyst :


 A catalyst does not initiate the reaction. It simply
fastens it.
 Only a small amount of catalyst can catalyse the
reaction.
 A catalyst does not alter the position of equilibrium
i.e. magnitude of equilibrium constant and hence Gº.
It simply lowers the time needed to attain equilibrium.
This means if a reversible reaction in absence of
The energy required to form this intermediate, called catalyst completes to go to the extent of 75% till
activated complex (C), is known as activation energy (Ea). attainment of equilibrium, and this state of equilibrium
This diagram is obtained by plotting potential energy vs is attained in 20 minutes then in presence of a
reaction coordinate. Reaction coordinate represents the catalyst also the reaction will go to 75% of completion
profile of energy change when reactants change into before the attainment of equilibrium but the time
products. Some energy is released when the complex needed for this will be less than 20 minutes.
decomposes to form products. So, the final heat of the
 A catalyst drives the reaction through a low energy
reaction depends upon the nature of reactants and
path and hence Ea is less. That is, the function of the
products.
catalyst is to lower down the activation energy.

Catalyst and catalysis Ea = Energy of activation in absence of catalyst.


E’a = Energy of activation in presence of catalyst.
A catalyst is a substance, which increases the rate
Ea – E’a = lowering of activation energy by catalyst.
of a reaction without itself being consumed at the
end of the reaction, and the phenomenon is called
catalysis.
P.E. Ea
E'a
HR

HP
Products

Reaction Coordinate

Catalyst are generally foreign substances but


sometimes one of the product may act as a catalyst
and such catalyst is called “auto catalyst” and the
phenomena is called auto catalysis.

PAGE # 184
Molecularity and Order : The rate law for the elementary reaction
The number of molecules that react in an elementary aA + bB  products rate = k[A]a[B]b,
step is the molecularity of the elementary reaction. where a + b = 1, 2 or 3.
Molecularity is defined only for the elementary For an elementary reaction, the orders in the
reactions and not for complex reactions. No rate law equal the coefficients of the reactants.
elementary reactions involving more than three While, the order is defined for complex as well as
molecules are known, because of very low probability elementary reactions and is always experimentally
of near-simultaneous collision of more than three calculated by the mechanism of the reaction, usually
molecules. by the slowest step of the mechanism known as rate
determining step of the reaction.

Comparison B/W Molecularity and order of reaction

Molecularity of Reaction Order of Reaction

2. It is always a whole number 2. It may be zero , fractional or integer.


It can neither be zero nor fractional.

3. It is derived from RDS in the mechanism of reaction. 3. It is derived from rate expression.

4. It is theoretical value. 4. It is experimental value.

5. Reactions with molecularity 4 are rare. 5. Reactions with O.R are also rare.

6. Molecularity is in independent of 6.OR. depends upon pressure and temperature.


Pressure and temperature.

PAGE # 185
6. ln the reaction; A + 2B  3C + D, which of the
EXERCISE-1
following expression does not describe changes in the
I. Rate of reaction concentration of various species as a function of time :
(A) {d [C] / dt} = – {3d [A] / dt}
d[ A ] d[B] d[C ] (B) {3d [D] / dt} = {d [C] / dt}
1. xA + yB  zC. If  =  = 1.5 = then x,y
dt dt dt (C) {3d [B] / dt} = – {2d [C] / dt}
and z can be : (D) {2d [B] / dt} = – {d [A] / dt}
(A) 1,1,1 (B) 3,2,3
(C) 3,3,2 (D) 2,2,3 7. In the following reaction :
xA  yB
2. The rate of a reaction is expressed in different ways as
follows ;  d[ A ]   d[B ] 
+ 1/2(d[C]/dt) = – 1/3 (d[D]/dt) = + 1/4 (d[A]/dt) = – (d[B]/dt) log    = log  dt  + log 2
 dt   
The reaction can be :
where –ve sign indicates rate of disappearance of the
(A) 4 A + B  2C + 3D reactant. Thus, x : y is :
(A) 1 : 2 (B) 2 : 1
(B) B + 3D  4A + 2C (C) 3 : 1 (D) 3 : 10

(C) 4A + 2B  2C + 3D II. Rate law

(D) B + (1/2) D  4A + 3C 8 . aA + bB  Product, dx/dt = k [A]a [B]b . If conc of A is


doubled, rate is four times. If conc. of B is made four
3. Consider the chemical reaction : times, rate is doubled. What is relation between rate of
disappearance of A and that of B ?
N2(g) + 3H2(g)  2NH3(g) (A) {– d [A] / dt} = {– d [B] / dtK
The rate of this reaction can be expressed ;in terms of (B) – {d [A] / dt} = – 4 {d [B] / dt}
time and of concentration of N2(g), H2(g) or NH3(g). Identify (C) – 4 {d [A] / dt) = {– d [B]/ dt}
the correct relationship amongest the rate expressions. (D) None of these
d [N 2 ] 1 d [H 2 ] 1 d [NH3 ] 9. For a reaction pA + qB  products, the rate law
(A) Rate = – =– =
dt 3 dt 2 dt expression is r = k[A]1 [B]m , then :
(A) (p+1) < (1+ m)
d [N2 ] 3d [H2 ] 2d [NH3 ] (B) (p + q) > (1 + m)
(B) Rate = – =– =
dt dt dt (C) (p + q) may or may not be equal to (1+ m)
(D) (p + q) = (1 + m)
d [N2 ] 1 d [H2 ] d [NH3 ]
(C) Rate = – =– =
dt 3 dt dt 10. For the reaction H2 + Br2  2 HBr overall order is found to
be 3/2. The rate of reaction can be expressed as:
d [N2 ] d [H2 ] d [NH3 ] (A) [H2][Br2]1/2 (B) [H2]1/2 [Br2]
(D) Rate = – = = 3/2 0
dt dt dt (C) [H2] [Br2] (D) All of these

11. If concentration of reactants is increased by a factor x


4. In a reaction N2(g) + 3H2(g)  2NH3(g) the rate of then the rate constant k becomes :
appearance of NH3 is 2.5  10–4 mol L–1 sec–1. The rate of k k
reaction & rate of disappearance of H 2 will be (A) In (B)
x x
(in mol L–1 sec–1)
(A) 3.75  10–4 , 1.25  10–4 (C) k + x (D) k
(B) 1.25  10 , 2.5  10–4
–4

(C) 1.25  10–4, 3.75  10–4 12. The rate of certain hypothetical reaction A + B + C 
(D) 5.0  10–4, 3.75  10–4 d [ A ]
products is given by r = = K [A] 1/2 [B] 1/3
dt
 d [B]
5.   3A  2B, rate of reaction is equal to : [C]1/4 The order of the reaction :
dt
1
3 d [ A] 2 d [ A] (A) 1 (B)
(A) – (B) – 2
2 dt 3 dt
13
(C) 2 (D)
1 d [ A] d [ A] 12
(C) – (D) +2
3 dt dt

PAGE # 186
13. 2A  B + C it would be a zero order reaction when : 21. For a reaction A + B  products, the rate of the reaction
(A) The rate of reaction is proportional to square of conc. was doubled when the concentration of. A was doubled,
of A the rate was again doubled when the conc. of A & B were
(B) The rate of reaction remains same at any conc. of A doubled the order of the reaction with respect to A & B
(C) The rate remains unchanged at any conc. of B and C are :
(D) The rate of reaction doubles if conc. of B is increased (A) 1,1 (B) 2,0
to double (C) 1,0 (D) 0,1

14. The rate equation for the reaction 2A + B  C is found to III. The integrated rate laws
be : rate k[A][B]. The correct statement in relation to this
22. The rate constant of reaction 2 A + B  C is 2.57 ×
reaction is that the
(A) Rate of formation of C is twice the rate of 10–5 It mole–1 sec–1 after 10 sec. 2.65 × 10–5 It. mole–1
disappearance of A. sec–1 after 20 sec. and 2.55 × 10–5 It. mole–1 sec–1 after
(B) Half life is a constant 30 sec. The order of the reaction is:
(C) Unit of k must be s–1 (A) 0 (B) 1
(D) Value of k is independent of the initial concentration (C) 2 (D) 3
of A and B
23. For a first order reaction, the plot of ‘t’ against log C gives
15. The rate constant for a reaction is 10.8  10–5 mol L–1 S–1 a straight line with slope equal to :
The reaction obeys : (A) (k / 2.303) (B) (– k / 2.303)
(A) First order (B) Zero order (C) (ln k / 2.303) (D) – k.
(C) Second order (D) All are wrong
24. In a first order reaction the amount of reactant decayed
in three half lives (let a be is initial amount) would be:
16. For the reaction, 2NO(g) + 2H2(g)  N2(g) + 2H2O(g) (A) 7a / 8 (B) a / 8
the rate expression can be written in the following ways: (C) a / 6 (D) 5a / 6
{dt [N2] / dt} = k1 [NO][H2] ; {d[H2O] / dt) = k[NO][H2] ; {– d[NO]
/ dt} = k1 [NO] [H2] ; {–d[H2] / dt} = k1 [NO][H2] 25. Graph between concentration of the product and time of
The relationship between k, k1 , k1 and k1. is :
(A) k = k1 = k1 = k1 (B) k = 2k1 = k1 = k1
(C) k = 2k1 = k1 = k1 (D) k = k1 = k1 = 2 k1 the reaction A  B is of the type Hence graph

17. If a reaction gets completed in finite time then its order between – d[A]/dt and time will be of the type :
can be :
(A) 3 (B) 2 (–d[A]/dt)
(C) 1 (D) Zero (A) (B)
Time
18. For the irreversible process, A + B  products, the
rate is first–order w.r.t. A and second–order w.r.t. B.If 1.0
mol each of A and B introduced into a 1.0 L vessel, and (C) (D)
the initial rate was 1.0 × 10–2 mol L–1 s–1 , rate when half
reactants have been turned into products is :
(A) 1.25 × 10–3 mol L–1 s–1 26. A graph plotted between log t50% vs. log concentration is
(B) 1.0 × 10–2 mol L–1 s–1 a straight line. What conclusion can you draw from this
(C) 2.50 × 10–3 mol L–1 s–1 graph.
(D) 2.0 × 10–2 mol L–1 s–1

19. The rate law for the single step reaction


2A + B  2C, is given by
(A) Rate = K [A][B] (B) Rate = K [A]2[B] (A) n = 1 ; t1/2  a (B) n = 2, t1/2  1/a
(C) Rate = K [2A][B] (D) Rate = K[A]2[B]0 (C) n = 1 ; t1/2 = (0.693 / k) (D) None of these

20. For the reaction A + B  Products, it is found that the


order of A is 2 and of B is 3 in the rate expression. When 27. K for a zero order reaction is 2 10–2 mol L–1 sec–1 . If the
concentration of both is doubled the rate will increase by concentration of the reactant after 25 sec is 0.5 M, the
a factor of : initial concentration must have been.
(A) 10 (B) 6 (A) 0.5 M (B) 1.25 M
(C) 32 (D) 16 (C) 12.5 M (D) 1.0 M

PAGE # 187
28. Plot of log(a–x) vs time t is straight line. This indicates 37. In a I order reaction A  products, the concentration of
that the reaction is of the reactant decrases to 6.25% of its initial value in 80
(A) Second order (B) First order minutes. What is (i) the rate constant and (ii) the rate of
(C) Zero order (D) Third order the reaction, 100 minutes after the start, if the initial
concentration is 0.2 mole/litre?
29. In a first order reaction the concentration of the reactant (A) 2.17 × 102 min1, 3.47 × 104 mol.litre1 min1
is decreased from 1.0 M to 0.25 M in 20 min. The rate (B) 3.465 × 102 min1, 2.166 × 104 mol.litre1 min1
constant of the reaction would be – (C) 3.465 × 103 min1, 2.17 × 103 mol.litre1 min1
(A) 10 min–1 (B) 6.931 min–1 (D) 2.166 × 103 min1, 2.667 × 104 mol.litre1 min1
–`1
(C) 0.6931 min (D) 0.06931 min–1
IV. Methods to determine the rate law
30. In a certain reaction, 10% of the reactant decomposes
in one hour, 20 % in two hours, 30% in three hours and 38. The data for the reaction A + B  C is
so on the dimensions of the rate constant is : Exp. [A]0 [B]0 initial rate
(A) hour–1 (B) mole litre–1 sec–1 1 0.012 0.035 0.10
(C) litre mole–1 sec–1 (D) mole sec–1 2 0.024 0.035 0.80
3 0.012 0.070 0.10
31. If a first order reaction is completed to the extent of 75% 4 0.024 0.070 0.80
and 50% in time intervals, t1 and t2, what is the ratio, t1 : (A) r = k [B]3 (B) r = k [A]3
4
t2 ? (C) r = k [A] [B] (D) r = k [A]2 [B]2 .

ln (3 / 4)
(A) ln 2 (B) dx
ln 2 39. A + B  Product, =k [A]a [B]b
dt
(C) 2 (D) 1/2
 dx 
32. The rate constant for the reaction A  B is 2 × 10–4 It. mol– If   = k, then order is :
1  dt 
min–1. The concentration of A at which rate of the reaction
is (1/12) × 10–5 M sec–1 is : (A) 4 (B) 2
(C) 1 (D) 0
(A) 0.25 M (B) (1/20) 5 / 3 M
(C) 0.5 M (D) None of these
 dx   dx 
40. A  Product and  dt  = k[A]2 . If log  dt  is plotted
33. What will be the order of reaction and rate constant for a    
chemical change having log t50% vs log concentration of against log [A],then graph is of the type :
(A) curves as :

(A) (B)
(A) 0, 1/2 (B) 1, 1
(C) 2, 2 (D) 3, 1

ln 2
34. The rate constant for a reaction is min–1. What will
10
be the order of reaction and time taken to change (C) (D)
concentration from 1 M to 0.25 M.
(A) one, 10 min (B) zero, 10 min
(C) one, 20 min (D) two, 20 min
d [A ]
35. In the first order reaction 75% of the reactant disappeared 41. For the reaction A  Products, – = k and at
dt
in 1.388 hrs. Calculate the rate constant of the reaction : different time interval, [A] values are :
(A) 1 s–1 (B) 2.8  10–4 s–1
–3 –1
(C) 17.2  10 s (D) 1.8  10–3 s–1
Time 0 5 min 10 min 15 min
36. In the case of zero order reaction, the ratio of time required [A] 20 mol 18 mol 16 mol 14 mol
for 75% completion to 50% completion is : At 20 minute, rate will be :
(A) ln 2 (B) 2 (A) 12 mol /min (B) 10 mol/min
(C) 1.5 (D) None (C) 8 mol/min (D) 0.4 mol/min

PAGE # 188
42. The rate law for a reaction A + B  product is rate = 48. The rate constant, the activation energy and the frequency
K[A]1[B]2. Then which one of the following statement is factor of a chemical reaction at 25°C are 3.0 × 10–4 s–1,
false : 104.4 KJ mol–1 and 6.0 × 1014 s–1 respectively. The value
(A) If [B] is held constant while [A] is doubled, the reaction of the rate constant as T   is :
will proceed twice as fast (A) 2.0 × 1018 s–1 (B) 6.0 × 1014 s–1
(B) If [A] is held constant while [B] is reduced to one (C)  (D) 3.6 × 1030 s–1
quater, the rate will be halved 49. According to collision theory of reaction rates –
(C) If [A] and [B] are both doubled, the reaction will (A) Every collision between reactant leads to chemical
proceed 8 times as fast reaction
(D) This is a third order reaction (B) Rate of reaction is proportional to velocity of
molecules
43. The conversion of vinyl allyl ether to pent-4-enol follows (C) All reactions which occur in gaseous phase are zero
a certain kinetics. The following plot is obtained for such order reaction
a reaction. (D) Rate of reaction is directly proportional to collision
frequency.
50. Activation energy of a reaction is –
(A) The energy released during the reaction
(B) The energy evolved when activated complex is formed
(C) Minimum amount of energy needed to overcome the
potential barrier of reaction
(D) The energy needed to form one mole of the product.
51. The minimum energy for molecules to enter into
The order for the reaction is chemical reaction is called.
(A) zero (B) –1 (A) Kinetic energy (B) Potential energy
(C) 1 (D) 2 (C) Threshold energy (D) Activation energy

44. The reaction A(s)  2 B(g) + C(g) is first order. The 52. The activation energy for the forward reaction X  Y is
pressure after 20 min. and after very long time are 150 60 KJ mol–1 and H is –20 KJ mol–1. The activation energy
mm Hg and 225 mm Hg. The value of rate constant and for the backard reaction Y  X is :
pressure after 40 min. are : (A) 80 KJ mol–1 (B) 40 KJ mol–1
–1
(A) 0.05 In 1.5 min–1 ,200 mm (C) 60 KJ mol (D) 20 KJ mol–1
(B) 0.5 ln 2 min–1 ,300 mm
(C) 0.05 In 3 min–1, 300 mm 53. For producing the effective collisions, the colloiding
(D) 0.05 In 3 min–1 , 200 mm molecules must posses :
(A) A certain minimum amount of energy
V. Effect of Temperature (B) Energy equal to or greater than threshold energy
(C) Proper orientation
45. Rate of which reactions increases with temperature : (D) Threshold energy as well as proper orientation of
(A) of any collision.
(B) of exothermic reactions
(C) of endothermic
54. In a gaseous state reaction, A2 (g)  B(g) + (1/2)C
(D) of none.
(g), The increase in pressure from 100 mm to 120 mm
46. For a zero order reaction. W hich of the following is notices in 5 minutes. The average rate of disapearance
statement is false : of A2 in mm min–1 is :
(A) the rate is independent of the temperature of the (A) 4 (B) 8
reaction. (C) 16 (D) 2
(B) the rate is independent of the concentration of the
reactants.
(C) the half life depends as the concentration of the EXERCISE-2
reactants.
(D) the rate constant has the unit mole It–1 sec–1. 1. For the reaction, 2A + B  C which relationship is
correct ? [IJSO Stage-1/2013-14]
47. A large increase in the rate of a reaction for a rise in (A) [A] = [C] (B) –[A] = [C]
temperature is due to (C) –2[A] = [C] (D) –[A] = 2[C]
(A) increase in the number of collisions
(B) the increase in the number of activated molecules 2. For a first order reaction, the ratio of the times taken for
(C) The shortening of mean free path completion of 99.9% and 50% of the reaction is
(D) the lowering of activation energy [IJSO Stage-1/2014-15]
(A) 8 (B) 9
(C) 12 (D) 10

PAGE # 189
3. The following data was recorded for the reaction
A + B  Product at 298 K.

Experiment No. [A] [B] Rate of reaction


1 1.00M 0.15M 4.20  10 – 3
2 2.00M 0.15M 8.40  10 – 3
3 1.00M 0.30M 8.40  10 – 3

From the above data one can conclude that


[IJSO Stage-1/2014-15]
(A) Rate  [A]2[B] (B) Rate  [A][B]2
2 2
(C) Rate  [A] [B] (D) Rate  [A][B]

PAGE # 190
NUCLEAR CHEMISTRY

(b) On the basis of stability :


INTRODUCTION (i) Stable nuclei : Those nuclei which are permanent
& their proton and neutron contents remain
Atoms have three fundamental particles that are unchanged forever (can be changed only under severe
electrons, protons and neutrons. Protons and conditions of bombardment by external radition) are
neutrons are present inside the nucleus and electrons called stable nuclei.
are present in the extranuclear region. Changes
(ii) Unstable nuclei : Those nuclei in which no. of
occurring in the nucleus which are a source of
protons and neutrons change with time.
tremendous energy are called nuclear reactions. The
branch of science which deals with the study of atomic
nucleus and nuclear changes is called nuclear
STABILITY OF NUCLEUS
chemistry. Stability of a nucleus can be explained by following
Nucleus term was firstly introduced by Rutherford . It is two factors.
defined as the central part of an atom which contain (a) On the basis of (n/p) ratio :
all the protons and neutrons.
Neutrons help to hold protons together within the
rn  A1/ 3  rn = RoA1/3 nucleus. The number of neutrons necessary to create
a stable nucleus increases rapidly as the number of
Ro = 1.2 × 10–15 m protons increases. The number of neutron to proton
where : A =mass number, ratio (n/p) of stable nuclei increases with increasing
rn = radius of nucleus atomic number . The area of graph in which all stable
R0 = Rutherford constant nuclei are found is known as the belt of stability.
Density of nucleus = 1017 kg/m3 Radioactive nuclei occur outside this belt.

(a) On the basis of their Z and n values :


(i) Isotopes : The isotopes of an element have the
same atomic number but different atomic masses
due to the presence of different number of neutrons.
1 2 3 22 23 24
e.g. 1H , 1 H , 1 H ; 11 Na , 11 Na , 11 Na

(ii) Isobars : The atoms of different elements with


different atomic numbers, but same mass number
are called isobars.
e.g. 14 , 14 ; 40 and 40 are isobars.
6 C 7N 20 Ca 18 Ar
(iii) Isotones : The isotones may be defined as the
atoms of different elements containing same
number of neutrons.
3
e.g. 13
6C
and 14 ;
7N 1
H and 24 He
A1 A2 The type of radioactive decay that a particular radio
(iv) Mirror nuclei : z1 X & z2 Y are mirror nuclei if
isotope will undergo depends to a large extent on its
Z1 = n2 and Z2 = n1 neutrons to protons ratio compared to those of nearby
Z1 = A1 – n1 = n2 ..... (1) nuclei that are within the belt of stability.
Z2 = A2 – n2 = n1 ...... (2)
(i) A nucleus whose high n/p ratio places it above the
from equation(1) n1 = A1 – n2
belt of stability emits a -particle in order to lower n/p
by putting the value of n1 in equation (2) ratio and move towards the belt of stability.
A2 – n 2 = A1 – n 2 1
0n 11 p  –10 e  –
A1 = A2
So these are defined as those isobars in which the ( – is Anti neutrino)
Z and n values are interchanged
 Note :
e.g. 13 H and 32 He , 13 13
6 C and 7 N Antineutrino is the antiparticle of neutrino, which is
neutral particle produced in nuclear beta decay.

PAGE # 191
(ii) A nucleus which has lower n/p ratio , is placed below Radioactivity is a nuclear phenomenon i.e., the kind
the belt of stability either emits positrons or undergoes of intensity of the radiation emitted by any radioactive
electron capture. Both modes of decay decrease the substance is absolutely the same whether the
number of protons and increase the number of neutrons element is present as such or in any one of its
in the nucleus and thus, positron emission or electron compounds.
capture results in an increase in n/p ratio. e.g. Elements like uranium (U) , thorium (Th) ,
e.g. polonium (Po), radium (Ra) etc. are radioactive in
nature.
1 1
1P  0 n  01 e  
(a) History of the Discovery of Radioactivity :
(Positron emission)
In 1895, Henri Becquerel was studying the effect of
1 1
 1p  –10 e  0 n  X  ray sunlight on various phosphorescent minerals, one
of the substance being studied was uranium ore.
(Electron capture)
He accidently left a crystal of uranium sample ;
 Note : Potassium uranyl sulphate [K2UO2 (SO4)2. 2H2O] in
A positron has same mass as electron but carries a drawer along with some photographic plate
opposite charge. The positron has a very short life wrapped in black paper. Much to his surprise, he
because it is annihilated when it collides with discovered that the photographic plate had been
an electron, producing gamma rays. This phenomenon fogged by exposure to some invisible radiations
is known as pair production. from uranium. He called this mysterious property of
the ore as ‘radioactivity’ (Radioactivity means ray-
(iii) The nuclei with atomic number > 83, outside the emitting activity). A year later, in 1896, Marie Curie
belt of stability, undergo -emissions. Emission of an found that besides uranium and its compounds,
-particle decreases both the number of protons and thorium was another element which possessed the
neutrons and thereby increases n/p ratio. property of radioactivity. 1898 Marie Curie and her
Thus, husband Pierrie Curie isolated two new radioactive
elements polonium and radium.
For lighter element if Z = n then nuclei will be stable
For nuclie Z > 20, stability condition is 1.6  n/p > 1 (b) Natural & Artificial Radioactivity :
Unstable nuclei emit radiations to achieve stability, this
property is known as radioactivity. If a substance emits radiations by itself it possesses

(b) On the basis of even and odd nature of the natural radioactivity but if a substance does not
number of protons and neutrons : possess radioactivity and starts emitting radiations
(i) The number of stable nuclides is maximum when on exposure to rays from a natural radioactive
both Z and n are even numbers. About 60% of stable
substance, it is called induced or artificial
nuclides have both Z and n even.
radioactivity.
(ii) The number of stable nuclides in which either the Z
or n is odd is about one third of those, where both are e.g.
even. When aluminium is bombarded with - particles , a

MAGIC NUMBERS radioactive isotope of phosphorus is formed which


disintegrate spontaneously with the emission of
Just as certain numbers of electrons (2,8,18,36,54 and
positrons (which are positively charged electron, e0).
+1
86) correspond to stable closed shell electron
configuration, certain number of nucleons leads to
closed shell in nuclei. The protons and neutrons can
achieve closed shell. Nuclei with 2, 8, 20, 28, 50 or 82
protons or 2,8,20, 28, 50, 82, or 126 neutrons
correspond to closed nuclear shell. Closed shell nuclei
are more stable than those that do not have closed
shells. These numbers of nucleons that correspond to
closed nuclear shells are called magic numbers.

 Note :
RADIOACTIVITY
Natural radioactivity was discovered by Becquerel
Radioactivity is a process in which nuclei of certain
elements undergo spontaneous disintegration without while artificial radioactivity was discovered by Irene
excitation by any external means. Curie and Joliot.
All heavy elements from bismuth (Bi) to uranium and a
few of lighter elements have naturally occurring (c ) Analys is of Radioa ctive Radiat ions :
isotopes which possess the property of radioactivity.
All those substances which have the tendency to emit In 1904, Rutherford and his co-workers observed
these radiations are termed as radioactive materials. that when radioactive radiations were subjected to
a magnetic field or a strong electric field, these were

PAGE # 192
split into three types, as shown in the figure. The rays -rays -rays
+

-
which are attracted towards the negative plate, are

r ay
+

s
+  -rays
positively charged , and called alpha () rays. The +
rays which are deflected towards the positive plate +
ys
ra +
-
are negatively charged and are called beta () rays. +
-rays

+
The third type of rays which are not deflected on any + Magnetic
field
side but move straight are known as gamma () rays.
Radioactive
substances

(A) (B)
Figure :
(A) Deflection of radioactive rays in electric field and,
(B) Emission of radioactive rays and their deflection
in a magnetic field. (The direction of magnetic field is
inward perpendicular to the page).

The important properties of- rays ,  rays and -rays are as follows :

is called parent element and the new element formed


( d ) Units of Radioactivity : is called daughter element.
(i) SI unit is Becquerel (Bq) which is defined as (i) When an _ particle is emitted, the new element
disintegration per sec (dps). formed is displaced two positions to the left in the
(ii) Earlier radioactivity was given in terms of Curie periodic table than that of parent element (because
(Ci). the atomic number decreases by 2).
238
1 Ci refers to the activity of Radium. e.g. 92 U  90 Th234  2 He 4
1 Ci = 3.7 × 1010 dps = 3.7 × 1010 Bq.
1 Milli Ci = 3.7 × 107 Bq. (ii) W hen a  _ particle is emitted the new element
1 Micro Ci = 3.7 × 104 Bq. formed is displaced one position to the right in the
periodic table than that of parent element (because
(iii) Another unit is Rutherford (Rd).
the atomic number increases by 1).
1 Rd = 106 dps e.g.
The emission of  _ particle by 6 C 14 may be
represented as follows:
These laws were given by Soddy,Fajans and Russel
6
C 14  N14 +-1e0
7
(1911_1913).The element emitting the  or particle

PAGE # 193
(a) Explanation : 16 – 10
=6
The results of the group displacement laws may be
Thus, no. of -particles emitted out = 8
explained as follows:
No. of -particles emitted out = 6
Since an  _ particle is simply a helium nucleus Ex.3 90Th234 disintegrates to give 82Pb206 as final product .
(containing two neutrons and two protons) therefore, How many alpha and beta particles are emitted during
loss of _ particle means loss of two neutrons and this process ?
two protons . Thus, the new element formed has Sol. Suppose the no. of  particles emitted = x and no. of
atomic number less by 2 unit and mass number less _ particles emitted = y.
by 4 unit. Then
The _ particle is simply an electron and there are no Th234  Pb206 + x 2He4 + y-1e0
90 82
electrons present in nucleus .However , the loss of _
particle is also found to be a nuclear phenomenon Equating the mass number on both sides ,we get
because the change in external conditions 234 = 206 + 4x + 0y
or 4x = 28 or x=7
(temperature etc.) has no effect on the rate of the
Equating the atomic number on both sides ,we get
emission of _ particle. It is therefore, believed that 90=82+2x-y y=6
for emission of _ particle to occur, a neutron changes Ans. 7 and 6  particles will be emitted.
to a proton and an electron i.e.
Alternative Method:
Neutron  Proton + Electron (_ particle)
No. of -particles =
As a result ,the number of protons in the nucleus
increases by 1 and so does the atomic number. Difference in atomic mass of reactants and products

 Note : 4
234 – 206 28
(i) Increase or decrease in the number of protons in = = =7
4 4
the nucleus (due to loss of  particle or _ particle)
is accompanied simultaneously by the loss or gain No. of -particle=( 2 × no. of -particles) – (ZA – ZB)
of electrons in the extranuclear part (from the where , ZA = Atomic number of reactant
surroundings) so that the electrical neutrality is ZB = Atomic number of product
= (2 × 7) – (90 – 82) = 14 – 8 = 6
maintained in the new atom formed.
Ans.7 and 6  particles will be emitted.
(ii) _ decay produces isodiaphers i.e.parent and the
daughter nuclides have same isotopic mass (which RATE OF RADIOACTIVE DECAY
is the difference between number of neutrons and
protons) . Radioactive disintegration is an example of first order
238
E.g. 92 U  90 Th234  2 He4 reaction, i.e., the rate of decay is directly proportional
to the no. of atoms (amount) of the element present
No. of neutrons: 146 144
No. of protons: 92 90 at the particular time.
Difference 54 54 A  Decay product
(iii)  decay produces isobars i.e.parent and the
_ No. of atoms at t = 0  N0
daughter nuclides have different atomic numbers but No. of atoms left after t = t  N
same mass number .
E.g. Hence, rate  [N 0 – N]/t because rate continuously
decreases with time. Let dN be the change in no. of
6
C 14
 14
N +-1e
7
0
atoms in an infinitesimal small time dt, then rate of
(iv) Emission of 1  and 2  particles produces an decay can be written as -
isotope of parent element. dN
e.g. –   [N]1 = N. The negative sign indicates the
   dt
92
U238   91Pa234 
 90Th 234   92U234
decreasing trend of N with increasing time.
 Note : where  is the proportionality constant.
The emission of  and _ particles is also known as
Integration of this equation finally gives -
_decay and _ decay.
2.303 N0
Ex-2 Calculate number of  and  - particles emitted when or  = log10
t N
238
92 U changes into radioactive 206
82 Pb .
 Note :  is also known as decay or disintegration or
Change in mass number
Sol. No. of -particles = radioactive constant.
4
238 – 206 32
= = =8
4 4 CHARACTERISTICS OF RATE OF DISINTEGRATION
No. of -particles = 2 × -particles - (ZA – ZB)
(i) Rate of disintegration continuously decreases with
Here ZA and ZB are atomic no. of parent and daugther time.
nuclei respectively.
= 2 × 8 – (92 – 82) (ii) Rate of disintegration as well as  are independent
of P and T.

PAGE # 194
(iii) (a) Unit of rate of decay : disintegration per time  1n
(b) Unit of decay constant : time–1 N = No  
(iv) Time required to complete a definite fraction is 2
independent of initial no. of atoms (amount) of 1
According to question , N = N
radioactive species. 8 0
1  1n
HALF-LIFE PERIOD or N0 = No  
8 2
The time required for the decay of radioactive element 1 1
or 3 = n  n = 3
to half of the original amount is called half-life period. ( 2) (2)
(a) Characteristics of Half-Life Period : So, time taken by the sample to reduce to 1/8th of its
• It is denoted by t1/2 . reactivity will be -

• Each radioactive element has a characteristic half- T= n×t½


life period .
T= 3×27.96
• Half-life period for an element is a constant.
= 83.88 days
0.693
• t1/2 = Ex.6 Half- life period of a radioactive element is 100

seconds. Calculate the disintegration constant.
W here  is a constant known as disintegration
constant or decay constant. It is the characteristic of Sol. t1/2 = 100 Seconds,
the nature of the radioactive element.
0.693 0.693
 Note :  = = = 0.00693 s–1
t 100
Half-life period does not depend upon initial amount 1/2
of element. = 6.93 × 10–3 sec–1
(b) Significance of Half-Life Period :
AVERAGE LIFE
(i) Stability of nuclei : The value of half-life period can
give an idea about relative stability of radio isotopes. Evidently , the whole of the radioactive element can
All isotopes with longer t1/2 are more stable. never disintegrate or in other words
, the time required for the disintegration of the whole of
(ii) The amount of substance left after ‘n’ number of
a radioactive element will be infinity.
half lives can be given as :- Thus, it is meaningless to talk of the total life of a
 1n radioactive element . However, sometimes another
N = No   term is used ,called average life ( ) which is the
2
reciprocal of the disintegration constant () i.e.
Where ;
1 t
N = Amount of the substance left after ‘n’ half-lives. Average life ()= ½ = 1.44 t .
 0.693 1/2

No = Initial amount of the substance.

Ex.4 The half-life period of 53I125 is 60 days .What percent ISODIAPHERS


of the original radioactivity would be present after 180 (i) Atoms having the same difference of neutrons and
days ? proton.
Sol. t½ = 60 days, t = 180 days
(ii) Nuclide and its decay product after -emission
Total time( t ) 180 are called isodiaphers.
n= = =3
Half  life peiod( t ½ ) 60 m m–4
(iii) e.g., Z A Z–2 B
N0
Applying the formula N= p=Z p=Z–2
2n n=m–Z n=m–Z–2
N0 1 n – p = m – 2Z n – p = m – 2Z
we get N = n = × 100 = 12.5%
2 8  Note :
Ex.5 The half -life period of a radioactive element is 27.96 Isotopic no. n – p = m – 2Z
days . Calculate the time taken by a given sample to
reduce to 1/8th of its activity.
ISOSTERS
Sol. The amount of substance left after ‘n’ number of half
lives can be given as :- (i) Molecules having same no. of atoms and same
no. of electrons are called isosters.
e.g., CO 2 and N 2O (There are three atoms and 22
electrons in both the molecules.)

PAGE # 195
235 1 144 90 1
NUCLEAR ISOMERS 92 U + 0n  56 Ba
+ 36 Kr + 20 n

235
.118
1 .009

 143
 .881
 89
 .947
 2.018
 
(i) Nuclides having identical atomic no. and mass
236 .127 amu 235.846
no. but differing in radioactive properties are known
as nuclear isomers.  m = 236.127 – 235.846
= 0.281 amu
(ii) Nuclear isomers differ in their energy state and
spins. E(in MeV) = 0.281 × 931.5 = 261.75 MeV
60 60m 69 69m 80 80m
e.g., Co and Co, Zn and Zn, Br and Br etc. Energy released in one fission is equal to 261.75
MeV.
The symbol m with mass no. represents the
metastable state of parent element. (i) Chain reaction : Whatever are the primary products
Isomeric
60m
Co Co + -rays
60 of fission of uranium, it is certain that neutrons are
Transition
set free.If the conditions are so arranged that each of
(iii) Nuclear isomers, thus have different rate of decay, these neutrons can, in turn, bring about the fission,
decay constant, half life, average life and binding the number of neutrons will increase at a continuously
energy. accelerating rate until whole of the material is
exhausted. Such type of reaction is called chain
 Note :
reaction. It takes very small time and is uncontrolled.
In a metastable state, a system is in equilibrium (not It ends in terrible explosion due to release of
changing with time),but is susceptible to fall into lower enormous amount of energy.
energy states with only slight interaction. 235 1
92 U + 0 n   141 92 1
56 Ba  36 Kr  3 0 n + Energy
NUCLEAR REACTIONS The chain reaction is shown in the figure .
Ba
The reactions in which nuclei of atoms interact with n
other nuclei or elementary particles such as alpha 235 n
92 U n
Ba
particle, proton, deutron, neutron etc. resulting in the n
E Kr
formation of a new nucleus and one or more
235 n
elementary particles are called nuclear reactions. 92U Ba
Ba n
Nuclear reactions are expressed in the same fashion n
E n 235 n
U
as chemical reactions. In a nuclear reaction ,atomic Kr
92
n
n n
number and mass number are conserved. U235
92 E Kr
e.g. the nuclear reaction : n Ba Ba
E n
14 4 Kr n n
 178 O  11H
235
U
7 N  2He 
92 n
U235
92 n
E
(a ) Nuclear Fission : Kr
E n
Kr Ba
The process of artificial transmutation in which heavy n
n
nucleus is broken down into two lighter nuclei of nearly 92U
235
n
comparable masses with release of large amount of
E Kr
energy is termed as nuclear fission.
e.g. (ii) Critical mass : The minimum mass which the
238 239 239 fissionable material must have so that one of the
(i) 92 U is converted into 93 Np and 94 Pu
neutrons released in every fission hits another
238 239 239 nucleus and causes fission so that the chain reaction
92 U 10 n 
– –
 93 Np 
 94 Pu continues at a constant rate is called critical mass .If
Uranium Neptunium Plutonium the mass is less than the critical mass , it is called
235 sub-critical mass . If the mass is more than critical
(ii) 92 U captures slow neutron and splits up into
mass, it is called super-critical mass.
fragments.
(iii) Applications of Nuclear Fission : Three practical
235 1 236 144 90 1
92 U + 0 n  92 U  56 Ba 36 Kr 2 0 n + Energy applications of nuclear fission are as follows -
Unstable (A) Atomic bomb (B) Nuclear reactor
During fission, there is always loss of mass, known (C) Nuclear power plants
as mass defect ,which is converted into energy (A) Atomic Bomb :
according to Einstein equation i.e.
• The basic principle of atomic bomb is uncontrolled
E = mc2.
nuclear fission reaction (chain reaction).
e.g.
• It requires several small samples of U-235 or Pu-239.

PAGE # 196
• An explosive like TNT (Trinitrotoluene) is placed 1. Reactor core
behind the samples which explodes to initiate the 2. Heat exchanger
reaction which causes the small samples to join and 3. Steam turbine
form large mass.
4. Steam condensing system
• Neutron from Ra-Ba source (s) initiate the reaction
Reactor core is the main part of nuclear reactor. It
which starts the chain reaction finally leading to
explosion and release of large amount of energy. consists of the following parts :
• The rapid release of energy raises the temperature Fuel rod :
enormously and generates a very high pressure front The fissionable material used in the reactor is called
in the atmosphere.
fuel. The fuel used is enriched uranium -235 . This is
obtained from the naturally occurring U-235
(containing about 0.7% of U-235 ) by raising the
percentage of U-235 to about 2-3%.

• Control rods : Cadmium or boron rods are used to


Atomic Bomb
 Note : raise or lower and control the fission process.
Because they can absorb neutrons.
The first atomic bomb dropped over Hiroshima city
during the second world war in 1945 utilized 235U and • Moderator : The material used to slow down the
the second atomic bomb dropped on Nagasaki neutrons (without absorbing them so that they can
made use of 239Pu. India exploded her first atomic be easily captured by the fuel, is known as moderator.
bomb at Rajasthan in May 1974,and used 239Pu as Heavy water (D2O) or graphite is used as moderator
the fissionable material. material in nuclear power plant.
(B) Nuclear reactor :
• Coolant : To carry away the heat produced during
• An equipment in which nuclear chain reaction is fission, a liquid is used. This liquid is known as coolant.
carried out in a controlled manner is called a nuclear Usually heavy water is used as coolant so that it also
reactor. acts as a moderator.
• The energy thus liberated can be used for • Shield : To prevent the losses of heat and to protect
constructive purposes like generation of steam to run the persons operating the reactor from the radiation
turbines and produce electricity. and heat, the entire reactor core is enclosed, in a
• In a nuclear reactor, fission is controlled by controlling heavy steel or concrete dome, called the shield.
the number of neutrons released. Steam

• In a nuclear reactor, fission is based on the fact that Electricity


Generator
Turbine
cadmium and boron can absorb neutrons thus
forming corresponding isotopes which are not
radioactive.

113 1
 114
48 Cd  0n  48 Cd    rays Reactor Primary
Heat D2O
exchanger
coolant
10 1
 115B 
5 B  0n    rays Condenser

 Note :
D2O

The first nuclear reactor was assembled by Fermi Pump


and his coworkers at the University of Chicago in the
United states of America, in 1942. In India, the first Heavy water

nuclear reactor was put into operation at Trombay Nuclear Reactor


(Mumbai) in 1956.
(b) Nuclear Fusion :
(C ) Nuclear Power Plants :
A nuclear reaction in which two lighter nuclei are fused
When a nuclear reactor is used for the production of together to form a heavier nucleus is called nuclear
electricity it is termed as a nuclear power plant.The fusion. A fusion reaction is difficult to occur because
positively charged nuclei repel each other. At very high
heat produced during a nuclear reaction is utilized in
temperature of the order of 106 to 107 K, the nuclei
generating steam which runs the steam turbines. The
may have sufficient energy to overcome the repulsive
electric generator is connected to the turbine. The forces and fuse.Therefore, fusion reactions are also
electric power is then obtained from the generator. called thermonuclear reactions. Fusion reaction are
Thus, a nuclear power plant consists essentially of highly exothermic in nature because loss of mass
the following four parts: occurs when heavier nucleus is formed from the two

PAGE # 197
lighter nuclei.
2 2 4
1 H 1 H 2 He  23.85 MeV
3 3 4
1 H 1 H 2 He  210 n  11.3 MeV
1 3 4
1H 1 H 2 He  20 .0 MeV
4 note that the first two reactions should occur twice to
2 3
1 H 1 H  2 He 10 n  17.6 MeV 3
produce two 2 He
nuclei and initiate the third reaction.
Hydrogen bomb is based on fusion reaction. Energy
released is so enormous that it is about 1000 times As a result of this cycle, effectively, four hydrogen nuclei
that of an atomic bomb. combine to form a helium nucleus. About 26.7 MeV
It is believed that the high temperatures of stars energy is released in the cycle. Thus, hydrogen is the
including the sun is due to fusion reactions. fuel which ‘burns’ into helium to release energy. The
sun is estimated to have been radiating energy for
(i) Applications of Nuclear Fusion :
the last 3.5 × 109 years and will continue to do so till
(A) Hydrogen bomb : all the hydrogen in it is used up. It is estimated that
• Its principle is nuclear fusion. the present store of hydrogen in the sun is sufficient
• It consists of an arrangement of nuclear fission in for the next 5 × 109 years.
the centre surrounded by a mixture of deuterium (12 H) In hotter stars where the temperature is  10 8 K,
another cycle known as CNO (Carbon-nitrogen-
and lithium isotopes ( 36 Li) .
oxygen cycle) cycle takes places.
• The nuclear fission provides heat and neutrons.
6
• Neutrons convert to tritium 13 H and the heat
3 Li 1 13
1H  12
6 C  7 N   + 1.95 MeV
liberated is used for fusion between 12 H & 13 H .
• The reactions occurring are : 13 13 C  0e   + 2.22 MeV
7 N  6 1
Fission (in the centre)  heat + 10 n 1
1H 13
6 C
 14
7 N  + 7.54 MeV
6 1  3 4
3 Li + 0n 1H + 2 He + 3.78 MeV
2
1
1H 14
7 N
 158 O   + 7.35 MeV
1H + 13 H  24He + 10n + 17.6 MeV
15 15 0
2 8O  7 N + 1 e ++ 2.75 MeV
1H + 12 H  32 He + 10n + 3.2 MeV
1
3
1H + 13 H  24 He + 2 10n + 13.14 MeV 1H 15
7 N 
12 4
 6 C  2 He + 4.96 MeV

(B) Fusion in sun : Among the celestial bodies in which


energy is produced, the sun is relatively cooler. There 411H  24He  2 1e0   rays + 26.7 MeV
are stars with temperature around 10 8 K inside. In
sun and other stars, where the temperature is less The end result of this cycle is again the fusion of four
than or around 107 K, fusion takes place dominantly hydrogen nuclei into a helium nucleus. Carbon
by proton-proton cycle as follows - nucleus acts only as a catalyst.

DIFFERENCES BETWEEN NUCLEAR FISSION AND NUCLEAR FUSION

S.No. Nuclear fission Nuclear fusion

1 This process occurs in heavy nuclei. This process occurs in lighter nuclei.

The heavy nucleus splits into lighter nuclei The lighter nuclei fuse together to form a
2
of comparable masses. heavy nucleus.

3 The reaction occurs at ordinary temperature. This occurs at very high temperature.

The energy liberated in one fission is about The energy liberated in one fusion is about
4
200 MeV. 24 MeV.

5 This can be controlled. This cannot be controlled.

Products of fission are usually unstable Products of fusion are usually stable and
6
and radioactive in nature. non-radioactive in nature.

7 The links of fission reactions are neutrons. The links of fusion reactions are protons.

PAGE # 198
DIFFERENCES BETWEEN NUCLEAR REACTIONS AND CHEMICAL REACTIONS

Some of the characteristics that differentiate between nuclear reactions and ordinary chemical reactions are
summarized ahead :
Nuclear reactions Chemical reactions

Involve conversion of one nuclide into Involve rearrangement of atoms and not
another. change in the nucleus.

Particles within the nucleus are


Only outermost electrons participate.
involved .

Often accompanied by release Accompanied by release or absorption of


of tremendous amount of energy. relatively small amount of energy.

Rate of reaction is independent of


Rate of reaction is influenced by external
external factors such as temperature,
factors.
pressure and catalyst.
No breaking or making of bonds
Involves breaking or making of bonds.
involved.
Irreversible. Can be reversible or irreversible.

APPLICATIONS OF RADIOACTIVITY DATING


AND RADIOISOTOPES
(i) The determination of age of minerals and rocks,
(a ) In Medicine : an important part of geological studies involves
determination of either a species formed during a
Radioisotopes are used to diagnose many diseases. radioactive decay or the residual activity of an isotope
E.g. arsenic - 74 tracer is used to detect the presence which is undergoing decay. For example
of tumour, sodium -24 tracer is used to detect the 238 undergoes
presence of blood clots , iodine-131 tracer is used to 92 U
a decay (t1/2 =4.5 × 109 years) series
study the activity of the thyroid glands and cobalt-60 forming a stable isotope 206
82 Pb and He. Helium
is used in the treatment of cancer . It should be noted
that the radioactive isotopes used in medicine have obtained as a result of decay of 238
92 U
has almost
very short half life periods. certainly been formed from  -particles. Thus, if
238 and He contents are known in a rock we can
(b) In Agriculture : 92 U
The use of radioactive phosphorus 32P in fertilizers has
revealed how phosphorus is absorbed by plants. This determine the age of rock sample (1g of 238 92 U
in
study has led to an improvement in the preparation of equilibrium with its decay products produces about
fertilizers. 14C is used to study the kinetics of 10 –7 g He in a year). Also by assuming that initially
photosynthesis. rock does not contain 206
82 Pb and it is present in rocks
(c ) In Industry :
due to decay of 238
92 U
, we can calculate the age of
(i) The thickness of a material (e.g. cigarettes, metal rocks and minerals by measuring the ratio of
plates etc.) can be determined by placing a radioactive
source on one side of the material and a counting 238 and 206 .The amount of 206
92 U 82 Pb 82 Pb is supposed
device on the other. From the amount of radiation
238
reaching the counter, the thickness of the material to be obtained by decay of U . Thus,
can be calculated.
206
(ii) When a single pipe line is used to transfer more 238
92 U  82 Pb + 8 42 He + 6 –01e
than one petroleum derivative, a small amount of 238
radioactive isotope is placed in last portion of one Mole of U left = N at time t i.e., Nt
substance to signal its end and the start of another. Mole of
206
Pb formed = N’ at time t
(d) In Geological Dating : 238
The age of the earth and rocks can be predicted by  Initial mole of U = N + N’ (at time 0) i.e., (N0)
geological dating. Age of a rock sample can be Thus, time t can be evaluated by-
calculated by finding out the amounts of the parent
radioactive element and the isotope of lead (e.g. 2.303 N0
t= log N
238 206  t
92 U and 82 Pb ) in rock sample.
(ii) To determine the age of animals or objects of
(e ) In Radio Carbon Dating : vegetable origin such as wood, charcoal and textiles
The age of a fallen tree or dead animal can be by radio carbon dating technique.
predicted by measuring the amount of C-14 in dead
plants or animals.  Note :
Radio carbon dating technique was given by W.F.
Libby and was awarded Nobel Prize.

PAGE # 199
Carbon-14 has been used to determine the age of radiation pollution.
organic material. The procedure is based on the
(a) Effects of Radioactive Pollution :
formation of 14C by neutron capture in the upper
atmosphere. (i) Radiations induce mutations and breaks in
14 1 14 1 chromosomes particularly at the time of cell division.
7 N + 0n
 6 C + 1H
(ii) Higher doses of radiations can cause cancer,
This reaction provides a small, but reasonably leukaemia, anaemia and sterility. Excessive use of X-
constant source of 14 C . The 14 C isotope is rays causes death of tissues.
radioactive, undergoing  - decay with a half life of (iii) Radiations induce mutations in plants also.
5730 years. Morphological deformities occur.
14  14 0 (b) Control of Radiation Pollution :
6C 7 N + –1e
In using radio carbon dating, we generally assume (i) Manufacture and use of nuclear weapons should
that the ratio of 14C to 12C in the atmosphere has been be stopped.
constant for at least 50,000 years. The 14 C is
(ii) Nuclear tests and further development should be
incorporated into CO2, which is in turn incorporated
suspended.
through photosynthesis, into more complex carbon
containing molecules within plants. W hen these (iii) Ocean dumping of nuclear wastes should be
plants are eaten by animals, the 14 C becomes suspended.
incorporated within them. Because a living plant or (iii) Proper handling of radio isotopes during their use
animal has a constant intake of carbon compounds, in various fields should be done.
it also has to maintain 14C to 12C ratio that is identical
with that of atmosphere . However, once the organism
dies, it no longer ingests carbon compounds to
compensate 14C which is lost through radioactive
EXERCISE-1
decay. The ratio of 14 C to 12 C therefore,
decreases.Thus, by knowing the equilibrium I. Types of nucleus
concentration of 14C in a living matter as well as in a
dead piece of matter at a particular time, the age of 1. Radioactivity is due to -
material can be determined. (A) stable electronic configuration.
(B) unstable electronic configuration.
HAZARDS OF RADIATIONS (C) stable nucleus .
(D) unstable nucleus.
(i) Radioactive radiations cause atmospheric
pollution. 2. The activity of radioisotope changes with -
(ii) When living organisms are exposed to radiations, (A) temperature
the complex organic molecules get ionized, break up (B) pressure
and disrupt the normal functioning of the organisms.
(C) chemical environment
(iii) Effects of radiations : (D) none of the above
(A) Pathological damage : i.e. permanent damage
to living body which causes death and development 3. The rays are given off by a radioactive element from -
of diseases e.g. cancers or leukemia etc. (A) nucleus (B) valence electrons
(B) Genetic damage : i.e. effect on chromosomes (C) all the orbits (D) outer orbit
causing mutations.
4. Radium is a radioactive substance. It dissolves in
RADIOACTIVE POLLUTION dilute H2SO4 and forms compound radium sulphate.
The compound is -
Radioactive pollution is a special form of physical
pollution, relating to all systems air, water and soil. (A) no longer radioactive.
This type of pollution is not only harmful for the present (B) half as radioactive as radium content.
generation but also for future generations. The (C) as radioactive as the radium content.
radioactive substances with long half-life are usually (D) twice as radioactive as the radium content.
the main sources of environmental concern.
Neutrons released during nuclear tests make other 5. A radioactive disintegration differs from a chemical
materials radioactive in the surrounding. These
change in being -
materials include 90Sr, 137Cr and 131. The radioactive
materials are converted into gases. These gases (A) an exothermic change.
and fine particles are thrown high up into the air and (B) a spontaneous process.
carried away by wind to distant areas. They ultimately (C) a nuclear process.
settle down and cause pollution to water and soil. (D) none of these
From soil the radioactive substances enter in the food
chain and thus affect all forms of life including man. 6. In treatment of cancer, which of the following is used ?
Cosmic radiations and explosion of a hydrogen bomb
produce 14C in air. (A) 131
53 I
32
(B) 15 P
Nuclear power plants and reprocessing plants
discharge 90Sr, 137Cs, 131, 140 Ba, 140La, 144Rh, etc. Coal (C) 60
27 Co (D) 12 H
based thermal power stations release radioactive
gases such as 85Kr, 133Xe and particulates such as
137
, 60Co, 54Mn and 137 Cs through chimney.
Nuclear dumping within land or in ocean leads to

PAGE # 200
7. 11
6 C
 11
5 B decay produces - 16. In the nuclear reaction 92U238  Pb206 the number
82

of  and  particles emitted is :


(A) positron (B)  -particle
(A) 7,5 (B)6,4
(C) -particle (D) None of these
(C)4,3 (D)8,6
8. Isodiaphers are atoms having -
234 234
(A) p/n constant (B) (p + n) constant 17. In radioactive decay 90 Th 92 U the number
(C) (n–p) constant (D) (n–p) different of  and particles emitted respectively are -
(A)1, 2 (B) 2, 0
60
9. 27 Co is radioactive because - (C) 0, 2 (D) 2, 1
(A) its atomic no. is high
18. Consider the nuclear change,
(B) it has high p/n ratio
237 – – –
(C) it has high n/p ratio 93 Np
 A  B 
  C
(D) None of these
Which of the following statements is not correct ?
10. 23
Na is the more stable isotope of Na. By which (A) Mass number of B is 233.
(B) Atomic number of A is 91.
24
process 11 Na can undergo radioactive decay ? (C) C is an isotope of A.
(D) B is an isobar of A.
(A)  –-emission (B) -emission
(C)  +-emission (D) None of these 19. W hen 13 Al 27 is bombarded with  -particles a
11. Which of the following isotopes is likely to be most radioactive isotope of 15 P30 with the emission of.........
stable ? is formed.
(A) protons (B) neutrons
(A) 71
30 Zn (B) 66
30 Zn (C)positrons (D)electrons
40
(C) 20 Ca (D) None of these
20. 210 206 4
84 Po  82 Pb + 2 He . In this reaction predict

12. The number of neutrons in the element L in the the position of group of Po when lead is in the IV A
following nuclear changes is - group.
(A) II A (B) IV B
238 x
 y N  24 (C) VI B (D) VI A
92 M  He
X A  21. Th a member of III group on losing -particles forms
90
YN
 B L  2 a new element belonging to -
(A) 146 (B) 144 (A) I group (B) III group
(C) 140 (D) 142 (C) II group (D) IV group

13. As compared to 12C atom, 14C atom has - 22. Number of neutrons in a parent nucleus X, which
(A) two extra protons and two extra electrons. gives 14
7 N after two successive  -emission, would
(B) two extra protons and no extra electron. be -
(C) two extra neutrons and no extra electron. (A) 6 (B) 7
(D) two extra neutrons and two extra electrons. (C) 8 (D) 9

II. Radioactivity, Group displacement laws III. Rate of radioactive decay, Half life period, Average
life
14. Which does not take place by -disintegration ?
238 234
23. The half -life period of radioactive element depends
(A) 92 U  90 Th upon -
(B) 232 228 (A) the amount of the element.
90 Th  88 Ra
(B) the temperature.
226 222
(C) 88 Ra  86 Rn (C) the pressure.
213 213 (D) none of these.
(D) 83 Bi  84 Po
24. If 2 g of an isotope has a half-life period of 7 days, the
15. W hen one  and one  particle are emitted by an half -life period of 1 g sample is -
element, the new element formed will have: (A) 3.5 days (B) 7 days
(A)atomic number greater by 1 and mass number (C)14 days (D) 28 days
greater by 4
25. A radioactive sample has a half-life period 1500 years
(B) atomic number less by 1 and mass number less
. A sealed tube containing 1 g of the sample will contain
by 4
after 3000 year -
(C)atomic number greater by 1 and mass number
(A) 1 g of the sample
less by 4
(B) 0.5 g of the sample
(D)atomic number less by 1 and mass number
(C) 0.25 g of the sample
greater by 4
(D) 0.025g of the sample

PAGE # 201
26. The half-life period of a radioactive substance is 20 36. Boron rods in a nuclear reactor are used to -
minutes. The time taken for 1 g of the substance to (A) absorb excess neutrons.
reduce to 0.25g will be: (B) absorb alpha particles.
(A) 30 minutes (B) 40 minutes
(C) slow down the speed of neutrons .
(C) 60 minutes (D) 10 minutes
(D) speed up the reaction.
27. Half life of a Radioactive element is 100 years its
average life will be - 37. In the nuclear reaction -
14 4  17
(A) 13.4 years (B) 1.44 years 7 N + 2 He 8 O + ----------
(C) 144 years (D) None of these
The missing particle is -
28. Two radioactive elements X and Y have half lives of
(A) 24 He (B) 11H
50 and 100 minutes respectively. Initial sample of
both the elements have same no. of atoms. The ratio (C) 10 n (D) 
of the remaining number of atoms of X and Y after
200 minute is- 38. Nuclear fusion takes place in -
(A) 2 (B) 1/2 (A) atom bomb (B) hydrogen bomb
(C) 4 (D) 1/4 (C) neutron bomb (D) nuclear reactor
29. If 12 g of sample is taken, then 6 g of a sample decays 39. An example of nuclear fusion reaction is -
in 1 hour. The amount of sample showing decay in 233 1 234
(A)
90 Th  0 n  90 Th
next hour is -
113 1
(A) 6 g (B) 3 g (B)  114
48 Cd  0n  48 Cd    rays
(C) 2g (D) 1 g 3 3 4
(C)
2 He  2 He  2 He  211H
30. Proportionality constant  is also known as - 239 239 0
(D) 92 U  93 Np 
1 e
(A) decay constant
2 3
(B) disintegration constant 40. The reaction 1H 12 H  2 He 10 n is called -
(C) radioactive constant (A) fusion.
(D) All of these (B) fission.
(C) endothermic reaction.
31. Rate of decay-
(D) spontaneous reaction .
(A) increases with time
(B) decreases with time 41. The first nuclear explosion by India was carried out at -
(A) Puskar (B) Narora
(C) remains constant
(D) first increases, then decreases with time (C) Pokhran (D) Mumbai

32. Unit of decay constant is- EXERCISE-2


(A) time2 (B) time–2
(C) time–1 (D) time3
COMPETITIVE EXAM PREVIOUS YEARS’ QUESTIONS :
60
33. The half-life of 27 Co is 5.3 years. How much of 20 g
1. The half-life period of radioactive element is 14 hours.
of 60
27 Co will remain radioactive after 21.2 years ? The fraction of the radioactive element that
disintegrates in 56 hours is - [IJSO- 2009]
(A) 10 g (B) 1.25 g (A) 0.125 (B) 0.625
(C) 2.5 g (D) 3.0 g (C) 0.9025 (D) 0.9375

IV. Nuclear reaction, Dating


2. H2 + H2 He3 + X : X stands for :
[IJSO-Stage-I/2012]
34. Energy in the sun is mainly generated by -
0 1
(A) fusion of radioactive materials. (A)  1e (B) 0n
(B) fission of helium nuclei.
0
(C) fusion of hydrogen nuclei. (C) – 1e (D) H1
(D) chemical reaction .
3. A radioactive element 90R 232 emits one alpha (a)
35. In a nuclear reactor chain reaction is controlled by particle and then two beta (b) particles. The daughter
introducing - element will have - [IJSO-Stage-I/2012]
(A) iron rods (B) silicon rods (A) Atomic no 90, Mass No. 228
(C) cadmium rods (D) platinum rods (B) Atomic no. 90, Mass no. 232
(C) Atomic no. 88, Mass No. 228
(D) Atomic no. 88, Mass no. 232

PAGE # 202
4. In a nuclear reactor the fission process of each mU-
atom gives out an energy of 200MeV. According to
Einstein's equation the amount of mass getting
converted to energy in this process is -
[IJSO-Stage-I/2012]
(A) 3.55X10–30 Kg (B) 3.55X10–28Kg
–30
(C) 3.55X10 Kg (D) 3.55X10–27Kg

5. Which radiation is the easiest to shield ?


[IJSO-Stage-I/2013]
(A) Alpha (B) Beta
(C) gamma (D) neutron

PAGE # 203
THERMODYNAMICS & THERMOCHEMISTRY

 Open system : System which can exchange energy &


matter both with the surroundings.
THERMODYNAMICS & THERMOCHEMISTRY E.g. : Living systems( any living organism) are open
systems, air in an open room
Introduction :
Thermodynamics : The branch of science which deals Closed system : System which can exchange only energy
with different forms of energy & their interconversion. but cannot exchange matter with the surrounding is called
Application of thermo dynamics closed system.
 W e can predict feasibility of the reaction that is if two E.g : any matter in a closed container.
substances are mixed then the reaction between them  Isolated system : System which cannot exchange energy
will takes place or not. and matter both with the surroundings.
 If Reaction does take place then what are the energy
changes involved during the reaction. E.g. : Water in thermos flask.(Though not a perfectly
 If in a chemical reaction, equilibrium is going to get isolated system but can be taken as, for
attained then what will be the equilibrium concentrations small interval of time as the energy exchanges are
of different reactants & products, can be calculated with negligible).
thermodynamics.
Limitations of Thermodynamics :  Whole of universe is a perfect isolated system.
 Laws of thermodynamics are applicable to matter in bulk
or on system as a whole, these can not be applied on
individual particles(temperature, pressure, enthalpy etc
have meanings only for system as a whole).
 Using thermodynamics we cannot calculate the time
taken for completion of a reaction or for attainment of .
chemical equilibrium.

Terms to be used in Thermodynamics : Open System Close System


 System : Part of the universe which is under study for
energy changes.

Ex. Air in a room, water in a bottle, any living body.

 Surrounding : Rest of the universe.


 Universe : Universe = System + Surroundings Isolated system

STATE OF SYSTEM
 It means the condition in which the system is present.
  It can be specified/defined by measuring/ specifying
some observable/measurable properties of the system
like pressure, volume, temperature, amount of
substance, elasticity, heat capacity etc.
 Boundary : Anything which separates system &
surroundings is called boundary. e.g. For an ideal gaseous system state of the system
 Boundary can be real or imaginary. can be defined by specifying volume, temperature and
 Boundary can be flexible or rigid pressure.
e.g. - air in a flexible balloon (flexible boundary) while  We may have to specify more properties of the system
air in a room (fixed boundary). depending on the complexity of the system.
 Boundary can be adiabatic (non-conducting) or
diathermic(conducting). State function :
 Property of a system which is dependent only on the
Types of system : state of the system i.e. it is a point function
 It is independent of the path adopted to attain a particular

PAGE # 204
state. system is called an Intensive property. W hile the
properties which have values different from the values
e.g. In Mechanics, Displacement of any object will be a for whole of the system are called Extensive Properties.
state function but distance travelled by the object will be
a path function.
for any thermodynamic system,
Temperature, Pressure, Volume, Total internal energy
(E or U), Enthapy(H), Gibbs free energy (G), Entropy (S)
are all state functions.
e.g.

 For example consider air in a room at temp of 300K, 1


atm pressure. Now if the room is divided by some
boundary( imaginary or real) into two parts
(equal or unequal) then in these two parts :
  The temperature, pressure, density of the gas,
concentration of gaseous molecules etc. will have the
same value as that of for whole of the system. (intensive)
 While the volume of two parts, mass of gas in two
parts, total energy of the gaseous molecules in the two
parts, entropy the two parts etc. will be different from the
In the above example the final temperature, pressure,
values of these properties as for the whole of the system
and the volume will be same in both the above ways but
initially. (extensive)
the work involved and the heat exchanged during the
Examples
processes will be different.
1. Categorize these property into state and path function
(a) Internal energy (b) Volume
Path function :
(c) Heat (d) Enthalpy
  Quantities which are dependent on the path/way the
(e) Temperature (f) Work
system has achieved a particular state.
(g) Molar heat capacity
e.g. Heat, work, Heat capacities(Molar heat capacities,
Sol. State function : (a) (b) (d) (e) ; Path function (c) (f) (g)
specific heat capacities etc.).
  These quantities are define when there is a process
2. Categorize these property into extensive and intensive
going on.
property
 These can not have any definite (particular) value in any
(a) Temperature (b) Internal energy
particular state of the system.
(c) Heat (d) Density
(e) Molar volume (f) molar enthalpy( g )
 Types of properties
viscosity
Extensive properties :
Sol. Extensive Property : (b) (c) ;
  Functions or properties of the system which are
Intensive Property : (a) (d) (e) (f) (g)
dependent on mass or on size of the system are called
Extensive Properties .
 Extensive functions are additive in nature( The addition THERMODYNAMIC EQUILIBRIUM
of the volumes of the two parts equals the volume of the
whole of the room.) When there is no change in any observable or measurable
e.g. Volume, Mass, Total heat capacity, Total internal property of a system with time then the system is said to
energy (E), Enthalpy(H), Gibbs Free be in thermodynamic equilibrium.
Energy(G), Entropy(S).   Thermodynamic equilibrium consist of three types of
equilibrium.
Intensive properties : (1) Mechanical equilibrium
 Functions or properties which are not mass dependent (2) Thermal equilibrium
or size dependent are called intensive function.
 Intensive properties are not additive in nature.

eg. Temperature, pressure, molar heat capacity, specific heat


capacity, density,concentration, vapour pressure.

How to identify extensive or intensive properties (3) Chemical equilirbrium


If a system in a particular state is divided into two equal
or unequal parts, the properties which have value equal Mechanical equilibrium :
to the original value of that property for the whole of the There should not be any pressure gradient (with time or
with space (for any ideal gaseous system, for a liquid

PAGE # 205
system there can be pressure gradient with space as rate.
pressure at the bottom of the container in which a liquid Fdriving = Fopposing – dF and dF  0
is filled will be greater than the pressure at the surface
of the liquid.) in the system.  An ideal reversible process will take infinite time to
get completed.

 It is carried out infinitesimally slowly.

 Strictly speaking there is no ideal reversible process


in universe.

To get an idea of a reversible process we can consider


the following system.
 Thermal equilibrium : There should not be any An ideal gas is enclosed in a container and a massless
temperature gradient (difference). piston is put on the gas on which a pile of sand is placed
Temperature may have different values at different places/ having particles of negligible mass. To carry out a
locations in a system but it should remain constant with reversible expansion we will slowing decrease the mass
time. of the sand lets say by removing the particles one by
one, so the expansion of the gas will take place at
infinitesimally small rate and we can always assume
the system to in thermodynamic equilibrium. So the
expansion will be of reversible type.

 Chemical equilibrium : There should not be any


concentration gradient of any of the species in the
system.

 Types of thermodynamic process on basis of state/


conditions
Thermodynamic process : Any method/process by
which system can change its state from one state of
FBD of piston
thermodynamic equilibrium to another state of
thermodynamic equilibrium.
There can be infinite type of thermodynamic processes,
out of these the following are important ones:

1. Isothermal process : T = constant


dT = 0
T = 0
2. Isochoric process : V = constant for piston to be in equilibrium : Pgas = Patm + Mg/A
dV = 0
V = 0  Irreversible process : The process can not be reversed
3. Isobaric process : P = constant by a small change is known as irreversible.
dP = 0  If a process is carried out in such a manner so that
P = 0 the system is in thermodynamic equilibrium (I) Only at
4. Adiabatic process : q = constant initial & final state of the process but not at the
or heat exchange with intermediate stages.
the surrounding = 0(zero) (System may be in thermodynamic equilibrium state
at some finite number of intermediate
 Types of thermodynamics processes on basis of the stages only - for example - n step irreversible expansion
way the processes are carried out : of a gas)
 If during the process there is a finite difference in
Reversible process : driving force and opposing force so that process takes
The process that can be reversed by a very small change place with a finite rate.
is known as reversible process.  Irreversible processes will get completed in finite
 If a process is carried out in such a manner so that time.
the system is always in thermodynamic equilibrium at  At intermediate stages of the irreversible process,
every stage of the process. different state function such as Pressure, temperature
etc. are not defined.
 If the process is carried out such that the difference  All real process are irreversible.
in driving force and opposing force is infinitesimally Consider the above system. If the stopper placed over
small so that process takes place at infinitesimally slow the piston is removed then the piston will move with

PAGE # 206
almost infinite acceleration and will keep moving to a
position where the pressure of the gas becomes equal Work : dW = Fext dx
to the external pressure. Since the process will get P-V Work :
completed in finite time and there was a finite difference Assuming that under an external force F ext, the piston
between the driving force and the opposing force so moves down by a distance ‘dx’.
process is irreversible.During the process the pressure
of the gas can not be defined as it will be having different Fext
values at different locations.
dx

Fext
dW  ( A dx )  dW = Pext (dV)
A
As the work done in the above case by the external agent
 Modes of energy exchange : is +ve and as the expression conveys otherwise, hence
These are two ways by which a system can interact or a -ve sign is introduced.
can exchange energy with its surroundings.
(i) Heat & (ii) Work  dW = – Pext. dV
Heat & Work both are forms of energy.
 IUPAC Sign convention about Heat and Work
Any energy given to system is taken positive so heat
Heat : When the energy transfer takes place because of
temperature difference between system & surroundings. given to system = positive
It is known as heat. heat given out from system / taken out from system =
Negative
Work : Energy transfer which is not heat or which is not Work done on the system = Positive
because of temperature difference is called work.
Work can be of many types : Mechanical work, Electrical
work, Magnetic work, Gravitational work etc.

WORK DONE (W)

Energy that is transmitted from one system to another


in such a way that difference of temperature is not
direc tly involved is kn own as w ork. It is a pa th Work done by the system = Negative
function. The sign convention is different from physics, but the
This definition is consistent with our understanding meaning always comes out to
of work as dw = Fdx. The force F can arise from be same only in equation we have to use a different sign
electrical, magnetic, gravitational & other sources. convention for work.
Units : So if in any problem, w = – 10 J
Heat & work both are forms of energy . Hence, their It means system has done work of 10 Joule on
units are units of energy. i.e.S system: surroundings.
Joules (J). Much data is available in the old units of According to Chemistry :
calories (cal) as well.
P × V = (litre. atmosphere) term which has units of
Q Q
energy . It is useful to remember the conversion
 In Physics
1 litre. atm = 101.3 Joules = 24.206 cal
 For irreversible processes, state parameters such w w
as P,T etc cannot be defined. Hence work cannot be U = Q + W U = Q – W
estimated using P gas. But by the work energy theorem Work done on the system = +ve
W gas = – W ext + K pisston IUPAC convention of Heat
When the piston comes to rest again Kpiston = 0 1. Heat given to the system = +ve
2. Heat coming out of the system = –ve
 W gas = – W ext = – P
ext dv 3. Work done on the system = +ve
4. Work done by the system = –ve
as the external pressure is always defined hence for all
 Calculation of work for different type of process for an
processes work can be calculated using
ideal gas.
(1) Isothermal expansion : There are many ways in
W ext = – P
ext .dv which a gas can be expanded isothermally.
 Work done in irreversible isothermal expansion

PAGE # 207
From some intermediate state of volume ‘v’ the work
50
done is slight expansion from = – 2.303 × 2 × 2 × 298 × log = – 1436 calories.
15
V  (V + dv) 2. If a gas at a pressure of 10 atm at 300 k expands against
dw = – Pext . dv (IUPAC sign convention) a constant external pressure of 2 atm from a vol. of 10
Vf litres to 20 litres find work done ? [Isothermal process]
Sol Process is irreversible
so W =  dw    P Vi
ext .dv
20

W = – Pext (Vf – Vi) 


w  – 2dv  –[ 20 – 10] = – 20 J.
10

Only initial and final states can be located (as at 1 litre atm = 101.3 J
intemediate stages pressure of the gas is not defined)
 Work done in reversible isothermal expansion  Thermodynamic definition of an ideal gas :
 If for a gas the internal energy is directly proportional to
Pext = Pgas – dp its absolute temperature then the gas is termed as an
v2 ideal gas.

 W ext = – P
v1
ext .dv
so 
 E   E 
 =0,   =0
 V  T  P  T
v2
Internal Energy (E, also denoted by U) :
=–
v1
 (Pgas  dp)dv
 Every system having some quantity of matter is
associated with a definite amount of energy, called
v2
internal energy .
=–  (P
v1
gas dv )  It is the sum of all forms of energies present in
the system.
E = E Translational + E Rotational + E Vibrational + E bonding + .....
v2
nRT E = E Final  E Initial .
=  dv = – nRT ln v 1
V v2
v1 E = q v, heat supplied to a gas at constant volume,
since all the heat supplied goes to increase the
W = – nRT ln (Vf/Vi)
internal energy of the gas .

 It is an extensive property & a state function . It is


(2) isothermal compression of an ideal gas :
exclusively a function of temperature.
 Work done in irreversible isothermal compression
If T = 0 ; E = 0 as well.
To compress gas a mass m 0 is suddenly placed on
massless piston
 Degrees of freedom  The total no of modes on which
dw = – Pext. dv
a molecule of an ideal gas can exchange energy during
= – (Patm + m0g/A) dv
collisons is known as its degrees of freedom.
so. to calculate total work done on the gas
Translational degree of freedom = 3 for all type of gases.
Vf Rotational degree of freedom
W=  dw    Vi
Pext . dv ; W = – Pext (Vf – Vi)
= 0 for monoatomic gases
= 2 for diatomic or linear polyatomic gases
 Reversible isothermal compression of an ideal gas = 3 for non-linear polyatomic gases.
This can be achieved by placing particles of sand one by If “f” is initial degrees of freedom for that gas.
one at a very slow take in the assembly which keeps the f=3 for monoatomic
temperature of gas constant in this case the expression =5 for diatomic or linear polyatomic
of work done will be exactly similar to as obtained in =6 for non - linear polyatomic
case of reversible expansion of gas
W = – nRT ln(Vf/Vi) Law of equipartition of energy :
This will automatically come out to be +ve as Vf < Vi 1
Solved Example Energy equal to KT is associated with each degree of
2
1. Calculate the work performed when 2 moles of hydrogen
expand isothermally and reversibly at 25ºC form 15 to freedom per ideal gas molecule
50 litres. 1
Sol.We have, U/molecule = f x KT
2
V2
W = – 2.303 n RT log f f
V1  U/mole = x KT  U/mole = RT
2 2

PAGE # 208
f q dq
 For n moles = nRT 
2 Mathematically CT = = J/ºC
T dT
f
U= nRT only for ideal gas.
2  It is extensive properties and path function
E nt ha lp y
So dq = cdT
Heat content of a system at constant pressure is called
enthalpy denoted by 'H'.
From first law of thermodynamics;
on integrating q=  cdT
Q = E + PV ... (i)
Heat change at constant pressure can be given as  Molar heat capacity
Q = E + PV ... (ii) Heat required to raise temperature of 1 mole of a
At constant pressure heat can be replaced by enthalpy. subtance by 1ºC
H = E + PV ... (iii)
q dq
Mathematically C=  = J mole–1 K–1
Constant pressures are common in chemistry as most nT ndT
of the reactions are carried out in open vessels. So. dq = ncdT

At constant volume, V = 0 ; thus equation (ii) can be q=  ncdT  ncT


written as
for an ideal gas CP and CV are frequently used.
Q = E
 Specific heat capacity (s) :
H = Heat change or heat of reaction (in chemical
Heat required to raise temperature of unit mass
process) at constant pressure
(generally 1 g) of a substance by 1ºC.
E = Heat change or heat of reaction at constant volume.
(i) In case of solids and liquids participating in a reaction, q dq
H = E ( PV = 0) S=  = Jgm–1 K–1
m T mdT
(ii) Difference between H and E is significant when so dq = msdT
gases are involved in a chemical reaction.
H = E + PV
q=  dq   ms dT msT
H = E + nRT We can define specific heat capacity at constant pressure
and at constant volume.(sP & sV )
Here, PV = nRT
Total heat capacity, molar heat capacity & specific heat
n = Number of gaseous moles of products - number of capacity of a process on a substance are related as
gaseous moles of reactants. CT = nC = ms & C = Ms
Where m – weight of substance
Using the above relation we can interrelate heats of M – molar mass of substance
reaction at constant pressure and at constant volume. n – no. of moles of the substance
CALCULATION OF HEAT (q)
For isothermal process C = 
 Calculation of Heat (q) For isobaric process C = Cp
For isochoric process C = Cv
 Heat is a path function and is generally calculated For adiabatic process C = 0
indirectly using Ist Law of thermodynamics Heat capacity can have value from –  to +  depending
 First calculate E and W & then q or heat can be calculated on the process.
if heat capacity of any process is g ive n
to us.
 Heat capacity is a path function and different type of heat
When two bodies are in thermal equilibrium with a third
capacities are defined
Remember heat capacity of a substance is not fixed it is body, then the two bodies will also be in thermal
dependent on type of process which is being performed equilibrium with each other, and all three bodies are
on that substance said to be at the same temperature.
An important derived concept from the zeroth law is that
of equality of temperature . When two bodies at the same
TOTAL HEAT CAPACITY temperature are brought into thermal contact, there is
Heat required to raise the temperature of system by 1ºC no discernible thermal effect on either body by the other.
under the given process is known as total heat capacity. In more rigorous expositions of the zeroth law, the nature
of the thermal contact is qualified as being through a
diathermic or nonadiabatic wall. An adiabatic wall does
not allow discernible thermal effect of one body on the

PAGE # 209
other. Materials called insulators are close to the idea of  For any SUBSTANCE or ION IN SOLUTION – the
adiabatic media when interposed between two bodies. species should be in unit molality (can also be taken as
The diathermic wall, however, is more like the materials 1M concentration), at one bar pressure and at any given
classified as conductors. temperature.
 Molar standard enthalpy of water vapours at 398 K will
be represented as H°(H2O, g , 398 K) and molar standard
FIRST LAW OF THERMODYNAMICS (FLOT) enthalpy of liquid water at 398 K will be represented as
Hm° (H2O, l, 398 K)
(It is hypothetical but can be calculated).
Law of energy conservation :
 W e cannot exactly calculate enthalpy content of a
Energy of total universe is always conserved.
substance only the change in enthalpy can be calculated
or
when substance is taken from one state to other.
Total energy of an isolated system is always conserved.
For example :
or
Hence absolute value of E can never be calculated only
change in value of E can be calculated for a particular
process.

Mathematical form of First Law of thermodynamics.


If a system is intially in a particular state in which its total
internal energy is E1. Now q amount of heat is given to it
and w amount of work is done on it so that in new state
its total internal energy becomes E2. Then according to
1st Law of thermodynamics.
E2 = E1 + q + w
so E = (E2 – E1) = q + w

0
Let enthalpy contant initially be Hm ,1 & finally entalpy
0
THEMOCHEMISTRY contant be Hm , 2
Basically it is study of transfer of heat between chemical 0 0
Then H° = Hm , 2 – Hm ,1
system & surrounding.
In lab mostly Reactions occur at constant atmospheric = heat added at constant pressure to change
pressure. So heat exchange is equal to change in temperature from 25°C to 50°C.
enthalpy. = CPT = (18 cal/mole °C) (25°C) = 450 cal/mol
W hile if reaction occur at constant volume then heat Example
exchange is equal to change in internal energy. 1. W hy does heat get released/absorbed during
chemical reactions?
Sol. because the reactants have a fixed enthalpy content
ENTHALPY OF SUBSTANCE before the reaction and when these are converted
into the products which have a different enthalpy
 Every substance has a fixed value of enthalpy under any content heat gets released or absorbed. Even if
particular state. Though, its exact value cannot be temperature of reactions remains constant yet due
calculated but it has some finite fixed value. to change in bonding energies E  0.
 The enthalpy of one mole of a substance called molar If Hproducts > Hreactants
enthalpy in any particular state can be assigned symbol  Reaction should be endothermic as we have to
 Hm(substance) : For example molar enthalpy of water give extra heat to reactants to get these converted
vapours at 398 K and 1 atm pressure may be into products and if Hproducts < Hreactants
represented as Hm (H2O, g, 398 K, 1 atm). In very simple  Reaction will be exothermic as extra heat content
works enthalpy can be considered as heat constant of reactants will be released during the reaction.
(amount) of substance, and during reaction this heat is Enthalpy change of a reaction :
being released or absorbed. Hreaction = Hproducts – Hreactants = positive–endothermic
 Molar enthalpy of substance under standard conditions H°reactions = H°products – H°reactants = negative – exothermic
is called standard molar enthalpy of a substance.
Standard state of any substance means.
 For a GAS standard state means ideal gas at 1 bar ENTHALPY OF FORMATION
partial pressure at any given temperature.
 For a LIQUID – pure liquid at one bar pressure at It is not possible to determine the absolute value of the
any given temperature. molar enthalpy of a substance. However, based on the
 For a PURE CRYSTALLINE SOLID – pure crystalline following convention, the relative values of standard
solid at 1 bar pressure and at any given temperature molar enthalpies of formation of various substances
can be find.

PAGE # 210
 “The standard enthalpy of formation of every element in
its stable state of aggregation at one bar pressure and HESS'S LAW OF CONTANT HEAT SUMMATION
at specified temperature is assigned a zero value”.The
specified temperature is usually taken as 25 °C.  The heat absorbed or evolved in a given chemical
A few exmaples are Hf° (O2 , g) = 0 equation is the same whether the process occurs in
Hf° (C, graphite) = 0 Hf° (C, diamond)  0 one step or several steps.
Hf° (Br2 , ) = 0
Hf° (S, rhombic) = 0 Hf° (S, monoclinic)  0  The chemical equation can be treated as ordinary
Hf° (P, white) = 0 Hf° (P, black)  0 algebraic expressions and can be added or subtracted
to yield the required equation. The corresponding
 “The standard enthalpy of formation of a compound is enthalpies of reactions are also manipulated in the same
the change in the standard enthalpy when one mole of way so as to give the enthalpy of reaction for the desired
the compound is formed starting from the requisite chemical equation.
amounts of elements in their stable state of aggregation”.
The chemical equations corresponding to enthalpy of  Since  rH stands for the change of enthalpy when
formation of few substances are given below. reactants (substances on the left hand side of the arrow)
are converted into products (substances on the right
Enthalpy of formation of HBr(g) : hand side of the arrow) at the same temperature and
1 1 pressure, if the reaction is reversed (i.e., products are
H2 (g) + Br (l)  HBr(g) written on the left hand side and reactants on the right
2 2 2
hand side), then the numerical value of rH remains the
0
Hf°(HBr, g) =  B Hm (2) = same, but its sign changes.
 The utility of Hess’s law is considerable. In almost all
0
1 0 1 0
Hm (HBr, g) – Hm (H2, g) – Hm (Br2, 1) ...(1) the thermochemical numericals, Hess’s law is used.
2 2
 One of the important applications of Hess’s law is to
Enthalpy of formation of SO2 (g) :
determine enthalpy of reaction which is difficult to
S (rhombic) + O2 (g)  SO2 (g) determine experimentally. For example, the value rH for
0 0 0 the reaction
Hf° (SO2, g) = Hm (SO2, g) – Hm (S, rhombic) – Hm (O2, g)
...(2) 1
C(graphite) + O (g)  CO(g)
But above equations cannot be for calculation of enthalpy 2 2
of reaction as the molar enthalpies of different species
can not be exactly known. W hich is difficult to determine experimentally, can be
estimated from the following two reactions for which rH
 Enthalpy of Reaction from Enthalpies of Formation: can be determined experimentally.
The enthalpy of reaction can be calculated by
Hr° =  B Hf°,products –  B Hf°,reactants   B is C(graphite) + O2(g)  CO2(g) rH1
the stoichiometric coefficient
1
above equation holds good for any reaction as the same CO(g) + O (g)  CO2(g) rH2
2 2
reference state is used for reactants and products
(shown in figure). Substracting the latter from the former, we get
1
C(graphite) + O (g)  CO(g)
2 2
Consequently, rH = rH1 – rH2

Example

7. Calculate heat of the following reaction at constant


pressure,
F2O(g) + H2O(g)  O2 (g) + 2HF(g)
The heats of formation of F2O (g), H2O(g) and HF (g) are
5.5 kcal, –57kcal and 64 kcal respectively.
Sol. Given that
Example 1
(i) F2 (g) + O (g)  F2O (g); H = 5.5 kcal
2 2
1. Calculate the standard enthalpy of reaction ZnO(s) +
CO(g)  Zn(s) + CO2 (g). 1
(ii) H2 (g) + O (g)  H2O (g); H = – 57 kcal
Given, Hf° (ZnO, s) = – 350 KJ / mole–1 2 2
Hf° (CO2, g) = – 390 KJ / mole
Hf° (CO, g) = – 110 KJ / mole

PAGE # 211
1
1 1 2 [H2(g) + O (g)  H2O()] rH° = 2(– 285) kJ mol–1
(iii) H (g) + F (g)  HF (g); H = – 64 kcal 2 2
2 2 2 2
– [CH4(g) + 2O2 (g)  CO2 (g) + 2H2O()]
F2O and H2O in eqns. (i) and (ii) and in the eqn. given in
rH° = –(– 890) kJ mol–1
the problem are on the opposite sides, while HF in eqn.
–––––––––––––––––––––––––––––––––––––––------------
(iii) and in the eqn. given in the problem is on the same
sides. C(graphite) + 2H2(g)  CH4 (g)fH° = – 73 kJ mol–1
Thus applying,
[ – Eqn. (i) –Eqn. (ii) + 2 × Eqn. (iii) ], we get
MEASUREMENT OF ENTHALPY OF COMBUSTION
1 1
–F2 (g) – O (g) – H2 (g) – O2(g) + H2 + F2 (g)
2 2 2
Enthalpies of combustion are usually measured by
 –F2O (g) – H2O(g) + 2HF(g) ;
placing a known mass of the compound in a closed
H = – 5.5 – (– 57) + 2 × ( – 64) =
steel container (known as bomb calorimeter) which is
or F2O (g) + H2O (g)  O2 (g) + 2HF(g); H = – 76.5
filled with oxygen at about 30 bar pressure.
kcal.
The calorimetre is surrounded by a known
Thermometer
 Enthalpy of Combustion : Stirrer
Ignition wire
 It is the enthalpy change when one mole of a compound
combines with the requisite amount of oxygen to give
products in their stable forms.
For example, the standard enthalpy of combustion of
methane at 298 K is – 890 kJ mol–1. This implies the Insulated jacket
following reaction :
Calorimeter bucket
CH4 (g) + 2O2(g)  CO2(g) + 2H2O ()
H° = – 890 kJ mol–1 Water
The standard enthalpy of combustion of methane at 298
K may be written as
Hc° (CH4, g, 298 K) = – 890 kJ mol–1
 The data on the enthalpy of combustion can be
determined experimentally.
 With the help of such data, we can determine the enthalpy
of formation of a compound, which otherwise is difficult Bomb Sample cup
or impossible to determine experimentally. Consider for Bomb calorimeter to determine enthalpy of combustion
example, the enthalpy of formation of CH4(g):
mass of water. The entire apparatus is kept in an
C(graphite) + 2H2(g)  CH4(g)
insulated jacket to prevent heat entering into or leaving
First of all, the combination of carbon and hydrogen does from the container, as shown in figure. The sample is
not occur readily. Secondly, if the reaction is even ignited electrically to bring about the combustion
completed, the end product would not be pure methane. reaction. The heat evolved is used in raising the
Therefore, the enthalpy of formation of methane can be temperature of water and the calorimetre.
determined indirectly through the enthalpy of combustion
of methane : If total heat capacity of calorimeter and all of its contents
CH4(g) + 2O2(g)  CO2(g) + 2H2O() = C, rise in temperature = T
HC°(CH4, g) = Hf°(CO2, g) + 2Hf°(H2O , ) – Hf° (CH4, g) then heat released = q = CT, this heat is because
therefore of m mass of substance.
Hf°(CH4, g) = Hf°(CO2, g) + 2Hf°(H2O , ) – HC° (CH4, g) M
due to 1 mole heat released =   q = EC° (constant
m
 The enthalpies of formation of CO 2 and H 2O can be
volume reactions).
determined experimentally by the combustion of carbon
Now HC° can be calculated by using HC° = EC° +
(graphite) and hydrogen. Thus, knowing the measured
ng RT.
value of HC°(CH4, g), the enthalpy of formation of CH4
W here ng is the change in stoichiometric number of
can be calculated. The value is
gaseous species in the balanced chemical equation
Hf°(CH4, g) = Hf°(CO2, g) + 2 Hf°(H2O , ) – HC° (CH4, g)
representing the combustion process.
= [– 393 + 2 (–285) – (–890)] kJ mol–1 = – 73 kJ mol–1
or, equivalently, we may add the following three chemical
Example
equations.
8. A gas mixture of 4 litres of ethylene and methane on
complete combustion at 25ºC produces 6 litres of CO2 .
C(graphite) + O2 (g)  CO2 (g) CH° = – 393 kJ mol–1 Find out the amount of heat evolved on burning one litre
of the gas mixture. The heats of combustion of ethylene
and methane are – 1464 and –976 kJ mol-1 at 25ºC.

PAGE # 212
3.67 lit. hydrated salt is the enthalpy change when it combines
with the requisite amount of water to form a new hydrated
Sol. + O2  CO2 + H2O stable salt.
C2H4 + CH4
For example, the hydration of anhydrous cupric sulphate
x lit. (4 – x)lit. 6 lit. is represented by
(say) CuSO4 (s) + 5H2O ()  CuSO4 . 5 H2O(s)
or x moles (4 – x) moles 6 moles There is a almost invariably a liberation of heat in such
Applying POAC for C atoms, reactions, i.e. the value of H is negative.
2 × x + 1 × ( 4 – x ) = 1 × 6 ; x = 2 lit. CuSO4(s) + 800 H2O ()  CuSO4 (800 H2O)
Thus, the volume of C2H4 = 2 lit., and volume of CH2 = 2 lit. Hr° = – 68 kJ mol–1
 volume of C2H4 in a 1–litre mixture = 2/4 = 0.5 lit. CuSO4 . 5H2O (s) + 795 H2O ()  CuSO4 (800 H2O)
and volume of CH4 in a 1– litre mixture = 1 –0.5 = 0.5 lit. Hr° = + 10 kJ mol–1
Now, thermochemical reactions for C2H4 and CH4 are by subtraction, we get
C2H4 + 3O2  2CO2 + 2H2O; H = – 1464 kJ CuSO4(s) + 5H2O ()  CuSO4 . 5 H2O(s)Hr°
CH4 + 2O2  CO2 + 2H2O; H = – 976 kJ = – 78 kJ mol–1
As H values given are at 25ºC, let us first calculate the volume  Enthalpy of hydration of gaseous ions.
occupied by one mole of any gas at 25ºC (supposing pressure Enthalpy of hydration of any gaseous ion is the enthalpy
as 1atm) change when 1 mole of the gaseous ion is hydrated in
298 large amount of water to form aqeous ion.
Volume per mole at 25ºC = × 22.4 = 24.4 lit. By convention, the standard enthalpy of formation of
273
H+(aq) is taken to be zero.
Thus, heat evolved in the combustion of 0.665 lit. of C2H4 = – Enthalpy of hydration of Cl¯ gaseous ions will be
1464 represented by :
× 0.5 = – 30 kJ
24.4 Cl¯(g) + aq.  Cl¯(aq) HHydration
and heat evolved in the combustion of 0.335 lit.of CH 4 =  Enthalpy of Neutralization :
 The amount of heat released when one gram equivalent
976
× 0.5 = –20 kJ. of an acid is neutralised by one gram equivalent of a
24 .4 base.
 total heat evolved in the combustion of 1 litre of the or
mixture = –30 + (– 20) = – 50 kJ.  The amount of heat released in formation of one mole of
The integral enthalpy of solution at the given concentration is water when an acid is neutralised by a base.
the enthalpy change when one mole of the solute is dissolved or
in a definite quantity of solvent to produce a solution of a  Enthalpy of neutralization is defined as the enthalpy
desired concentration. change when one mole of H+ in dilute solution combines
with one mole of OH¯ to give rise to undissociated water,
While recording integral enthalpies of solution it is a general i.e.
practice to state the amount of the solvent in which 1 mole of H+(aq) + OH¯(aq)  H2O()
solute is dissolved ; Thus
H= –57.1 kJ/mole = –13.7 kcal/mol
HCl(g) + 10H2O()  HCl (10H2O)H1 = –69.5 kJ mol–1
indicates that when 1 mol of hydrogen chloride gas is Remember :
dissolved in 10 mol of water, there is an evolution of 69.5 kJ  For Strong Acid + Strong Base, heat of neutralisation is
of heat. Other values are always equal to –13.7 kcal/mole or –57.1 kJ/mole.
(i) HCl(g) + 25 H2O()  HCl (25H2O)  For any other combination of acid and base this heat is
H2 = –72.3 kJ mol–1 less than –13.7 kcal/mole or –57.1 kJ/mole.
(ii) HCl(g) + 40 H2O()  HCl (40H2O)
Enthalpy of Ionization :
H3 = –73.0 kJ mol–1
   Whenever a weak acid (or base) reacts with a strong
(iii) HCl(g) + 200 H2O()  HCl (200H2O) base (or acid), the release of heat is less than 57.1 kJ
H4 = –74.2 kJ mol–1 mol–1.
(iv) HCl(g) + aq  HCl (aq) H5 = –75.0 kJ mol–1    It is because of the fact that these acids or bases are
not completely ionized in solution. Some of the heat is
Whenever amount of solvent is not specified then take its consumed in ionizing there acids and bases this heat is
amount to be very large just like in equation no. (iv). known as enthalpy of ionization. Examples are :
HCN + Na+ OH–  Na+ + CN– + H2O
ENTHALPY OF HYDRATION rH° = – 12 kJ mol–1
CH3COOH + Na+OH–  Na+ + CH3COO– + H2O
Enthalpy of hydration is used in following to ways. rH° = – 49 kJ mol–1
 Enthalpy of hydration of anhydrous or partially hydrated The enthalpy of ionization can be calculated as follows.
salts : The neutralization of a weak acid, say HCN, may be
Enthalpy of hydration of a given anhydrous or partially represented in two steps, namely,
Ionization HCN  H+ + CN¯ H°1 = x

PAGE # 213
Neutralization H+ + OH¯  H2O H°2 = – 57.1 kJ/mole = –65 kJ mol–1
The complete reaction is obtained by adding the above
two steps. Thus  Enthalpy of Formation of ions :
The enthalpy change when one mole of hydrated ions is
HCN + OH¯  H2O + CN¯ obtained from element in its standard state as. ½ Cl 2
H° = – 12 kJ/mole (g) + aq  Cl– (aq) Hr° = Hf° (Cl–, aq)
By convention, the standard enthlpy of formation of
Obviously, H° = H°1 + H°2 H+(aq) is taken to be zero.
H°1 = H°H°2 = [–12 – (–57.1)] = 45.1 kJ/mole
We have seen that H+ (aq) + OH–(aq)  H2O(l)
  Greater the enthalpy of ionization of any weak acid or rH0 = – 57.1 kJ mol–1
weak base, weaker will be the acid or base. For this reaction, Hr0 = Hf0 (H2O,l) – {Hf0 (H+, aq)
0 –
+ Hf (OH ,aq)}
Examples
Hence at 25ºC, we get
1. Enthalpy of neutralization of HCl by NaOH is –57.1 kJ/
Hf0 (H+, aq) + Hf0 (OH– ,aq) = Hf0 (H2O, l) – Hr0
mol and by NH4OH is –51.1 kJ/mol. Calculate the
s o Hf0 (OH– ,aq) = {– 286.1 – (– 57.1)} kJ mol–1 = –
enthalpy of dissociation of NH4OH.
229.00 kJ mol–1
Sol. Given that

H+(aq) + NH4OH(aq)  NH4+(aq) + H2O() H = –  With the enthalpies of formation of these two ions, the
51.1 kJ/mole enthalpy of formation of any other ion can be found from
We may consider neutralization in two steps. the enthalpies of formation and solution of its pure
(i) Ionization compound with H+ or OH¯. for example, the enthalpy of
formation of Na+ can be calculated from the enthalpy of
NH4OH(aq)  NH4+(aq) + OH¯(aq) H1 = ? formation and enthalpy of infinite dilute solution of NaOH.
(ii) Neutralization The two values are :

H+(aq) + OH¯(aq)  H2O() H2 = – 57.1 kJ/mole  The chemical equation for the formation of infinite dilute
Thus, H = H1 + H2 solution of NaOH(s) is
Therefore,
NaOH(s) + nH2O()  Na+(aq) + OH¯(aq)
H1 = H – H2 = – 51.1 kJ/mol + 57.1 kJ mol–1 = 6.0 kJ/mol
aqH°(NaOH, s) = –44.50 kJ mol–1
Since there are equal amounts of water on both sides of
Enthalpy of Transition :
the above equation, the two enthalpies give no net effect
Enthalpy of transition is the enthalpy change when one
and thus
mole of one allotropic form changes to another.
 aqH°(NaOH, s) =  fH°(Na + , aq) +  f H°(OH¯, aq) –
For example : C(graphite)  C(diamond)
fH°(NaOH, s)
Htrs0 = 1.90 kJ mol–1
or fH° (Na+, aq) = aqH° (NaOH, s) – fH°(OH°, aq)
so if C(graphite) + O2(g)  CO2(g)
+ fH°(NaOH, s)
HC0 = – 393.51 kJ mol-1
= [–44.50 – (–229.99) + (–425.61)] kJ mol–1
and C(diamond) + O2(g)  CO2(g)
= –240.12 kJ mol–1
HC0 = – 395.41 kJ mol–1
Subtracting, we have, C(graphite)  C(diamond)
 Similarly, from NaCl(aq) or HCl(aq), the enthalpy of
Htrs0 = 1.90 kJ mol–1
formation of Cl¯(aq) can be determined, and so on.
These are recorded in Appendix-II. The changes in
Enthalpy of Precipitation :
enthalpy of any ionic reaction can then be found from
Enthalpy of precipitation is the enthalpy change when
these ionic enthalpies of formation and the usual
one mole of a precipitate is formed. For example :
enthalpies of formation of compounds.
BaCl2(aq.) + Na2SO4(aq)  BaSO4(s) + 2NaCl(aq)
rH0=–24.27 kJ mol–1
Example
Examples
11. The enthalpy of formation of H2O(l) is –285 kJ mol–1 and
9. Calculate H° for the reaction,
enthalpy of neutralization of a strong acid and a strong
Ag+(aq) + Cl¯(aq)  AgCl(s) base is –55 kJ mol–1. What is the enthalpy of formation
at 25°C. Given fH° (Ag+, aq) = 105 kJ mol–1, of OH¯ ions ?
fH° (Cl°, aq) = –167 kJ mol–1 Sol. Given that,
and fH°(AgCl, s) = – 127 kJ mol–1
H+(aq) + OH¯(aq)  H2O()
Sol.For the reaction
fH 0 –285 kJ mol–1
Ag+(aq) + Cl¯(aq)  AgCl(s) neutH = fH(H2O, ) – fH(OH¯, aq)
we have Hence fH (OH¯, aq) = fH(H2O, ) – neutH
H° = fH° (AgCl, s) – fH°(Ag+, aq) – fH°(Cl¯, aq) = [–285 – (–55 )] kJ mol–1 = –230 kJ mol–1
= [–127 – 105 – (–167 )] kJ mol–1

PAGE # 214
12. Calculate the enthalpy change when one mole of HCl(g)
is dissolved in a very large amount of water at 25°C. The Data : Bond Bond enthalpy
change in state is : C–C 336 kJ mol–1
HCl(g) + aq  H+(aq) + Cl¯(aq) C=C 606 kJ mol–1
Given : fH¯(HCl, g) = – 92 kJ mol–1 and fH° (Cl¯, aq) = C–H 410 kJ mol–1
– 167 kJ mol–1 H–H 431 kJ mol–1
Sol.For the reaction, HCl(g) + aq  H+(aq) + Cl¯(aq) Sol. Diagrammatically, we any represent the given reaction
as follows :
we have H° = H°(Cl¯,aq) – fH°(HCl, g)
H° = [–167 – (–92 )] kJ mol–1 = – 75 kJ mol–1
H H H H
| |
BOND ENTHALPIES C=C (g) + H H(g) H H C C H
| |
The bond enthalpy is the average of enthalpies required H H H H
to dissociate the said bond present in different gaseous Break Break
make bond
compounds into free atoms or radicals in the gaseous bonds bonds
state. While bond dissociation enthalpy is the enthalpy H H
required to dissociate a given bond of some specific
compound.for example the enthalpy of dissociation of 2C(g) + 4H(g) + 2H(g) H C C H
the O–H bond depends on the nature of molecular
species from which the H atom is being separated. For H H
example, in the water molecule.
H2O(g)  H(g) + OH(g) Hr0 = 501.87 kJ mol–1 The heat required to dissociate C2H 4(g) and H2(g) into
However, to break the O–H bond in the hydroxyl radical the gaseous atoms is
required a different quantity of heat : For breaking 1C=C 606 kJ mol–1
OH(g)  O(g) + H(g) Hr0 = 423.38 kJ mol–1 For breaking 4C–H 4 x 410 kJ mol–1
The bond enthalpy, OH , is defined as the average of For breaking 1H–H 431 kJ mol–1
these two values, that is :
Total 2677 kJ mol–1
501.87mol1  423 .38kJmol1
HOH = = 462.625 kJ mol–1 Thus H = –2802 kJ mol–1 + 2677 kJ mol–1 = –125 kJ mol–1
2
In the case of diatomic molecules, such as H2, the bond 14. Using the bond enthalpy data given below, estimate the
enthalpy and bond dissociation enthalpy are identical enthalpy of formation of gaseous isoprene
because each refers to the reaction.
CH2  C  CH  CH2
H2(g)  2H(g)HH – H = Hr0 = 435.93 kJ mol–1
|
Thus, the bond enthalpy given for any particular pair of
CH3
atoms is the average value of the dissociation enthaplies
Data Bond enthalpy of C – H bond = 413.38 kJ mol–1
of the bond for a number of molecules in which the pair
Bond enthalpy of C – C bond = 347.69 kJ mol–1
of atoms appears.
Bond enthalpy of C = C bond = 615.05 kJ mol–1
Enthalpy of sublimation of carbon (graphite) = 718.39 kJ
 Estimation of Enthalpy of a reaction from bond
mol–1
Enthalpies :
Enthalpy of dissociation of H2(g) = 435.97 kJ mol–1
Let the enthalpy change for the gaseous reaction
C2H4(g) + HCl(g)  C2H5Cl(g) (g)
Sol For isoprene, we have to form
be required from the bond enthalpy data. This may be
2C – C bonds ; 2C = C bonds and 8C – H bonds
calculated as follows :
For which energy released is
 Enthalpy required to break reactants  [2(–347.69) + 2(–615.05) + 8(–413.38)] kJ mol–1
H =  into gasesous atoms 
  = 5 232.52 kJ mol–1
that is, H (from gaseous atoms) = –5 232.52 kJ mol–1
  Enthalpy released to form products  The reaction corresponding to this is
 from the gasesous atoms  5C(g) + 8H(g)  C5H8(g) fH1 = – 5232.52 kJ mol–1
= [4HC–H + HC=C + HH–Cl] + [–5HC–H – HC–C – HC–Cl] But we want fH corresponding to the following equation
= (HC=C + HH–Cl) – (HC–H + HC–C + HC–Cl) 5C(graphite) + 4H2(g)  C5H8(g) f H = ?
This can be obtained by the following manipulations :
Example 5C(g) + 8H(g) C5H8(g) rH2 = – 5232.52 kJ mol–1
13. Using the bond enthalpy data given below, calculate the 5C(graphite)  5C(g)
enthalpy change for the reaction rH3 = 5 × 718.39 kJ mol–1
C2H4(g) + H2(g)  C2H6(g) 4H2(g)  8H(g)
rH4 = 4 × 435.97 kJ mol–1
Adding, we get
5C(graphite) + 4H2(g)  C5H8(g) fH = 103.31 kJ mol–1

PAGE # 215
15. Find the bond enthalpy of S – S bond from the following
data :
C2H5 – S – C2H5(g)
fH° = – 147 kJ mol–1
C2H5 – S – S – C2H5(g)
fH° = –201 kJ mol–1
S(g)
fH° = 222 kJ mol–1

Sol. Given that


(i) 4C(s) + 5H2(g) + S(s)  C2H5 – S – C2H5(g)
fH° = –147 kJ mol–1
(ii) 4C(s) + 5H2(g) + 2S(s)  C2H5 – S – S – C2H5(g)
fH° = –201 kJ mol–1
Subtracting Eq. (i) from Eq. (ii), we get
C2H5 – S – C2H5(g) + S(s)  C2H5 – S – S – C2H5(g)
fH° = –54 kJ mol–1
Adding to this, the following equation
S(g)  S(s) fH° = –222 kJ mol–1
We get
C2H5 – S – C2H5(g) + S(g)  C2H5 – S – S – C2H5(g)
fH° = –277 kJ mol–1
In the last equation 277.49 kJ of heat id evolved
because of the S–S bond formation.
Hence, the bond enthalpy of S – S is 277 kJ mol–1.

RESONANCE ENERGY

H°resonance = H°f, experimental – H°f, calclulated

= H°c, calclulated  – H°c, experimental

Example
16. Calculate the heat of formation of benzene from the
following data, assuming no resonance. Bond energies :
C – C = 83 kcal C C = 140 kcal
C – H = 99 kcal
Heat of atomisation of C = 170 .9 kcal
Heat of atomisation of H = 52.1 kcal

Sol. We have to calculate H for the reaction


6C (s) + 3H2(g)  C6 H6 (g)

For reactants :
Heat of atomisation of 6 moles of C = 6 ×170.9 kcal
heat of atomisation of 6 moles of H = 6 × 52.1 kcal
For products :
Heat of formation of 6 moles of C – H bonds = – 6 × 99
Heat of formation of 3 moles of C–C = – 3 × 83
Heat of formation of 3 moles of C C bonds = –3 × 140
on adding, we get heat of formation of C6H6,i.e.,
H = 6 ×170.9 + 6 × 52.1– 6 × 99 – 3 × 83 = –75 kcal

PAGE # 216
W hen the partition is removed, the gases mix. The
EXERCISE-1 changes in enthalpy (H) and entropy (S) in the
process, respectively, are :
I. Types of system, Properties, Function, Processes (A) zero, positive (B) zero, negative
(C) positive, zero (D) negative, zero
1. Thermodynamics is concerned with
(A) total energy of a system 10. Warming ammonium chloride with sodium hydroxide in
a test tube is an example of :
(B) energy changes in a system
(A) Closed system (B) Isolated system
(C) rate of a chemical change (C) Open system (D) None of these
(D) mass changes in nuclear reaction
11. Among the following, an intensive property is :
2. Identify the intensive property from the following : (A) mass (B) volume
(A) Volume (B) mass (C) surface tension (D) enthalpy
(C) enthalpy (D) temperature
12. Out of boiling point (), entropy (), pH () and e.m.f. of a
3. Which one of the following is an extensive property ? cell (V), intensive properties are -
(A) ,  (B) , , 
(A) Enthalpy (B) Concentration
(C) , , V (D) All of the above
(C) Density (D) Viscosity
13. Thermodynamic equilibrium involves
4. For an adiabatic process, which of the following (A) Chemical equilibrium
relations is correct ? (B) Thermal equilibrium
(A) E = 0 (B) PV = 0 (C) Mechanical
(D) All the above
(C) dq = 0 (D) q = +w

5. For a cyclic process the condition is : 14. An isolated system is that system in which
(A) there is no exchange of energy with the surroundings
(A) E = 0 (B) there is exchange of mass and energy with the
(B) H = 0 surrounding
(C) E = 0 and H > 0 (C) There is no exchange of mass and energy with the
surrounding
(D) both E = 0 and H = 0
(D) There is exchange of mass with surroundings.

6. Heat change of a process, in which volume remains


II. Heat capacity, Laws of thermodynamics
constant, is represented by :
(A) E (B) H 15. Internal energy of a system of molecules is determined
(C) G (D) n by taking into consideration its :
(A) kinetic energy
7. Which of the following is an endothermic reaction ?
(B) vibrational energy
(A) 2H2(g) + O2(g)  2H2O (l)
(C) rotational energy
(B) N2(g) + O2(g)  2NO(g)
(D) all kinds of energies present in the molecules
(C) NaOH (aq) + HCl(aq)  NaCl(aq) + H2O(l)
16. A system absorbs 500 kJ heat and performs 250 kJ
(D) C2H5OH(aq) + 3O2(g)  2CO2(g) + 3H2O(l)
works on the surroundings. The increase in internal
8. Which of the following statement is false ?
energy of the system is :
(A) Work is state function
(A) 750 kJ (B) 250 kJ
(B) Temperature is state function
(C) 500 kJ (D) 1000 kJ
(C) Change of state is completely defined when initial
and final states are specified 17. Which one of the following is not applicable for a
(D) Work appears at the boundary of the system thermochemical equation ?
(A) It tells about physical state of reactants and products
9. An isolated box, equally partitioned contains two ideal
gases A and B as shown (B) It tells whether the reaction is spontaneous
(C) It tells whether the reaction is exothermic or endot-
hermic
(D) It tells about the allotropic form (if any) of the reac-
tants

PAGE # 217
18. For which of the following reaction, will H be equal to 27. Given
E : NH3(g) + 3Cl2(g) NCl3(g) + 3HCl(g) ; – H1
1
(A) H2(g) + O (g)  H2O(l) N2(g) + 3H2(g) 2NH3(g) ; –H2
2 2
(B) H2(g) +I2(g)  2HI(g)
H2(g) + Cl2(g) 2HCl(g) ; –H3
(C) 2NO2(g)  N2O4(g)
(D) 2SO3(g)  2SO2(g) + O2(g) The heat of formation of NCl3(g) in terms of H1, H2
and H3 is :
19. The absolute enthalpy of neutralisation of the reaction, 1 3
(A) H1 = H1 + H – H
MgO (s) + 2HCl(aq)  MgCl2(aq) + H2O(l) will be - 2 2 2 3
(A) –57.33 kJ mol–1 1 3
(B) Hf = –H1 + H2 – H3
(B) greater than –57.33 kJ mol–1 2 2
(C) less than –57.33 kJ mol–1 1 3
(C) Hf =H – H – H
(D) 57.33 kJ mol–1 2 2 2 3
(D) None of these
20. The first law of thermodynamics is represented by the 28. In the exothermic reaction the enthalpy of reaction is
equation : always :
(A) E = Q – W (B) E = Q + W (A) zero (B) positive
(C) W = Q + E (D) Q = W + E
(C) negative (D) none of these
21. An ideal gas receives 10 J of heat in a reversible
isothermal expansion. Then the work done by the gas: 29. Which of the reaction defines molar Hf°?
(A) would be more than 10 J
(B) 10 J (A) CaO(s) + CO2(g)  CaCO3 (s)
(C) would be less than 10 J
(D) cannot be determined 1 1
(B) Br (g) + H (g)  HBr(g)
2 2 2 2
22. As per the First Law of thermodynamics, which of the
following statement would be appropriate:
3
(A) Energy of the system remains constant (C) N2 (g) + 2H2 (g) + O (g)  NH4 NO3 (s)
(B) Energy of the surroundings remains constant 2 2
(C) Entropy of the universe remains constant
(D) 2 (s) + H2 (g)  2H (g)
(D) Energy of the universe remains constant
23. A gas expands isothermally against a constant external 30. In the reaction, CO2(g) + H2(g) CO(g) + H2O(g);
pressure of 1 atm from a volume of 10 dm3 to a volume H = 2.8 kJ, H represents
of 20 dm3. It absorbs 300 J of thermal energy from its (A) heat of reaction (B) heat of combustion
surroundings. The U is : (C) heat of formation (D) heat of solution
(A) – 312 J (B) + 123 J
(C) – 213 J (D) + 231 J 31. The enthalpy of formation of ammonia is
24. Two molecules of an ideal gas expand spotaneously = 46.0 kJ mol–1. The enthalpy change for following
into a vaccum. The work done is : reaction is :
(A) 2 J (B) 4 J 2NH3  N2 + 3H2
(C) 8 J (D) zero (A) 42.0 kJ (B) 64.0 kJ
(C) 86.0 kJ (D) 92.0 kJ
III. Heat of reaction, Hess law, Bond Energy

25. If H+ + OH–  H2O + 13.7 kcal, then heat of complete 32. The difference between H and E on a molar basis for
neutralisation of one mole of H 2SO 4 with strong base the combustion of Methane gas at T K (kelvin) would be :
(A) zero (B) – RT
will be :
(C) -2RT (D) -3RT
(A) 13.7 kcal (B) 27.4 kcal
(C) 6.85 kcal (D) 3.425 kcal 33. Given that the molar heat capacity of ice is more than the
molar heat capacity of water vapour. Let x and y be the
26. Heat of reaction at constant volume is measured in : magnitudes of the enthalpies of sublimation of ice at T1
the apparatus : K and T2 K such that T1 < T2. Choose the correct options:
(A) bomb calorimeter (B) calorimeter (A) x = y (B) x > y
(C) x < y (D) cannot be determined
(C) pyknometer (D) pyrometer

PAGE # 218
34. The standard heat of combustion of solid boron is equal 2. W hen a certain amount of ethylene (C 2H 4) was
to -
combusted , 6226 kJ heat was evolved. If heat of
(A) H°f (B2O3) (B) 1/2 H°f (B2O3)
(C) 2H°f (B2O3) (D) 1/2 H°f (B2O3) combustion of ethylene is 1411 kJ/mole , the volume of
oxygen that entered into the reaction at STP is nearly :
35. The enthalpy of combustion of C6H6 is – 3250 kJ when [IJSO-STAGE-II/2011]
0.39g of C6H 6 is burnt in excess of oxgyen in an open
(A) 296 ml (B) 296 litre
vessel, the amount of heat envolved is :
(A) 8.32 kJ (B) 12.36 kJ (C) 6226 × 22.4 litre (D) 22.4 litre
(C) 16.25 kJ (D) 20.74 kJ
36. Enthalpy of neutralisation of CH3COOH by NaOH is – 3. At constant temperature and pressure which of the fol-
50.6 kJ/mol and the heat of neutralisation of a strong
lowing statements is true for the reaction.
acid with NaOH is – 55.9 kJ/mol. The value of H for the
ionisation of CH3COOH is : [IJSO-STAGE-I/2012]
(A) 3.5 kJ / mol (B) 4.6 kJ / mol CO2 (g) + ½ O2 (g)  CO2 (g)
(C) 5.3 kJ / mol (D) 6.4 kJ / mol
(A) E = H

37. The bond dissociation energy of gaseous H 2, Cl2 and (B) E > H
HCl are 104, 58 and 103 kcal mol –1 (C) E < H
respectively. The enthalpy of formation for HCl gas will
(D) H and E are independent of each other
be
(A) – 44.0 kcal (B) – 22.0 kcal
(C) 22.0 kcal (D) 44.0 kcal 4. Which of the following species has standard enthalpy of

38. AB, A 2 and B 2 are diatomic molecules. If the bond formation as 0 KJ mol–1 ? [IJSO-STAGE-I/2013]
enthalpies of A 2 , AB & B 2 are in the ratio
(A) H2O(I) (B) Na(g)
1 : 1 : 0.5 and enthalpy of formation of AB from A2 and B2
is – 100 kJ/mol–1. What is the bond enthalpy of A2. (C) Na(s) (D) CO2(g)
(A) 400 kJ/mol (B) 200 kJ/mol
(C) 100 kJ/mol (D) 300 kJ/mol 5. 60g of ice at 0ºC is added to 20g of water at 40 °C. The
final temperature gained by the mixture is (given latent
heat of melting of ice is 80 cal/g and specific heat of
EXERCISE-2 water is 1 cal/g°C) [IJSO-STAGE-2/2012-13]
(A) 0oC (B) 20ºC
COMPETITIVE EXAM PREVIOUS YEARS’ QUESTIONS : (C) 10ºC (D) 5ºC

1. Sulphur exhibits two allotropic forms which are 6. If temperature of a certain mass of aluminum having
interconvertible. These two allotrope of sulphur are rhom- specific heat capacity of 0.8 j/g0 C is lowered by 6ºC and
heat lost is 96J, then mass of aluminum is
bic and monoclinic sulphur. W hen both of them are
[IJSO-STAGE-2/2013-14]
heated in excess of oxygen, rhombic sulphur releases
(A) 16g (B) 48g
297.681 kJ whereas monoclinic sulphur releases (C) 60g (D) 20g
300.193 kJ of heat. Which of the following is true ?
[IJSO-STAGE-II/2011] 7. The heat of formation of carbon dioxide is X1 and that of
(A) The heat of transformation of rhombic to monoclinic water is X2. Any hydrocarbon on combustion gives car-
sulphur is - 2.512 kJ rhombic and monoclinic both bon dioxide and water. Ethyne is a hydrocarbon whose
form SO2 and SO3 on oxidation. formula is C2H2. Heat of combustion of ethyne is X3. With
(B) The heat of transformation of rhombic to monoclinic the above data predict what will be the heat of formation
sulphur is +2.512 kJ rhombic and monoclinic, both
of ethyne. [IJSO-STAGE-II/2014]
form SO2 on oxidation.
(A) X1 + X2 – X3 (B) 2X1 + X2 – X3
(C) The heat of transformation of rhombic to monoclinic
(C) –2 X1 – X2 + X3 (D) – 2X1 –2 X2 + X3
sulphur is - 597.875 kJ. Rhombic and monoclinic,
both form SO2 and SO3 on oxidation.
(D) The heat of transformation of rhombic to monoclinic
sulphur is + 597.875 kJ. Rhombic and monoclinic,
both form SO2 on oxidation.

PAGE # 219
8. Hess' Law states that 'the heat evolved or absorbed in a
chemical reaction is the same whether the process takes
place in one or in several steps'

Heat evolved in the combustion of one mole C(s) in excess


oxygen is x kJ/mol, and that for the combustion of H2(g) is
y kJ/mol and for that of C6H6 (I) is z kJ/mol.Therefore, the
heat change (kJ/mol) involved in the formation one mole
of C6H6(A) from the given equation is
[IJSO-STAGE-I/2015]
(A) x+y+z (B) 6x+3y-z
(C) 6x+3y+z (D) x+y-z/6

9. Evaporation of (Sweat) water is an essential


mechanism in human beings for maintaining normal
body temperature. For human being, heat of vaporization
of water at a body temperature of 37ºC is nearly 2.3 × 106
J/kg and specific heat capacity is 3500 J/kg –1 K –1. On
consuming a certain prescribed diet, the body
temperature of an althlete of mass 82 kg is expected to
increase by 2ºC. In order to prevent this, he drinks N
bottles of mineral water (250 ml water in each) at 37ºC
Assume that the entire amount of this water is given out
as sweat, which vaporizes. N is nearly
[IJSO-STAGE-2/2016]
(A) 4 (B) 3
(C) 2 (D) 1

PAGE # 220
CHEMICAL BONDING
Element Outer most shell
INTRODUCTION configuration
2 6
It is a well known fact that none of the elements Ne 2s 2p
2 6
exist as independent atoms, except for noble Ar 3s 3p
2 6
gases, Kr 4s 4p
2 6
Most of the elements exist as molecules which are Xe 5s 5p
2 6
in a cluster. The questions that arise are. Rn 6s 6p
2 6
 Why do atoms combine and form bonds ? (iv) It is therefore concluded that s p configuration
How do atoms combine to form molecules ? in the outer energy level constitues a structure of
Why is a molecule more stable than the individual maximum stability or minimum energy
atom ?
(v) “Tendency of atoms to have eight electrons in
Such doubts will be discussed in this chapter.
their outermost shell is known as Lewis octet
rule".To achieve inert gas configuration atoms lose,
DEFINATION :
gain or share electrons.
(i) An attractive force that acts between two or
more particles (atoms, ions or molecules) to hold Limitations of the Octet Rule :
them together, is known as a chemical bond. The octet rule, though useful, is not universal. It is
(ii) It is a union of two or more atoms to aquire quite useful for understanding .the structures of most
2 6
stable inert gas configuration s p . of the organic compounds and it applies mainly to
the second period elements of the periodic table.
To explain nature of chemical bond Different There are three types of exceptions to the octet rule.
theories are given at first KOSSEL & LEWIS
1. The incomplete octet of the central atom
proposed the theory.
In some compounds, the number of electrons
Kossel & Lewis approch chemical bonding surrounding the central atom Is less than eight. This
: In early days , the ability if various elements to is especially the case with elements having less than
combine with one another was expressed in terms four valence electrons. Examples are LiCl. BeH2 and
of their valency. The concept of valency was not BCl3.
based on any logi cal understanding. The
development in the field of atomic structure provide
foundations for various theories of valency. The Li. Be and B have 1,2 and 3 valence electrons only.
initial contributions in this field came from. W. Some other such compounds are AlCl3 and BF3.
Kossel and G. N. Lewis in 1916. They were the first
to provide some logical explanation of valency which 2. Odd-electron molecules
based on the inertness of noble gases. This view, In molecules with an odd number of electrons like
later on, came to be known as octete rule. nitric oxide. NO and nitrogen dioxide. NO2, the octet
rule is not satisfied for all the atoms
OCTET RULE .
Cl O 2
(Tendency to acquire noble gas configuration)
(i) It has been observed that atoms of noble gases
3. The expanded octet/ super octet / hypervalent
have little or no tendency to combine with each other
compound
or with atoms of other elements.
Elements in and beyond the third period of the periodic
(ii) It means that these atoms must have a stable table have, apart from 3s and 3p orbitals, 3d orbitals
electronic configuration. also available for bonding. In a number of compounds
(iii) These elements (noble gases) have 8 electrons of these elements there are more than eight valence
2 6 electrons around the central atom. This is termed as
(ns np ) except helium which has 2 electrons
2 the expanded octet. Obviously the octet rule does
(1s ) in their outer most shell.
not apply in such cases.
Some of the examples of such compounds are: PF5
SF 6 , H 2 SO 4 and a number of coordination
compounds.

PAGE # 221
ionic or electrovalent bond.
Electrovalent bond is not possible between similar
atoms. This type of bonding requires two atoms of
different nature. One atom should have the tendancy
to loose electrons i.e. electropositive in nature and
the other atom should have the tendency to accept
electrons i.e. electronegative in nature.
Electropositive atom looses electrons (group IA to
IIIA)
Interestingly, sulphur also forms many compounds Electronegative atom gains electron (group IVA to
in which the octet rule is obeyed. In sulphur dichloride, VII A)
the S atom has an octet of electrons around it. Example : IA and VII A group elements form
strong ionic compound.
+ –
Na + Cl  Na + Cl
2, 8, 1 2, 8, 7 2, 8 2, 8, 8
4. Other drawbacks of the octet theory
•It is clear that octet rule is based upon the chemical More the distance between two elements in the
Inertness of noble gases. However, some noble gases periodic table more will be the ionic character of the
(for example xenon and krypton) also combine with bond.
oxygen and fluorine to form a number of compounds Total number of electrons lost or gained is called
like XeF2 , KrF2 , XeOF2 etc., electrovalency.
• This theory does not account for the shape of Example :
molecules.
(1) Mg + O  MgO
• It does not explain the relative stability of the
molecules being totally silent about the energy of a
molecule.
CLASSIFICATION OF CHEMICAL BONDS :
On the basis of electronic valency theory and
structure, chemical bonds can be classified as electrovalency of Mg = 2
follows. electrovalency of O = 2
(B) Ca + 2Cl  CaCl2

Chemical Bond +2
Ca Ca Cl Cl
2, 8, 8 2, 8, 8, 2 2, 8, 7 2, 8, 8
Interatomic bond
Inter molecular bond One e
(weak in nature) Cl Cl
(strong in nature) 2, 8, 7 2, 8, 8
One e

Electrovalent Covalent Metallic Hydrogen Vander electrovalency of Ca = 2


or ionic bond Bond Bond Bond waal's bond electrovalency of Cl = 1
(C) Ca + O CaO
Bond Bond Energy per mole
Ionic, Covalent & 200 kJ
Co–ordinate
Hydrogen bonding 10 – 40 kJ
Vander–waal’s 2 – 10 kJ electrovalency of Ca = 2
electrovalency of O = 2
ELECTROVALENT OR IONIC BOND
Electrostatic force of attraction between cation and
The chemical bond formed between two or more anion is called ionic bond or electrovalent bond.
atoms as a result of complete transfer of one or Force of attraction is equal in all direction so ionic
more electrons from one atom to another is called bond is non–directional.
A definite three dimensional structure called crystal
lattice is formed .

PAGE # 222
Energy released during the formation of one mole (ii) Boiling point and melting point :
crystal lattice is called lattice energy. High boiling point and melting points are due to
23 + 23 – strong electrostatic force of attraction.
e.g. (6.023 × 10 )Na + (6.023 × 10 ) Cl 
23 (iii) Electrical conductivity :
(6.023 × 10 ) NaCl + 94.5 K.cal (Lattice energy).
It depends on ionic mobility. In solid state there are
Ionic compounds do not have a molecular formula,
no free ions so they are bad conductors of electricity
they only have empirical formula.
e.g. NaCl is the empirical formula of sodium chloride. In fused state or aqueous solution free ions are
present so they are good conductors of electricity
Factors favouring formation of Ionic (Conductivity order) Solid state < Fused state <
bonds : Aqueous solution
Formation of ionic bond depends upon three factors : (iv) Ionic reactions :
(A) Ionisation energy (IE) :  Ionic compounds show ionic reactions &
Amount of energy required to remove an electron
covalent compounds show molecular reactions.
from the outermost orbit of an isolated gaseous
Ionic reactions are fast reactions.
atom to form positve ion or cation is called ionzation
Example : When NaCl is added to AgNO 3
energy [energy is absorbed so it is an endothermic
process] solution, white ppt of AgCl is formed at once.
+ – + + –
+ – Ag NO3 + Na Cl Na NO3 + AgCl 
M + I.E. M + e white ppt.
Less Ionisation energy Greater tendency to
form cation. COVALENT BOND
Example : A covalent bond may be defined as the bond formed
+3 by mutual sharing of electrons between the
Na+ > Mg+2 > Cation Al participating atoms (which are short of electrons) of
formation
+ + + + + comparable electronegativity.
Cs > Rb > K > Na > Li tendency
(B) Electron affinity : 




Amount of energy released when an electron is H H   
O O
   
added to an isotated gaseous atom to form negative
ion or anion is called electron affinity [energy is
released so it is an exothermic process] H 2 molecule O2
– –
X + e X + EA H H O=O
High electron affinity  Greater tendency to
form anions
  
  


N N
Cl > F > Br > I
Anion formation tendency
2 3 N2
F > O > N
N N
Characteristics of ionic compounds : Covalency :
(i) Physical state
(a) Electrov alent compounds are generally (i) It is def ined as the number of electrons
crystalline, hard & brittle in nature. contributed by an atom of the element for sharing
with other atom so as to achiev e noble gas
(b) These compounds are generally made from ions configuration.
which are arranged in a regular way as a lattice (ii) It can also be defined as the number of covalent
structure. bonds formed by the atom of the element with other
(c) Thus electrovalent compounds exist as three atoms.
dimensional solid aggregates.
(iii) The usual covalency of an element except
(d) Normally each ion is surrounded by a number of
hydrogen is equal to 8 minus the number of group
oppositively charged ions and this number is called
to which the element belongs.
co-ordination number
Lewis structure and covalent bond :
+ + + + + +
(i) Structures in which valency electrons are
+ + + +
represented by dots are called Lewis structures.
+ + + + + +
+ + + + (ii) All atoms in the formulae will have eight
electrons in it’s valence shell.H atom is an
Attraction Repulsion
exception for it can form large number of bonds with
{Same charged ions comes near. the atom present at the center of any crystal
So they repell each other}
structure.
PAGE # 223
Other atoms surround it to complete their octet. charged particles or free electrons.
(iii) Lewis dot formulae shows only the number of (b) Substances which have polar character like HCl
valency electrons, the number and kinds of bonds, in a solution, can conduct electricity.
but does not depict the three dimensional (c) Graphit e can conduct elect ricit y si nce
shapes of molecules and polyatomic ions. electrons can pass from one layer to other.
(iv) Lewis formulae are based on the fact that the (d) Some show conductivity due to self ionisation.
representative elements achieve a noble gas example Liq NH3
+ –
configuration in most of their compounds i.e. 8 NH3 + NH3 NH4 + NH2
+ –
electrons in their outermost shell (except for H2, Li
+ H2O + H2O  H3O + OH
2+
and Be ions which have only 2 electrons) (v) Chemical reactions :
Covalent substances give molecular reactions.
Characteristics of covalent compounds :
Reaction rate is usually low because it involves two
(i) Physical state : steps (i) breaking of covalent bonds of the reactants
(a) Under normal temperature and pressure, they (ii) establishing new bonds. While in ionic
exist as gases or liquids of low boiling points. reactions there is only regrouping of ions.
(b) This is due to the fact that very weak forces of
attraction (Vander waal’s forces) exist between the (vi) Solubility :
molecules due to which molecules are far from (a) Non polar compounds are soluble in non polar
solvents. Non polar solvents are CCl4, Benzene,
each other.
CS2 etc.
(c) If their molecular masses are high they exist as
(b) Polar compounds are soluble in polar solvents
soft solids ex. Sulphur, phosphorus and iodine
(i.e. EN 0.9 to 1.8). Polar solvents are H2O, CHCl3,
(ii) Crystal structures : Alcohol etc.
Various types of structures that are present in a
(vii)Isomerism :
covalent compound are as follows.
Covalent bonds are rigid and directional. On account
(a) Covalent solid – In this type of structure every of this there is a possibility of different
atom is bonded to four other atoms by single arrangem ent of at oms i n space. Cov al ent
covalent bonds resulting in the formation of a giant compounds can thus shows isomerism (structural
structure e.g. SiC, AIN and diamond these and spatial)
crystals are very hard and possess high melting
point. ORBITAL OVERLAP CONCEPT
3
Diamond is sp hybridized carbon atom and it In the formation of hydrogen molecule, there is a
forms a tetrahedral structure. minimum energy state when two hydrogen atoms are
2
Graphite is sp hybridized carbon atomsand it so near that their atomic orbltals undergo partial
forms hexagonal layers which can slide over each interpenetration. This partial merging of atomic orbitals
other due to weak Vander waal’s forces of
is called overlapping of atomic orbitals which results
attraction.Distance between C–C atom is 1.42 Å
in the pairing of electrons. The extent of overlap
distance between layers is 3.6 Å. Graphite is
decides the strength of a covalent bond. In general,
more stable than diamond due to high value of
greater the overlap the stronger is the bond formed
change in enthalpy.
between two atoms. Therefore, according to orbital
(b) Molecular solids : They are formed when one overlap concept, the formation of a covalent bond
atom combines with another by a covalent bond between two atoms results by pairing of electrons
and then the molecule combines with another present In the valence shell having opposite spins.
similar molecule with the help of Vander waal’s force
of attraction or hydrogen bond Main points of valency bond theory :
Example : CH4(Solid), dry ice, ice (i) A covalent bond is formed by partial overlapping of
two atomic orbitals
(iii) Melting and boiling points :
(a) With the exception of a few which have giant
three dimensional structures such as diamond,
carborundum (SiC), Silica (SiO2), others have
relatively low melting and boiling points.
(b) This is due to the presence of weak attractive
forces between the molecules.
(iv) Electrical conductivity :
(a) In general covalent substances are bad
(ii) More is the extent of overlapping between the two
conductors of electricity. Since they do not contain
atomic orbital, stronger will be bond.

PAGE # 224
2p-2p 3p-3p 4p-4p 5p-5p
(Type of overlapped orbitals)
< < Types of Overlapping and Nature of
[Principal Quantum no. same, n = 2]
Covalent Bonds :
() () ()
 s orbital are spherical in nature so they are least The covalent bond may be classified into two types
diffused hence it will provide less area for overlapping. depending upon the types of overlapping :
(iii) Orbitals which are undergoing overlapping must (i) Sigma () bond, and (ii) pi () bond
be such that
(i) Sigma () bond : This type of covalent bond is
(a) Each orbital should have one electron with opposite
spin (for formation of covalent bond) formed by the end to end (hand-on) overlap of bonding
orbitals along the intemuclear axis. This is called as
(b) One orbital have pair of electron and the other
orbital have no electron head on overlap or axial overlap. This can be formed
by any one of the following types of combinations of
(for formation of co-ordinate bond)
atomic orbitals.
(iv) If the overlapping is along the molecular axis then
bond will be sigma () & in the perpendicular direction,  s-s overlapping : In this case, there is overlap of two
it will be pi() bond. half filled s-orbitals along the intemuclear axis as
shown below :

+
S–orbital S–orbital S–S overlapping
 Examples of overlapping of pure atomic
orbitals.  s-p overlapping: This type of overlap occurs between
(i) H2 (s–s) H = H= half filled s-orbitals of one atom and half filled p-orbitals
of another atom.

+
(ii) HCl gas molecule (s-p)
S–orbital P–orbital S–P orbital
 p-p overlapping : This type of overlap takes place
between half filled p-orbitals of the two approaching
3s 3p atoms.

+
P–orbital P–orbital
(iii) F2, Cl2, Br2, 2 (p-p)

P–P overlaping

F2 Cl2 Br2 2

PAGE # 225
(iii) pi() bond : In the formation of  bond the atomic
orbitals overlap in such a way that their axes remain Strength of Sigma and pi Bonds :
parallel to each other and perpendicular to the
Basically the strength of a bond depends upon the
intemuclear axis. The orbitals formed due to sidewise
extent of overlapping- In case of sigma bond, the
overlapping consists of two saucer type charged
overlaping of orbitals takes place to a larger extent.
clouds above and below the plane of the participating
Hence, it is stronger as compared to the pi bond where
atoms.
the extent of overlapping occurs to a smaller extent.
Further, it is important to note that pi bond . between
two atoms is formed in addition to a sigma bond. It is
always present in the molecules containing multiple

+ Or
bond (double or triple bonds)

P–orbital P–orbital P–P overlaping

Diffenence in  and  bonds :


Sigma () bond Pi () bond

1. It results from the end to end overlapping It result from the sidewise (lateral)
of two s orbitals or two p–orbitals or overlapping of two p orbitals.
one s and one p–orbital.

2. Its bonded orbital consists of a single Its bonded orbital consists of two electron
electron cloud symmetrical about clouds one above and the other below
internuclear Axis the plane of symimetry

3. Strong Weak

4. Bond energy 80 Kcals 65 Kcals

5. More stable Less Stable

6. Less reactive More reactive

7. Can independently exis Always exists along with a  bond hybridsation

8. Hybridization depends upon  bond doesn't depend on  bond

9. The groups or atoms can undergo Due to resistance to rotation around


bond rotation about single sigma () the  bond the groups attached to it
bonds are not free to rotate.

10. The  electrons are referred as in  bond the electrons are held less
localised firmly bond thus can be easily dissociated or polarised

by an external charge and hence the  e are referred
as mobile electrons.

11. Shape of the molecule is determined  bonding does not affect the shape
by the  bonds present in the molecule of the molecule

PAGE # 226
HYBRIDISATION
The valence bond theory (overlapping concept)
explains satisfactorily the formation of various
molecules but it fails to account the geometry and
shapes of various molecules. It does not give the
explanation why BeCl2 is linear, BF3 is planar, CH4 is
tetrahedral, NH3 is pyramidal and water is V– shaped
molecule.
In order to explain these cases, the valence bond
theory has been supplemented by the concept of
hybridization. This is a hypothetical concept and was
introduced by Pauling & Slater.
According to them the atomic orbitals combine to
form new set of equivalent orbitals known as hybrid
orbitals. Unlike pure orbitals, the hybrid orbitals are
used in bond formation. The phenomenon is known
as hybridisation which can be defined as the process
of intermixing of the orbitals of slightly different
energies in the formation of new set of orbitals of
equivalent energies and shape. For example when
one 2s and three 2p-orbitals of carbon hybridise, there
is the formation of four new sp3 hybrid orbitals.

Salient features of hybridisation : The main features


of hybridisation are as under :
1. The number of hybrid orbitals is equal to the number
of the atomic orbitals that get hybridised.
2. The hybridised orbitals are always equivalent in energy
and shape.
3. The hybrid orbitals are more effective in forming stable
bonds than the pure atomic orbitals.
4. These hybrid orbitals are directed in space in some
preferred direction to have minimum repulsion
between electron pairs and thus a stable arrangement
is obtained. Therefore, the type of hybridisation
indicates the geometry of the molecules.
Important conditions for hybridisation :
(i) The orbitals present in the valence shell of the
atom are hybridised.
(ii) The orbitals undergoing hybridisation should have
almost equal energy.
(iii) Promotion of electron is not essential condition
prior to hybridisation.
(iv) It is the orbital that undergo hybridization and
not the electrons. For example, for orbitals of nitrogen
atom ( 2s 2 2p1x 2p1y 2p1z ) belonging to valency shell
when hybridize to form four hybrid orbitals , one of
which has two electrons (as before) and other three
have one electron each. It is not necessary that only
half filled orbitals participate in hybridisation. In some
cases, even filled orbitals of valence shell ake part in
hybridisation.

PAGE # 227
Determination of hybridisation of an atom (ii) sp2 hybridisation :
in a molecule or ion: Examples
Steric number rule (given by Gillespie) :
Type AB3 AB2L
Steric No. of an atom = number of atom bonded with
that atom + number of lone pair(s) left on that atom.
Shape Trigonal V-shape (bent)
Note : This rule is not applicable to molecules/ions
which have odd e– (ClO2, NO, NO2 ), free radicals and
• ••
compounds like B2H6 which involve 3 centre 2e– bond Example , CH3 , C H3 – C– , SO2, SnCl2
(banana bond). NOCl, O3, NO2
C6H6, CO32– , HCO3–
H2CO3, graphite, BF3 NO2–
For example : O = C = O S.No. = 2 + 0 = 2
B(OH)3 , SO3 , NO3–
C60 (Fullerene)

S.No. = 2 + 1 = 3 (iii) sp3 hybridisation :

Type AB4 AB3 L AB2 L2 ABL3


S.No. = 3 + 0 = 3 Shape tetrahedral pyramidal V-shape Linear
or bent
Example CH4 XeO3 OBr2 –
OCl

HYBRIDIZATION INVOLVING d-ORBITAL :


S.No. = 3 + 1 = 4 Type of ‘d’ orbital involved
sp3 d dz2
Steric number Types of Geometry sp3 d2 dx2 – y2 & dz2
Hybridisation sp3 d3 dxy , dyx, dzx
2 sp Linear d x2  y2
dsp2
3 sp2 Trigonal planar
4 sp3 Tetrahedral (iv) sp3 d hybridiation :
5 sp3 d Trigonal Type AB5 AB4L AB3 L2 AB2L3
bipyramidal Trigonal
Shape bipyramidal See-saw T-shape Linear
6 sp3 d2 Octahedral
7 sp3 d3 Pentagonal Example PCl5 , PBr5, SF4 , ClF3 , [XeF3]+ XeF2 , 3

, [ Cl2 ]–
bipyramidal PF5 etc. XeO2 F2

(i) sp hybridisation : (v) sp3 d2 hybridization :


Examples :
Type AB6 AB5 L AB4 L2
(a) —C H – C  N, H–C  C–H
Shape Octahedral Square Square
(b) = C = O=C=O H2C = C = CH2 Planar
Pyramidal
(c) N3– (azide ion), BeF2, HgCl2, NO2+ (nitronium ion), N2O
Example SF6, PF6–, [SiF6]2–, BrF5 , F5 Cl4–

Note : In N there is a total of two  and two bonds
3 [AlF6]3– , [XeO6]4– XeOF4 XeF4
giving N = N = N and a bond order of two. Both N – N
bonds are the same length, 1.16 Å. The hydrogen Solved Example
azide molecules has a bent structure and two N – N Ex.3 Which is linear PH4+ , H3S+ or NO2+ ?
bond length are different. (A) Phosphonium ion (PH4+)
(B) Sulphonium (H3S+)
(C) Nitronium ion (NO2+) .
(D) None of these
Sol. Phosphonium ion (PH 4 +) is tetrahedral,
sulphonium (H3S+) is pyramidal, but nitronium ion
(NO2+) is linear ; (sp hybridisation).
Ans. (C)

PAGE # 228
Ex.4 The triodide ion (I 3–) in which the I atoms are
arranged in a straight line is stable, but the
corresponding F 3– ion does not exist.
Sol. F 3– and I 3– are of same group. Iodine can
expand its octet but F cannot and thus, in I 3–
octet rule is not violated, but in F 3– octet rule
is violated.

INTERMOLECULAR FORCES
Intermolecular forces are the forces of attraction
and repulsion between interacting particles (atoms
and molecules). This term does not include the elec-
trostatic forces that exist between the two oppositely
charged ions and the forces that hold atoms of a
molecule together i.e., covalent bonds. Attractive in-
termolecular forces are known as van der Waals
forces, in honour of Dutch scientist Johannes van
der Waals (1837- 1923), who explained the deviation
of real gases from the ideal behaviour through these
forces. It is important to note that attractive forces
between an ion and a dipole are known as ion-dipole
forces and these are not van der Waals forces. We
will now learn about There are different types of van
der Waals forces.
Fig : Dispersion forces or London forces between
(i) Dispersion Forces or London Forces atoms.
Atoms and nonpolar molecules are electrically sym-
metrical and have no dipole moment because their
(ii) Dipole-dipole forces
electronic charge cloud is symmetrically distributed.
But a dipole may develop momentarily even in such Dipole-dipole forcesact between the molecules pos-
atoms and molecules. This can be understood as sessing permanent dipole. Ends of the dipoles pos-
follows. sess partial charges and these charges are shown
Suppose we have two atoms ‘A’ and ‘B’ in the close by Greek letter delta . Partial charges are always
vicinity of each other (Fig. a). It may so happen that less than the unit electronic charge (1.6 ~10-19 C).
momentarily electronic charge distribution in one of The polar molecules interact with neighbouring mol-
the atoms, say ‘A’, becomes unsymmetrical i.e., the ecules. Fig (a) shows electron cloud distribution in
charge cloud is more on one side than the other (Fig.
the dipole of hydrogen chloride and Fig. (b) shows
b and c). This results in the development of instanta-
dipole-dipole interaction between two HCl molecules.
neous dipole on the atom ‘A’ for a very short time.
This instantaneous or transient dipole distorts the This interaction is stronger than the London forces
electron density of the other atom ‘B’, which is close but is weaker than ion-ion interaction because only
to it and as a consequence a dipole is induced in the partial charges are involved. The attractive force de-
atom ‘B’. The temporary dipoles of atom ‘A’ and ‘B’ creases with the increase of distance between the
attract each other. Similarly temporary dipoles are dipoles. As in the above case here also, the interac-
induced in molecules also. tion energy is inversely proportional to distance be-
This force of attraction was first proposed by the tween polar molecules. Dipole-dipole interaction en-
German physicist Fritz London, and for this reason ergy between stationary polar molecules (as in sol-
force of attraction between two temporary dipoles is ids) is proportional to 1/r3 and that between rotating
known as London force. Another name for this force polar molecules is proportional to 1/r6, where r is the
is dispersion force. These forces are always attrac- distance between polar molecules. Besides
tive and interaction energy is inversely proportional
dipoledipole interaction, polar molecules can inter-
to the sixth power of the distance between two inter-
act by London forces also. Thus cumulative effect is
acting particles (i.e., 1/r6 where r is the distance
that the total of intermolecular forces in polar mol-
between two particles). These forces are important
ecules increase.
only at short distances (~500 pm) and their magni-
tude depends on the polarisability of the particle.

PAGE # 229
Fig: Dipole - induced dipole interaction between
permanent dipole and induced dipole
In this case also cumulative effect of dispersion forces
and dipole-induced dipole interactions exists.

POLARITY OF BOND
The existence of a hundred percent ionic or covalent
bond represents an ideal situation. In reality no bond
or a compound is either completely covalent or ionic.
Even in case of covalent bond between two hydrogen
atoms, there is some ionic character. When covalent
bond is formed between two similar atoms, for ex-
ample in H2O2, Cl2, N2 or F2, the shared pair of elec-
trons is equally attracted by the two atoms. As a
result electron pair is situated exactly between the
two identical nuclei. The bond so formed is called
nonpolar covalent bond.
Fig : (a) Distribution of electron cloud in HCl- a polar Contrary to this in case of a heteronuclear molecule
molecule. like HF, the shared electron pair between the two
(b) Dipole-dipole interaction between two HCl atoms gets displaced more towards fluorine since
molecules the electronegativity of fluorine is far greater than
that of hydrogen. The resultant covalent bond is a
(ii) Dipole–Induced Dipole Forces polar covalent bond. As a result of polarisation, the
This type of attractive forces operate between the polar molecule possesses the dipole moment (depicted
molecules having permanent dipole and the molecules below) which can be defined as the product of the
lacking permanent dipole. Permanent dipole of the magnitude of the charge and the distance between
polar molecule induces dipole on the electrically neu- the centres of positive and negative charge. It is usu-
tral molecule by deforming its electronic cloud. Thus ally designated by a Greek letter ‘ì’. Mathematically,
an induced dipole is developed in the other molecule. it is expressed as follows :
In this case also interaction energy is proportional to Dipole moment (ì) = charge (Q) × distance of
1/r6 where r is the distance between two molecules. separation (r)
Induced dipole moment depends upon the dipole Dipole moment is usually expressed in Debye units
moment present in the permanent dipole and the (D).The conversion factor is 1 D = 3.33564 × 10–30 Cm
polarisability of the electrically neutral molecule. where C is coulomb and m is meter. Further dipole
Molecules of larger size can be easily polarized. High moment is a vector quantity and is depicted by a
polarisability increases the strength of attractive in- small arrow with tail on the positive centre and head
teractions. pointing towards the negative centre. For example
the dipole moment of HF may be represented as :

The shift in electron density is symbolised by crossed


arrow ( ) above the Lewis structure to indi-
cate the direction of the shift. In case of polyatomic
molecules the dipole moment not only depend upon
the individual dipole moments of bonds known as bond
dipoles but also on the spatial arrangement of vari-
ous bonds in the molecule. In such case, the dipole
moment of a molecule is the vector sum of the dipole
moments of various bonds.
For example in H2O molecule, which has a bent struc-
ture, the two O–H bonds are oriented at an angle of
104.50. Net dipole moment of 6.17 × 10–30 C m (1D
= 3.33564 × 10–30 C m) is the resultant of the dipole
moments of two O–H bonds.

PAGE # 230
Net Dipole moment, ì = 1.85 D Solve Example
= 1.85 × 3.33564 × 10–30 C m = 6.17 ×10–30 C m
The dipole moment in case of BeF2 is zero. This is Ex.10 What is the increasing order of ionic character
because the two equal bond dipoles point in in H2Se, H2S, H2O
opposite directions and cancel the effect of each (1) H2Se < H2S < H2O
other. (B) H2Se > H2S > H2O
(C) H2Se < H2S > H2O
(D) None of these
Ans. (1)
Ex.11 Which of the following molecules should not
have  = 0 :–
(1) H2 (B) CO2
In tetra-atomic molecule, for example in BF3, the di- (C) Cl2 (D) SO2
pole moment is zero although the B – F bonds are Ans. (D)
oriented at an angle of 120° to one another, the three Ex.12 Which of the following compounds should have
bond moments give a net sum of zero as the result-
higher dipole moment than the remaining three :–
ant of any two is equal and opposite to the third.
(1) HF (B) H2O
(C) NH3 (D) NF3
Ans. (B)

HYDROGEN BOND
This is special case of dipole-dipole interaction. This
is found in the molecules in which highly polar N–H,
O–H or H–F bonds are present. Although hydrogen
bonding is regarded as being limited to N, O and F;
but species such as Cl may also participate in hy-
drogen bonding. Energy of hydrogen bond varies be-
tween 10 to 100 kJ mol–1. This is quite a significant
Let us study an interesting case of NH3 and NF3
amount of energy; therefore, hydrogen bonds are
molecule. Both the molecules have pyramidal shape
powerful force in determining the structure and prop-
with a lone pair of electrons on nitrogen atom. Al-
erties of many compounds, for example proteins and
though fluorine is more electronegative than nitrogen,
nucleic acids. Strength of the hydrogen bond is de-
the resultant dipole moment of NH3 ( 4.90 × 10–30
termined by the coulombic interaction between the
Cm) is greater than that of NF3 (0.8 × 10–30 C m). This
lone-pair electrons of the electronegative atom of one
is because, in case of NH3 the orbital dipole due to
molecule and the hydrogen atom of other molecule.
lone pair is in the same direction as the resultant
Following diagram shows the formation of hydrogen
dipole moment of the N – H bonds, whereas in NF3
bond.
the orbital dipole is in the direction opposite to the
resultant dipole moment of the three N–F bonds. The
orbital dipole because of lone pair decreases the ef-
fect of the resultant N – F bond moments, which re-
sults in the low dipole moment of NF3 as represented
below : Intermolecular forces discussed so far are all attrac-
tive. Molecules also exert repulsive forces on one
another. When two molecules are brought into close
contact with each other, the repulsion between the
electron clouds and that between the nuclei of two

PAGE # 231
molecules comes into play. Magnitude of the repul- place using intermolecular H-bonding) because it
sion rises very rapidly as the distance separating exists as discrete molecules.
the molecules decreases. This is the reason that liq-
uids and solids are hard to compress. In these states
molecules are already in close contact; therefore they
resist further compression; as that would result in (c) Chloral hydrate
the increase of repulsive interactions.
Conditions required for H-bond :
(i) Molecule should have more electronegative atom
(F, O, N) linked to H-atom. Chloral hydrate is stable only on account of
(ii) Size of electronegative atom should be smaller. intramolecular hydrogen bonding.
(iii) A lone pair should be present on electronegative
(B) Intermolecular H-Bonding :
atom.
 Order of H-bond strength Exists between the negative and positive ends of
diff erent molecules of the same or different
substances i.e., it is formed between two different
>
molecules of the same or different compounds.
(a) In water molecules
N H - - - - - - :N Due to polar nature of H2O there is association of
O H - - - - - - :O >
water molecules giving a liquid state of abnormally
high boiling point.
> N H - - - - - - :O

Types of Hydrogen Bonds :


(A) Intramolecular H-Bonding :
 When ice is formed from liquid water, some air
This type of H-bonding occurs when polar H and gap is formed (in tetrahedral packing of water
electronegative atom are present in the same molecules). The tetrahedral structure around each
molecule i.e., it is formed when hydrogen atom is oxygen atom with two regular bonds to hydrogen and
present in between the two highly electronegative (F, two hydrogen bonds to other molecules requires a
O, N) atoms within the same molecule. very open structure with large spaces between ice
molecules. Due to this volume of ice is greater than
liquid water and thus ice is lighter than water. We
can say that density decreases when ice is formed.
Reversely when ice melts, density increases but only
(a) upto 4oC, after this intermolecular H-bonding between
water molecules breaks hence, volume increases and
hence density decreases. Thus, water has maximum
density at 4oC.
(b) The hydrogen bonds in HF link the F atom of one
It has lower boiling point (i.e. more volatile) than its molecule with the H-atom of another molecule, thus
para-derivative (where association of molecules takes forming a zig-zag chain (HF)n in both the solid and
place using intermolecular H-bonding) because it also in the liquid.
exists as discrete molecules.

(b)
Some hydrogen bonding also occurs in the gas,
which consists of a mixture of cyclic (HF)6 polymers,
dimeric (HF)2, and monomeric HF.
Very strong hydrogen bonding occurs in the alkali
metal hydrogen fluorides of formula M[HF2]; in KHF2,
for example, an X-ray diffraction study together with
It has lower boiling point (i.e. more volatile) than its
a neutrons diffraction study shows that there is a liner
para-derivative (where association of molecules takes
symmetrical anion having an over all, F–H–F distance

PAGE # 232
of 2.26 Å, which may be compared with the H–F bond

length of 0.92Å in hydrogen fluoride monomer.  CH3 
 |  –
(c) There is also similar H-bonding in alcohol (R— CH3  N  CH3  O H
OH) ammonia (NH3) and phenol (C6H5OH) molecules.  | 
 CH 
 3 
(d) Carboxylic acid dimerises in gaseous state due
to H-bonding In the trimethyl compound the O–H group is hydrogen
bonded to Me3NH group and this makes it more
difficult for the OH group to ionize and hence it is a
weak base.
In the tetramethyl compound, hydrogen bonding can
not occur, so the OH– group ionizes easily and thus
it is a much stronger base.

(e) Alcohol is said to be highly soluble in water due


to crossed intermolecular H-bonding (between H2O EXERCISE-1
and R—OH molecules).
I. Ionic bond
1. Two element have electronegativity of 1.2 and 3.0. Bond
formed between them would be :
(A) ionic (B) polar covalent
(C) co-ordinate (D) metallic
However isomeric ether is less soluble in water
due to less polar nature of ether. 2. Which forms a crystal of NaCl ?
(A) NaCl molecules (B) Na+ and Cl– ions
(C) Na and Cl atoms (D) None of these

3. Which of the following pair of elements form a compound


Solved Example with maximum ionic character ?
(A) Na and F (B) Cs and F
Ex.13 C2H2 is not soluble in H2O but it is highly soluble (C) Na and Br (D) Cs and I
in acetone.
4. Which of the following shows the highest lattice energy
?
(A) RbF (B) CsF
Sol. (C) NaF (D) KF

In hybridisation as %S character increase II. Covalent Bond, Coordinate Bond and Octet Rule
electronegativity increase hence C2H2 forms H–bonds
with O–atom of acetone and get dissolved. But H2O 5. The maximum covalency of representative
elements is equal to (excluding 1st and 2nd period) :
molecules are so much associated that it is not
(A) the number of unpaired p-electrons
possible for C2H2 molecules to break that association, (B) the number of paired d-electrons
hence C2 H2 is not soluble in H2O. (C) the number of unpaired s and p-electrons
(D) the actual number of s and p-electrons in the
Ex.14 Explain that tetramethyl ammonium hydroxide outermost shell.
is a stronger base than that of trimethyl ammonium
hydroxide. 6. Which of the following contains both electrovalent and
covalent bonds ?
CH3 (A) MgCl2 (B) H2O
| (C) NH4Cl (D) none
CH3  N  H    O  H
Sol. | 7. Example of super octet molecule is :
CH3
(A) SF6 (B) PCl5
(C) IF7 (D) All of these

PAGE # 233
8. The number of electrons involved in the bond formation 18. Choose the molecules in which hybridisation occurs in
in N2 molecule is : the ground state ?
(A) 2 (B) 4 (1) BCl3 (2) NH3
(C) 10 (D) 6 (3) PCl3 (4) BeF2
The correct answer is -
9. The octet rule is not obeyed in : (A) 1, 2, 4 (B) 1, 2, 3
(A) CO 2 (B) BCl3 (C) 2, 3 (D) 3, 4

(C) PCl5 (D) (B) and (C) 19. Which of the following compounds have bond angle as
Lewis Structure and Formal Charge nearly 90º ?
(A) CH4 (B) CO2
10. If the atomic number of element X is 7 the lewis diagram (C) H2O (D) SF6
for the element is :
20. sp2 – hybridisation is shown by :
 (A) BeCl2 (B)) BF3
(A) X  (B)  X 
 (C) NH3 (D) XeF2

  21. The hybridisation of carbon in diamond, graphite and


(C)  X  (D)  X
  acetylene is –

(A) sp3, sp2, sp (B) sp3, sp, sp2
2 3
(C) sp , sp , sp (D) sp, sp3, sp2
11. Which of the following Lewis diagram is incorrect ?
Polarity of Bond / Dipole Moment

(A) (B) 22. Of the following molecules, the one, which has
permanent dipole moment, is :
(A) SiF4 (B) BF3
(C) PF3 (D) PF5

23. The dipole moment of the given molecules are such


(C) (D)
that :
(A) BF3 > NF3 > NH3 (B) NF3 > BF3 > NH3
12. W hat are the formal charges on central sulphur and
(C) NH3 > NF3 > BF3 (D) NH3 > BF3 > NF3
each terminal oxygen atoms in SO2?
(A) 0, 0, 0 (B) + 2, 0, – 1
24. Which of the following has the least dipole moment ?
(C) 0, – 1, + 1 (D) + 2, + 2, + 2
(A) NF3 (B) SO3
(C) XeO3 (D) NH3
V.B.T. and Hybridisation
25. Among the following compounds the one that is polar
13. The total number of  and  bonds in C2(CN)4 are :
and has central atom with sp3 hybridisation is :
(A) 9  and 9  (B) 9  and 18
(A) H2CO3 (B) SiF4
(C) 18  and 9 (D) 18  and 18
(C) BF3 (D) HClO2
14. Acetylene consists of :
26. The correct order of dipole moment is :
(A) both sigma and pi bonds (B) sigma bond only
(A) CH4 < NF3 < NH3 < H2O
(C) pi bond only (D) none of these
(B) NF3 < CH4 < NH3 < H2O
(C) NH3 < NF3 < CH4 < H2O
15. The correct order towards bond angle is :
(D) H2O < NH3 < NF3 < CH4
(A) Bond angle does not depend on hybridisation.
(B) sp < sp2 < sp3
Hydrogen bonding
(C) sp2 < sp < sp3
(D) sp3 < sp2 < sp
27. Pure phosphoric acid is very viscous because :
(A) it is a strong acid
16. Which of the following has been arranged in increasing
(B) it is tribasic acid
order of % p-character?
(C) it is hygroscopic
(A) sp < sp2 < sp3 (B) sp3 < sp2 < sp
2 3 (D) it has PO 43- groups which are bonded by many
(C) sp < sp < sp (D) sp2 < sp < sp3
hydrogen bonds
17. Which is not true about CH4 molecule ?
28. Which of the following is least volatile ?
(A) Tetrahedral hybridisation
(A) HF (B) HCl
(B) 109.5º bond angle
(C) HBr (D) HI
(C) Four sigma bonds
(D) One Ione pair of electrons on carbon

PAGE # 234
29. H – bonding is not present in : 6. Greenhouse gases absorb (and trap) outgoing infared
(A) NH3 (B) H2O rediation (heat) from Earth and contribute to global
(C) H2S (D) HF warming. A molecular that acts as a greenhouse gas,
generally has a permanent dipole moment and
30. Hydrogen bonding would not affect the boiling point of : sometimes for other reasons. Going only by the condition
(A) HI (B) NH3 of permanent dipole moment, in the list of gases given
(C) CH3OH (D) H2O below, how many can be potential greenhouse gases ?
W ater, Sulphur dioxide, Boron trifluoride, Carbon
31. W hich of the following compound has maximum monoxide, Carbon dioxide, Nitrogen, Oxygen, Methane
number of H-bonds per mole ? hydrogen sulphite, ammonia.
(A) HF (B) PH3 [IJSO stage-1/2016-17]
(C) H2O (D) OF2 (A) Five (B) Six
(C) Seven (D) Four
32. Water (H2O) is liquid while hydrogen sulphide (H2S) is a
gas because :
(A) water has higher molecular weight
(B) hydrogen sulphide is week acid
(C) water molecular associate through hydrogen
bonding
(D) sulphur has high electronegativity than oxygen

EXERCISE-2
1. The property which is characteristic of an electrovalent
compound is that [IJSO stage-1/2011-12]
(A) it is easily vapourised
(B) it has a high melting point
(C) it is a weak electrolyte.
(D) it often exists as a liquid.

2. Which of the following elements forms polyatomic


molecules ? [IJSO stage-1/2011-12]
(A) Nitrogen (B) Chlorine (C) Argon
(D) Boron

3. The compound which contains both ionic and covalent


bonds is [IJSO stage-1/2012-13]
(A) KCI (B) CS2
(C) C2H6 (D) KCN

4. W hich species below has the same general shape as


NH3 ? [IJSO stage-1/2013-14]
(A) SO23  (B) CO32

(C) NO3 (D) SO3

5 What occurs when H20(l) evaporates


[IJSO stage-1/2016-16]
(i) Covalent bonds are broken.
(ii) Only dipole-dipole forces are overcome.
(iii) Heat is absorbed by water from the surroundings.
(iv) lt becomes oxygen and hydrogen gas.
(A) Only (i) and (iii) occurs
(B) Only (ii) and (iii) occurs
(C) (i), (iii) and (iv) occurs
(D) (i), (ii), (iii) and (iv) take place

PAGE # 235
QUALITATIVE ANALYSIS

Introduction :
Qualitative analysis involves the detection of cation(s) and anion(s) of a salt or a mixture of salts.
The systematic procedure for qualitative analysis of an inorganic salt involves the following steps :
(a) Preliminary tests
 Physical appearance (colour and smell).  Dry heating test.
 Charcoal cavity test.  Charcoal cavity and cobalt nitrate test.
 Flame test.  Borax bead test.
 Dilute sulphuric acid test.  Potassium permanganate test.
 Concentrated sulphuric acid test.  Tests for sulphate, phosphate and borate.
(b) Wet tests for acid radicals.
(c) Wet tests (group analysis) for basic radicals.

Analysis of ANIONS (Acidic Radicals) :


(a) DILUTE SULPHURIC ACID/DILUTE HYDROCHLORIC ACID GROUP :
1. CARBONATE ION (CO32–) :
 Dilute H2SO4 test : A colourless odourless gas is evolved with brisk effervescence.
CaCO3 + H2SO4  CaSO4 + H2O + CO2 
 Lime water/Baryta water (Ba(OH)2) test : The liberated gas can be identified by its property of rendering lime
water (or baryta water) turbid.
CO2 + Ca(OH)2  CaCO3  milky+ H2O
On prolonged passage of CO2 the milkiness disappears.
CaCO3 + CO2 + H2O  Ca(HCO3)2 (soluble)
 Soluble bicarbonates give white precipitate with MgSO4 (aq) / MgCl2(aq) only on heating.

Mg2+ + 2HCO3–  Mg(HCO3)2  MgCO3  + H2O + CO2
2. SULPHITE ION (SO32–) :
 Dilute H2SO4 test : Decomposition of salt is more rapidly on warming, with the evolution of sulphur dioxide.
CaSO3 + H2SO4  CaSO4 + H2O + SO2 
SO2 has suffocating odour of burning sulphur.
 Acidified potassium dichromate test : The filter paper dipped in acidified K2Cr2O7 turns green.
Cr2O72– + 2H+ + 3SO2  2Cr3+ (green) + 3SO42– + H2O.
 Barium chloride/Strontium chloride solution : White precipitate of barium (or strontium) sulphite is obtained.
SO32– + Ba2+/Sr2+  BaSO3/SrSO3  (white).
 White precipitate is soluble in dilute HCl forming SO2.
BaSO3 + 2HCl  BaCl2 + SO2 + H2O
 Lime water test : A white precipitate is formed. The precipitate dissolves on prolonged passage of the gas, due
to the formation of hydrogen sulphite ions.
Ca(OH)2 + SO2  CaSO3 (milky) + H2O
CaSO3  + SO2 + H2O  Ca(HSO3)2 (soluble)
3. SULPHIDE ION (S2–) :
 Dilute H2SO4 test : Pungent smelling gas like that of rotten egg is obtained.
S2– + 2H+  H2S 
 Lead acetate test : Filter paper moistened with lead acetate solution turns black.
(CH3COO)2Pb + H2S  PbS  (black) + 2CH3COOH.

PAGE # 236
 Sodium nitroprusside test : Purple coloration is obtained.
S2– + [Fe(CN)5 (NO)]2–  [Fe(CN)5NOS]4– (violet).
 Cadmium carbonate suspension/ Cadmium acetate solution : Yellow precipitate is formed.
Na2S + CdCO3  CdS + Na2CO3
4. NITRITE ION (NO2¯ ) :
 Dilute H2SO4 test : Solid nitrite in cold produces a transient pale blue liquid (due to the presence of free nitrous
acid, HNO2 or its anhydride, N2O3) first and then evolution of pungent smelling reddish brown vapours of NO2
takes place.
NO2– + H+  HNO2 ; (2HNO2  H2O + N2O3);
3HNO2  HNO3 + 2NO + H2O ; 2NO + O2  2NO2 
 Starch iodide test : The addition of a nitrite solution to a solution of potassium iodide, followed by acidification
with acetic acid or with dilute sulphuric acid, results in the liberation of iodine, which may be identified by the blue
colour produced with starch paste. A similar result is obtained by dipping potassium iodide–starch paper moistened
with a little dilute acid into the solution.
2NO2– + 3I– + 4CH3COOH  I3– + 2NO + 4CH3COO– + 2H2O
Starch + I3–  Blue (starch iodine adsorption complex)
 Ferrous sulphate test (Brown ring test) : When the nitrite solution is added carefully to a concentrated
solution of iron(II) sulphate acidified with dilute acetic acid or dilute sulphuric acid, a brown ring appears due to
the formation of [Fe(H2O)5NO]SO4 at the junction of the two liquids. If the addition has not been made slowly and
caustiously, a brown colouration results.
NO2– + CH3COOH  HNO2 + CH3COO–
3HNO2  H2O + HNO3 + 2NO 
Fe2+ + SO42– + NO   [Fe, NO]SO4
 Thiourea test : When a dilute acetic acid solution of a nitrite is treated with a little solid thiourea, nitrogen is
evolved and thiocyanic acid is produced. The latter may be identified by the red colour produced with dilute HCl
and FeCl3 solution.
NaNO2 + CH3COOH  HNO2 + CH3COONa
HNO2 + H2NCSNH2(s) (thiourea)  N2 + HSCN + 2H2O
dil HCl
FeCl3 + 3HSCN   Fe(SCN)3 (blood red colouration) + 3HCl
5. ACETATE ION (CH3COO¯)
 With dilute H2SO4 a vinegar like smell is obtained.
(CH3COO)2Ca + H2SO4  2CH3COOH + CaSO4
 Neutral ferric chloride test : A deep red/ blood red colouration (no precipitate) indicates the presence of acetate.
6CH3COO– + 3Fe3+ + 2H2O  [Fe3(OH)2(CH3COO)6]+ + 2H+
 When solution is diluted with water and boiled, brownish red precipitate of basic iron (III) acetate is obtained.
Boil
[Fe3(OH)2(CH3COO)6]+ + 4H2O  3Fe(OH)2CH3COO ¯ + 3CH3COOH + H+
Solved Example
Ex.1 An aqueous solution of salt containing an anion Xn– gives the following reactions :
(i) It gives the purple or violet colouration with sodium nitroprusside solution.
(ii) It liberates a colourless unpleasant smelling gas with dilute H2SO4 which turns lead acetate paper black.
Identify the anion (Xn–) and write the chemical reactions involved.
Sol. Xn– is S2– because
(i) [Fe(CN)5NO]2– + S2–  [Fe(CN)5NOS]4– (purple or violet colouration)
(ii) S2– + H2SO4  H2S  (colourless unpleasant smelling) + SO42–
H2S + Pb(CH3COO)2  PbS  (black) + 2CH3COOH
Ex.2 Sulphite on treatment with dil. H2SO4 liberates a gas which :
(A) turns lead acetate paper black
(B) burns with blue flame
(C) smells like vinegar
(D) turns acidified K2Cr2O7 solution green

PAGE # 237
Sol. SO32– + H2SO4  SO2 + SO42– + H2O
SO2 turns acidified K2Cr2O7 solution green.
K2Cr2O7+ H2SO4 + 3SO2  Cr2(SO4)3 (Green) + K2SO4 + H2O
Therefore, (D) option is correct.

(b) CONC . H2SO4 GROUP :


1. CHLORIDE ION (Cl¯) :
 Concentrated H2SO4 test : Colourless pungent smelling gas is evolved which gives fumes of NH4Cl when a
glass rod dipped in dil. HCl is brought in contact with evolving gas.
Cl– + H2SO4  HCl + HSO4–
 NH4OH + HCl  NH4Cl  (white fumes) + H2O.
 2NaCl + MnO2 + 2H2SO4 (conc.)  Na2SO4 + MnSO4 + 2H2O + Cl2
 Silver nitrate test :
Cl– + Ag+  AgCl  (white)
 White precipitate is soluble in aqueous ammonia and precipitate reappears with HNO3.
AgCl + 2NH4OH  [ Ag(NH3)2]Cl (Soluble) + 2H2O
[Ag(NH3)2]Cl + 2H+  AgCl  + 2NH4+.
 Chromyl Chloride test :
4Cl– + Cr2O72– + 6H+ (conc.)  2CrO2Cl2 (deep red vapours) + 3H2O
When deep red vapours are passed into sodium hydroxide solution, a yellow solution of sodium chromate is
formed, which when treated with lead acetate gives yellow precipitate of lead chromate.
CrO2Cl2 + 4OH–  CrO42– + 2Cl– + 2H2O
CrO42– + Pb+2  PbCrO4 (yellow)
 Heavy metal chlorides such as Hg2Cl2, HgCl2, SnCl2, AgCl, PbCl2 and SbCl3 do not respond to this test
as they are partially dissociated. This test is given generally by ionic chlorides.
 Test should be carried out in a dry test tube otherwise chromic acid will be formed.
CrO2Cl2 + 2H2O  H2CrO4 + 2HCl
2. BROMIDE ION (Br¯) :
 Concentrated H2SO4 test : First a reddish-brown solution is formed, then reddish-brown bromine vapour
accompanies the hydrogen bromide (fuming in moist air) is evolved.
2NaBr + H2SO4  Na2SO4 + 2HBr 2HBr + H2SO4  Br2 + 2H2O + SO2
 2KBr + MnO2 + 2H2SO4  Br2+ K2SO4 + MnSO4 + 2H2O
 Silver Nitrate test : Pale yellow precipitate is formed
NaBr + AgNO3  AgBr  + NaNO3
 Yellow precipitate is partially soluble in dilute aqueous ammonia but readily dissolves in concentrated ammonia
solution.
AgBr + 2NH4OH  [Ag(NH3)2] Br + H2O
 Chlorine water test (organic layer test) : When to a sodium carbonate extract of metal bromide containing
CCl4, CHCl3 or CS2, chlorine water is added and the content is shaken and then allow to settle down reddish
brown colour is obtained in organic layer.
2Br– + Cl2  2Cl– + Br2  .
Br2 + CHCl3 / CCl4  Br2 dissolve to give reddish brown colour in organic layer..
With excess of chlorine water, the bromine is converted into yellow bromine monochloride and a pale yellow
solution results.
Br2 + Cl2   2BrCl
3. IODIDE ION ¯) :
 Concentrated H2SO4 test : Pungent smelling violet vapours are evolved.
2Na + H2SO4  Na2SO4 + 2HI
2HI + H2SO4  I2  (dark violet) + 2H2O + SO2

PAGE # 238
 Evolution of dark violet fumes intensifies on adding a pinch of MnO2.
3I– + MnO2 + 2H2SO4  I3–  + Mn2+ + 2SO42– + 2H2S
 Starch paper test : Iodides are readily oxidised in acid solution to free iodine; the free iodine may than be
identified by deep blue colouration produced with starch solution.
3I– + 2NO2– + 4H+  I3– + 2NO  + 2H2O.
 Silver nitrate test : Bright yellow precipitate is formed.
I– + Ag+  AgI 
 Bright yellow precipitate is insoluble in dilute aqueous ammonia but is partially soluble in concentrated ammonia
solution.
 Chlorine water test (organic layer test) : When chlorine water is added to a solution of iodide, free iodine
is liberated which colours the solution brown and on shaking with CS2, CHCl3 or CCl4, it dissolves in organic layer
forming a violet solution, which settles below the aqueous layer.
2NaI + Cl2  2NaCl + I2
I2 + CHCl3  I2 dissolves to give violet colour in organic layer..
If excess of chlorine water is added, I2 is oxidised to iodic acid (colourless).
I3– + 8Cl2 + 9H2O  3O3– + 16Cl– + 18 H+
4. NITRATE ION (NO3¯) :
 Concentrated H2SO4 test : Pungent smelling reddish brown vapours are evolved.
4NO3– + 2H2SO4  4NO2  + O2 + 2SO42– + 2H2O
 Addition of bright copper turnings or paper pellets intensifies the evolution of reddish brown gas.
2NO3– + 4H2SO4 + 3Cu  3Cu2+ + 2NO  + 4SO42– + 4H2O ; 2NO  + O2  2NO2 
4 C (paper pellet) + 4HNO3  2H2O + 4NO2 + 4CO2.
 Brown ring test : When a freshly prepared saturated solution of iron (II) sulphate is added to nitrate solution and
then concentrated H2SO4 is added slowly from the side of the test tube, a brown ring is obtained at the junction
of two layers.
NaNO3 + H2SO4  NaHSO4 + HNO3
6FeSO4 + 2HNO3 + 3H2SO4  3Fe2(SO4)3 + 2NO + 4H2O
or 2NO3– + 4H2SO4 + 6Fe2+  6Fe3+ + 2NO  + 4SO42– + 4H2O.
Fe2+ + NO  + 5H2O  [Fe (H2O)5 NO+]2+ (brown ring).

 On shaking and warming the mixture, NO escapes and a yellow solution of iron(iii) ions is obtained.
(B) GROUP 'B' RADICALS :
Group of anions which do not give any gas with dilute as well as concentrated H2SO4 in cold but give
precipitate with certain reagents :
These acid radicals are identified in inorganic salts by their individual tests as given below
1. SULPHATE ION (SO42–) :
 Barium chloride test :
W.E. or S.E. + Barium chloride (aq)  White precipitate
Na2SO4 + BaCl2  BaSO4 white+ 2NaCl.
 White precipitate is insoluble in warm dil. HNO3 as well as HCl but moderately soluble in boiling concentrated
hydrochloric acid.
 Lead acetate test :
W.E. or S.E. + Lead acetate  white precipitate

PAGE # 239
Na2SO4 + (CH3COO)2Pb  PbSO4 White + 2CH3COONa
 White precipitate soluble in excess of hot ammonium acetate.
PbSO4 + 2CH3COONH4  (CH3COO)2Pb (soluble) + (NH4)2SO4
 Match stick test :
(a) W.E. or S.E. + Barium chloride  white precipitate
Na2SO4 + BaCl2  2NaCl + BaSO4  (white)
(b) White precipitate + Na2 CO3(s) mix and apply the paste on the end of the carbonized match stick or a wooden
splinter. Put it in the reducing flame.
BaSO4 (s) + Na2CO3(s)  Na2SO4 + BaCO3  (white)
Na2SO4 + 4C  Na2 S + 4CO
(c) Now dip the match stick in sodium nitroprusside solution, purple colour near the fused mass is developed.
Na2S + Na2 [Fe(CN)5 NO]  Na4 [Fe(CN)5 NOS] (purple)
2. PHOSPHATE ION (PO43– ) :
 Ammonium molybdate test :
Na2 HPO4 (aq) + 12(NH4)2MoO4 + 23HNO3  (NH4)3PMo12O40  (canary yellow) + 2NaNO3 + 21NH4NO3 +
12H2O
 Some times ammonium phosphomolybdate is also represented by the formula (NH4)3 PO4 . 12MoO3
 Magnesium nitrate or magnesia mixture test : W.E. or S.E + Magnesium nitrate reagent (3-4 mL) and allows
to stand for 4-5 minutes, white crystalline precipitate is formed.
Na2HPO4 (aq) + Mg(NO3)2 (aq) + NH4OH(aq)  Mg(NH4) PO4  (white) + 2NaNO3 + H2O
Magnesia mixture is a solution containing MgCl2, NH4Cl and a little aqueous NH3.
 PO43– also gives BaCl2 test due to the formation of white precipitate of Ba3 (PO4)2 . So phosphate test should be
carried out first and then conclude if PO43– is present or absent before proceeding with the test for SO42–.
 Silver nitrate solution : Yellow precipitate is formed which is soluble in dilute ammonia and in dilute nitric acid.
PO43– + 3Ag+  Ag3PO4 
Ag3PO4 + 6NH3  3[Ag(NH3)2]+ + PO43– ; Ag3PO4 + 2H+  H2PO4– + 3Ag+
3. BORATE ION (BO33–) :
Salt (0.2 g) + conc. H2SO4 (1 mL) + Ethyl alcohol (4-5 mL) mix in a test tube and then heat. Ignite the evolved
vapours with the help of Bunsen flame, green edged flame is obtained.
Na3BO3 + 3H2SO4  3Na2 SO4 + 2H3BO3
3C2H5OH + H3BO3  (C2 H5)3 BO3 + 3H2O
SOLVED EXAMPLE
Ex.1 A compound (A) of S, Cl and O has vapour density of 67.5 (approx.). It reacts with water to form two acids and
reacts with KOH to form two salts (B) and (C) while (B) gives white precipitate with AgNO3 solution and (C) gives
white precipitate with BaCl2 solution. Identify (A), (B) & (C).
Sol. As mixture give white precipitate with BaCl2 and AgNO3, it should contain SO42– and Cl– ions. As SO2Cl2 when
dissolved in water gives, a mixture of H2SO4 & HCl which then react with KOH to form KCl and K2SO4 . Therefore,
(A) is SO2Cl2 and (B) & (C) are K2SO4 and KCl respectively.
Vapour density of SO2Cl2 = molecular weight / 2.
Vapour density of SO2Cl2 = 135 / 2 = 67.5.
Ex.2 Bromine vapours turn moist starch iodide paper :
(1) brown (2) red
(3) blue (4) colourless
Sol. 2I– + Br2  I2 + 2Br– ; I2 + starch  blue starch iodine adsorption complex.
Therefore, (C) option is correct.
Ex.3 Na2S2O3 + 2  NaI + .......... [X], [X] is :
(1) Na2S4O6 (2) Na2SO4
(3) Na2S (4) Na3SO4
Sol. 2Na2S2O3 + 2  2NaI + Na2S4O6 .
Therefore, (A) option is correct.

PAGE # 240
Analysis of CATIONS (Basic Radicals) :
Group Group reagent Basic radical Composition and colour of
precipitate
Zero NaOH or Ca(OH)2 , heat if required NH4+ Ammonia gas is evolved.
1. Dil HCl Ag+ AgCl ; White
Hg22+ Hg2Cl2 ; White
Pb2+ PbCl2 ; White
2. (A) H2S in presence of dil HCl Hg2+ HgS ; Black
(Insoluble in YAS) Pb2+ PbS ; Black
Bi3+ Bi2S3 ; Black
Cu2+ CuS ; Black
Cd2+ CdS ; Yellow
2. (B) H2S in presence of dil HCl As3+ As2S3 ; Yellow
(Soluble in YAS) Sb3+ Sb2S3 ; Orange
Sn2+ SnS ; Brown
Sn4+ SnS2 ; Yellow
3. NH4OH in presence of NH4Cl Fe3+ Fe(OH)3 ; Reddish brown
Cr3+ Cr(OH)3 ; Green
Al3+ Al(OH)3 ; Gelatinous white
4. H2S in presence of NH4OH Zn2+ ZnS ; White
and NH4Cl Mn2+ MnS ; Buff (or Pink)
Co2+ CoS ; Black
Ni2+ NiS ; Black
5. (NH4)2CO3 in presence of NH4OH Ba2+ BaCO3 ; White
Sr2+ SrCO3 ; White
Ca2+ CaCO3 ; White
6. Na2HPO4 in presence of NH4OH Mg2+ Mg(NH4)PO4 ; White

 [YAS = Yellow ammonium sulphide. (NH4)2Sx].


ZERO GROUP :
1. AMMONIUM ION (NH4+) :
Sodium hydroxide solution : Ammonia gas is evolved on warming the solution containing ammonium salt and
sodium hydroxide.
NH4Cl + NaOH  NH3 + H2O + NaCl
 The gas can be identified by the following characteristics / reactions.
— Its characteristics smell.
— The evolution of the white fumes of ammonium chloride when a glass rod dipped in dilute HCl is held in the
vapour.
NH3 + HCl  NH4Cl  (white fumes)

— Its ability to turn filter paper moistened with Hg2(NO3)2 solution black.
2HgNO3 + 2NH3  Hg (NH2 ) NO 3  Hg + NH4NO3

black
— Its ability to turns filter paper moistened with CuSO4 solution deep blue.
CuSO4 + 4NH3  [Cu(NH3)4]SO4
 Nessler's reagent (Alkaline solution of potassium tetraidomercurate(II) :
Brown precipitate or brown or yellow colouration is obtained according to the amount of ammonia or ammonium
ions present. The precipitate is a basic mercury (II) amido–iodide.

PAGE # 241
NH4+ + 2[HgI4]2– + 4OH –  HgO Hg (NH2)I + 7I– + 3H2O

Ist GROUP (Pb2+, Hg22+, Ag+) :

1. LEAD ION (Pb2+) :


 Dilute HCl solution : White precipitate is formed in cold solution.
Pb2+ + HCl  PbCI2  (white) + 2H+
White precipitate is soluble in hot water. White precipitate is also soluble in concentrated HCl or concentrated
KCl.
PbCl2 + 2Cl–  [PbCl4]2– (colourless)
 Sodium hydroxide solution : White precipitate is formed which is soluble in excess of the reagent.
Pb2+ + 2OH–  Pb(OH)2  ; Pb(OH)2 + 2OH–  [Pb(OH)4]2–

 Potassium iodide solution : A yellow precipitate is formed.


PbCl2 + 2KI  PbI2  + 2KCl ; Pbl2 + KI (6M) K2[PbI4] 
Yellow precipitate reappears on dilution with water.
 Potassium chromate solution (in neutral, acetic acid or ammonia solution) : A yellow precipitate is formed.
PbCl2 + K2CrO4  PbCrO4  + 2KCl
Yellow precipitate is soluble in sodium hydroxide and HNO3 (nitric acid).

PAGE # 242
 Ammonia solution : With ammonia solution, Pb2+ gives a white precipitate of lead hydroxide.
Pb+2 + 2NH4OH  Pb(OH)2  + 2NH4+

2. MERCURY(I) ION (Hg22+) :

 Dilute HCl solution : White precipitate is formed in cold solution.


Hg2+2 + 2HCl  Hg2Cl2  (white) + 2H+

 Ammonia solution : A mixture of mercury metal (black precipitate) and basic mercury (II) amido chloride (white
precipitate) is formed.
2Hg2Cl2 + 4NH4OH  HgO.Hg(NH2 ) Cl  Hg  + 3NH4Cl + 3H2O



black

Dissolution of white precipitate (Hg2Cl2) in aquaregia :


3Hg2Cl2 + 2HNO3 + 6HCl  6HgCl2 +2 NO+ 4H2O

(a) Stannous chloride test : White precipitate is formed which finally turns to black.
2HgCl2 + SnCl2  Hg2Cl2  + SnCl4 ; Hg2Cl2 + SnCl2  2Hg  (black) + 2SnCl4

(b) Potassium iodide test : Scarlet/red precipitate is formed which is soluble in excess of the reagent.
HgCl2 + KI  HgI2 + 2KCl ; Hg2 + KI (excess)  K2[Hg4] (soluble)

(c) Copper chips test : Shining grey deposition of mercury on copper chips is formed.
HgCl2 + Cu  Hg (grey) + CuCl2
 Reaction with H2S gas :
Hg22+ + H2S  HgS  (black) + Hg  (grey) + 2H+

3. SILVER ION (Ag+) :

 Dilute hydrochloric acid/soluble chlorides : White precipitate is formed.


Ag+ + HCl  AgCl  + H+
The precipitate obtained after filtration is soluble in concentrated HCl.
AgCl + Cl– [AgCl2]–
Dilute ammonia solution dissolves the precipitate forming a soluble complex.
AgCl + 2NH3 [Ag (NH3)2 ]+ + Cl–
Dilute nitric acid or hydrochloric acid neutralizes the excess ammonia and the precipitate reappears because the
equilibrium is shifted backwards.
[Ag(NH3)2]Cl + 2HNO3  AgCl (white) + 2NH4NO3.
 Potassium iodide solution : A bright yellow precipitate is formed which is insoluble in dilute ammonia b u t
partially soluble in concentrated ammonia.
Ag+ + I–  AgI
The yellow precipitate is soluble in KCN and in Na2S2O3.
AgI + 2CN–  [Ag(CN)2]– + I– ; AgI + 2S2O32–  [Ag(S2O3)2]3– + I–
 Potassium chromate solution : Red precipitate is formed which is soluble in dilute HNO3 and in ammonia solution.
2Ag+ + CrO42–  Ag2CrO4 

IInd Group (Hg2+, Pb2+, Bi3+, Cu2+, Cd2+, As3+, Sb3+, Sn2+)
On the basis of the solubility of the precipitates of the sulphides of II group cations in yellow ammonium sulphide,
they have been classified into two subgroups as given below :

A : HgS, PbS, CuS, Bi2S3, all black but CdS is yellow. All insoluble in yellow ammonium sulphide.
B : SnS2, As2S3 are yellow, Sb2S3 is orange & SnS is dark brown All soluble in yellow ammonium sulphide.

PAGE # 243
IIA Group (Hg2+, Pb2+, Bi3+, Cu2+, Cd2+)

1. MERCURY (II) ION (Hg2+) :


 Precipitation with H2S in acidic medium : Black precipitate is formed. Precipitate insoluble in water, hot
dilute HNO3 ,alkali hydroxides, or colourless ammonium sulphide.
H
Hg2+ + H2S 
 HgS  + 2H+
Aqua regia dissolves the precipitate.
3 HgS + 6 HCl + 2HNO3  3 HgCl2 + 3S  + 2NO  + 4 H2O
2 HNO3 + S   SO42– + 2 H+ + 2NO 
 Potassium iodide solution : On slow addition red precipitate is formed.
Hg2+ + 2I–  HgI2 
Precipitate dissolves in excess of KI forming colourless soluble complex.
HgI2 + 2I–  [HgI4]2–

PAGE # 244
 Ammonia solution : White precipitate of mixed composition (Mercury (II) oxide + Mercury (II) amido nitrate) is
formed with metal nitrate.
2 Hg2+ + NO3– + 4NH3 + H2O  HgO.Hg (NH2)NO3  + 3 NH4+
 Cobalt (II) thiocyanate test : When reagent is added to an aqueous solution of Hg2+ ions and the walls of the
test tube is stirred with a glass rod, deep–blue crystalline precipitate is formed.
Hg2+ + Co2+ + 4 SCN–  Co+2 [Hg(SCN)4]–2 
 In place of Cobalt (II) thiocyanate, Co(CH3COO)2 and NH4SCN can be added to the aqueous solution
2+
of Hg ions.

2. COPPER ION (Cu2+) :


 Precipitation with H2S in acidic medium : Black precipitate is formed.
H
Cu2+ + H2S   CuS  + 2 H+
Precipitate is insoluble in boiling dilute (M) H2SO4 (distinction from cadmium), in NaOH, Na2S and (NH4)2S.
Precipitate dissolves in hot concentrated HNO3
3 CuS + 8HNO3  3Cu(NO3)2 (blue) + 2NO + 4H2O + 3S
When boiled for longer S is oxidised to H2SO4 and a clear solution of Cu(NO3)2 is obtained.
 Ammonia solution : When added sparingly a blue precipitate of basic salt (basic copper sulphate) is formed
with CuSO4.
2Cu2+ + SO42– + 2 NH3 + 2 H2O  Cu(OH)2 .CuSO4  + 2NH4+
It is soluble in excess of reagent forming a deep blue colouration.
Cu(OH)2 .CuSO4 + 8NH3  2 [Cu(NH3)4]2+ + SO42– + 2 OH–
 Sodium hydroxide in cold solution : A blue precipitate is formed.
Cu2+ + 2 OH –  Cu (OH)2 
Heat
Cu(OH)2   CuO(red) + H2O
Potassium iodide : It gives a white precipitate of Cu(I) iodide but the solution is intensely brown because of the
formation of tri–iodide ions (or iodine).
2 Cu2+ + 5 I–  Cu2I2  + I3–
The solution becomes colourless and a white precipitate is visible when excess of sodium thiosulphate solution
is added.
I3– + 2 S2O32–  3 I– + S4O62–
 These reactions are used in quantitative analysis for the iodometric determination of copper.
 Potassium ferrocyanide (Potassium hexacyanidoferrate (II) ) solution : Cu2+ ions gives brown/chocolate
brown precipitate.
2Cu2+ + K4Fe(CN)6  Cu2[Fe(CN)6]  + 4K+
(vi) Potassium cyanide : When added sparingly forms first a yellow precipitate.
Cu2+ + 2CN–  Cu(CN)2 (yellow)
Precipitate quickly decomposes into CuCN and cyanogen.
2 Cu(CN)2   CuCN (white) + (CN)2  (highly poisonous)
Excess reagent dissolves the precipitate forming a colourless soluble complex.
CuCN  + 3 CN–  [Cu(CN)4]3–
Complex is so stable that H2S cannot precipitate Cu (I) sulphide (distinction from cadmium).
3. BISMUTH ION (Bi3+) :
 Precipitation with H2S in acidic medium : Black precipitate is formed which is soluble in cold dilute HNO3
and ammonium sulphide.

H
2Bi3+ + 3H2S 
 Bi2 S3  (black) + 6H+
Bi2 S3 + 8HNO3  2Bi (NO3)3 + 2NO + 3S + 4H2O
 Bi S  + 6 HCl (boiling concentrated)
2 3
 2 Bi3+ + 6 Cl– + 3 H2S

PAGE # 245
 Alkaline sodium stannite (Sodium tetrahydroxidostannate (II)) : A black precipitate of metallic bismuth is
obtained.
Bi3+ + 3 OH–  Bi(OH)3 
2Bi(OH)3  + [Sn(OH)4]2–  2Bi  + 3 [Sn(OH)6]2–
(OA) (RA)
 The reagent must be freshly prepared and test must be carried out in cold solution.
 Dilution with water : Solution of bismuth salts gives white precipitate when water is added in larger quantity.
Bi3+ + NO3– + H2O  BiO(NO3)  + 2H+
Bi3+ + Cl– + H2O  BiOCl  (bismuth oxychloride or bismuthyl chloride) + 2H+
Soluble in mineral acids (dilute) but insoluble in tartaric acid (distinction from antimony) and in alkali hydroxide
(distinction from tin).
 Potassium iodide : When the reagent is added dropwise to a solution containing Bi3+ ions , a black precipitate
is formed.
Bi3+ + 3–  Bi3 
The precipitate dissolves in excess KI forming orange coloured soluble complex.
Bi3 + – [Bi4]–

4. CADMIUM ION (Cd2+) :


 Precipitation with H2S in acidic medium : Yellow precipitate is formed which dissolves in hot dilute HNO3 .
H
Cd2+ + H2S   CdS  + 2H+
CdS + 8HNO3  3Cd (NO3)2 + 4H2O + 2NO + 3S

 Ammonia solution (Dropwise addition) : Ammonium hydroxide first gives white precipitate of Cd(OH)2 which
gets dissolve in excess of reagent forming a soluble complex.
Cd2+ + 2 NH3 + 2 H2O Cd(OH)2  + 2 NH4+
Cd(OH)2 + 4 NH3  [Cd(NH3)4]2+ + 2 OH–
.

 Potassium cyanide : Initially a white precipitate of Cd(CN)2 is formed which in excess of reagent dissolves
forming a soluble complex.
Cd2+ + 2 CN–  Cd(CN)2 
Cd(CN)2  + 2 CN–  [Cd(CN)4]2–
The colourless soluble complex is not too stable, therefore, reacts with H2S gas forming a yellow precipitate of
CdS.
[Cd(CN)4]2– + H2S  CdS  + 2 H+ + 4 CN–

 KI forms no precipitate (distinction from Copper)


5. LEAD ION (Pb2+) :
 Precipitation with H2S in acidic medium : Black precipitate is formed which is soluble in hot dilute HNO3 .
Pb2+ + H2S  PbS  (black) + 2H+
3PbS + 8HNO3  3Pb(NO3)2 + 2NO + 4H2O + 3S
 Dilute H2SO4 : White precipitate is formed which is soluble in ammonium acetate.
Pb(NO3)2 + H2SO4  PbSO4  (white) + 2HNO3
PbSO4 + 2CH3 COONH4  (CH3COO)2 Pb + (NH4)2 SO4
 Potassium iodide : Yellow precipitate is formed which is soluble in excess more concentrated (6M) solution of
the reagent. Yellow precipitate of PbI2 is moderately soluble in boiling water to give a colourless solution.
(CH3COO)2 Pb + 2KI  2CH3 COOK + PbI2 yellow

 Potassium dichromate : Yellow precipitate is formed.


(CH3COO)2 Pb + K2CrO4  2CH3 COOK + PbCrO4 (yellow)

PAGE # 246
IIIrd Group (Al+3, Cr+3, Fe+3)

 Group Boil off


Filtrate   

Precipitate. Filtrate, move


for IVth group.

If gelatinous white If reddish brown If green


precipitate (Al(OH)3). precipitate (Fe(OH)3). precipitate (Cr(OH)3).

Dissolve in dilute
HCl (AlCl3). Dissolve in dilute Fuse the precipitate with
HCl (FeCl3) fusion mixture
& divide the (KNO3 + Na2CO3)
Add sodium solution into two parts. & extract with
hydroxide solution. water (Na2CrO4).
Divide solution into two parts.
White precipitate
dissolves in
excess of precipitant Ist part + IInd part +
(NaAlO2). K4[Fe(CN)6]. KSCN.
Ist part + IInd part +BaCl2.
Add NH4Cl and heat. CH3COOH + (CH3COO)2Pb.

Prussian blue (aq)/ Blood red (aq)


Gelatinous precipitate of of Fe(SCN)3. Yellow precipitate Yellow precipitate
white precipitate Fe4[Fe(CN)6]3. of PbCrO4. of BaCrO4.
of Al(OH)3.

1. ALUMINIMUM ION (Al3+) :


 Precipitation with NH4OH in presence of NH4Cl : White gelatinous precipitate is formed.
NH 4 Cl
Al3+ + 3NH4OH    Al(OH)3  + 3NH4+
 Sodium hydroxide : A solution containing Al +3 ions give a white precipitate with the reagent.
Al3+ + 3OH–  Al(OH)3 
White precipitate dissolves in excess sodium hydroxide according to following reaction.
Al(OH)3 + OH– [Al(OH)4]–
The reaction is reversible and any reagent, which will reduce the hydroxyl ion concentration sufficiently should
cause the reaction to proceed from right to left with the consequent precipitation of aluminimum hydroxide. This
may be effected with a solution of ammonium chloride (the hydroxyl ion concentration is reduced owing to the
formation of the weak base ammonia , which can be readily removed as ammonia gas by heating) or by the
addition of acid ; in the latter case, a large excess of acid causes the precipitated hydroxide to redissolve.
[Al(OH)4]– + NH4+  Al(OH)3  + NH3 + H2O
[Al(OH)4]– + H+ Al(OH)3  + H2O
Al(OH)3 + 3H+ (excess) Al3+ + 3 H2O

 Lake Test :
AlCl3 + 3NH4OH  Al(OH)3  (white) + 3 NH4Cl
The precipitation of aluminium hydroxide is done in presence of blue litmus. Originally the solution will appear red
when the group precipitate is dissolved in HCl as blue litmus turns red in acid medium. On adding large excess
of NH4OH, Al(OH)3 will be reprecipitated and will adsorb the litmus as it is flocculent by nature. After a while a
blue mass will be seen floating in a colourless solution as all the colour from the solution will adsorbed.

PAGE # 247
 Dry test : Aluminium compounds when heated with sodium carbonate upon charcoal gives a white infusible solid,
which glows when hot. If the residue is moistened with a little cobalt nitrate solution and again heated, a blue
infusible mass is obtained.
2 Al2O3 + 2 Co2+ + 4 NO3–  2 CoAl2O4 (thenard blue) + 4NO2  + O2 
Use of excess cobalt nitrate solution should be avoided since this will produce black cobalt oxide (Co3O4) upon
ignition, which will mask the blue colour.

2. IRON ION (Fe3+) :


 Precipitation with NH4OH in presence of NH4Cl : Gelatinous reddish brown precipitate is formed which is
insoluble in excess reagent but soluble in acids.
NH4 Cl
Fe3+ + 3NH4OH   Fe(OH)3  + 3NH4+
 Potassium ferrocyanide (Potassium hexacyanidoferrate(II) ) : Intense blue precipitate (Prussian blue) of
iron(III) hexacyanidoferrate(II) is formed.
4Fe3+ + 3 [Fe(CN)6]4–  Fe4[Fe(CN)6]3 

 Potassium ferricyanide (Potassium hexacyanidoferrate(III)) : A brown colouration is formed.


Fe3+ + [Fe(CN)6]3–  Fe[Fe(CN)6]

3Fe2+ + 2K3 [Fe(CN)6]  Fe3 [Fe (CN)6]2 (ferrous ferric cyanide) + 6K+
Turnbull's blue
 Potassium thiocyanate (Potassium sulphocyanide) : In slightly acidic medium, a deep red colouration is
produced due to the formation of a non-dissociated iron(III) thiocyanate complex.
Fe3+ + 3 SCN–  Fe(SCN)3
This neutral molecule can be extracted by ether or amyl alcohol.

3. CHROMIUM ION (Cr3+) :

 Precipitation with NH4OH in presence of NH4Cl : A grey-green to green gelatinous precipitate is formed.
Cr3+ + 3 NH3 + 3 H2O  Cr(OH)3 + 3 NH4+

 Sodium hydroxide solution : A green precipitate is formed.


Cr3+ + 3 OH– 
 Cr(OH)3 
 The green precipitate of Cr(OH)3 can be dissolved by using any of the following methods.
(A) Fusion with fusion mixture (Na2CO3 + KNO3)
2 Cr(OH)3 + 2Na2CO3 + 3 KNO3  2 Na2CrO4 + 3 KNO2 + 2 CO2 + 3 H2O
Fused mass on extraction with water gives yellow solution of Na2CrO4 .
(B) 2 Cr(OH)3 + 3 Na2O2  2Na2 CrO4 + 2NaOH + H2O
(C) Precipitate is heated with NaOH and bromine water.
2NaOH + Br2  NaOBr + NaBr + H2O ; NaOBr  NaBr + [O]
2Cr(OH)3 + 4NaOH + 3[O]  2Na2 CrO4 + 5H2O.

The yellow solution of Na2CrO4 gave the following reactions with lead acetate, barium chloride and silver nitrate
solutions.

(a) Lead acetate solution : Yellow precipitate is formed.


CH3 OOH
CrO42– + Pb2+   PbCrO4 
(b) Barium chloride solution : Yellow precipitate is formed owing to the formation of barium chromate which is
insoluble in acetic acid.
CrO42– + Ba2+  BaCrO4 

PAGE # 248
IV th GROUP (Zn 2+ , Mn 2+ , Ni 2+ , Co 2+ ) :

1. MANGANESE ION (Mn2+) :


 Precipitation with H2S in presence of NH4OH + NH4Cl : A buff coloured (light pink) precipitate is formed.

Mn2+ + S2–  MnS 


It is readily soluble in mineral acids (distinction with Ni and Co) and even in acetic acid (distinction with Ni, Co
and Zn).
MnS  + 2H+  Mn2+ + H2S ; MnS  + 2 CH3COOH  Mn2+ + H2S  + 2 CH3COO–
 Sodium hydroxide solution : Initially white precipitate of Mn(OH)2 is formed which is insoluble in excess
reagent and rapidly oxidised on exposure to air, becoming brown.
Mn2+ + 2 OH–  Mn(OH)2 
Mn(OH)2  + O2  2 MnO(OH)2 (hydrated manganese dioxide).
 With sodium hydroxide and bromine water initially a white precipitate is formed which immediately turns to
black owing to the formation of MnO2 .
Mn(OH)2  + Br2 + 2NaOH  MnO2(black) + 2NaBr + 2H2O
 Lead dioxide(PbO2) and concentrated nitric acid : On boiling a dilute solution of manganese(II) ions with
lead dioxide and a little concentrated nitric acid and allowing the suspended solid containing unattacked lead
dioxide to settle, the supernatant liquid acquired a violet-red (or purple) colour due to permanganic acid.
5 PbO2 + 2 Mn2+ + 4H+  2 MnO4– + 5Pb2+ + 2 H2O

PAGE # 249
2. ZINC ION (Zn2+) :
 Precipitation with H2S in presence of NH4OH + NH4Cl : A white precipitate is formed. In neutral solutions,
precipitation is partial as H+ ions concentration produced depressed the ionisation of H2S .
Zn2+ + H2S ZnS 
The precipitate is soluble in dilute HCl.
ZnS + 2H+  Zn2+ + H2S 
 Sodium hydroxide solution : A white gelatinous precipitate is formed.
Zn2+ + 2 OH–  Zn(OH)2 
The precipitate is soluble in acids as well as in excess of the reagent.
Zn(OH)2 + 2 H+  Zn2+ + 2 H2O
Zn(OH)2 + 2 OH–  [Zn(OH)4]2–
 Ammonia solution : A white gelatinous precipitate is formed which is readily soluble in excess of the reagent
and in solutions of ammonium salts forming the tetraamminezinc(II).
Non–precipitation of Zn(OH)2 by ammonia solution in the presence of NH4Cl is due to the lowering of OH– ion
concentration to such a value that the Ksp of Zn(OH)2 is not attained.
Zn2+ + 2 NH3 + 2 H2O Zn(OH)2  + 2 NH4+
Zn(OH)2  + 4 NH3 [Zn(NH3)4]2+ + 2 OH–
 Potassium ferrocyanide (Potassium hexacyanidoferrate(II)) solution : A white precipitate of variable composition
is formed. If excess reagent is added, the composition of precipitate is K2Zn3[Fe(CN)6]2 .
3Zn2+ + 2K++2 [Fe(CN)6]4–  K2Zn3[Fe(CN)6]2 
The precipitate is insoluble in dilute acids, but dissolves in sodium hydroxide readily.
K2Zn3 [Fe(CN)6]2 + 12 OH–  2 [Fe(CN)6]4– + 3 [Zn(OH)4]2–
This reaction can be used to distinguish zinc from aluminium.
3. COBALT ION (Co2+) :
 Precipitation with H2S in presence of NH4OH + NH4Cl : A black precipitate is formed. The black precipitate of
CoS is insoluble in dilute HCl or acetic acid but hot concentrated HNO3 or aquaregia dissolves it and white
sulphur remains. On longer heating the mixture becomes clear as because sulphur is oxidised to sulphate.
NH 4OH
CoCl2 + H2S  
 CoS  + 2HCl
3 CoS + 2 HNO3 + 6 H+  3 Co2+ + 3 S  + 2NO  + 4 H2O
2CoS + 6HCl + 2HNO3  3CoCl2 + 2NO + 3S + 4H2O
 Solution on evaporation to dryness gives blue residue (CoCl2) which turns pink on adding water.
 Potassium nitrite solution : A yellow precipitate is formed from neutral solution of cobalt(II) ions.
CoCl2 + 7KNO2 + 2CH3COOH  K3[Co(NO2)6]  + H2O + 2KCl + 2CH3COOK + NO 
or Co2+ + 7NO2– + 2H+ + 3K+  K3[Co(NO2)6]  + NO  + H2O.
 Ammonium thiocyanate solution : A neutral or acid solution of cobalt(II) gives a blue colouration in amyl
alcohol or ether layer when a few crystals of ammonium thiocyanate are added.
Co2+ + 4 SCN–  [Co(SCN)4]2–
 In amyl alcohol or ether, the free acid H2[Co(SCN)4] is formed and dissolved by the organic solvent
(distinction from nickel)
 Potassium cyanide solution :
Co2+ + 2 CN–  Co(CN)2  (reddish brown / buff coloured)
In excess of reagent, a brown solution of a soluble complex is formed.
Co(CN)2  + 4 CN–  [Co(CN)6]4–
On acidification precipitate reappears.
[Co(CN)6]4– + 4 H+ (cold and dilute)  Co(CN)2  + 4HCN 
 If brown solution is boiled for a longer time in air or some H 2O 2 is added and solution is heated it
turns yellow.
4 [Co(CN)6]4– + O2 + 2 H2O  4 [Co(CN)6]3– (yellow solution) + 4 OH–
2 [Co(CN)6]4– + H2O2  2 [Co(CN)6]3– (yellow solution) + 2 OH–

PAGE # 250
4. NICKEL ION (Ni2+) :
 Precipitation with H2S in presence of NH4OH + NH4Cl :
A black precipitate is formed which is insoluble in cold dilute HCl and CH3COOH but dissolves in hot concentrated
HNO3 and in aquaregia.
NH 4OH
NiCl2 + H2S  
 NiS  (Black) + 2HCl
3 NiS  + 2 HNO3 + 6 H+  3 Ni2+ + 2 NO  + 3 S  + 4 H2O
NiS + HNO3 + 3HCl  Ni2+ + S  + NOCl  + 2Cl– + 2H2O.
 Dimethylglyoxime reagent : A red precipitate is obtained from the solution just made alkaline or acid solutions
buffered with sodium acetate.
NiCl2+ 2NH4OH + 2CH3 – C = NOH  (C4H7N2O2)2Ni + 2NH4Cl + 2H2O
| CH3 – C = NOH
 Bromine water test (alkaline medium) : A black precipitate of Ni2O3 is formed.
NiCl2 + 2NaOH (excess)  Ni(OH)2  (green) + 2 NaCl
Green precipitate is formed which gradually turns black on adding bromine water.
Br2 + H2O  2HBr + [O] ; 2Ni(OH)2 + H2O + [O]  2 Ni(OH)3 

Vth Group (Ba2+, Sr2+, Ca2+) :


IV Group filtrate  Boil off H2S then add (NH4)2CO3 (aq), NH4OH & NH4Cl (s)

White precipitate move for


Filtrate, VI group.
(BaCO , SrCO
3 3

or CaCO ).3

Dissolve in CH COOH 3

and divide into three parts


and test in the sequence
given below.

I part + K CrO4.
2 II Part + (NH4)2SO . III part + (NH4)2C2O4.
4

White precipitate
(SrSO4).
Yellow precipitate White precipitate
(BaCrO4 insoluble (CaC2 O4).
in CH3COOH).

1.BARIUM ION ( Ba2+) :


 Precipitation with (NH4)2 CO3 in presence of NH4OH + NH4Cl : A white precipitate is formed which is
soluble in acetic acid and dilute mineral acidsBaCl2 + (NH4)2CO3  BaCO3 + 2NH4Cl
BaCO3 + 2CH3COOH  Ba (CH3COO)2 + H2O + CO2
 Potassium chromate test : A yellow precipitate is formed, practically insoluble in water
Ba(CH3COO)2 + K2CrO4  BaCrO4  + 2CH3COOK
 Precipitate is insoluble in dilute acetic acid (distinction from strontium and calcium) but readily soluble in mineral
acids.

PAGE # 251
2. STRONTIUM ION (Sr2+) :
 Precipitation with (NH4)2 CO3 in presence of NH4OH + NH4Cl :
A white precipitate is formed which is soluble in acetic acid.
SrCl2 + (NH4)2CO3  SrCO3 + 2NH4Cl
SrCO3 + 2CH3COOH  Sr(CH3COO)2 + H2O + CO2
 Ammonium sulphate solution : A white precipitate is formed which is slightly soluble in boiling hydrochloric
acid.
Sr(CH3COO)2 + (NH4)2SO4  SrSO4  + 2CH3COONH4
3. CALCIUM ION (Ca2+) :
 Precipitation with (NH4)2 CO3 in presence of NH4OH + NH4Cl : A white precipitate is formed. The precipitate
is soluble in water which contains excess carbonic acid (e.g freshly prepared soda water) because of the formation
of soluble hydrogen carbonate. On boiling precipitate reappears again, as CO2 is removed. The precipitate is also
soluble in acetic acid.
CaCl2 + (NH4)2CO3  CaCO3  + 2NH4Cl
CaCO3 + 2CH3COOH  Ca(CH3COO)2 + H2O + CO2
 Ammonium oxalate solution (concentrated) : A white precipitate is formed. The precipitation is facilitated by
making the solution alkaline. The precipitate is practically insoluble in water (Ksp = 2.6 × 10–9), insoluble in acetic
acid but readily soluble in mineral acids.
Ca(CH3COO)2 + (NH4)2C2O4  CaC2O4 + 2CH3COONH4
 Potassium hexacyanidoferrate (II) solution : White precipitate of a mixed salt is produced.
Ca2+ + 2 K+ + [Fe(CN)6]4–  K2Ca[Fe(CN)6] 
 The confirmatory test for the basic radicals of (V) group have to be done in the order of Ba2+, Sr2+ then Ca2+ (i.e
BSC) because Ba2+ give positive test with all the reagents used in the confirmatory test of these group radicals,
K2CrO4, (NH4)2 SO4 & (NH4)2 C2O4. So performs the test for Sr2+ with (NH4)2SO4 only when Ba2+ is absent.
Similarly Sr2+ gives the test with both (NH4)2SO4 and (NH4)2C2O4. So proceeds with Ca2+ only when Sr2+ is
absent otherwise it will respond to ammonium oxalate test.

Vth GROUP :
1. MAGNESIUM ION (Mg2+) :
 Disodium hydrogen phosphate solution : To the filtrate of V group or Mg2+ ions solution add 1 ml (NH4)2C2O4
solution and heat if white precipitate is formed then filter it. Now to filtrate add a solution of disodium hydrogen
phosphate. A white crystalline precipitate is formed in the presence of NH4Cl (prevent precipitation of Mg(OH)2)
and ammonia solution.
Mg2+ + NH3 + HPO42–  Mg(NH4)PO4 

 Ammonia solution : A white gelatinous precipitate is formed.


Mg2+ + 2 NH4OH  Mg(OH)2 + 2 NH4+
 Titan Yelllow (a water soluble yellow dyestuff) : It is adsorbed by Mg(OH)2 producing a deep red colour or
precipitate.
Dissolved the precipitate in dilute HCl (minimum qty.) and to 1 drop of this add 1 drop of NaOH solution (2 M)
followed by 1 drop of titan yellow solution a deep red colour solution or precipitate is obtained.
Solved Example
Ex.1 A compound on heating with an excess of caustic soda solution liberates a gas (B) which gives white fumes on
exposure of HCl. The resultant alkaline solution thus obtained after heating again liberates the same gas (B)
when heated with zinc powder. Compound (A) on heating alone gives a neutral oxide of nitrogen not nitrogen gas.
Identify (A) and (B) and give the relevant chemical reactions.
Sol. As NH3 gives white fumes with HCl, therefore, (B) should be NH3 and (A) should be the salt of ammonium.
Further we know that nitrite of ammonium gives NH3 with Zn and alkali and when heated alone gives neutral
oxide (N2O) not N2. Hence the salt should be ammonium nitrate not ammonium nitrite.
NH4 NO3 (A) + NaOH  NaNO3 + H2O + NH3  (B) ; NH3 + HCl  NH4Cl (white fumes)
NaNO3 + 8[H] Zn NaOH
    
 NaOH + 2H2O + NH3 ; NH4 NO3  N2O (neutral) + 2H2O

PAGE # 252
Ex.2 A certain metal (A) is boiled with dilute HNO3 to give a salt (B) and an neutral oxide of nitrogen (C). An aqueous
solution of (B) gives a white precipitate (D) with brine which is soluble in ammonium hydroxide. An aqueous
solution of (B) also gives red / brick red precipitate, (E) with potassium chromate solution. Identify (A) to (E)
and write the chemical reactions involved.
Sol. As solution of (B) gives white precipitate with NaCl (aq) and precipitate is soluble in ammonium hydroxide,
it may be of silver salt. Further it gives brick red precipitate with K2CrO4, therefore, metal (A) may be silver.
3Ag (A) + 4HNO3  3AgNO3 (B) + NO(C) + 2H2O;
AgNO3 + NaCl  AgCl  (white) (D) + NaNO3
AgCl + 2NH4OH  [Ag(NH3)]2Cl (soluble) + 2H2O
2AgNO3 + K2CrO4  Ag2CrO4  (red / brick red) (E) + 2KNO3
Ex.3 Which of the following salt will give white precipitate with the solution containing Pb2+ ions ?
(1) Na2CO3 (2) NaCl
(3) Na2SO3 (4) All of these
Sol. Pb2+ + CO32–  PbCO3  (white)
Pb2+ + 2Cl–  PbCl2  (white)
Pb2+ + SO32–  PbSO3  (white) ; Therefore, (D) option is correct.
Ex.4 Why Na2S cannot be used in place of H2S (in presence of HCl) as a reagent for IInd group cations ?
Sol. With Na2S, along with 2nd group cations, some cations of higher groups i.e., IIIrd and IVth groups may be
precipitated because the higher concentration of S2– ions is obtained according to following reaction.
Na2S 2 Na+ + S2–
To precipitate IInd group cation lower concentration of S2– ions is required (because of low Ksp of IInd group
sulphides) and this can be obtained easily by H2S in presence of dilute HCl.
H2S 2 H+ + S2– ; HCl H+ + Cl–
Due to common ion effect the ionisation of H2S is suppressed and the concentration of S2– ions obtained is
just sufficient to precipitate the cations of IInd group.
Ex.5 A black coloured compound (A) on reaction with dil. H2SO4 gives a gas (B) and a yellow colour solution. The
gas (B) on passing in a solution of an acid (C) gives a white/yellow turbidity (D). Gas (B) when passed in
acidified solution of (E) gives a precipitate (F) soluble in dil HNO3. After boiling this solution when excess of
NH4OH is added, a blue coloured compound (G) is formed. To this solution on addition of acetic acid and
aqueous potassium ferrocyanide a chocolate coloured precipitate (H) is obtained. On addition of an aqueous
solution of BaCl2 to an aqueous solution of (E), a white precipitate insoluble in HNO3 is obtained. Yellow colour
solution on reaction with ammonium hydroxide in presence of air gives reddish brown precipitate. Identify (A)
to (H).
Sol. FeS (A) + H2SO4  FeSO4 + H2S (B)
HNO3 (C)  NO2 + H2O + O ; H2S + O  H2O + S  (D)
CuSO4 (E) + H2S  CuS  (F) + H2SO4.
3CuS + 8HNO3  3Cu(NO3)2 + 2NO  + 4H2O + 3S 
Cu2+ + 4NH3  [Cu(NH3)4]+2 (G).
2Cu2+ + K4Fe(CN)6  Cu2[Fe(CN)6]  (H) + 4K+
Ba2+ + SO42–  BaSO4  (white)
Fe2+ + 2H+ + O  Fe3+ + H2O
Fe3+ + 3NH3 + 3H2O  Fe(OH)3  (reddish brown) + 3NH4+
Hence, (A) = FeS ; (B) = H2S ; (C) = HNO3 ; (D) = S ; (E) = CuSO4 ; (F) = CuS ; (G) [Cu(NH3)4](NO3)2 ;
(H) Cu2 [Fe(CN)6]
Ex.6 Thenard's blue is :
(1) CoAl2O4 (2) Fe4(Fe(CN)6)3 (3) Fe2[Fe(CN)6]3 (4) [Ni(NH3)6](OH)2

Sol.CoO + Al2O3  CoO . Al2O3 (Thenard's blue or cobalt meta-aluminate)
It is cobalt nitrate test.
Therefore, (A) option is correct.

PAGE # 253
Ex.7 Why the volume of Al(OH)3 precipitate decreases when excess of ammonium salts is added ?
Sol.A small portion of Al(OH)3 precipitate passes into the solution as colloidal Al(OH)3 i.e. aluminium hydroxide sol.
Ex.8 Why Zn(II) salt is not precipitated as Zn(OH)2 by ammonia solution in the presence of excess of ammonium
chloride ?
Sol.Zn2+ + 2 NH3 + 2 H2O Zn(OH)2 + 2 NH4+
It is due to the lowering of OH– ion concentration because of common ion effect of NH4+ to such a value that
the Ksp of Zn(OH)2 is not attained.
Ex.9 Explain the following.
(a) Reaction between NiCl2 and excess KCN solution.
(b) Reaction between cobalt chloride and sodium bicarbonate and bromine water.
(c) When manganese nitrate, sodium bismuthate (NaBiO3) solution and concentrated HNO3 is boiled.
(d) When precipitate of Zn(NH4)PO4 reacts with ammonia.
Sol.(a) A green precipitate is formed which dissolves in excess of reagent forming a yellow coloured solution.
NiCl2 + 2KCN  Ni(CN)2  + 2KCl ;
Ni(CN)2 + 2KCN  K2[Ni(CN)4]
(b) When sodium bicarbonate is added in excess to cobalt chloride solution followed by the bromine water and
then mixture is slightly heated, the solution turns apple green.
CoCl2 + 2NaHCO3  Co(HCO3)2 + 2NaCl
Co(HCO3)2 + 4 NaHCO3  Na4 [Co(CO3)3] + 3H2O + 3CO2
Br2 + H2O  2HBr + [O]
2Na4 [Co(CO3)3] + H2O + [O]  2Na3 [Co(CO3)3 ] (green) + 2NaOH
(c) 2 Mn2+ + 5 NaBiO3 + 14 H+  2MnO4– (violet–red/purple) + 5 Bi3+ + 5 Na+ + 7 H2O
(d) Zn(NH4)PO4  + 3NH3  [Zn(NH3)4]2+ + HPO42–.
Ex.10 Column I and column II contains four entries each. Entries of column I are to be matched with some entries
of column II. Each entry of column I may have the matching with one or more than one entries of column II.
Column I
(A) Forms coloured precipitate on addition of CrO42– ions but precipitate dissolves in ammonia solution.
(B) Forms coloured precipitate with ammonia solution but dissolves forming
(C) Forms coloured precipitate on addition of I– ions but in excess of precipitant precipitate dissolves forming
coloured solution.
(D) Forms white precipitate, when salt (chloride or nitrate) is poured into a large volume of water.
Column II
(p) Ag+
(q) Cu2+ coloured solution in excess of precipitant.
(r) Ni2+
(s) Bi3+
Sol.(A - p) ; (B - q, r) ; (C - s) ; (D - s)
(A) 2Ag+ + CrO42–  Ag2CrO4  (red)
Ag2CrO4 + 4NH3  2[Ag(NH3)2]+ + CrO42–
(B) 2Cu2+ + SO42– + 2NH3 + H2O  Cu(OH2)2. CuSO4  (blue) + 2NH4+
Cu(OH)2 . CuSO4  + 8NH3  2 [Cu(NH3)4]2+]SO42– (intense blue) + 2OH–
Ni2+ + 2NH3 + 2H2O  Ni(OH)2  (green) + 2NH4+
Ni(OH)2  + 6NH3  [Ni(NH3)6]2+ (deep blue) + 2OH–
(C) Bi3+ + 3I–  BiI3  (black)
BiI3 + I–  [BiI4]– (orange colouration)
(D) Bi3+ + NO3– + H2O  BiO(NO3)  (white) + 2H+
basic salt

PAGE # 254
MISCELLANEOUS SOLVED PROBLEMS (MSPs)
1. BaCl2 solution gives a white precipitate with a solution of a salt, which dissolves in dilute hydrochloric acid
with the evolution of colourless, pungent smelling gas. The gas as well as the salt both are used as bleaching
agent in the textile industries. The salt contains:
(1) sulphite (2) sulphide (3) acetate (4) carbonate
Ans. (1)
Sol. Ba2+ + SO32–  BaSO3  (white)
BaSO3 + 2HCl  BaCl2 + SO2(colourless pungent smelling gas) + H2O
SO32– and SO2 both act as bleaching agent.
2. Which of the following precipitate(s) does / do not dissolve in excess of ammonia solution ?
(1) Zn(OH)2 (2) Ni(OH)2 (3) Al(OH)3 (4) (B) and (C) both
Ans. (3)
Sol. (1) Zn(OH)2 + 4 NH3  [Zn(NH3)4]2+ (colourless solution) + 2OH–
(2) Ni(OH)2  + 6NH3  [Ni(NH3)6]2+
(deep blue solution) + 2OH–
(3) Al(OH)3 + NH3  No reaction.
3. Chocolate brown precipitate is formed with :
(1) Cu2+ ions and [Fe (CN)6]3– (2) Cu2+ ions and [Fe(CN)6]4–
(3) Fe2+ ions and [Fe (CN)6]4– (4) Fe2+ ions and dimethylglyoxime
Ans. (2)
Sol (1) Cu3 [Fe(CN)6]2 (green) ; (2) Cu2 [Fe(CN)6]  ( chocolate brown)
(3) Fe4[Fe(CN)6]3 (Prussian blue) ; (4) red solution of iron(II) dimethylglyoxime.
4. Pink colour of acidified KMnO4 is decolourised but there is no evolution of any gas. This may happen with the
compound containing the following acid radical.
(1) SO 23 – (2) NO 2– (3) S2– (4) All of these
Ans. (4)
Sol. (1) 5SO32–+ 2MnO4– + 6H+  2Mn2+ + 5SO42– + 3H2O
(2) 2MnO4– + 5NO2– + 6H+  2Mn2+ + 5NO3– + 3H2O
(3) 2MnO4– + H2S + 6H+  2Mn2+ + 5S  + 8H2O
5. Which of the following gives a precipitate with Pb(NO3)2 but not with Ba(NO3)2?
(1) Sodium chloride (2) Sodium acetate
(3) Sodium nitrate (4) Disodium hydrogen phosphate
Ans. (A)
Ans. (1) Pb2+ + 2Cl–  PbCl2(white) ;
Ba2+ + 2Cl–  BaCl2(water soluble)
(2) (CH3COO)2 Pb and (CH3COO)2Ba both are water soluble salts.
(3) Nitrates are mostly soluble in water
(4) 3Pb2+ + 2HPO42–  Pb3(PO4)2  (white) + 2H+ ; Ba2+ + HPO42–  BaHPO4  (white)
6. Colour of cobalt chloride solution is :
(1) pink (2) black (3) colourless (4) green
Ans. (1)
Sol. Anhydrous Co(II) salts are blue in colour while hydrated Co(II) salts are pink/red.
7. A red colouration or precipitate is not obtained when :
(1) Fe3+ reacts with potassium thiocyanate (2) Fe2+ reacts with dimethylglyoxime.
2+
(3) Hg reacts with potassium iodide. (4) None
Ans. (4)
Sol. (1) Fe3+ + 3SCN–  Fe(SCN)3 (red solution)
(2) Red solution of iron(II) dimethylglyoxime.
(3) Hg2– + 2I–  HgI2  (red).
8. When H2S gas is passed through an ammonical salt solution X, a slightly white precipitate is formed. The X
can be :
(1) a cobalt salt (2) a lead salt (3) a zinc salt (4) a silver salt

PAGE # 255
Ans. (3)
Sol. Zn2+ + H2S  ZnS  (white) + 2H+
9. Consider the following statement :
S1 : Cu2+ ions are reduced to Cu+ by potassium iodide and potassium cyanide both, when taken in excess
S2 : H2S will precipitate the sulphide of all the metals from the solutions of chlorides of Cu, Zn and Cd if the
solution is aqueous.
S3 : The presence of magnesium is confirmed in qualitative analysis by the formation of a white crystal
line precipitate of MgNH4 PO4.
S4 : Calomel on reaction with potassium iodide gives red precipitate.
and arrange in the order of true /false.
(1) T T F F (2) T F T F (3) T T T T (4) T T T F
Ans. (4)
Sol. S1, S2 and S3 are correct statements.
S4 : Hg22+ + 2I–  Hg2I2  (green)

10. Statement - 1 : Addition of NH4OH to an aqueous solution of BaCl2 in presence of NH4Cl (excess) precipi
tates Ba(OH)2 .
Statement - 2 : Ba(OH)2 is water soluble.
(1) Both Statement-1 and Statement-2 are true and Statement-2 is the correct explanation of Statement-1.
(2) Both Statement-1 and Statement-2 are true but Statement-2 is not correct explanation of Statement-1.
(3) Statement-1 is true but Statement-2 is false.
(4) Statement-1 is false but Statement-2 is true
Ans. (4)
Sol. Ba2+ ions does not give any precipitate with NH4OH solution in excess of NH4Cl because product formed,
Ba(OH)2 is soluble in water
11. Statement - 1 : Sodium meta aluminate on boiling with ammonium chloride produces white gelatinous precipitate.
Statement - 2 :Aluminium hydroxide is formed which is not soluble in water
(1) Both Statement-1 and Statement-2 are true and Statement-2 is the correct explanation of Statement-1.
(2) Both Statement-1 and Statement-2 are true but Statement-2 is not correct explanation of Statement-1.
(3) Statement-1 is true but Statement-2 is false.
(4) Statement-1 is false but Statement-2 is true
Ans. (1)
NH4 Cl
Sol. [Al(OH)4]–   Al(OH)3  + OH–
boil

12. Which of the following statement(s) is (are) incorrect?


(1) Fe2+ ions give a dark blue precipitate with potassium hexacyanidoferrate (III) solution.
(2) Fe3+ ions give intense blue precipitate with potassium hexacyanidoferrate (II) solution.
(3) Fe3+ ions give a brown colouration with potassium hexacyanidoferrate (III) solution.
(4) Fe2+ ions give a deep red colouration with ammonium thiocyanate.
Ans. (4)
Sol. (1) Fe2+ + [Fe(CN)6]3–  Fe3+ + [Fe(CN)6]4–
4Fe + 3 [Fe(CN)6]4–  Fe4[Fe(CN)6]3(turnbull's blue)
3+

(2) 4Fe3+ + 3[Fe(CN)6]4–  Fe4[Fe(CN)6]3


(intense blue)
3+ 3–
(3) Fe + [Fe(CN)6]  Fe[Fe(CN)6]
(brown colouration)
3+ –
(4) Fe + 4SCN  Fe (SCN)3 (deep red colouration)
Fe2+ + 4 SCN–  No reaction
13. Which of the following pair (s) of ions would be expected to form precipitate when dilute solutions are mixed?
(1) NH4+, [Co(NO2)6]3– (2) NH4+, CO32– (3) Fe3+, OH– (4) Ba2+, SO42–
Ans. (1,3,4)

PAGE # 256
Sol. (1) NH4+ + [Co(NO2)6]3–  (NH4)3 [Co(NO2)6]  (yellow)
(2) Ammonium and alkali metal carbonates are water soluble.
(3) Fe3+ + OH–  Fe(OH)3  (reddish - brown)
(4) Ba2+ + SO42–  BaSO4  (white)

Aqueous solution of salt (A)

NaOH(aq)/warm

Gas(B) Solution of salt (C)


Colourless gas which is FeSO4
alkaline in nature conc. H2SO4 Zn/NaOH /heat
dil.HCI
White fumes Brown ring (D) Gas(B)
at the junction
of the two layers
Salt (A) on heating gives a colourless neutral gas which supports combustion.
From the a fore said, flow diagram, answer the following questions.
14. The compound (A) contains the following acid radical.
(1) NO2– (2) NO3– (3) Br– (4) SO32–
Ans. (2)
Sol. NO3– and NO2– both give brown ring test and reduction of NO3– and NO2– both give ammonia which with dilute
HCl gives dense white fumes.

if NH4NO3  N2O + 2H2O ; N2O supports th e combustion

but NH4NO2  N2 + 2H2O ; Nitrogen does not supports combustion.
Hence, the anion is NO3–.
15. The basic radical of salt (A) and gas B both gives brown precipitate with Nessler’s reagent. The composition
of the brown precipitate is :
(1) (NH4)2 [HgI4] (2) Hg(NH2) NO3 (3) HgO. Hg (NH2)I (4) (NH4)3[Co(NO2)6]
Ans. (3)
Sol. NH4+ + 2[HgI4] + OH–  HgO. Hg (NH2)I + 7I– + 3H2O
Hence the cation is NH4+.
16. Which of the following statement is correct ?
(1) Salt (A) gives yellow precipitate with chloroplatinic acid as well as with sodium cobaltinitrite.
(2) The brown ring is formed due to the formation of nitroso ferrous sulphate [Fe(NO)]2+SO4–.
(3) Salt ‘C’ reacts with silver nitrate solution to form white precipitate.
(4) (A) and (B) both.
Ans. (4)
Sol. (1) 2NH4+ + [PtCI6]4–  (NH4)2 [PtCl6]  (yellow)
3NH4+ + [Co(NO2)]3–  (NH4)3 [Co(NO2)6]  (yellow)
(2) 2NO3– + 4H2SO4 + 6Fe2+  6Fe3+ + 2NO  + 4SO42– + 4H2O
SO42– + Fe2+ + NO  [Fe(NO)]2+ SO42–
(3) Ag NO3 + NaNO3  No reaction.
If the anion is NO2– then Ag+ + NO2–  Ag NO2  (white)
Reactions :
NH4NO3 + NaOH  NH3 + NaNO3
(A) (B) (C)
NH3+ HCI  NH4CI (White)
NO3– + 4Zn + 7OH– + 6H2O  NH3 + 4[Zn(OH)4]2–

PAGE # 257
True/False :
17. Magnesium is precipitated from its salt solution as only magnesium ammonium phosphate by adding diso-
dium hydrogen phosphate solution in absence of ammonium chloride and aqueous ammonia.
Sol. (False) Precipitation is carried out in presence of ammonium chloride and aqueous ammonia as they prevent
precipitation of magnesium hydroxide.
Mg2+ + NH3 + HPO42–  Mg (NH4) PO4  (white)
18. When a solution of nitrite acidified with dilute hydrochloric acid is treated with solid urea, the nitrite is
decomposed, and nitrogen and carbon dioxide are evolved.
Sol. (True) CO(NH2)2 +HNO2  2N2  + CO2  + 3H2O.
19. Solution of alkali metal cyanide containing freshly prepared iron (II) sulphate solution and dilute H2SO4 on
exposure to air produces Prussian blue precipitate
Sol. (True) Fe2+ + 2 CN–  Fe(CN)2  ; Fe(CN)2  + 4CN–  [Fe(CN)6]4–
4Fe2+ + O2 + 4H+  4Fe3+ + 2H2O ; Fe3+ [Fe(CN)6]4–  Fe4[Fe(CN)6]3 
Subjective :
20. What happens when ?
(A) Aqueous solution of CrCl3 is added to ammonia solution.
(B) Ammonium carbonates reacts with MgCl2 (i) in absence of ammonium salts and (ii) in presence of
ammonium salts :
Sol. (A) Cr3+ + 3NH3 + 3H2O  Cr(OH)3  (green) + 3NH4+
Cr(OH)3 precipitate formed becomes slightly soluble in excess of precipitant in cold forming a violet or pink
solution containing [Cr(NH3)6] 3+ complex ions.
Cr(OH)3  + 6 NH3  [Cr(NH3)6]3+ + 3OH–
(B) (i) 5Mg2+ + 6 CO23 + 7H2O  4MgCO3. Mg(OH)2. 5 H2O  + 2HCO3–
White precipitate of basic magnesium carbonate is formed.
(ii) In presence of ammonium salts no precipitation occurs, because the equilibrium
NH4+ + CO32–  NH3+ HCO3– is shifted towards the formation of HCO3– ions.
21. Salts given in column (I) reacts with the excess of reagents given in column (II) and form white /coloured
precipitates. Select the correct options for the salts given in column (I) with the reagent(s) given in the
column (II)
Column - I Column - II
(1) Zn (NO3)2 (p) Sodium hydroxide
(2) Cu (NO3)2 (q) Ammonia solution
(3) Fe(NO3)3 (r) Disodium hydrogen phosphate
(4) Ag(NO3)2 (s) Potassium ferrocyanide
Ans. (1 - r, s) ; (2 - p, r, s) ; (3 - p, q, r, s) ; (4 - p, q, r, s)
Sol. (1) Zn2+ + 2OH–  Zn(OH)2 (white) ;
Zn (OH)2 + 2OH–  [Zn(OH)4]2– (colourless soluble complex)
Zn2+ + 2NH3 + 2H2O  Zn (OH)2  (white) + 2NH4+
Zn(OH)2  + 4NH3  [Zn(NH3)4]2+ (colour less soluble complex) + 2OH–
3Zn2+ + 2HPO42–  Zn3 (PO4)2  (white) + 2H+
3 Zn2+ + 2K+ + 2[Fe(CN)6]4–  K2 Zn3 [Fe(CN)6]2  (bluish white)
(2) Cu2+ + 2OH–  Cu(OH)2  (blue)
Cu2+ + 4NH3  [Cu(NH3)4]2+ (deep blue soluble complex)
Cu2+ + 2HPO42–  Cu3(PO4)2 (blue) + 2H+
Cu2+ [Fe(CN)6]4–  Cu2[Fe(CN)6]  (chocolate brown)
(3) Fe3+ + 3OH–  Fe (OH)3  (reddish brown)
Fe3+ + 3NH3 + 3H2O  Fe(OH)3  (reddish brown) + 3NH4+
Fe3+ + HPO42–  FePO4  (yellowish-white) + H+
4Fe3+ + 3 [Fe(CN)6]4–  Fe4 [Fe(CN)6]3  (intense blue)
(4) 2Ag+ + 2OH–  Ag2O  (brown) + H2O
2Ag+ + 2NH3 + H2O  Ag2O (brown) + 2NH4+
4Ag+ + HPO42–  Ag3PO4  (yellow) + H+
4Ag+ + 3 [Fe(CN)6]4–  Ag4[Fe(CN)6]  (white)

PAGE # 258
10. When a mixture of solid NH4Cl, solid K2Cr2O7 is heated
EXERCISE-1 with concentrated H2SO4, deep red vapours are obtained.
This is due to the formation of :
I. Dilute H2SO4 group (A) chromous chloride (B) chromyl chloride
(C) chromic chloride (D) chromic sulphate
1. The carbonate of which of the following cation is
11. AgCl dissolves in ammonia solution giving :
insoluble in water ?
(A) Ag+, NH4+ and Cl– (B) Ag(NH3)+ and Cl–
(A) Cs+ (B) K+ 2+ –
(C) Ag2(NH3) and Cl (D) Ag(NH3)2+ and Cl–
(C) NH4+ (D) Ba2+
12. A solution of a salt with concentrated H 2SO 4 acid
2. A substance on treatment with dilute H2SO4 liberates a
produces violet colour vapours which turns starch paste
colourless gas which produces (i) turbidity with baryta
blue. The salt may be :
water and (ii) turns acidified dichromate solution green.
(A) chloride (B) nitrate
The reaction indicates the presence of :
(C) bromide (D) iodide
(A) CO32– (B) S2–
2–
(C) SO3 (D) NO2–
13. A solution of a salt in concentrated H2SO4 acid produced
a deep blue colour with starch iodide solution. The salt
3. Colourless salt (A) + dil. H2SO4 or CH3COOH + Kl  may be
blue colour with starch. (A) can be (A) chloride (B) carbonate
(A) K2SO3 (B) Na2CO3 (C) acetate (D) bromide
(C) NaNO2 (D) NH4Cl
14. A colourless solution of a compound gives a precipitate
4. Sodium sulphide reacts with dilute H2SO4. Gas liberated with AgNO3 solution but no precipitate with a solution of
can : Na2CO 3 . The action of concentrated H 2SO 4 on the
(A) turn lead acetate paper black compound liberates a suffocating reddish brown gas.
(B) turn lime water milky The compound is :
(C) give both of the above tests (A) Ba(CH3COO)2 (B) CaCl2
(D) give none of the above tests (C) NaI (D) NaBr

5. An aqueous solution of substance 'X' with freshly 15. Nitrate is confirmed by ring test. The brown colour of the
prepared FeSO 4 solution on adding acetic acid slowly ring is due to formation of :
along the walls of test tube develops brown ring at the (A) ferrous nitrite (B) nitroso ferrous sulphate
junction of to layers. The anion present in 'X' is : (C) ferrous nitrate (D) FeSO4 .NO2 .
(A) NO2– (B) SO32–
(C) NO3– (D) Br– 16. Nitrates of all the metals except mercury and bismuth
are :
6. A mixture when rubbed with dilute acid smells like (A) coloured (B) unstable
vinegar. It contains : (C) soluble in water (D) insoluble in water
(A) sulphite (B) nitrate
(C) nitrite (D) acetate 17. An inorganic salt when heated with concentrated H2SO4
evolves a colourless pungent smelling gas but with
7. W hen a salt is heated with dilute H 2SO 4 and KMnO 4 concentrated H 2 SO 4 and MnO 2 evolves a coloured
solution, the pink colour of KMnO 4 is discharged, the pungent smelling gas which bleaches moist litmus
salt is : paper. The coloured gas is :
(A) a sulphite (B) a carbonate (A) NO2 (B) Cl2
(C) a nitrate (D) a bicarbonate (C) Br2 (D) I2

8. Solution of a salt in dilute H2SO4 or acetic acid produces 18. Mix the AlCl3 thoroughly with about three times as much
deep blue colour with starch iodide solution. The salt powdered potassium dichromate, and place the mixture
contains : in distillation flask. Add an equal volume of concentrated
(A) Br– (B) I– sulphuric acid and warm the mixture gently. Vapours of
(C) NO3– (D) NO2– the compound (X) are formed. The colour of the vapours
II. Concentrated H2SO4 group of compound (X) is :
(A) deep red (B) greenish-yellow
9. An inorganic salt when heated with concentrated H2SO4 (C) purple (D) dark brown
evolves a colourless pungent smelling gas but with
concentrated H 2 SO 4 and MnO 2 evolves a coloured 19. Chromyl chloride vapours are dissolved in water and
pungent smelling gas which bleaches moist litmus acetic acid and barium acetate solution is added, then:
paper. The coloured gas is : (A) the solution will remain colourless.
(A) NO2 (B) Cl2 (B) the solution will become dark green.
(C) Br2 (D) I2 (C) a yellow solution will be obtained.
(D) a yellow precipitate will be obtained.

PAGE # 259
20. When CS2 layer containing both Br2 and I2 (2 : 1) is shaken V. Ist Group
with excess of chlorine (Cl2) water, the violet colour due
to I2 disappears and a pale yellow colour appears in the 29. Group reagent for Ist group radicals is :
solution. The disappearance of violet colour and (A) KCl (concentrated)
appearance of pale yellow colour is due to the formation (B) HCl (concentrated)
of : (C) HCl (dilute)
(A) I3– and Br2 respectively. (D) none of these
(B) HIO3 and BrCl respectively.
(C) ICl and BrCl respectively. 30. Which one of the following salts will not produce clear
(D) I– and Br– respectively. and transparent original solution in 2M HCl ?
(A) Ag2CO3 (B) Pb(CO3)2
21. A solution of compound (X) on heating with zinc and (C) Hg2CO3 (D) All of these
sodium hydroxide gives ammonia gas. The anionic
species of compound (X) may be : 31. A metal nitrate reacts with KI solution to give yellow
(A) NO2– (B) NO3– precipitate which on addition of excess of more
(C) both (A) and (B) (D) none of these concentrated solution (6 M) of KI dissolves forming a
solution. The cation of metal nitrate is :
III. Precipitation Reactions (A) Hg22+ (B) Ag+
2+
(C) Pb (D) Cu2+
22. W hich one of the following reagents gives white
precipitated with SO42– ions? 32. Cu2+ and Ag+ both are present in the same solution . To
(A) Ba(NO3)2 (B) NH4NO3 precipitate one of the ions and leaves the other in
(C) NaNO3 (D) Be(NO3)2 solution, add :
(A) H2S (aq) (B) HCl (aq)
23. Precipitate of PbSO4 is soluble in : (C) HNO3(aq) (D) NH4NO3(aq)
(A) ammonium acetate (6M) (B) dilute HCl
(C) dilute H2SO4 (D) none 33. AgNO3 gives red precipitate with :
(A) NaI (B) NaOH
24. A metal salt solution gives a yellow precipitate with silver (C) NaCl (D) Na2CrO4
nitrate. The precipitate dissolves in dilute nitric acid as well
as in dilute ammonia solution. The solution contains : 34. Consider the following observation :
(A) bromide (B) iodide 
Mn+ + HCl (dilute)  white precipitate  water
(C) phosphate (D) chromate 2–
CrO 4
soluble   Yellow precipitate.
IV. Zero Group The metal ion Mn+ will be :
(A) Hg2+ (B) Ag+
25. Nessler’s reagent is : (C) Pb2+ (D) Sn2+
(A) K2 Hg4 (B) K2 Hg4 + KOH
(C) K2Hg 2 + KOH (D) K2 Hg4 + K 35. Identify the compound which turns black with ammonia
solution.
26. Ammonia/ammonium ion gives yellow precipitate with : (A) Lead chloride
(A) H2PtCl6 (B) HgCl2 (B) Mercurous chloride
(C) Na3[Co(NO2)6] (D) (A) and (C) both (C) Mercuric chloride
(D) Silver chloride
KOH
27. NH4Cl + Nessler’s reagent 
 Brown precipitate 36. To a solution of a substance, gradual addition of
(X) ammonium hydroxide results in a brownish black
Chemical composition of (X) is : precipitate which does not dissolve in excess of NH4OH.
(A) Hg(NH2)Cl (B) Hg(NH2)Cl + Hg However, when KI (not in excess) is added to the original
(C) HgO.Hg(NH2)I (D) HgO.Hg(NH2)NO3 solution, a green precipitate is formed. The solution
contained :
28. Ammonium salts on heating with slaked lime liberates (A) Lead salt (B) Silver salt
a colourless gas (X). Identify the incorrect statement for (C) Mercurous salt (D) Copper salt.
gas (X).
(A) (X) turns red litmus blue and produces dense white 37. A white crystalline substance dissolves in water. On
fumes in contact with dilute HCl. passing H 2S in this solution, a black precipitate is
(B) (X) turns filter paper moistened with mercurous obtained. The black precipitate dissolves completely in
nitrate black. hot HNO3. On adding a few drops of concentrated H2SO4,
(C) (X) when passed through Nessler's reagent a white precipitate is obtained which is soluble in
produces a red colour precipitate. ammonium acetate. The white precipitate is that of:
(D) (X) gives intense blue coloured solution with aqueous (A) BaSO4 (B) SrSO4
solution of CuSO4. (C) PbSO4 (D) Ag2SO4

PAGE # 260
38. Three separate samples of a solution of a single salt 48. When excess of dilute NH4OH is added to an aqueous
gave these results. One formed a white precipitate with solution of copper sulphate an intense blue colour is
excess ammonia solution, one formed a white precipitate developed. This is due to the formation of :
with dilute NaCl solution and one formed a black (A) [Cu(NH3)6]2+ (B) Cu(OH)2
precipitate with H2S. The salt could be (C) [Cu(NH3)4]2+ (D) (NH4)2SO4
(A) AgNO3 (B) Pb(NO3)2
(C) Hg(NO3)2 (D) Mn(NO3)2 49. Precipitation of II group cations takes place when H2S
gas passed is : (in presence of dilute HCl)
39. A metal nitrate solution does not give white precipitate (A) highly ionised (B) not ionised
with concentrated hydrochloric acid but on dilution with (C) less ionised (D) none of these
water produces a white precipitate. The metal nitrate
solution with K2CrO4 and Na2HPO 4 reagents gives red 50. A black sulphide is formed by the action of H2S on :
and yellow precipitates respectively which are soluble (A) cupric chloride (B) cadmium chloride
in ammonia solution. The cation of the metal nitrate is : (C) zinc chloride (D) ferric chloride.
(A) Pb2+ (B) Ag+
(C) Cu2+ (D) Bi3+ 51. When H2S gas is passed through Hg2+ :
(A) only HgS is formed
VI. IInd Group (B) both HgS and Hg2S are formed
(C) both HgS and Hg are formed
40. H2S in the presence of HCl precipitates  group but not (D) only Hg2S is formed
V group because
(A) HCl activates H2S 52. The chemical composition schweitzer's reagent is :
(B) HCl increases concentration of Cl– (A) CuSO4.5H2O (B) Cu2I2
(C) HCl decreases concentration of S2– (C) CuCO 3.Cu(OH)2 (D) Cu(NH3)4.SO 4
(D) HCl lowers the solubility of H2S in solution
53. In borax bead test, the red colour of bead in reducing
41. Which of the following metal ions is precipitated when flame is due to the formation of :
H2S gas is passed in presence of HCl? (A) Cu(BO2)2 (B) CuBO2
(A) Co2+ (B) Al3+ (C) Cu (D) Cu2O
3+
(C) Bi (D) Mn2+
54. Which of the following metal cation is reduced from its
42. Which compound does not dissolve in hot 50% HNO3 ? higher oxidation state (+2) to (+1) by both KI solution and
(A) AgS (B) CuS excess of KCN solution ?
(C) Bi2S3 (D) HgS (A) Zn2+ (B) Hg2+
(C) Cu2+ (D) None
43. Yellow ammonium sulphide solution is a suitable
reagent for the separation of : 55. W hich of the following reagent gives white precipitate
(A) HgS and PbS (B) PbS and Bi2 S3 with Hg(NO3)2 solution ?
(C) Bi2 S3 and CuS (D) CuS and As2 S3 (A) Cobalt (II) thiocyanate
(B) Tin (II) chloride (excess)
44. In which of the pair the precipitates are red and yellow (C) Ammonia solution
coloured respectively? (D) Potassium cyanide solution
(A) HgI2 , Cu2I2 (B) HgI2 , PbI2
(C) Cu2I2 , AgI (D) PbI2, AgI 56. A metal nitrate solution (X) responds to the following
tests.
45. A metal chloride original solution (i.e. O.S) on mixing (I) It gives blue precipitate with cold solution of sodium
with K 2CrO4 solution gives a yellow precipitate soluble hydroxide.
in aqueous sodium hydroxide. The metal may be : (II) It gives brown precipitate with potassium ferrocyanide.
(A) mercury (B) Iron (III) It gives intense brown colour solution containing white
(C) silver (D) lead precipitate with potassium iodide solution.
The cation of metal nitrate (X) is :
46. W hich of the following is insoluble in dil. HNO 3 but (A) Fe3+ (B) Cu2+
+
dissolves in aquaregia ? (C) NH4 (D) Ni2+
(A) HgS (B) PbS
(C) Bi2 S3 (D) CuS. VII : IIIrd Group

47. When small amount of SnCl2 is added to a solution of 57. When NH4Cl is added to a solution of NH4OH :
Hg2+ ions, a silky while precipitate is obtained. The silky (A) the dissociation of NH4OH increases.
white precipitate is due to the formation of : (B) the concentration of OH– increases.
(A) Hg2Cl2 (B) SnCl4 (C) the concentrations of both OH– and NH4+ increase.
(C) Sn (D) Hg (D) the concentration of OH– ion decreases.

PAGE # 261
58. The solution of sodium meta aluminate on diluting with 70. An original solution of an inorganic salt in dilute HCl
water and then boiling with ammonium chloride gives: gives a brown colouration with potassium
(A) [Al(H2O)5OH]2+ (B) AlCl3 hexacyanidoferrate (III) and reddish brown colouration
(C) Al (OH)3 (D) NaAl(OH)4 with sodium acetate solution. The cation of the salt is :
(A) Ni2+ (B) Fe3+
59. W hich one among the following is insoluble in excess (C) Cu2+ (D) none
of NaOH solution ?
(A) Al(OH)3 (B) Zn(OH)2 71. Select the correct statement with respect to Fe3+ ions.
(C) Fe(OH)3 (D) Pb(OH)2 (A) Iron (III) ions react with H2S in acidic solution to give
a black precipitate of Fe2S3.
60. Turnbull’s blue is a .................... . (B) Iron (III) ions react with ammonium sulphide to give
(A) Ferricyanide (B) Ferrous ferricyanide the black precipitate of Fe2S3.
(C) Ferrous cyanide (D) Ferri ferrocyanide (C) Iron (III) ions react with ammonium thiocyanate
solution to produce deep red colouration.
61. Fe(OH)3 and Cr(OH)3 precipitates can be completely (D) All of these
separated by :
(A) Aq. NH3 (B) HCl VIII : IVth Group
(C) NaOH/H2O2 (D) H2SO4
72. In qualitative analysis Ni is under :
62. NH4SCN can be used to test one or more out of Fe3+ , (A) IInd group (B) IIIrd group
Cr3+, Cu2+ : (C) IVth group (D) VIth group
(A) Fe3+ only (B) Cr3+ , Cu2+ only
3+ 2+
(C) Fe Cu only (D) All 73. W hich of the following sulphides is buff coloured i.e.
light pink coloured?
63. Concentrated nitric acid is added before proceeding to (A) ZnS (B) MnS
test for group III members. This is to : (C) NiS (D) CdS
(A) oxidise any remaining H2S.
(B) convert ferrous ions to ferric ions as K sp of Fe(II) 74. Which one of the following sulphide is white?
hydroxide is higher. (A) MnS (B) ZnS
(C) form nitrates which gives granular precipitate. (C) HgS (D) CdS
(D) increase ionisation of ammonium hydroxide.
75. HgS can be separated from ZnS by treating with :
64. Which one of the following metal salts produces a blue
(A) HCl (B) NaOH
coloured bead in cobalt nitrate charcoal cavity test ?
(C) aqua-regia (D) NH3
(A) Zn2+ (B) Mg2+ (C) Sn2+ (D) Al3+
76. A metal salt solution when treated with potassium
65. To avoid the precipitation of hydroxides of Ni2+, Co2+, Mn2+
cyanide solution, a green precipitate is obtained. The
along with those of the third group cations, the solutions
metal is :
should be :
(A) Ni (B) Zn
(A) heated with few drops of concentrated HNO3.
(B) boiled with excess of ammonium chloride. (C) Co (D) Mn.
(C) concentrated to small volume.
(D) none of these. 77. An aqueous solution of colourless metal sulphate M,
gives a white precipitate with NH4OH. This was soluble
66. Concentrated sodium hydroxide can separate a mixture in excess of NH 4OH. On passing H 2S through this
of : solution a white precipitate is formed. The metal M in the
(A) Al3+ and Cr3+ (B) Cr3+ and Fe3+ salt is :
3+ 2+
(C) Al and Zn (D) Cu2+ and Mn2+ (A) Ca (B) Ba
(C) Al (D) Zn
67. A dark green bead in the borax bead test (in oxidising
flame) indicates the presence of :
78. W hich one of the following cation will give a green
(A) Cr3+ (B) Mn2+
2+
coloured ash when a piece of filter paper dipped in a
(C) Co (D) Ni2+
solution containing its salt and Co(NO3)2 is burned ?
68. Which one of the following is not correctly matched? (A) Cu2+ (B) Mg2+
3+
(A) [Fe(H2O)5SCN]2+ – deep blue solution (C) Al (D) Zn2+
(B) [Cu(NH3)4]2+ – deep blue solution
(C) Fe[Fe(CN)6] – brown solution 79. Zn(OH)2 is soluble in :
(D) [Ni(CN)4]2– – yellow solution (A) excess of sodium hydroxide
(B) excess of ammonia solution
69. W hat product is formed by mixing the solution of K 4 (C) solutions of ammonium salts
[Fe(CN)6] with the solution of FeCl2 in complete absence (D) all of these
of air ?
(A) Ferro ferricyanide (B) Ferric ferrocyanide
(C) Ferric ferricyanide (D) None

PAGE # 262
80. A dilute solution of manganese(II) nitrate and lead dioxide 90. Select the correct statement with respect to Ca2+ ions.
is boiled with a little concentrated nitric acid. A coloured (A) K 2CrO 4 gives white precipitate in the presence of
compound (X) is formed. Compound (X) is : acetic acid.
(A) H2MnO4 (B) HMnO4 (B) Potassium hexacyanidoferrate (II) solution gives
(C) Pb3OH (D) MnO2 white precipitate.
(C) Ammonia solution gives white precipitate.
81. An aqueous solution of a substance (X) gives a black (D) Prolonged passage of carbon dioxide gas through
precipitate on treatment with H 2S gas in presence of its aqueous solution produces white precipitate.
NH 4OH and NH 4Cl which dissolves in aqua regia on
heating. The ammonical solution of substance (X) gives 91. Potassium chromate solution is added to an aqueous
red precipitate with dimethylglyoxime. The substance solution of a metal chloride. The precipitate thus obtained
(X) is : is insoluble in acetic acid. The precipitate is subjected
(A) Cu2+ salt (B) Fe3+ salt to flame test, the colour of the flame is :
2+
(C) Ni salt (D) Pb2+ salt (A) lilac (B) apple green
(C) crimson red (D) brick red
82. Concentrated solution of sodium hydroxide in water can
separate a mixture of :
(A) Zn2+ and Pb2+ (B) Al3+ and Pb2+ PART - II : COMPREHENSION
2+ 2+
(C) Pb and Ni (D) Al3+ and Zn2+
Comprehension
th th
IX : V and VI Group Read the following comprehension carefully and
answer the questions :
83. In fifth group, (NH4)2CO3 is added to precipitate out the Comprehension # 1
carbonates in presence of NH4Cl. We do not add Na2CO3 A chemist opened a cupboard and found four bottles
because : containing water solutions, each of which had lost its
(A) CaCO3 is soluble in Na2CO3 label. Bottles 1, 2, 3 contained colourless solutions, while
(B) Na 2CO 3 increases the solubility of fifth group bottle 4 contained a blue solution. The labels from the
carbonates bottles were lying scattered on the floor of the cupboard.
(C) MgCO3 will be precipitated out in fifth group They were :
(D) Mg(OH)2 will be precipitated copper (II) sulphate, hydrochloric acid
lead nitrate, sodium carbonate
84. K4[Fe(CN)6] can be used to detect one or more out of By mixing samples of the contents of the bottles, in pairs,
Fe2+ , Fe3+, Zn2+, Cu2+, Ag+ , Ca2+ : the chemist made the following observations :
(A) Only Fe2+, Fe3+ (B) Only Fe3+,Zn2+,Cu2+ Bottle 1 + Bottle 2  white precipitate is formed.
(C) All but not Ca2+ (D) All of these.
Bottle 1 + Bottle 3  white precipitate is formed.
85. If crimson flame is given when an inorganic mixture is Bottle 1 + Bottle 4  white precipitate is formed.
tested by flame test, it may be due to the presence of
(A) potassium (B) strontium Bottle 2 + Bottle 3  colourless and odourless gas
(C) barium (D) calcium is evolved
Bottle 2 + Bottle 4  no visible reaction is observed.
86. A brick red colour is imparted to Bunsen flame by a :
Bottle 3 + Bottle 4  blue precipitate is formed.
(A) Ca salt (B) Sr salt
W ith the help of the above observations answer the
(C) Na salt (D) Co salt
following questions.
87. The presence of magnesium is confirmed in the
92. Bottle 3 contains :
qualitative analysis by :
(A) copper (II) sulphate (B) hydrochloric acid
(A) titan yellow solution + 2M NaOH solution
(B) disodium hydrogen phosphate +NH4Cl + NH3 (aq.) (C) lead nitrate (D) sodium carbonate
(C) magneson(I) reagent
(D) all of these 93. W hich of the following statement is correct for salts
contained in bottle 1 and 4?
88. W hich of the following solution gives white precipitate (A) Bottle 4 gives white precipitate with excess of KI
with Pb(NO3)2 as well as with Ba(NO3)2 ? solution.
(B) Bottle 4 gives white precipitate with excess of K4 [Fe
(A) Sodium chloride (B) Sodium sulphate
(CN)6] solution.
(C) Potassium iodide (D) All of these
(C) Bottle 1 and 4 both gives precipitate with excess of
NaOH solution.
89. An aqueous solution of salt gives white precipitate with
(D) Bottle 1 gives white precipitate with concentrated
AgNO3 solution as well as with dilute H2SO4. It may be :
HCl solution.
(A) Pb(NO3)2 (B) Ba(NO3)2
(C) BaCl2 (D) CuCl2

PAGE # 263
94. W hich one of the following bottles develops intensive
deep blue colour with aqueous ammonia ? EXERCISE-2
(A) Bottle 1 (B) Bottle 2
(C) Bottle 3 (D) Bottle 4 1. The metal that dissolves in liquid ammonia giving dark
blue coloured solution is - [IJSO-Stage-I/2011]
95. Which of the following bottle will give white precipitate (A) Sn. (B) Pb. (C) Na. (D) Ag.
with bottle 1?
2. The green coloured substance produced during the
(A) Bottle 2 (B) Bottle 3
(C) Bottle 4 (D) All of these burning of ammonium dichromate in fireworks is
[IJSO_Stage-I/ 2012]
Comprehension # 2 (A) CrO3 (B) Cr2O3
(C) CrO(O2)2 (D) Cr(OH)3
Yellow precipitate is obtained which is insoluble 3. An alkaline solution of K2Hgl4 is called
in NH3 solution but soluble in more concentrated [IJSO Stage-1/2012-13]
precipitant and boiling water. (A) Fehling's reagent (B) Benedict's reagent
(C) Nessler's reagent (D) Tollen's reagent
(C) aq. 4. W hich of the following solution will form a precipitate
with excess 0.1M KOH but not with excess 0.1M NH3 ?
Black (D) aq. Aqueous (B) aq. Red [IJSO-Stage-II/2014]
solution of precipitate (A) AgNO3 (B) AlCl3
precipitate
potassium is obtained (C) BaCl2 (D)MgCl2
is obtained
iodide as which is
which is 5. A white crystalline salt P reacts with dilute HCl to
precipitant. soluble in
soluble in liberate a suffocating gas Q and also forms a yellow
excess of excess of
(A) aq. precipitate. The gas Q turns potassium dichromate acidi-
precipitant. precipitant.
fied with H2SO4 to a green colored solution R. P,Q and R
are ? [IJSO-Stage-II/2014]
White precipitate in brown colour (A) P: Na2S2O3, Q: SO2, R: Cr2(SO4)3
solution is obtained. (B) P: Na2SO3, Q: Cl2, R: Cr(SO4)3
(C) P: Na2SO4, Q: SO3, R: Cr2(SO4)3
(A), (B), (C) and (D) are different metal nitrates.
(D) P: Na2S, Q: Cl2, R: Cr2(SO4)3
A mixture consists (A) (red solid) and (B) (colourless
solid) which gives lilac colour in flame. 6. A compound 'A' when treated with a dilute mineral acid
(a) Mixture gives black precipitate (C) on passing H 2S gives a gas which when passed through a solution of B
(g). regenerates A. Further, a gas (C) that is obtained from
(b) (C) is soluble in aquaregia and on evaporation of the addition of Concentrated HCI to KMnO4 crystals is
aquaregia and adding SnCl 2 gives greyish black
used to react with B to give D. Identify A, B, C and D.
precipitate (D).
[IJSO-Stage-I/2015]
The salt solution with NH4OH gives a brown precipitate. (A) Na2CO3, NaOH, O2, Na2O.
(i) The sodium extract of the salt with CCl4/FeCl3 gives a (B) MgCO3, Mg(OH)2 OCl2 , Mg(OCl)2
violet layer. (C) CaCO3. Ca(OH)2, Cl2, CaOCI2
(ii) The sodium extract gives yellow precipitate with (D) AI2(CO3)3, AI(OH)3, Cl2 , Al2O3.
AgNO 3 solution which is insoluble in dilute ammonia
7. An excess of NaOH solution is added gradually to an
solution.
aqueous solution of ZnSO4 . Which of the following will
Identify (A) and (B), and the precipitates (C) and (D)
happen [IJSO-Stage-1/2014-15]
96. Compounds (A) and (B) respectively are : (A) A white precipitate is formed which does not
(A) Cu(NO3)2 and Pb(NO3)2 (B) Cu(NO3)2 and Hg(NO3)2 dissolve in excess NaOH.
(C) Cu(NO3)2 and AgNO3 (D) Hg(NO3)2 and AgNO3
(B) A green precipitate is formed which dissolves in
excess NaOH.
97. Compounds (C) and (D) respectively are :
(A) AgNO3 and Hg(NO3)2 (B) AgNO3 and Cu(NO3)2 (C) No observable change occurs.
(C) Pb(NO3)2 and Bi(NO3)2 (D) AgNO3 and Bi(NO3)2 (D) A white precipitate is formed which dissolves in
excess NaOH.
98. Select incorrect statement about (A) and (C) compounds. 8. A colorless solution of compound ‘A’ gives white
(A) Aqueous solution of (C) produces white precipitate
precipitate, ‘B’, when treated with sodium hydroxide
with dilute HCl (2M).
solution. The white precipitate dissolves in excess of
(B) Aqueous solution of (A) gives black precipitate with
H2S gas and dilute HCl. sodium hydroxide solution. The clear solution thus
(C) Aqueous solution of (C) produces red precipitate obtained when treated with hydrogen sulphide gas gives
with K2CrO4 solution. white precipitate (C) Identify A, B and C.
(D) Aqueous solution of (A) develops deep blue (A) MgSO4, Mg(OH)2, MgS
colouration with excess NH3 solution. (B) Al2 (SO4)3, Al(OH)3, Al2S3
(C) ZnSO4, Zn(OH)2 ZnS
(D) (NH4)2SO4, NH4OH, (NH4)2S

PAGE # 264
Electrochemistry

INTRODUCTION

Batteries are everywhere in modern societies. They


provide the electric current to start our autombiles
and to power a host of products such as pocket
caculators, digital watches, heart pacemaker, radio,
and tape recorders.
Electrochemistry is the area of chemistry concerned
with the interconversion of chemical and electrical.A
battery is a an electrochemical cell, a device for
interconverting chemical and electrical energy. A
battery takes the energy relased by a spontaneous
Figure : 2
chemical reaction and uses it to produce electricity.

ELECTROCHEMICAL CELL

It is device for converting chemical energy in to


electrical energy.
Electrochemical cell are of two types

Figure : 3
Galvanic cells or Voltaic cell Electrolytic cell. The Zn atom or metal atoms will move in the solution
A spontaneous chemical reaction   An electric current drives a to form Zn+2. After some time following equilibrium
generates an electric current. nonspontaneous reaction.
will be established. Zn(s) Zn2+ +2e–
The two types of cells are therefore reverse of There will be accumulation of sufficient negative charge
each other on the rod which will not allow extra zinc ions to move
in the solution. i.e. solution will be saturated with Zn+2
Construction/ Working principle
ions.
When ever an metal strip is put in an electrolyte the
The positive charge will be more concentrated nearly
process of oxidation and reduction takes place
the rod.
simultaneously within the system. Due to this there
The extra positive charge of the solution will be more
is a potential difference between the metal phase and
concentrated around the negatively charged rod. An
the liquid phase.
electrical double layer is developed in the system
On joining the metal strips through a wire (of negligible
and hence a potential difference is created between
resistence) the current flows as long as the potential
the rod and the solution which is known as electrode
difference exists between the metal phase and the
potential
liquid phase.
This particular electrode is known as anode :
 Anode :  On anode oxidation will take place. (release of
Some metals (which are reactive) are found to have electron).
tendency to go into the solution phase when these  To act as source of electrons.
are placed in contact with their ions or their salt  It is of negative polarity.
solutions.  The electrode potential is represented by EZn(s) / Zn2+
For example : Zn rod is placed in ZnSO4solution . (aq)

 Cathode :
Zn rod Cu rod
+
+
+
+
+
+
+
+
+
ZnSO4 + CuSO 4
+
+
+
+
+ +2
Zn+2
+
+ Cu
Figure-1
Figure -1

PAGE # 265
 The metal strips are called electrodes and are
connected by an conducting wire.
 Two solutions are connected by a salt bridge.
 The oxidation and reduction half reactions occur at a
separate electrodes and electric current flows
through the wire.

Figure : 2

+
+
+
Selection of electrolyte for Salt Bridge :
+
+
+  The electrolyte in salt bridge should be such that
+
+ Very soon a dynamic
+ speed of it's cation equals speed of it's anion in
+
+ equilibrium is created
+
+
+ and become a electrical field.
+
+
+ cylinderical capacitor  For that charge and sign of the ions should be almost
Figure -3 equal.
Transport number of cation = Transport number of
anion
Some metals(Cu, Ag, Au etc.,) are found to have the or
opposite tendency i.e. when placed in contact with Mobility of cation = Mobility of anion
their aqueous ions, the ions from the solution will get KCl is generally preffered but KNO3 or NH4NO3 can
deposited on the metal rod. also be used.
The following equilibrium will be established :  If Ag+, Hg2+, Pb2+, Tl+ ions are present in a cell then
Cu2+ +2e– Cu(s). in salt bridge KCl is not used because there can be
So rod will have deficiency of electron (positive formation of precipitate of AgCl, Hg2Cl2, PbCl2 or
charge).Extra negative charge will surround this TlCl at mouth of tube which will prevent the migration
positively charged rod and form double layer. An of ions and its functioning will stop.
electrical double layer is developed in the system
and hence a potential difference is created between Functions of Salt Bridge :
the rod and the solution which is known as electrode  A salt bridge is a U–shaped inverted tube that
potential. This will be known as cathode. contains a gel permeated with an inert electrolyte.
 It connects the solution of two half cell to complete
 At cathode reduction will take place.(gain of e– will
the circuit.
take place)
 It minimise the liquid junction potential. The potential
 To act as sink of electron. difference between the junction of two liquids.
 Positive polarity will be developed.  It maintains the electhical neutrality of the solution in
order to give continious flow or generation of current.
 Their electrode potential can be represented by : E " The simultaneous electrical neutrality of the anodic
Cu2+(aq)/Cu(s) oxidation chamber and cathodic reduction chamber
is due to same mobility or velocity of K+ and NO3–
Is where oxidation occurs
ions taken into salt bridge.
Anode : Is where electrons are produced
Has a negative sign  If the salt bridge is removed then voltage drops to
zero.
Is where reduction occurs  The ions of the inert electrolyte do not react with other
Cathode : Is where electrons are consumed ion in the solution and the ions are not
Has a positive sign
oxidised or reduced at the electrodes.
Construction of Cell :  Generally tube is filled with a paste of agar-agar
 It has two half–cells,each having a beaker powder with a natural electrolyte/generally not
containing a metal strip that dips in its aqueous common to anionic/cathodic compartment with porous
solution. plugs at each mouth of tube.
 It prevents mechanical mixing of two electrolytic
solution.
PAGE # 266
 Nature of the electrolyte.
ELECTRODE POTENTIAL  Pressure, temperature coditions.
 The potential difference developed between metal
Electrode Potential : electrodes and the solution of its ions at 1 M
 The driving force that pushes the negative charge concentration at 1 bar pressure and 298 K is known
electrons away from the anode and pulls them towards as standard electrode potential.
the cathode is an electrical potential called
electromotive force also known as cell potential
or the cell voltage. Its unit is volt
 The potential difference devepoled between metal Oxidation Potential Reduction Potential
electrode and its ions in solution in known as (O.P.) (R.P.)
electrode potential.  The electrode potential   The electrode potential for
 Electrode potential depends upon : oxidation half reaction reduction half reaction
 Concentration of the solution.  Tendency to get oxidised.  Tendency to get reduced.
 Nature of the metal.  Greater the O.P. greater  Greater the R.P.
will be tendency greater will be tendency
to get oxidised. be tendency to get
reduced.

Type of Electrode Electrode reaction in standard condition Representation

1. Metal electrode Reduction : Zn2+ + 2e–  Zn(s) E 0Zn2  / Zn(s ) ( SRP)

(Zn electrode, Oxidation : Zn(s)  Zn2+ + 2e– E 0Zn(s ) / Zn2  (SOP)


Cu electrode etc.)
2. Hydrogen peroxide Reduction : 2e– + 2H+ + H2O2  2H2O E H0 2O 2 / H2 O

electrode Oxidation : H2O2  O2 + 2H+ + 2e– E H0 2O 2 / O 2

0
3. Redox electrode Reduction : MnO4– + 8H+ + 5e–  Mn2+ + 4H2O EMnO 
/ Mn 2 
4

4. Metal Metal Reduction : AgCl(s) + e–  Ag(s) + Cl– E 0AgCl( s ) / Ag( s ) / Cl 

salt insoluable Oxidation : Ag(s) +Cl–  AgCl(s) + e– E 0Ag( s ) / AgCl( s ) / Cl 

electrode

Reference electrode : cell) is known as emf of the cell or cell potential.


 The potential of a singal electode cannot be The emf of the cell or cell potential can be calculated
determined what were the potential difference between from the values of electrode potential of the two half
two electrodes can be accurately measured using a cell constituning the cell. The following three methode
reference electrode. are in use :
 An electrode is chosen as a reference with respect  When oxidation potential of anode and reduction
to which all other electrodes are valued. potential of cathode are taken into account :
 Standard Hydrogen Electrode (SHE) is taken as E°cell = oxidation potential of anode + reduction
standard reference electrode. Its electrode potential is potential of cathode
arbitrarily assumed to be 0.00 volt. E°ox (anode) + E°red(cathode)
 Standard Hydrogen Electrode (SHE) consists of a  When reduction potential of both electrodes are
platinum electrode in contact with H2 gas and aqueous
taken into account :
H+ ions at standard state conditions (1 atm H2 gas, 1 M
E°cell = Reduction potential of cathode – Reduction
H+ (aq), 25°C).
potential of anode
2H+ (aq, 1M) + 2e–  H2 (g, 1 atm) E° = 0V
H2(g, 1atm)  2H+ (aq, 1M) + 2e– E° = 0V = E°cathode – E°anode C both are reduction potential.
 When oxidation potential of both electrodes are
Cell potential :
taken into account :
 The difference in electrode potentials of the two half
E°cell = oxidation potential of anode – Oxidation
cell reactions (oxidation half cell and reduction half
potential of cathode
= E°ox (anode) – E°ox (cathode)

PAGE # 267
 The standard cell potential E° is the cell potential Solved Example
when both reactants and products are in their standard
states – solutes at 1 M concentration, gases at a
Ex.1 Write short hand notation for the following reaction,
potential pressure of 1 atm, solids and liquids in pure
Sn2+ (aq) + 2Ag+ (aq)  Sn4+ (aq) + 2Ag(s).
from, with all at a specified temperature, usually 25°
C. Sol. The cell consists of a platinum wire anode dipping
into an Sn+2 solution and a silver cathode dipping
 E°cell is intensive property so on multiplying/Dividing into an Ag+ solution therefore Pt(s) | Sn2+(aq),
cell reaction reaction by any number, the E°cell value Sn4+ (aq) || Ag+ (aq) | Ag(s).
would not change.
Ex.2 Write the electrode reaction and the net cell
Free energy changes for cell reaction : reaction for the following cells. Which electrode
 The free energy change G (a thermochemical would be the positive terminal in each cell ?
quantity) and the cell potential E(an electrochemical (a) Zn | Zn2+ || Br–, Br2 | Pt
quantity) both measure the driving force of a chemical (b) Cr| Cr3+ || I– , I2 | Pt
reaction. (c) Pt | H2, H+ || Cu2+ | Cu
 The values of G and E are directly proportional and
(d) Cd | Cd2+ || Cl– , AgCl | Ag
are related by the equation.
G = –nFE Sol.(a) Oxidation half cell reaction,
where n = Number of moles of electron Zn  Zn2+ + 2e–
transfered in the reaction.
reduction half cell reaction,
F = Faraday constant = 96485 C/mole e– 
Br2 + 2e–  2Br–
96500 C/mole e–
Net cell reaction
Shorthand Notation for Galvanic Cells Zn + Br2  Zn2+ + 2Br–
 We require two half cells to produce an electrochemical (Positive terminal : cathode Pt)
cell, which can be represented by follwing few rules;
(b) Oxidation half reaction,
 The anode half-cell is always written on the left [Cr  Cr3+ + 3e–] x 2
followed on the right by cathode half cell.
reduction half reaction,
 The separation of two phases (state of matter) is [I2 + 2e–  2I–] x 3
shown by a vertical line.
Net cell reaction
 The various materials present in the same phase 2Cr + 3I2  2Cr3+ + 6I–
are shown together using commas. (Positive terminal : cathode Pt)
 The salt bridge is represented by a double slash (c) Oxidation half reaction,
(||). H2  2H+ + 2e–
reduction half reaction,
 The significant features of the substance viz.
pressure of a gas, concentration of ions etc. are Cu2+ + 2e–  Cu
indicated in brackets immediately after writing the Net cell reaction
substance. H2 + Cu2+  Cu + 2H+
 For a gas electrode, the gas is indicated after (Positive terminal : cathode Cu)
the electrode for anode and before the electrode in (d) Oxidation half reaction,
case of cathode. (i.e Pt H2 / H+ or H+ /H2 Pt) Cd  Cd2+ + 2e–
reduction half reaction,
[AgCl + e–  Ag + Cl–] x 2
Net cell reaction
Cd + 2AgCl  Cd2+ + 2Ag + 2Cl–
(Positive terminal : cathode Ag)

PAGE # 268
Calculation of electrode potential :

At Anode At Cathode
H2(g) 2H+ + 2e– 2H+ + 2e– H2(g)
Oxidation potential O.P. = EH ( g) / H ( aq.)  Reduction Potential (R.P.) EH / H ( g) = RP
2 2
under standard state  under standard state.
0
 E 0H2 ( g) / H (aq.) = SOP  E
H / H2 ( g)
= SRP

 For SHE reference potential is taken to be zero at all temperature.


SOP = – SRP = 0 for SHE.
To calculate standard potential of any other electrode a cell is coupled with standard hydrogen electrode
(SHE) and it's potential is measured that gives the value of electrode potential of that electrode.

Galvanometer

Salt bridge
Cl– Na+ H2
Inlet

Zn

H2O2 H2O2 Pt
H2
Zn
2+
H3O+
2– Cl

Cathode
Anode SO4

A galvanic cell measuring the


2+
Zn |Zn half-cell potential
Anode : Zinc electrode
Cathode : SHE
Cell : Zinc electrode || SHE
Cell potential :
Ecell = EH / H2 ( g) – E°Zn2+/Zn
= 0.76 V (at 298 K experimentaly)
So, E0Zn2+/Zn = – 0.76 V (SRP)
E0 Zn/Zn2+(aq) = 0.76 V(SOP)

PAGE # 269
 So, w.r.t. H2 , Zn has greater tendency to get oxidised.
In similar manner reduction potentials (SRP) at 298 K for many other electrodes are calculated and are
arranged in a series increasing order known as electro chemical series.

Electrochemical Series :
Electrode Reaction SRP (at 298 K)
* Li Li+ + e–  Li(s) – 3.05 V
K K+ + e–  K (s) – 2.93 V
Ba
Ca Ca+2 + 2e–  Ca(s) – 2.87 V
Na Na+ + e–  Na(s) – 2.71 V
Mg Mg+2 + 2e–  Mg(s) – 2.37 V
Al
* Zn Zn+2 + 2e–  Zn(s) – 0.76 V
Cr Cr+3 + 3e–  Cr(s) – 0.74 V
* Fe Fe2+ + 2e-  Fe – 0.44 V
1
* Electrolytes (H2O) H2O(l) + e–  2
H2 + OH– – 0.41 V
Cd Cd+2 +2e–  Cd(s) – 0.40 V
Co
Ni Ni+2 + 2e–  Ni(s) – 0.24 V
Sn Sn+2 + 2e–  Sn(s) – 0.14 V
Pb Pb+2 + 2e–  Pb(s) – 0.13 V
* H2 2H+ + 2e–  H2(g) 0.00 V
Cu Cu2+ + 2e–  Cu(s) 0.34 V
2
Fe Fe3+ + e–  Fe2+ 0.77 V
Hg Hg22+ + 2e-  Hg(l) 0.79 V
Ag Ag+ + e–  Ag 0.80 V
Hg Hg2+  Hg(l) 0.85 V
Br2 Br2 + 2e–  2Br– 1.06 V
1
* Electrolytes 2
O2 + 2H+ + 2e–  H2O() 1.23 V
* Cr2O72– + 14H+ + 6e–  2Cr+3 + 7H2O 1.33 V
* Cl2  Cl2 +2e–  2 Cl– 1.36 V

* MnO4– + 8H+ + 5e–  Mn2+ + 4H2O (a) Write half reaction occurring at the anode.
1.51 V (b) Write half reaction occurring at the cathode.
(c) Write the net ionic equation of the reaction.
* F2 + 2e–  2F– (d) calculate E°cell at 25°C.
2.87 V (e) Does the cell reaction go spontaneous as
written ?
Solved Example
0
(Given E°AgCl,Cl = + 0.22 volt) ; E Ag / Ag = + 0.80 volt)
Ex.3 Calculate E0cell of (at 298 K), Sol. (a) LHS electrode is anode and half reaction is
Zn(s) / ZnSO4(aq) || CuSO4(aq) / Cu(s) oxidation.
given that E0Zn/Zn2+(aq) = 0.76 V ;
0 Ag+ + Cl–  AgCl(s) + e– ... (i)
E Cu(s) / Cu (aq) = – 0.34 V
2+

(b) RHS electrode is cathode and half reaction is


Sol. E0cell = (S.R.P)cathode – (S.R.P)anode reduction.
= 0.34 – (– 0.76) = 1.1 V Ag + e  Ag(s) ... (ii)
Ex.4 Given the cell Ag AgCl(s) | NaCl (0.05 M) || Ag (c) From equation (i) and (ii) cell reaction is : Cl–
NO3 (0.30) | Ag (0.05 M) + Ag+ (0.30 M)  AgCl(s)
(d) E°cell = E°right – E°left = (0.80 – 0.22 volt = 0.58
volt
PAGE # 270
(e) Yes, the e.m.f. value is positive, the reaction
will be spontaneous as written in the cell reaction. ELECTROLYSIS & ELECTROLYTIC CELL

Electrolysis :
 Calculation of Electrode Potential of unknown  Electrolyte is a combination of cations and anions which
electrode with the help of given (two) electrode. in fused state can conduct electricity.
 This is possible due to the movement of ions from which
Obtain the reaction of the 3rd electrode with the help
it is made of and electrolyte.
of some algebraic operations on reactions of the given  The process of using an electric current to bring about
electrodes. chemical change is called electrolysis.
Then calculate G of the 3rd reaction with the help of  Electrolysis is a process of oxidation and readuction
some algebaric operations of G0 of 1st and 2nd due to current in the electrolyte.
reactions.  The product obtained during electrolysis depends on
Use G0 = –nF E0elec. to calculate unknown E.P. following factors.
 The nature of the electrolyte
 E0cell is intensive property so if we multiply/Devide  The concentration of electrolyte
 The charge density flowing during electrolysis.
electrode reaction by any number the E0cell value  The nature of the electrode
would not changed Active vs Inactive electrodes :
i.e. Zn2+ + 2e–  Zn(s) E° = – 0.76 V  The electrodes in the cell that are active because the
Multiply by 2 metals themselves are components of the half
2Zn2+ + 4e–  2Zn(s) E° = – 0.76 V (remain same) reactions.
 As the cell operates, the mass of the zinc electrode
Solved Example gradually decreases, and the [Zn2+] in the anode half
Ex.6 Given that E0Cu2+/Cu = 0.337 V and E0Cu+ /Cu2+ = – – cell increases. At the same time, the mass of the
0.153 V. Then calculate E0 Cu+/Cu. copper electrode increases and the [Cu2+] in the
Sol. (i) Cu2+ + 2e–  Cu G1 cathode half – cell decreases; we say that the Cu2+
(ii) Cu+  Cu2+ + e– G2 "plates out" on the electrode.
 For many redox reactions, however, there are no
after adding Cu++ e–  Cu reactants or products capable of serving as
G1 + G2 = G3 electrodes. Inactive electrodes are used, most
commonly rods of graphite or platinum, materials that
–2F E10 – F E 02 = – F E 03 conduct electrons into or out of the cell but cannot
E3 = 2 E10 + E 02 take part in the half -reactions.
 In a voltaic cell based on the following half reactions, for
= 2 x 0.337 – 0.153 = 0.674 – 0.153 = 0.521 V instance, the species cannot act as electrodes :
Ex.7 E0 Mn2 / MnO4 = –1.51 V ; E 0MnO2 / Mn2 = + 1.23 V 2I–(aq)  I2(s) +2e– [anode ; oxidation]

E0 MnO4 / MnO2 = ? (All in acidic medium) MnO4– (aq) + 8H+ (aq) + 5e–  Mn2+ (aq) +
4H2O() [cathode ; reduction]
Sol. 4H2O + Mn2+  Mn O 4 + 8H+ + 5e– Therefore, each half – cell consists of inactive
G1 electrodes immersed in an electrolyte solution that
contains all the species involved in that half -reaction.
(i) Mn O 4 + 8H + + 5e–  4H2O + Mn2+ –G1
In the anode half-cell, I– ions are oxidized to solid I2.
2e– + MnO2 + 4H+
 Mn2+
+ 2H2O G2 The electrons released flow into the graphite anode,
(ii) 2H2O + Mn2+  MnO2 + 4H+ + 2e– –G2 through the wire, and into the graphite cathode. From
(iii) 4H+ + Mn O 4 + 3e–  MnO2 + 2H2O G3 there, the electrons are consumed by MnO4– ions as
they are reduced to Mn2+ ions.
(i) + (ii) = (iii)
Examples of Electrolysis
G3 = – G1 – G2
 Using inert (pt/graphite) electrodes.
–3E3F = 5 E10 F + 2 E 02 F
Cathode (red) : Pb2+ + 2e–  Pb(s) E0 = 0.126V
[5E1  2E 2 ] [5( 1.51)  2(1.23 )] Anode : 2Br-  Br2 + 2e- E0 = – 1.08 V
E= =
3 3 Ecell = – 0.126 – (0.108) x 10 = – 1.206 V
[ 7.55  2.46] 5.09 Eext > 1.206 V
= = = 1.69 V
3 3  Electrolysis of CuSO4 molten

PAGE # 271
Cathode : Cu2+ + 2e–  Cu E0 = +0.34 V Cathode : Ag+ + e–  Ag(s)
E0 = 0.8 V
Anode :2 SO24  S2 O 28  + 2e– E0 = – 2.05 V
2H2O () + 2e–  H2(g) + 2OH– E0 = – 0.083 V
H2S2O8 – marchall's acid peroxy disulphuric acid. Anode :NO3–  X (No reaction)
Ecell = 0.34 – (2.05) = – 1.71 V (negative  not feasible) 2H2O()  O2 + 4H+ + 4e– E0 = – 1.23 V
O O Ag(s)  Ag+(aq) + e– E0 = – 0.80 V
H O S O O S O H
FARADAY'S LAW OF ELECTROLYSIS
O O
 Electrolysis of aq CuSO4  1st Law : The mass deposited/released/produced of
any substance during electrolysis is proportional to
Cathode : Cu2+ + 2e–  Cu(s) E0 = 0.34 V the amount of charge passed into the electrolyte.
2e + 2H2O()  H2(g) + 2OH–(aq) E0 = –0.83V WQ
Anode :2 SO24  S2 O 28  + 2e– E0 = –2.05 V W = ZQ
Z – electrochemical equivalent of the substance.
2H2O()  O2 + 4H+ + 4e– E0 = –1.23 V
mass
 Electrolysis of aq NaBr solution (initially PH = 7) Unit of Z = = Kg/C or g/C
coulomb
Cathode : Na+(aq) + e–  Na(s) E0 = – 2 V Z = Mass deposited when 1 C of charge is passsed
2e– + 2H2O()  H2 + 2OH– E0 = – 0.83 V into the solution.
Anode : 2Br-  Br2 + 2e– E 0OX= –1.08 V Equivalent mass (E) : mass of any substance
produced when 1 mole of e– are passed through the
2H2O ()  O2 + 4H+ + 4e– E 0OX = –1.23 V solution during electrolysis.
 Electrolysis of aq NaCl Molar mass
Cathode : Na+ + e–  Na E0 = – 2V E= 
no. of e involved in oxidation / reduction
2e– + 2H2O()  H2(g) + 2OH- E0 = –0.83 V
e.g.
Anode : 2Cl–  Cl2 + 2e– E 0OX = –1.30 V M
Ag+ + e–  Ag E=
2H2O()  O2 + 4H+ + 4e– E 0OX = –1.23 V 1
M
Rate of production of Cl2 is more than rate of production Cu2+ + 2e-  Cu(s) E=
2
of O2 gas.
M
Al3+ + 3e–  Al(s) E=
3
Note : According to thermodynamics, oxidation of H2O –
1 mole of e = 1 Faraday of charge = 96500 C
to produce O 2 should take place on anode but
 96500 C = Charge deposite E gram metal charge
experimentally (experiment from chemical kinetics)
the rate of oxidation of water is found to be very slow.  E  E
 1C   g Z=
To increase it's rate, the greater potential difference  96500  96500
is applied called over voltage or over potential but Molar mass
EQ Q
because of this oxidation of Cl– ions also become W= = x
96500 (no. of e  involved ) 96500
feasible and this takes place on anode.

 dQ = i  dt Q = it
Electrolysis using attachable (reactive)
electrodes. ixt Molar mass
W= x
96500 (no. of e  involved )
 Electrolysis of aq. CuSO4 using Cu electrode.
 2nd Law : When equal charge is passed through 2
Cathode (reduction) :Cu2+ + 2e–  Cu E0 = + 0.34 V
2H2O() + 2e–  H2(g) + 2OH– E0 = –0.83 V electrolytic cells and this cells are connected in
series then mass deposited at electrode will be in
Anode (oxidation) : SO 24  S2 O 82  + 2e– the ratio of their electrochemical equivalents or in the
ratio of their equivalent masses.
E 0OX = – 2.05 V
EQ
2H2O()  O2 + 2H+ + 4e– E0 = 1.23 V W = ZQ =
Cu(s)  Cu2+ + 2e– E0 = –0.34 V 96500
electrolytic refining W1 z1 E1
 AgNO3(aq) using Cu cathode & Ag anode. W2
= z2
= E2
( Q = same)

PAGE # 272
CURRENT EFFICIENCY EXERCISE-1
Current Efficiency : I. Basic Terminology and Construction of Electrochemical
charge actually used in electricit y Cell and Its Repsentation, Salt Bridge
current efficiency = ch arg e passed x 100
1. Which of the following has been universally accepeted
as a reference electrode at all temperatures and has
mass actually produced
current efficiency = x 100 been assigned a value of zero volt ?
mass that should have been produced
(A) platinum electrode
(B) copper electrode
Solved Example
(C) graphite electrode
Ex.13 Calculate volume of the gases liberated at STP if (D) standard hydrogen electrode
1 L of 0.2 molar solution of CuSO4 is electrolysed 2. W hich of the following statements is true for an
by 5.79 A current for 10000 seconds. electrochemical cell?
(A) H2 is anode and Cu is cathode
5.79  10000 579
Sol. No. of moles of e– = = = 0.6 (B) H2 is cathode and Cu is anode
96500 965 (C) reduction occurs at H2 electrode
Cathode : Cu2+ + 2e-  Cu(s) (D) oxidation occurs Cu electrode
0.2 mole 0.4 mole 3. The equation representing the process by which stan-
2H2O() + 2e–  H2 + 2OH– dard reduction potential of zinc can be defined is :
0.2 mole of e–  0.1 mole of H2 at S.T.P.. (A) Zn2+(s) + 2e–  Zn
Anode : 2H2O()  O2 + 4H+ + 4e– (B) Zn(g)  Zn2+(g) + 2e–
4 mole of e–  1 mole of O2
(C) Zn2+(g) + 2e–  Zn
0.6 mole of e–  0.15 mole of O2
(D) Zn2+(aq.) + 2e–  Zn(s)
so, total moles = 0.25 mole
Total volume = 5.6 Ltr. 4. Which is not true for a standard hydrogen electrode ?
(A) The hydrogen ion concentration is 1 M
Ex.14 How many cc of chlorine will be deposited by (B) Temperature is 25°C
100 amp. current flowing for 5 hours through (C) Pressure of hydrogen is 1 atmosphere
melted NaCl. (D) It contains a metallic conductor which does not
Sol. Q = It = 100  5  60  60 = 18  105 absorb hydrogen.

E 18 5. The chemical reaction


W = ZQ =  18  155 =  103 2AgCl(s) + H2 (g)  2HCl (aq) + 2Ag (s)
96500 96500
taking place in a galvanic cell is represented by the
18  35.5 notation
=  103 = 662.2 gm
965 2AgCl(s) + H2 (g)  2HCl (aq) + 2Ag (s)
 Volume of 71 gm Cl2 at NTP = 22.4 litre (A) Pt(s) | H2 (g), 1 bar | 1 M KCl (aq) | AgCl(s) | Ag (s)
(B) Pt(s) | H2 (g), 1 bar | 1 M HCl (aq) | 1 M Ag+ (aq) | Ag (s)
22.4 (C) Pt(s) | H2 (g), 1 bar | 1 M HCl (aq) | AgCl (s) | Ag (s)
 volume of 662.2 gm Cl2 at NTP = ´ (D) Pt(s) | H2 (g), 1 bar | 1 M HCl (aq) | Ag (s) | AgCl (s)
71
662.2 = 208.9 litre 6. In the galvanic cell Cu | Cu2+(1M) | | Ag+(1M) / Ag | the
electrons will travel in the external circuit :
Ex.15 The time required to coat a metal surface of 80 (A) from Ag to Cu
cm2 with 0.005 mm thick layer of silver (density (B) from Cu to Ag
= 10.5 gm cm–3) with the passage of 3A current (C) electrons do not travel in the external circuit
through silver nitrate solution is : (D) in any direction
Sol.  Volume of layer of silver = 0.005  10–1  80 7. Given : Eº(Cu2+ | Cu) = 0.337 V and Eº (Sn2+ | Sn) = –
= 0.04 cm3 0.136 V. Which of the following statements is correct?
 mass = Density  volume = 10.5  0.04 (A) Cu2+ ions can be reduced by H2(g)
= 0.42 gm (B) Cu can be oxidized by H+
(C) Sn2+ ions can be reduced by H2(g)
E (D) Cu can reduce Sn2+
So w =  It
96500 8. During electrolysis of fused sodium chloride the
reaction of the electrodes are
108 Anode Cathode
0.42 = 3t (A) Na+ + e Na Cl– ½ Cl + e
96500
(B) Na Na+ + e ½ Cl2 + e Cl–

(C) Cl ½ Cl2 + e Na+ + e Na
0.42  96500
t= = 125.09 seconds. (D) ½ Cl2 + e Cl – Na+ Na+ + e
108  3

PAGE # 273
9. Which of the following is not an anodic reaction : 19. The Eº M3+/M2+ values for Cr, Mn, Fe and Co are – 0.41, +
(A) Ag+ Ag – e– 1.57, + 0.77 and + +1.97 V respectively. For which one of
(B) Cu + Cu2+ + 2e– these metals the change in oxidation state from +2 to +3
(C) Fe2+ Fe3+ + e– is easiest?
(D) 4OH– 2H2O + O2 + 4e– (A) Co (B) Mn
(C) Fe (D) Cr
10. If a salt bridge is removed from the two half cells, the
voltage: 20. The standard electrode potential value of the elements
(A) drops to zero (B) does not change A,B and C 0.68, –2.50 and 0.50 V respectively. The order
(C) increases gradually (D) increases rapidly of their reducing power is :
(A) A > B > C (B) A > C > B
11. Which reaction occour at cathode during electrolysis of
(C) C > B > A (D) B > C > A
fused lead bromide ?
(A) Pb Pb2+ + 2e– (B) Br + e– Br¯
(C) Br¯ Br + e– (D) Pb2++ 2e– Pb
1
21. The standard potential of the reaction H2O + e–  H
2 2
II. Application of Electrochemical Series
+ OH– at 298 K is
12. Adding powdered Pb and Fe to a solution containing 1.0 (A) – 0.828 V (B) 0.828 V
M is each of Pb2+ and Fe2+ ions would result into the (C) 0 V (D) – 0.5 V
formation of
22. The reaction 1/2H2 (g) + AgCl(s) = H+(aq) + Cl–(aq) +
(A) More of Pb and Fe2+ ions
Ag(s) occurs in the galvanic cell :
(B) More of Fe and Pb2+ ions
(A) Ag | AgCl(s) | KCl(sol.) | | AgNO3) (sol.) | Ag
(C) More of Fe and Pb
(B) Pt| H2(g) | HCl(sol. | | AgNO3(sol.) | Ag
(D) More of Fe2+ and Pb2+ ions
(C) Pt | H2(g) |HCl (sol.) | | AgCl(s) | Ag
13. The standard oxidation potentials, Eº, for the half (D) Pt | H2(g) | KCl(sol.) | | AgCl(s) | Ag
reactions are as
0 0
Zn  Zn2+ + 2e– ; E0 = + 0.76 V 23. If E Au / Au is 1.69 V and E Au3  / Au is 1.40 V, then

Fe  Fe2+ + 2e– ; E0 = + 0.41 V E 0Au / Au3 


The EMF for the cell oxidation : (A) 0.19 V (B) 2.945 V
(A) –0.35 V (B) + 0.35 V (C) 1.255 V (D) None of these
(C) + 1.17 V (D) – 1.17 V 24. the standard oxidation potential E°, for the half reaction
14. Strongest reducing agent is : are as :
(A) K (B) Mg Zn  Zn2+ + 2e– E° = + 0.76 volt
(C) Al (D) I
Fe  Fe2+ + 2e– E° = + 0.41 volt
15. Zn can not displace following ions from their aqueous The emf of the cell Fe2+ + Zn  Zn2+ + Fe is :
solution :
(A) 0.35 volt (B) – 0.35 volt
(A) Ag+ (B) Cu2+
(C) + 1.17 volt (D) – 1.17 volt
(C) Fe2+ (D) Na+
III. Electrolysis (Faraday laws)
16. Which of the following displacement does not occur :
25. The electric charge for electrode deposition of one gram
(A) Zn + 2H+  Zn2+ + H2 
equivalent of a substance is :
(B) Fe + 2Ag+  Fe2+ + Ag  (A) one amp /sec (B) 96,500 C / sec
(C) one amp / hour (D) 96,500 C
(C) Cu + Fe2+  Cu2+ + Fe 
26. The amount of an ion discharged during electrolysis is
(D) Zn + Pb2+  ZN2+ + Pb  not directly proportional to :
(A) resistance
17. The oxidation potential of Zn, Cu, Ag, H2 and Ni are 0.76,
(B) time
– 0.34, – 0.80, 0, 0.55 volt respectively. Which of the fol-
(C) current strength
lowing reaction will provide maximum voltage ?
(D) electrochemical equivalent of the element
(A) Zn + Cu2+  Cu + Zn2+
27. W hich one is the correct equation that represents the
(B) Zn + 2Ag+  2Ag + Zn2+ first law of electrolysis ?
(m  mass, c  current, t  time)
(C) H2 + Cu2+ +
 2H + Cu (A) mz = ct (B) m = czt
(C) mc = zt (D) c = mzt
(D) H2 + Ni2+  2H+ + Ni
28. When one coulomb of electricity is passed through an
18. The metal that cannot be produced on reduction of its electrolytic solution the mass deposited on the elec-
oxide by aluminium is : trode is equal to :
(A) K (B) Mn (A) equivalent weight
(C) Cr (D) Fe (B) molecular weight
(C) electrochemical equivalent
(D) one gram

PAGE # 274
29. W g of copper deposited in a copper voltameter when an 41. How many electrons flow when a current of 5 amperes
electric current of 2 ampere is passed for 2 hours. If one is passed through a conductor for 200 seconds?
ampere of electric current is passed for 4 hours in the (A) 6.214 × 1021 (B) 6.0241 × 1021
same voltameter, copper doposited will be : (C) 6.241 × 1022 (D) 6.0241 × 1022
(A) W (B) W/2
(C) W/4 (D) 2W 42. The products formed when an aqueous solution of NaBr
is electrolyzed in a cell having inert eletrodes are:
30. The ratio of weights of hydrogen and magnesium de-
(A) Na and Br2 (B) Na and O2
posited by the same amount of electricity from aqueous
(C) H2, Br and NaOH (D) H2 and O2
H2SO4 and fused MgSO4 are :
(A) 1 : 8 (B) 1 : 12
43. The weight of silver (eq. wt = 108) displaced by that
(C) 1 : 16 (D) None of these
quantity of current which displaced 5600 ml. of hydro-
31. Faraday's law of electrolysis fails when : gen at STP is :
(A) Temperature is increased (A) 54 g (B) 108 g
(B) Inert electrodes are used (C) 5.4 g (D) None of these
(C) A mixture of electrolytes is used
44. Which process involves corrosion ?
(D) In none of the above cases
(A) Brown deposits on iron articles
32. An ion is reduced to the element when it absorbs 6 × (B) Green deposits on battary terminals
1020 electrons. The number of equivalents of the ion is: (C) Black deposits on silver coin
(A) 0.10 (B) 0.01 (D) All of the above
(C) 0.001 (D) 0.0001
33. Electrolysis can be used to determine atomic masses. 45. How many g of silver will be displaced from a solution of
A current of 0.550 A deposits 0.55 g of a certain metal in AgNO3 by 4 g of magnesium?
100 minutes. Calculate the atomic mass of the metal if (A) 18 g (B) 4 g
n=3: (C) 36 g (D) 16 g
(A) 100 (B) 45.0
(C) 48.25 (D) 144.75 46. Aluminium oxide may be electrolysed at 1000ºC to
furnish aluminium metal (At.Mass of Al = 27 amu ;
34. How many minutes will it take to plate out 5.0 g of Cr 1 Faraday = 96,500 Coulombs). The cathode reaction is
from a Cr2(SO 4)3 solution using a current of 1.50 A ? Al3+ + 3e–  Al0. To prepare 5.12 kg of aluminium metal
(Atomic weight : Cr = 52.0) by this method, one would require :
(A) 254 (B) 30 (A) 5.49 × 107 C of electricity
(C) 152 (D) 103 (B) 1.83 × 107 C of electricity
35. A spoon to be electroplated with gold should be : (C) 5.49 × 104 C of electricity
(A) cathode (B) anode (D) 5.49 × 1010 C of electricity
(C) electrolyte (D) none of these
47. Time required to deposite one millimole of aluminium
36. Which of the substances Na, Hg, S, Pt and graphite can metal by the passage of 9.65 ampere through aqueous
be used as electrodes in electrolytic cells having aque- solution of aluminium ion is
ous solution ? (A) 30 s (B) 10 s
(A) Hg and Pt (B) Hg, Pt and graphite (C) 30,000 s (D) 10,000 s
(C) Na, S (D) Na, Hg, S
37. How many coulomb of electricity are consumed when
100 mA current is passed through a solution of AgNO3 EXERCISE-2
for 30 minute during an electrolysis experiment :
(A) 108 (B) 18000 COMPETITIVE EXAM PREVIOUS YEARS’ QUESTIONS :
(C) 180 (D) 3000
38. A current of 9.65 amp. flowing for 10 minute deposits 3.0 1. When a dilute solution of sulphuric acid is electrolysed
g of a metal. The equivalent wt. of the metal is : using platinum electrodes the gas evolved at the posi-
(A) 10 (B) 30 tive electrode is [IJSO Stage-1/2012-13]
(C) 50 (D) 96.5 (A) SO2 (B) SO3
(C) H2 (D) O2
39. During the electrolysis of fused NaCl, the
reaction that occurs at the anode is :
2. An electrochemical cell constructed for the reaction : Cu2+
(A) Chloride ions are oxidized
+ M(s)  Cu(s) + M2+ (aq) has an Eº = 0.75 V. The standard
(B) Chloride ions are reduced (aq)
reduction potential for Cu2+(aq) is 0.34 V. What is the stan-
(C) Sodium ions are oxidized
dard reduction potential for M2+(aq)?
(D) Sodium ions are reduced
[IJSO Stage-1/2013-14]
40. 108 g fairly concentrate solution of AgNO3 is electrolyzed (A) 1.09V (B) 0.410 V
using 0.1 F of electricity. The weight or resulting solution (C) –0.410 V (D) –1.09V
is :
(A) 94 g (B) 11.6 g
(C) 96.4 g (D) None

PAGE # 275
ANSWER KEY

1. MOLE CONCEPT

EXERCISE#1
Ques. 1 2 3 4 5 6 7 8 9 10 11 12 13 14 15
Ans. D A B B A D C B B A C B C A D
Ques. 16 17 18 19 20 21 22 23 24 25 26 27 28 29 30
Ans. D B C A A B D A D A D B B A C
Ques. 31 32 33 34 35 36 37 38 39 40 41 42 43 44 45
Ans. A A A C A A D B C C D D A D A
Ques. 46 47 48 49 50 51 52 53 54 55 56 57 58 59 60
Ans. C A B A D C C D A B A D B D A
Ques. 61 62 63 64 65 66 67 68 69 70 71 72 73 74 75
Ans. B B B B B D B C C A B B A A A
Ques. 76 77 78 79 80 81 82 83 84 85 86 87 88 89 90
Ans. A C A A A B A C B A C A B C C
Ques. 91 92 93 94 95 96 97 98 99 100 101 102 103 104 105
Ans. C B C B D D D B D D B C A C C
Ques. 106 107 108 109 110 111 112 113 114 115 116 117 118 119 120
Ans. A D C B D B B D B C A A D C C
Ques. 121 122 123
Ans. C B C

EXERCISE#2
Ques. 1 2 3 4 5 6 7 8 9 10 11 12 13 14 15
Ans. A D B D B C B C C D D C A D C
Ques. 16 17 18 19 20 21 22 23 24 25 26 27 28 29 30
Ans. A B D D D C B B D C A D A C
Ques. 31 32 33 34 35 36 37 38 39 40 41 42 43 44 45
Ans. C B C D C B A D B B C B D B B
Ques. 46 47 48 49
Ans. B C C A

2. EQUIVALENT CONCEPT
EXERCISE#1
Ques. 1 2 3 4 5 6 7 8 9 10 11 12 13 14 15
Ans. D D A C B B D C A C A B A D B
Ques. 16 17 18 19 20 21 22 23 24 25 26 27 28 29 30
Ans. B A C B A D B B C C B D C B D
Ques. 31 32 33 34 35 36 37 38 39 40 41
Ans. A C B B B C A B B B B

EXERCISE # 2
Ques. 1 2 3 4 5
Ans. A D C B D

PAGE # 276
3. STRUCTURE OF ATOM
EXERCISE#1
Ques. 1 2 3 4 5 6 7 8 9 10 11 12 13 14 15
Ans. B B C B D A A C A A B C D C B
Ques. 16 17 18 19 20 21 22 23 24 25 26 27 28 29 30
Ans. C B B B A B A B B B C D B C C
Ques. 31 32 33 34 35 36 37 38 39 40 41 42 43 44 45
Ans. D A C D C A B A B C C D A C B
Ques. 46 47 48 49 50 51 52 53 54 55 56 57 58 59 60
Ans. A D D D B D C A D A D B C A A

EXERCISE # 2
Ques. 1 2 3 4
Ans. C D D A

4. PERIODIC TABLE
EXERCISE # 1
Que s. 1 2 3 4 5 6 7 8 9 10 11 12 13 14 15 16
Ans. B A B A D A B A B B C D B C C C
Que s. 17 18 19 20 21 22 23 24 25 26 27 28 29 30 31 32
Ans. A B D D B D C C B A C A D D D A
Que s. 33 34 35 36 37 38 39 40 41 42 43 44 45 46 47 48
Ans. A A D D A B C A B B C D A D B B
Que s. 49 50
Ans. A C

EXERCISE#2
Ques. 1 2 3 4 5 6 7 8 9 10 11 12 13 14 15
Ans. A D A C A A A D C C A A D D C
Ques. 16 17 18
Ans. C B C

5. Matter
EXERCISE#1
Ques 1 2 3 4 5 6 7 8 9 10 11 12 13 14 15
Ans C D C A A B A A C B B A D C C
Ques 16 17 18 19 20 21 22 23 24 25 26 27 28 29 30
Ans A D D C B C C A A D B C B C B

EXERCISE#2
Ques 1 2 3 4 5 6 7 8
Ans D B D D B D D C

PAGE # 277
6. ACIDS AND BASES
EXERCISE#1

Ques. 1 2 3 4 5 6 7 8 9 10 11 12 13 14 15
Ans. B A C B A D D A B C B A B C A
Ques. 16 17 18 19 20 21 22 23 24 25 26 27 28 29 30
Ans. C D B D A A B D A B A D A B B
Ques. 31 32 33 34 35 36 37 38 39 40 41
Ans. D A B D C B A B D A B

EXERCISE#2
Ques. 1 2 3 4 5 6 7 8 9 10 11 12 13
Ans. B D C B C B B,D C C C B B D

7. METALS AND NON-METALS


EXERCISE#1
Ques. 1 2 3 4 5 6 7 8 9 10 11 12 13 14 15
Ans. D A B A B C C C D C C D C C C
Ques. 16 17 18 19 20 21 22 23 24 25 26 27 28 29 30
Ans. B D B A A D B C C B B B B B C
Ques. 31 32 33 34 35 36 37 38 39 40 41 42 43 44 45
Ans. B C D D D B B A C C A C B D C
Ques. 46 47 48 49 50
Ans. C C C A D

EXERCISE#2
Ques. 1 2 3 4 5 6 7 8 9 10 11 12 13 14 15
Ans. A A A A A B D B A C D D B B A
Ques. 16 17 18 19 20 21
Ans. C A B C C C

8. CARBON
EXERCISE#1
Que s. 1 2 3 4 5 6 7 8 9 10 11 12 13 14 15
Ans. C B A D A C B B B A D C A B C
Que s. 16 17 18 19 20 21 22 23 24 25 26 27 28 29 30
Ans. C A C B B C D C A C B B B C C
Que s. 31 32 33 34 35 36 37 38 39
Ans. C B C B B C B C A

EXERCISE#2
Ques. 1 2 3 4 5 6 7 8 9 10 11 12 13 14
Ans. A C C C C A A A B D D C C C

PAGE # 278
9. STUDY OF GAS LAWS
EXERCISE#1
Que. 1 2 3 4 5 6 7 8 9 10 11 12 13 14 15
Ans. A C B C B A C D B C A C C B A
Que. 16 17 18 19 20 21 22 23 24 25 26 27 28 29 30
Ans. C D C D B B C D C C A D D C A
Que. 31 32 33 34 35 36 37 38 39 40 41 42 43 44 45
Ans. A B B A C A D C A B D A B B B
Que. 46 47 48 49 50 51 52 53 54 55 56 57 58 59 60
Ans. A A B B A A B B A D B D B A B
Que. 61 62 63 64 65
Ans. D D D C B

EXERCISE#2
Que. 1 2 3 4 5 6 7 8 9 10 11
Ans. C C D B A D B C C B A

10. CHEMICAL AND IONIC EQUILIBRIUM

EXERCISE#1
Ques. 1 2 3 4 5 6 7 8 9 10 11 12 13 14 15
Ans. D C B D B C A A D D B A B B B
Ques. 16 17 18 19 20 21 22 23 24 25 26 27 28 29 30
Ans. A A A B C C C A D C B A B A D
Ques. 31 32 33 34 35 36 37 38 39 40 41 42 43 44 45
Ans. D D D D A C D C A B D D D A B
Ques. 46 47 48 49 50 51 52 53 54 55 56 57 58 59 60
Ans. A D C C D B C B B A A A B C C
Ques. 61 62 63 64 65 66 67 68 69 70 71 72 73 74
Ans. B B C A D D A A D C B B C A

EXERCISE#2
Ques. 1 2 3 4 5 6 7 8 9 10 11 12 13
Ans. D C C B C C C B D B A D C

11. CHEMICAL KINETICS


EXERCISE#1
Que. 1 2 3 4 5 6 7 8 9 10 11 12 13 14 15
Ans. C B A C B D B B C D D D B D B
Que. 16 17 18 19 20 21 22 23 24 25 26 27 28 29 30
Ans. B D A B C C C B A C C D B D B
Que. 31 32 33 34 35 36 37 38 39 40 41 42 43 44 45
Ans. C C A C B C B B D C D B C D A
Que. 46 47 48 49 50 51 52 53 54
Ans. A B B D C C A D B
EXERCISE#2
Que. 1 2 3
Ans. D D D

PAGE # 279
12. NUCLEAR CHEMISTRY

EXERCISE#1
Que s. 1 2 3 4 5 6 7 8 9 10 11 12 13 14 15
Ans. D D A C C C A C C A C A C D B
Que s. 16 17 18 19 20 21 22 23 24 25 26 27 28 29 30
Ans. D C C B D C D D B C B C D B D
Que s. 31 32 33 34 35 36 37 38 39 40 41
Ans. B C B C C A B B C A C

EXERCISE#2
Que s. 1 2 3 4 5
Ans. D B A B A

13. THERMODYNAMICS

EXERCISE#1
Ques. 1 2 3 4 5 6 7 8 9 10 11 12 13 14 15
Ans. B D A C D A B A A C C C D C D
Ques. 16 17 18 19 20 21 22 23 24 25 26 27 28 29 30
Ans. B B B C B B C C D B A A C C A
Ques. 31 32 33 34 35 36 37 38
Ans. D C B B C C B A

EXERCISE#2
Ques. 1 2 3 4 5 6 7 8 9
Ans. B B B C A D B B D

14. CHEMICAL BONDING

EXERCISE#1
Ques. 1 2 3 4 5 6 7 8 9 10 11 12 13 14 15
Ans. A B B C D C D D D C C A A A
Ques. 16 17 18 19 20 21 22 23 24 25 26 27 28 29 30
Ans. A D C D B A C C B D A D A C A
Ques. 31 32
Ans. C C

EXERCISE#2
Ques. 1 2 3 4 5 6
Ans. B D D D B A

PAGE # 280
15. QUALITATIVE ANALYSIS

Ques. 1 2 3 4 5 6 7 8 9 10 11 12 13 14 15
Ans. D C C A A D A D B B D B B B C
Ques. 16 17 18 19 20 21 22 23 24 25 26 27 28 29 30
Ans. C B A D B C A A C B B C C C D
Ques. 31 32 33 34 35 36 37 38 39 40 41 42 43 44 45
Ans. C B D C B C C B B C C D D B D
Ques. 46 47 48 49 50 51 52 53 54 55 56 57 58 59 60
Ans. A A C C A A D C C C B D C C B
Ques. 61 62 63 64 65 66 67 68 69 70 71 72 73 74 75
Ans. C C B D B B A A D B C C B C A
Ques. 76 77 78 79 80 81 82 83 84 85 86 87 88 89 90
Ans. A D D D B C C C D B A D B C B
Ques. 91 92 93 94 95 96 97 98
Ans. B D A D D B C C

EXERCISE#2
Ques. 1 2 3 4 5 6 7 8
Ans. C B C A A C D C

16. ELECTROCHEMISTRY
EXERCISE#1
Ques. 1 2 3 4 5 6 7 8 9 10 11 12 13 14 15
Ans. D B D D B B A C A A D A A A B
Ques. 16 17 18 19 20 21 22 23 24 25 26 27 28 29 30
Ans. C B A D D A D D A B A B C A B
Ques. 31 32 33 34 35 36 37 38 39 40 41 42 43 44 45
Ans. B C C B A B C C A C C C A D C
Ques. 46 47
Ans. C A

EXERCISE#2
Ques. 1 2
Ans. D C

PAGE # 281

Вам также может понравиться